Sunteți pe pagina 1din 386

P

ICAP
Practice Kit

Advanced accounting
and financial reporting
Second edition published by
Emile Woolf Limited
Bracknell Enterprise & Innovation Hub
Ocean House, 12th Floor, The Ring
Bracknell, Berkshire, RG12 1AX United Kingdom
Email: info@ewiglobal.com
www.emilewoolf.com

© Emile Woolf International, May 2017

All rights reserved. No part of this publication may be reproduced, stored in a retrieval system, or transmitted,
in any form or by any means, electronic, mechanical, photocopying, recording, scanning or otherwise, without
the prior permission in writing of Emile Woolf Publishing Limited, or as expressly permitted by law, or under
the terms agreed with the appropriate reprographics rights organisation.

You must not circulate this book in any other binding or cover and you must impose the same condition on
any acquirer.

Notice
Emile Woolf International has made every effort to ensure that at the time of writing the contents of this study
text are accurate, but neither Emile Woolf International nor its directors or employees shall be under any
liability whatsoever for any inaccurate or misleading information this work could contain.

© Emile Woolf International ii The Institute of Chartered Accountants of Pakistan


Certified Finance and Accounting Professional

C
Advanced accounting and financial reporting

Contents
Page

Question and Answers Index v


Section A Questions 1
Section B Answers 137

© Emile Woolf International iii The Institute of Chartered Accountants of Pakistan


Advanced accounting and financial reporting

© Emile Woolf International iv The Institute of Chartered Accountants of Pakistan


Certified Finance and Accounting Professional

I
Advanced accounting and financial reporting

Index to questions and answers

Question Answer
page page
CHAPTER 1 – REGULATORY FRAMEWORK
1.1 GENERAL PURPOSE FINANCIAL
1 137
STATEMENTS
CHAPTER 2 – ACCOUNTING AND REPORTING CONCEPTS
2.1 DEFINITIONS 2 139
2.2 CONCEPTUAL FRAMEWORK 2 140
2.3 CARRIE 2 141
CHAPTER 3 – PRESENTATION OF FINANCIAL STATEMENTS
3.1 CLIFTON PHARMA LIMITED 3 142
3.2 BSZ LIMITED 4 144
3.3 YASIR INDUSTRIES LIMITED 6 147
3.4 FIGS PAKISTAN LIMITED 7 151
3.5 FAZAL LIMITED 8 154
3.6 BABER LIMITED 9 155
3.7 GOLDEN LIMITED 9 155
3.8 METAL LIMITED 10 156
3.9 ENGINA 10 157
3.10 SHAZAD INDUSTRIES LTD 11 159
3.11 AZ 11 160
3.12 J-MART LIMITED 12 162
3.13 QALLAT INDUSTRIES LIMITED 13 163
3.14 SKYLINE LIMITED 13 163
3.15 WALNUT LIMITED 14 164

© Emile Woolf International v The Institute of Chartered Accountants of Pakistan


Advanced accounting and financial reporting

Question Answer
page page

CHAPTER 4 – IAS 8: ACCOUNTING POLICIES, CHANGES IN


ACCOUNTING ESTIMATES AND ERRORS
4.1 WONDER LIMITED 15 165
4.2 DUNCAN 15 167
4.3 MOHANI MANUFACTURING LIMITED 16 168
CHAPTER 5 – IFRS 15: REVENUE FROM CONTRACT WITH
CUSTOMER
5.1 PARVEZ LIMITED 17 169
5.2 SACHAL LIMITED 17 169
5.3 BRILLIANT LIMITED 17 171
5.4 WAQAS LIMITED 18 172
CHAPTER 6 – IAS 16: PROPERTY, PLANT AND EQUIPMENT
6.1 FAM 19 174
6.2 GUJRAT CONSTRUCTION LIMITED 19 175
CHAPTER 7 – NON-CURRENT ASSETS: SUNDRY STANDARDS
7.1 SPIN INDUSTRIES LIMITED 21 178
7.2 QURESHI STEEL LIMITED 21 179
7.3 IMRAN LIMITED 22 180
7.4 KATIE 23 181
7.5 ALNUS LTD AND BUTEA LIMITED 23 182
7.6 VICTORIA 24 184
CHAPTER 8 – IAS 38: INTANGIBLE ASSETS
8.1 BROOKLYN 25 185
8.2 RAISIN INTERNATIONAL 25 186
8.3 OXTAIL LIMITED 26 187
8.4 SKY LIMITED 27 188
8.5 COMFORT SHOES LIMITED 27 189
CHAPTER 9 – IAS 36: IMPAIRMENT OF ASSETS
9.1 CHARLOTTE 29 190
9.2 ABA LIMITED 29 193
9.3 HUSSAIN ASSOCIATES LTD 30 194
9.4 IMPS 31 195
CHAPTER 10 – IFRS 5: NON-CURRENT ASSETS HELD FOR SALE AND
DISCONTINUED OPERATIONS
10.1 SAUL 32 197
10.2 SHAHID HOLDINGS 33 198
10.3 PRIMA 33 200

© Emile Woolf International vi The Institute of Chartered Accountants of Pakistan


Index to questions and answers

Question Answer
page page
CHAPTER 11 – IFRS 16: LEASES
11.1 X LTD 35 202
11.2 PROGRESS LIMITED 35 202
11.3 MIRACLE TEXTILE LIMITED 35 204
11.4 ACACIA LTD 36 205
11.5 SHOAIB LEASING LIMITED 36 206
11.6 AKBAR LIMITED 36 208
11.7 ALI LIMITED 37 208
11.8 MOAZZAM TEXTILE MILLS LIMITED 37 209
11.9 MODIFICATION THAT DECREASES THE
SCOPE OF THE LEASE (IFRS 16, 38 211
ILLUSTRATIVE EXAMPLE 17)
11.10 MODIFICATION THAT BOTH
INCREASES AND DECREASES THE
38 212
SCOPE OF THE LEASE (IFRS 16,
ILLUSTRATIVE EXAMPLE 18)
11.11 SUBLEASE CLASSIFIED AS A FINANCE
LEASE (IFRS 16, ILLUSTRATIVE 38 213
EXAMPLE 20)
11.12 SUBLEASE CLASSIFIED AS AN
OPERATING LEASE (IFRS 16, 38 213
ILLUSTRATIVE EXAMPLE 21)
CHAPTER 12 – IAS 37: PROVISIONS CONTINGENT LIABILITIES AND
CONTINGENT ASSETS
12.1 ROWSLEY 39 214
12.2 MULTAN PETROCHEM LTD 40 215
12.3 VIOLET POWER LIMITED 40 216
CHAPTER 13 – IAS 19: EMPLOYEE BENEFITS FLOWS
13.1 LABURNUM LIMITED 42 217
13.2 JABEL LIMITED 42 217
13.3 KAGHZI LIMITED 42 218
13.4 LASURA LTD 43 218
13.5 UNIVERSAL SOLUTIONS 43 219
13.6 DHA INTERIORS LTD 44 220
CHAPTER 14 – IFRS 2: SHARE BASED PAYMENTS
14.1 TOSHACK LTD 45 223
14.2 IFRS 2 45 223
14.3 SAVAGE LTD 45 225
14.4 YORATH LTD 46 225
14.5 QUALTECH LTD 46 228

© Emile Woolf International vii The Institute of Chartered Accountants of Pakistan


Advanced accounting and financial reporting

Question Answer
page page
14.6 BRIDGE LTD 46 228
14.7 CAPSTAN LTD 47 227
14.8 NEWTOWN LTD 47 227
14.9 SINDH TRANSIT LTD 47 228
CHAPTER 15 – IFRS 9: FINANCIAL INSTRUMENTS: RECOGNITION
AND MEASUREMENT
15.1 AJI PANCA LTD 49 229
15.2 PASSILA LTD 49 230
15.3 FINANCIAL INSTRUMENTS 50 230
15.4 ESPANOLA LTD 50 232
15.5 SANDIA LTD 51 233
15.6 GEO ALLOYS LTD 51 233
15.7 CASCABEL LTD 51 234
15.8 FAIR VALUE HEDGE ACCOUNTING 52 234
15.9 CASH FLOW HEDGE ACCOUNTING 52 236
15.10 WATERS LTD 52 237
15.11 ARIF INDUSTRIES LIMITED 53 239
15.12 QASMI INVESTMENT LIMITED 54 240
CHAPTER 16 – IFRS 7: FINANCIAL INSTRUMENTS: PRESENTATION
AND DISCLOSURE
16.1 SERRANO LIMITED 55 242
16.2 POBLANO LIMITED 55 243
16.3 PIQUIN LTD 55 243
16.4 AJI LTD 56 244
16.5 CHILTEPIN LTD 56 245
16.6 HABENERO LTD 56 246
CHAPTER 17 – IFRS 13: FAIR VALUE MEASUREMENT
There are no specific questions in this area.
CHAPTER 18 – IAS 12: INCOME TAXES
18.1 SHAKIR INDUSTRIES 57 247
18.2 DWAYNE LTD (PART 1) 58 248
18.3 DWAYNE LTD (PART 2) 58 248
18.4 COHORT 59 249
18.5 MODEL TOWN GROUP 60 250
CHAPTER 19 – BUSINESS COMBINATIONS AND CONSOLIDATION
19.1 HELLO 62 252
19.2 HASAN LIMITED 62 253

© Emile Woolf International viii The Institute of Chartered Accountants of Pakistan


Index to questions and answers

Question Answer
page page
19.3 FLAMSTEED LTD AND HALLEY LTD 64 256
19.4 BRADLEY LTD 65 257
19.5 X LTD 66 259
19.6 KHAN LIMITED 67 261
CHAPTER 20 – CONSOLIDATED ACCOUNTS: STATEMENTS OF
PROFIT OR LOSS AND OTHER COMPREHENSIVE INCOME
20.1 MILLARD LTD 69 264
20.2 SHERLOCK LIMITED 70 266
20.3 FAISAL LIMITED 71 269
20.4 GOLDEN LIMITED 73 273
CHAPTER 21 – IAS 28: ASSOCIATES AND JOINT VENTURES
21.1 JOINT ARRANGEMENTS 75 275
21.2 HELIUM 75 275
21.3 HAMACHI LTD 75 277
21.4 HIDE 77 279
21.5 HARK, SPARK AND ARK 77 280
21.6 P, S AND A 78 283
21.7 H LTD GROUP 79 285
CHAPTER 22 – IFRS 3: BUSINESS COMBINATIONS ACHIEVED IN
STAGES
22.1 STEP ACQUISITION 81 286
22.2 A LTD 81 286
22.3 X LTD GROUP 82 289
22.4 PLAIN LTD 84 291
22.5 MANGO LTD 85 294
CHAPTER 23 – COMPLEX GROUPS
23.1 PARVEZ LTD 87 297
23.2 HASAN, RIAZ AND SIDDIQ 88 300
CHAPTER 24 – DISPOSAL OF SUBSIDIARIES
24.1 PATCHE LTD 90 304
24.2 DISPOSAL 91 306
24.3 PART DISPOSAL 91 307
24.4 THE A GROUP 91 307
24.5 BARTLETT LTD 92 309
CHAPTER 25 – OTHER GROUP STANDARDS
There are no specific questions in this area. The topic is covered as parts of
other questions.

© Emile Woolf International ix The Institute of Chartered Accountants of Pakistan


Advanced accounting and financial reporting

Question Answer
page page
CHAPTER 26 – IAS 21: FOREIGN CURRENCY
26.1 DND LIMITED 93 311
26.2 STARLIGHT LIMITED 93 311
26.3 PERCEPT LTD 94 312
26.4 ORLANDO 95 313
26.5 MANCASTER AND STOCKPOT 95 313
26.6 A, B AND C 97 316
26.7 OMEGA LIMITED 98 319
26.8 PARENT COMPANY LIMITED 99 320
CHAPTER 27 – IAS 7: STATEMENTS OF CASH FLOWS
27.1 EVERNEW LTD 101 323
27.2 BELLA 102 325
27.3 BISHOP GROUP 103 326
27.4 THE GRAPE GROUP 105 329
CHAPTER 28 – IAS 33: EARNINGS PER SHARE
28.1 AIRCON LTD 108 331
28.2 CACHET LTD 109 333
28.3 MARY 109 333
28.4 MANDY 110 334
28.5 AAZ LIMITED 110 336
28.6 ABC LIMITED 111 337
28.7 ALPHA LIMITED 111 339
CHAPTER 29 – ANALYSIS AND INTERPRETATION OF FINANCIAL
STATEMENTS
29.1 ALPHA LIMITED AND OMEGA LIMITED 113 341
29.2 COOK LIMITED 114 342
29.3 FITZROY LIMITED 115 343
29.4 TRAVELWELL LTD 116 346
29.5 SACHAL LIMITED 118 348
29.6 OPAL INDUSTRIES LIMITED 118 349
CHAPTER 30 – SUNDRY STANDARDS AND INTERPRETATIONS
30.1 GUJRANWALA FOODS LIMITED 120 351
30.2 WAH AGRIPROD LTD 121 352
30.3 HELIOS GROUP 122 354
30.4 FASHION BLUE ENTERPRISES 123 355
30.5 KHAN LIMITED 124 356
30.6 AFRIDI 124 357

© Emile Woolf International x The Institute of Chartered Accountants of Pakistan


Index to questions and answers

Question Answer
page page
CHAPTER 31 – IFRS 1: FIRST TIME ADOPTION OF IFRS
31.1 IFRS 1 126 359
CHAPTER 32 – SPECIALISED FINANCIAL STATEMENTS
32.1 IFRS FOR SMES 127 361
32.2 AKMAL GENERAL INSURANCE LIMITED 127 362
32.3 MAHFOOZ GENERAL INSURANCE
128 363
LIMITED
32.4 DEE GENERAL INSURANCE LIMITED 128 364
32.5 BANK LATEEF BANK LIMITED 129 364
32.6 SECURED BANK LIMITED 129 365
32.7 AL-AMIN BANK LIMITED 130 366
32.8 BLUE-CHIP ASSET MANAGEMENT
130 366
LIMITED
32.9 A-ONE ASSET MANAGEMENT FUND
131 367
LIMITED
32.10 IAS 26 131 367
32.11 SOGO LIMITED 131 368
CHAPTER 33 – INTERNATIONAL PUBLIC SECTOR ACCOUNTING
STANDARDS
There are no specific questions in this area.
CHAPTER 34 – IAS 29: FINANCIAL REPORTING IN HYPERINFLATION
ECONOMIES
There are no specific questions in this area.
CHAPTER 35 – ISLAMIC ACCOUNTING STANDARDS
There are no specific questions in this area.
CHAPTER 36 – ETHICAL ISSUES IN FINANCIAL REPORTING
36.1 ETHICAL ISSUES 133 370
36.2 SINDH INDUSTRIES LTD 133 370
36.3 SOHAIB AND OMAR 134 373
36.4 ABBAS AND BASHIR 135 374

© Emile Woolf International xi The Institute of Chartered Accountants of Pakistan


Advanced accounting and financial reporting

© Emile Woolf International xii The Institute of Chartered Accountants of Pakistan


Certified Finance and Accounting Professional

A
Advanced accounting and financial reporting

SECTION
Questions
CHAPTER 1 – REGULATORY FRAMEWORK

1.1 GENERAL PURPOSE FINANCIAL STATEMENTS


The IFRS are generally accepted as accounting standards in the preparation of general purpose
financial statements in many countries of the world.
Required
(a) Briefly explain the meaning of general purpose financial statements in accordance with IAS
1 (Presentation of Financial Statements)
(b) Explain briefly any FOUR possible reasons for the prevalence of IFRS in many countries of
the world.
(c) Explain the arguments in support and against financial reporting standards.

© Emile Woolf International 1 The Institute of Chartered Accountants of Pakistan


Advanced accounting and financial reporting

CHAPTER 2 – ACCOUNTING AND REPORTING CONCEPTS

2.1 DEFINITIONS
“A statement of financial position is a snapshot of a business at a point in time. It shows the
assets that an entity owns and the liabilities that it owes. This is all that is required to convey a
business’s performance, position and adaptability.
As income generated and expenses incurred by a business are already reflected within the
assets and liabilities shown in the statement of financial position, a statement of profit or loss is a
superfluous statement.”
Required
Briefly appraise the validity of the above statement, defining the words underlined.

2.2 CONCEPTUAL FRAMEWORK


(a) Explain the term “Conceptual Framework” in relation to International Financial Reporting
Standards (IFRS).
(b) Define assets and liabilities.
(c) The International Accounting Standards Board’s framework for the preparation of financial
statements requires that entities should comply with certain accounting concepts and
underlying assumptions which include:
(i) Materiality;
(ii) Comparability; and
(iii) Faithful Representation.
Explain briefly the meaning of these concepts.
(d) Discuss the information needs of the following users of a company’s financial statements:
(i) Lenders;
(ii) Suppliers;
(iii) Customers ;
(iv) Employees; and
(v) Government and its agencies.

2.3 CARRIE
Carrie starts in business on 1 January Year 1. Carrie’s sole shareholder contributed capital of
Rs.1,000. Carrie purchased one item of inventory for Rs.1,000 and sold that inventory for cash of
Rs.1,400. At the end of Year 1 the replacement cost of the same item of inventory is Rs.1,100.
General inflation during the year was 7%.
Required
Calculate the profit for the year and set out a summary statement of financial position as of 31
December Year 1 under the following capital maintenance concepts.
(a) Physical capital maintenance
(b) Financial capital maintenance
(i) Historical cost accounting
(ii) Constant purchasing power accounting

© Emile Woolf International 2 The Institute of Chartered Accountants of Pakistan


Questions

CHAPTER 3: PRESENTATION OF FINANCIAL STATEMENTS

3.1 CLIFTON PHARMA LIMITED


The following trial balance relates to Clifton Pharma Limited, a public listed company, at 30
September 2016.
Rs.in ‘000
Dr Cr
Cost of sales 134,000
Operating expenses 35,000
Loan interest paid (note 1) 1,500
Rental of vehicles (note 2) 8,600
Revenue 338,300
Investment income 2,000
Leasehold property at cost (note 4) 250,000
Plant and equipment at cost 197,000
Accumulated depreciation at 1 October 2015:
- leasehold property 40,000
- plant and equipment 47,000
Investments 92,400
Share capital 280,000
Share premium 20,000
Retained earnings at 1 October 2015 19,300
Loan notes (note 1) 50,000
Deferred tax balance at 1 October 2015 (note 5) 20,000
Inventory at 30 September 2016 23,700
Trade receivables 76,400
Trade payables 14,100
Bank 12,100
830,700 830,700

The following notes are relevant:


(1) The effective interest rate on the loan notes is 6% per year.
(2) There are two separate contracts for rental of vehicles. A recent review by the finance
department of these contracts has reached the conclusion that Rs. 7 million of the total
rental cost of vehicles relates to a lease rather than a rental arrangement.
The lease was entered into on 1 October 2015 which was when the Rs. 7 million was paid:
the lease agreement is for a four-year period in total, and there will be three more annual
payments in advance of Rs. 7 million, payable on 1 October in each year. The vehicles in
the lease agreement had a fair value of Rs. 24 million at 1 October 2015 and they should
be depreciated using the straight line method to a nil residual value. The interest rate
implicit in the lease is 10% per year. The other contract for vehicle rental is a lease and the
rental payment should be charged to operating expenses. (Note: You are not required to
calculate the present value of the lease payments for the lease.)
(3) Other plant and equipment is depreciated at 20% per year by the reducing balance
method.
All depreciation of property, plant and equipment should be charged to cost of sales.

© Emile Woolf International 3 The Institute of Chartered Accountants of Pakistan


Advanced accounting and financial reporting

(4) The leasehold property has a 25-year life and is amortised at a straight-line rate. On 30
September 2016 the leasehold property was revalued to Rs. 220 million and the
directors wish to incorporate this revaluation in the financial statements.
(5) The provision for income tax for the year ended 30 September 2016 has been estimated at
Rs. 18 million. At 30 September 2016 there are taxable temporary differences of Rs. 92
million. The rate of income tax on profits is 25%.
Required
(a) Prepare a statement of profit or loss for Clifton Pharma Limited for the year to 30
September 2016
(b) Prepare a statement of financial position for Clifton Pharma Limited as at 30 September
2016

3.2 BSZ LIMITED


The post closing trial balance of BSZ Limited, a listed company, as at June 30, 2016 is given
below:
Debit Credit
Rs.in million
Cash at banks – current accounts 7
Cash at banks – in saving accounts 22
Stocks in trade – closing 90
Accounts receivable 60
Provision for bad debts 3
Advances to suppliers 16
Advances to staff 6
Short term deposits 11
Prepayments 4
Sales tax receivable 12
Freehold land – at revalued amount 375
Furniture and fixtures - cost 27
Accumulated depreciation – Furniture and fixtures 8
Machines - cost 85
Accumulated depreciation – Machines 27
Building on freehold land – cost 150
Accumulated depreciation – Building 26
Computer software – cost 10
Accumulated amortization – Computer software 2
Deferred taxation 40
Short term loan 85
Accounts payable 75
Accrued liabilities 7
Provision for taxation 17
Issued, subscribed and paid up capital (Rs. 10 each) 400
Surplus on revaluation of fixed assets 120
Accumulated profits 65
––––––– –––––––
875 875
═════ ═════

© Emile Woolf International 4 The Institute of Chartered Accountants of Pakistan


Questions

Additional Information:
(i) The first revaluation of freehold land was carried out in 2012 and resulted in a surplus
of Rs. 120 million. The valuation was carried out under market value basis by an
independent valuer, Mr. Dee, Chartered Civil Engineer of M/s SSS Consultants (Pvt.) Ltd.,
Islamabad.
(ii) The details relating to additions, disposal and depreciation/amortization of fixed assets,
during the year 2016 are given below:
 The company uses the straight line method for charging depreciation and
amortization. The building is depreciated at a rate of 5% whereas 10% is charged
on machines, furniture and fixtures and computer software.
 Construction on third floor of the building commenced on March 1, 2016 and is
expected to be completed on September 30, 2016. The cost incurred during the
year i.e. Rs. 20 million was capitalised on June 30, 2016.
 Furniture and fixtures worth Rs. 8 million were purchased on April 1, 2016.
 A machine was sold on February 29, 2016 to NJ Enterprise at a price of Rs. 13
million. At the time of disposal, the cost and written down value of the machine
was Rs. 15 million and Rs. 10 million respectively.
(iii) 50% of the accounts receivable were secured and considered good. 10% of the
unsecured accounts receivable were considered doubtful. Bad debts expenses for the
year amounted to Rs. 1.0 million. An amount of Rs. 1.4 million was written off during the
year.
(iv) All advances given to suppliers are considered good and include an amount of Rs. 4.0
million paid for goods which will be supplied on December 31, 2017.
(v) Cash at banks in saving accounts carry interest / mark-up ranging from 3% to 7% per
annum.
(vi) The authorised share capital of the company is Rs. 500 million.
Required
Prepare the statement of financial position as at June 30, 2016 along with the relevant notes
showing all possible disclosures as required under the International Accounting Standards
and the Companies Act, 2017.
(Comparative figures and the note on accounting policies are not required.)

© Emile Woolf International 5 The Institute of Chartered Accountants of Pakistan


Advanced accounting and financial reporting

3.3 YASIR INDUSTRIES LIMITED


The following trial balance related to Yasir Industries Limited (YIL) for the year ended June 30,
2016:
Dr Cr
Rs.in million
Ordinary share capital (Rs. 10 each) - 120.00
Retained earnings - 10.20
Sales - 472.40
Purchases 175.70 -
Production labour 61.00
Manufacturing overheads 39.00
Inventories (July 1, 2015) 38.90
Administrative expenses 40.00 -
Distribution expenses 19.80 -
Financial charges 0.30 -
Cash and bank - 13.25
Trade creditors - 30.40
Accrued expenses - 16.20
10% redeemable preference shares - 40.00
Debentures - 80.00
Deferred tax (July 1, 2015) - 6.00
Suspense account 30.00 -
Leasehold property - at cost 230.00 -
Machines – at cost 168.60 -
Software – at cost 20.00 -
Acc. depreciation – Leasehold property (June 30, 2016) - 40.25
Acc. depreciation – Machines (June 30, 2016) - 48.60
Acc. amortization – Software (June 30, 2016) - 12.00
Trade receivables 66.00 -
889.30 889.30

Additional Information:
(i) Sales include an amount of Rs. 27 million, made to a customer under sale or return
agreement. The sale has been made at cost plus 20% and the expiry date for the return of
these goods is July 31, 2016.
(ii) The value of inventories at June 30, 2016 was Rs. 42 million.
(iii) A fraud of Rs. 30 million was discovered in October 2015. A senior employee of the
company, who left in June 2015, had embezzled the funds from YIL’s bank account. The
chances of recovery are remote. The amount is presently appearing in the suspense
account.
(iv) On January 1, 2016 YIL issued debenture certificates which are repayable in 2021.
Interest is paid on these at 12% per annum.
(v) Financial charges comprise bank charges and bank commission.
(vi) The provision for current taxation for the year ended June 30, 2016 after making all the
above adjustments is estimated at Rs. 16.5 million.
(vii) The carrying value of YIL’s net assets as on June 30, 2016 exceeds their tax base by Rs.
30 million. The income tax rate applicable to the company is 30%.

© Emile Woolf International 6 The Institute of Chartered Accountants of Pakistan


Questions

(viii) On July 1, 2015, the leasehold property having a useful life of 40 years was revalued at
Rs. 238 million. No adjustment in this regard has been made in the books.
(ix) Depreciation of leasehold property is charged using the straight line method. 50% of
depreciation is allocated to manufacturing, 30% to administration and 20% to selling and
distribution.
Required
In accordance with the requirements of the Companies Act, 2017 and International Accounting
Standards, prepare the:
(a) statement of financial position as of June 30, 2016.
(b) statement of profit or loss and other comprehensive income for the year ended June 30,
2016.
(Comparative figures and notes to the financial statements are not required.)

3.4 FIGS PAKISTAN LIMITED


Figs Pakistan Limited is a listed company engaged in the business of manufacturing and
marketing of personal care and food products. Following is an extract from its trial balance for
the year ended 31 December 2016:
Debit Credit
Rs.in million
Sales - Manufactured goods 56,528
Sales - Imported goods 1,078
Scrap sales 16
Dividend income 12
Return on savings account 2
Sales tax - Imported goods 53
Sales tax - Manufactured goods 10,201
Sales discount 2,594
Raw material stock as on 1 January 2016 1,751
Work in process as on 1 January 2016 73
Finished goods (manufactured) as on 1 January 2016 1,210
Finished goods (imported) as on 1 January 2016 44
Purchases - Raw material 22,603
Purchases - Imported goods 658
Stores and spares consumed 180
Salaries, wages and benefits 2,367
Utilities 734
Depreciation and amortization 1,287
Stationery and office expenses 230
Repairs and maintenance 315
Advertisement and sales promotion 4,040
Outward freight and handling 1,279
Legal and professional charges 71
Auditor's remuneration 13
Donations 34
Workers Profit Participation Fund 257
Worker Welfare Fund 98
Loss on disposal of property, plant and equipment 10
Financial charges on short term borrowings 133
Exchange loss 22
Financial charges on lease 11

© Emile Woolf International 7 The Institute of Chartered Accountants of Pakistan


Advanced accounting and financial reporting

Additional information:
(i) The position of inventories as at 31 December 2016 was as follows:

Rs. m
Raw material 2,125
Work in process 125
Finished goods (manufactured) 1,153
Finished goods (imported) 66

(ii) The basis of allocation of various expenses among cost of sales, distribution costs and
administrative expenses are as follows:

Cost of Distribution Administrative


sales costs expenses
% % %
Salaries, wages and benefits 55 30 15
Depreciation and amortization 70 20 10
Stationery and office expenses 25 40 35
Repairs and maintenance / Utilities 85 5 10

(iii) Salaries, wages and benefits include contributions to provident fund (defined contribution
plan) and gratuity fund (defined benefit plan) amounting to Rs. 54 million and Rs. 44 million
respectively.
(iv) Auditor’s remuneration includes taxation services and out-of-pocket expenses amounting
to Rs. 4 million and Rs. 1 million respectively.
(v) Donations include Rs. 5 million given to Dates Cancer Foundation (DCF). One of the
company’s directors, Mr. Peanut is a trustee of DCF.
(vi) The tax charge for the current year after making all related adjustments is estimated at Rs.
1,440 million. Taxable temporary differences of Rs. 3,120 originated in the year million,
over the last year. The applicable income tax rate is 35%.
(vii) 274 million ordinary shares were outstanding as on 31 December 2016.
(viii) There is no other comprehensive income for the year.
Required
Prepare the statement of profit or loss and other comprehensive income for the year ended 31
December 2016 along with the relevant notes showing required disclosures as per the
Companies Act, 2017 and International Financial Reporting Standards. Comparatives are not
required.

3.5 FAZAL LIMITED


Fazal Limited is engaged in the manufacturing of specialized spare parts for automobile
assemblers. During the year 2016, the company has undertaken the following transactions with its
related parties:
(i) Sales of Rs. 500 million were made to its only subsidiary M/s Sami Motors Limited
(SML). Being the subsidiary, a special discount of Rs. 25 million was allowed to SML.
(ii) SML returned spare parts worth Rs. 5.5 million.
(iii) Raw materials of Rs. 5 million were purchased from Jalal Enterprises, which is owned
by the wife of the CFO of Fazal Limited.
(iv) Equipment worth Rs. 3 million was purchased from Khan Limited (KL). The wife of the
Production Director of the company is a director in KL.

© Emile Woolf International 8 The Institute of Chartered Accountants of Pakistan


Questions

(v) The company awarded a contract for supply of two machines amounting to Rs. 7 million
per machine to an associated company.
(vi) In 2014, an advance of Rs. 2 million was given to the Chief Executive of the
company. During the year 2016, he repaid Rs.0.3 million. The balance outstanding as on
December 31, 2016 was Rs. 1,100,000.
Required
Prepare a note for inclusion in the company’s financial statements in accordance with the
requirement of IAS 24: Related Party Disclosures.

3.6 BABER LIMITED


During the year ended June 30, 2016, Baber Limited (BL) has carried out several
transactions with the following individuals/entities:
(i) AK Associates provides information technology services to BL. One of the directors of BL
is also the partner in AK Associates.
(ii) SS Bank Limited is the main lender. By virtue of an agreement it has appointed a nominee
director on the Board of BL.
(iii) Mr. Zee who supplies raw materials to BL, is the brother of the Chief Executive Officer of
the company.
(iv) JB Limited is the distributor of BL’s products and has exclusive distribution rights for
the province of Punjab.
(v) Mr. Tee is the General Manager-Marketing of BL and is responsible for all major decisions
made in respect of sales prices and discounts.
(vi) BL’s gratuity fund is administered by the Trustees appointed by the company.
(vii) MM Limited is the leading supplier of BL and supplies 60% of BL’s raw materials.
(viii) Ms. Vee who conducted various training programmes for the employees of the
company, is the wife of BL’s Chief Executive Officer.
Required
Comment as to whether the above individuals/entities are ‘related parties’ of the company or not.
Support your arguments with references from International Accounting Standards.

3.7 GOLDEN LIMITED


The following related party transactions were carried out by Golden Limited (GL) during the
first year of its operation i.e. year ended December 31, 2016.
(i) Inventory costing Rs. 15 million was sold for Rs. 18 million to Platinum Limited (PL) which
owns 60% shares in GL. It is GL’s policy to add 30% margin on cost. Outstanding liability
at year end, in respect of these purchases was Rs. 6.5 million.
(ii) PL provided administrative services to GL. The cost of these services, if billed in the
open market, would have amounted to Rs. 350,000. No entries were made to record these
transactions, as it was agreed that the services would be provided free of charge.
(iii) A property was sold to Silver Limited (SL), an associated company, at its fair market value
of Rs. 10 million. 50% of the amount was settled prior to year end. GL reimbursed Rs.
500,000 to SL on account of transfer and other incidental charges related to this
property.
(iv) An interest free loan of Rs. 2 million was granted to an executive director of the company
under the terms of employment. During the year, Rs. 200,000 were repaid by the executive
director.
(v) On July 1, 2016 GL obtained a short term loan of Rs. 25 million from one of its major
shareholder, at the prevailing annual interest rate of 12%. The principal as well as the
accrued mark-up were outstanding at the close of the year.

© Emile Woolf International 9 The Institute of Chartered Accountants of Pakistan


Advanced accounting and financial reporting

Required
Prepare a note on related party transactions for inclusion in GL’s financial statements for the year
ended December 31, 2016 showing disclosures as required under IAS - 24 (Related Party
Disclosures).

3.8 METAL LIMITED


On 1 July 2015, Metal Limited (ML) acquired 80% shareholdings in Copper Limited (CL), 90%
shareholdings in Zinc Limited (ZL) and 55% shareholdings in Steel Limited (SL). The following
transactions took place among these companies, during the period up to 30 June 2016:
(i) On 1 May 2015, ML sold a machine to CL at 20% above the carrying amount of Rs. 16
million. CL paid the entire amount on 15 July 2015. The useful life of the machine is 10
years.
(ii) On 1 July 2015, ZL awarded a contract of Rs. 15 million to Iron Builders and Developers
(IBD) for the extension of its existing factory. One of the directors of ML is also a partner in
IBD.
(iii) Since the date of acquisition, ML has been providing management services to CL and ZL.
ML did not charge management fee for its services during the first year. However, with
effect from 1 July 2015, management fee has been charged from each company at the
rate of Rs.0.5 million per month. Payment is made on the 10th day of the next month.
(iv) On 1 January 2016, ML sold goods amounting to Rs. 10 million to Gold Limited (GL). The
wife of chief financial officer of ZL is a major shareholder in GL.
Required
Prepare a note on related party disclosure including comparative figures, for inclusion in the
individual financial statements of ML, CL, ZL and SL, for the year ended 30 June 2016.

3.9 ENGINA
Engina, a foreign company has approached a partner in your firm to assist in obtaining local
stock exchange listing (or stock market registration) for the company. Engina is registered in a
country where transactions between related parties are considered to be normal but where such
transactions are not disclosed. The directors of Engina are reluctant to disclose the nature of
their related party transactions as they feel that although they are a normal feature of business in
their part of the world, it could cause significant problems politically and culturally to disclose
such transactions.
The partner in your firm has requested a list of all transactions with parties connected with the
company and the directors of Engina have produced the following summary:
(a) Every month, Engina sells Rs. 50,000 of goods per month to Mr Satay, the financial
director. The financial director has set up a small retailing business for his son and the
goods are purchased at cost price for him. The annual turnover of Engina is Rs. 300
million. Additionally, Mr Satay has purchased his company car from the company for Rs.
45,000 (market value Rs. 80,000). The director, Mr Satay, earns a salary of Rs. 500,000 a
year, and has a personal fortune of many millions of pounds.
(b) A hotel property had been sold to a brother of Mr Soy, the Managing Director of Engina, for
Rs. 4 million (net of selling cost of Rs.0.2 million). The market value of the property was
Rs. 4.3 million but prices have been falling rapidly. The carrying value of the hotel was Rs.
5 million and its value in use was Rs. 3.6 million. There was an over-supply of hotel
accommodation due to government subsidies in an attempt to encourage hotel
development and the tourist industry.
(c) Mr Satay owns several companies and the structure of the group is outlined below. Engina
earns 60% of its profits from transactions with Car and 40% of its profits from transactions
with Wheel. All of the above companies are incorporated in the same country.

© Emile Woolf International 10 The Institute of Chartered Accountants of Pakistan


Questions

Required
Write a report to the directors of Engina setting out the reasons why it is important to disclose
related party transactions and the nature of any disclosure required for the above transactions
under IAS 24 Related Party Disclosures.

3.10 SHAZAD INDUSTRIES LIMITED


Shazad Industries Ltd has recently acquired four large subsidiaries. These subsidiaries
manufacture products which are of different lines from those of the parent company. The parent
company manufactures plastics and related products whereas the subsidiaries manufacture the
following:

Product Location
Subsidiary 1 Textiles Karachi
Subsidiary 2 Car products Lahore
Subsidiary 3 Fashion garments Peshawar
Subsidiary 4 Furniture items Multan

The directors have purchased these subsidiaries in order to diversify their product base but do
not have any knowledge of the information required in the financial statements regarding these
subsidiaries other than the statutory requirements.
Required
(a) Explain to the directors the purpose of segmental reporting of financial information.
(b) Explain to the directors the criteria which should be used to identify the separate reportable
segments. (You should illustrate your answer by reference to the above information)
(c) Critically evaluate IFRS 8, Operating segments, setting out any problems with the
standard.

3.11 AZ
(a) For enterprises that are engaged in different businesses with differing risks and
opportunities, the usefulness of financial information concerning these enterprises is
greatly enhanced if it is supplemented by information on individual business segments.
Required
(i) Explain why the information content of financial statements is improved by the
inclusion of segmental data on individual business segments.
(ii) Discuss how IFRS 8 requires that segments be analysed.

© Emile Woolf International 11 The Institute of Chartered Accountants of Pakistan


Advanced accounting and financial reporting

(b) AZ, a public limited company, operates in the global marketplace.


(i) The major revenue-earning asset is a fleet of aircraft which are registered locally
and its other main source of revenue comes from the sale of holidays. The directors
are unsure as to how to identify business segments.
(ii) The company also owns a small aircraft manufacturing plant which supplies aircraft
to its domestic airline and to third parties. The preferred method for determining
transfer prices for these aircraft between the group companies is market price, but
where the aircraft is of a specialised nature with no equivalent market price, the
companies negotiate a price for the aircraft.
(iii) The company has incurred an exceptional loss on the sale of several aircraft to a
foreign government. This loss occurred on a fixed price contract signed several
years ago for the sale of second hand aircraft and resulted from the fluctuation of
exchange rates between the two countries.
(iv) During the year, the company decided to discontinue its holiday business as a result
of competition in the sector. This plan had been approved by the board of directors
and announced in the press.
(v) The company owns 40% of the ordinary shares of Eurocat, an unquoted company
which specialises in the manufacture of aircraft engines and has operations in China
and Russia. The investment is accounted for by the equity method and it is
proposed to exclude the company’s results from segment assets and revenue.
Required
Discuss the implications of each of the above points for the determination of the segmental
information required to be prepared and disclosed under IFRS 8 Operating Segments and
other relevant International Accounting Standards.

3.12 J-MART LIMITED


(a) Explain the terms “adjusting events” and “non-adjusting events” and give three examples of
each.
(b) J-Mart Limited, a chain of departmental stores has distributed its operations into four
Divisions i.e. Food, Furniture, Clothing and Household Appliances. The following
information has been extracted from the records:
(i) The company allows the dissatisfied customers to return the goods within 30 days.
It is estimated that 5% of the sales made in June 2016 will be refunded in July 2016.
(ii) On June 2, 2016, three employees were seriously injured as a result of a fire at
the company’s warehouse. They have lodged claims seeking damages of Rs. 2.0
million from the company. The company’s lawyers have advised that it is probable
that the court may award compensation of Rs. 400,000.
(iii) Under a new legislation, the company is required to fit smoke detectors at all the
stores by December 31, 2016. The company has not yet installed the smoke
detectors.
(iv) On June 20, 2016, the board of directors decided to close down the Household
Appliances Division. However, the decision was made public after June 30,
2016.
(v) The company has a large warehouse in Lahore which was acquired under a
three-year rent agreement signed on April 1, 2015. The agreement is non-
cancellable and the company cannot sub-let the warehouse. However, due to
operational difficulties, the company shifted the warehouse to a new location.
(vi) A 15% cash dividend was declared on July 5, 2016.
Required
Describe how each of the above issue should be dealt with in the financial statements for
the year ended June 30, 2016. Support your point of view in the light of relevant
International Accounting Standards.

© Emile Woolf International 12 The Institute of Chartered Accountants of Pakistan


Questions

3.13 QALLAT INDUSTRIES LIMITED


The following information pertains to Qallat Industries Limited (QIL) for its financial year ended
June 30, 2016:
(i) QIL sells all its products on one-year warranty which covers all types of defects.
Previous history indicates that 2% of the products contain major defects whereas 10%
have minor defects. It is estimated that if major defects were detected in all the products
sold, repair cost of Rs. 150 million would result. If minor defects were detected in all
products sold, repair cost of Rs. 70 million would result. Total sales for the year are
amounted to Rs. 830 million.
(ii) QIL has two large warehouses, A and B. These were acquired under non-cancellable
lease agreements. Details are as follows:

Warehouse A Warehouse B

Effective date of agreement July 1, 2011 January 1, 2014

Lease period 10 years 8 years

Rental amount per month Rs. 450,000 Rs. 300,000

On account of serious operating difficulties, QIL vacated both the warehouses on January
1, 2016 and moved to a warehouse situated close to its factory. On the same day QIL
sub-let Warehouse A at Rs. 250,000 per month for the remaining lease period.
Warehouse B was sub-let on March 1, 2016 for Rs. 350,000 per month for the remaining
lease period.
(iii) On July 18, 2016, QIL was sued by an employee claiming damages for Rs. 6 million on
account of an injury caused to him due to alleged violation of safety regulations on the part
of the company, while he was working on the machine on June 15, 2016. Before filing the
suit, he contacted the management on June 29, 2016 and asked for compensation of Rs. 4
million which was turned down by the management. The lawyer of the company
anticipates that the court may award compensation ranging between Rs. 1.5 million to Rs.
3 million. However, in his view the most probable amount is Rs. 2 million.
(iv) On November 1, 2015 a new law was introduced requiring all factories to install
specialised safety equipment within four months. The Equipment costing Rs. 5.0 million
was ordered on December 15, 2015 against 100% advance payment but the supplier
delayed installation to July 31, 2016. On August 5, 2016 the company received a notice
from the authorities levying a penalty of Rs.0.4 million i.e. Rs.0.1 million for each month
during which the violation continued. QIL has lodged a claim for recovery of the penalty
from the supplier of the equipment.
Required
Describe how each of the above issues should be dealt with in the financial statements for the
year ended June 30, 2016. Support your answer in the light of relevant International Accounting
Standards and quantify the effect where possible.

3.14 SKYLINE LIMITED


The following information pertains to Skyline Limited (SL) for the financial year ended December
31, 2016:
(i) A customer who owed Rs. 1 million was declared bankrupt after his warehouse was
destroyed by fire on February 10, 2017. It is expected that the customer would be able to
recover 50% of the loss from the insurance company.
(ii) An employee of SL forged the signatures of directors and made cash withdrawals of Rs.
7.5 million from the bank. Of these, Rs. 1.5 million were withdrawn before December 31,
2016. Investigations revealed that an employee of the bank was also involved and
therefore, under a settlement arrangement, the bank paid 60% of the amount to SL on
January 27, 2017.

© Emile Woolf International 13 The Institute of Chartered Accountants of Pakistan


Advanced accounting and financial reporting

(iii) SL has filed a claim against one of its vendors for supplying defective goods. SL’s legal
consultant is confident that damages of Rs. 1 million would be paid to SL. The supplier has
already reimbursed the actual cost of the defective goods.
(iv) A suit for infringement of patents, seeking damages of Rs. 2 million, was filed by a third
party. SL’s legal consultant is of the opinion that an unfavourable outcome is most likely.
On the basis of past experience he has advised that there is 60% probability that the
amount of damages would be Rs. 1 million and 40% likelihood that the amount would be
Rs. 1.5 million.
Required
Advise SL about the amount of provision that should be incorporated and the disclosures that are
required to be made in the financial statements for the year ended December 31, 2016.

3.15 WALNUT LIMITED


Walnut Limited (WL) is engaged in the business of import and distribution of electronic
appliances.
The following events took place subsequent to the reporting period i.e. 31 December 2016:
(i) On 15 January 2017, one of WL’s competitors announced launching of an upgraded
version of DVD players. WL’s inventories include a large stock of existing version of DVD
players which are valued at Rs. 15 million. Because of the introduction of the upgraded
version, the net realizable value of the existing version in WL’s inventory at 31 December
2016 has reduced to Rs. 12.5 million.
(ii) On 20 December 2016, the board of directors decided to close down the division which
imports and sells mobile phones. This decision was made public on 29 December 2016.
However, the business was actually closed on 29 February 2017.
Net costs incurred in connection with the closure of this division were as follows:

Rs. m
Redundancy costs 1.50
Staff training 0.15
Future operating loss 0.80
Less: Profit on sale of remaining mobile phones (0.50)
1.95

(iii) On 16 January 2017, LED TV sets valuing Rs. 3 million were stolen from a warehouse.
These sets were included in WL’s inventory as at 31 December 2016.
(iv) WL owns 9,000 shares of a listed company whose price as on 31 December 2016 was Rs.
22 per share. During February 2017, the share price declined significantly after the
government announced a new legislation which would adversely affect the company’s
operations. No provision in this regard has been made in the draft financial statements.
(v) On 31 January 2017, a customer announced voluntary liquidation. On 31 December 2016,
this customer owed Rs. 1.5 million.
(vi) On 15 February 2017, WL announced final dividend for the year ended 31 December 2016
comprising 20% cash dividend and 10% bonus shares, for its ordinary shareholders.
Required
Describe how each of the above transactions should be accounted for in the financial
statements of Walnut Limited for the year ended 31 December 2016. Support your answer in
the light of relevant International Financial Reporting Standards.

© Emile Woolf International 14 The Institute of Chartered Accountants of Pakistan


Questions

CHAPTER 4: IAS 8: ACCOUNTING POLICIES, CHANGES IN ACCOUNTING


ESTIMATES AND ERRORS

4.1 WONDER LIMITED


Wonder Limited (WL) is engaged in the manufacturing and sale of textile machinery. Following
are the draft extracts of the statement of financial position and the statement of profit or loss for
the year ended 30 June 2016:
Statement of Financial Position

2016 2015
Rs. m Rs. m
Property, plant and equipment 189 130
Retained earnings 166 108
Deferred tax liability 45 27

Statement of profit or loss

2016 2015
Rs. m Rs. m
Profit before taxation 90 120
Taxation 32 42
Profit after taxation 58 78

Following additional information has not been taken into account in the preparation of the above
financial statements:
(i) Cost of repairs amounting to Rs. 20 million was erroneously debited to the machinery
account on 1 October 2014. The estimated useful life of the machine is 10 years.
(ii) On 1 July 2015, WL reviewed the estimated useful life of its plant and revised it from 5
years to 8 years. The plant was purchased on 1 July 2014 at a cost of Rs. 70 million.
Depreciation is provided under the straight line method. Applicable tax rate is 30%.
Required
Prepare relevant extracts (including comparative figures) for the year ended 30 June 2016
related to the following:
(a) Statement of financial position
(b) Statement of profit or loss
(c) Statement of changes in equity
(d) Correction of error note

4.2 DUNCAN
Duncan Company has previously written off any expenditure on borrowing costs in the period in
which it was incurred.
The company has appointed new auditors this year. They have expressed the view that the
previous recognition of borrowing costs in the statement of profit or loss was in error. The
company has decided to correct the error retrospectively in accordance with IAS 8.
The financial statements for 2015 and the 2016 draft financial statements, both reflecting the old
policy, show the following.

© Emile Woolf International 15 The Institute of Chartered Accountants of Pakistan


Advanced accounting and financial reporting

Statement of changes in equity (extract)

2015 2016
Retained earnings Retained earnings
Rs.000 Rs.000
Opening balance 22,500 23,950
Profit after tax for the period 3,200 4,712
Dividends paid (1,750) (2,500)
––––––– –––––––
Closing balance 23,950 26,162
═════ ═════
Borrowing costs written off were Rs. 500,000 in 2015 and Rs. 600,000 in 2016.
The directors have calculated that borrowing costs, net of depreciation which should have been
included in property, plant and equipment had the correct policy been applied, are as follows.

Rs.000
At 30 December 2014 400
At 31 December 2015 450
At 31 December 2016 180

Had the correct policy been in force depreciation of Rs. 450,000 would have been charged in
2015 and Rs. 870,000 in 2016.
Required
Show how the change in accounting policy must be reflected in the statement of changes in
equity for the year ended 31 December 2016. Work to the nearest Rs.000.

4.3 MOHANI MANUFACTURING LIMITED


Mohani Manufacturing Limited is engaged in manufacturing of spare parts for motor car
assemblers. The audited financial statements for the year ended December 31, 2015 disclosed
that the profit and retained earnings were Rs. 21 million and Rs. 89 million respectively. The draft
financial statements for the year show a profit of Rs. 15 million. However, following adjustments
are required to be made:
(i) The management of the company has decided to change the method for valuation of
raw materials from FIFO to weighted average. The value of inventory under each method
is as follows:

FIFO Weighted Average


Rs. m Rs. m
December 31, 2014 37.0 35.5
December 31, 2015 42.3 44.5
December 31, 2016 58.4 54.4
(ii) In 2015, the company purchased a plant for Rs. 100 million. Depreciation on plant was
recorded at Rs. 25 million instead of Rs. 10 million. This error was discovered after the
publication of financial statements for the year ended December 31, 2015. The error is
considered to be material.
Required
Produce an extract showing the movement in retained earnings, as would appear in the
statement of changes in equity for the year ended December 31, 2016.

© Emile Woolf International 16 The Institute of Chartered Accountants of Pakistan


Questions

CHAPTER 5: IFRS 15: REVENUE FROM CONTRACT WITH CUSTOMER

5.1 PARVEZ LIMITED


The following transactions took place at Parvez Limited (PL).
(1) On 5 March 2017 PL sold goods to a bank for Rs.18m cash and agreed to repurchase the
goods for Rs.19m cash on 5 July 2017. The goods will be shifted to a storage facility under
bank’s control and security.
(2) On 31 March PL’s car manufacturing division consigned several vehicles to independent
dealers for sale to third parties. The sales price to the dealer is PL’s list price at the date of
sale to third parties. If a vehicle is unsold after six months, the dealer has a right to return
the vehicle to PL within next fifteen days.
Required
Discuss how the above transactions should be accounted for in the books of accounts of Parvez
Limited.

5.2 SACHAL LIMITED


Sachal Limited (SL):
(a) Sells standard computer software package meant for small and medium sized restaurant
management. This software package is sold:
 at price of Rs. 1.5 million payable before delivery,
 with thirty days trial time, and
 without any maintenance support after trial time
As per practice, it takes around six months for the customers to use the package
independent of any support from SL. Practically, SL has to provide on-site support service
for at least six months to almost all customers free-of-cost. However, in case of customer’s
request for support beyond six months, SL provides services under a formal paid service
contract.
(b) Provides maintenance and support for the above standard software package at a price of
Rs. 0.3 million per annum.
(c) Provides designing and development of customized software to customers. Payment is
made monthly by customers on the basis of chargeable hours of developers of SL. First
year maintenance service is provided free-of-cost. Subsequent maintenance service is
provided at the rate of 10% of the total contract price. Thereafter, for next three years
maintenance service is provided at 5% of the contract price per annum.
Required
Explain the considerations to be taken into account in determining accounting for revenue by
Sachal Limited.

5.3 BRILLIANT LIMITED


Brilliant Limited (BL) manufactures and sells plastic card printing machines with laminators. A
machine-specific card printing software is provided as a must part of the printing machine. BL
also sells plastic cards imported from Thailand.
BL agreed to supply the following to, Proud Learners (PL), a country-wide school network:
 15 Card printing machines – Available in ready stock
 8 Laminators – Would require 30 days to deliver
 100,000 Plastic cards – Available in ready stock

© Emile Woolf International 17 The Institute of Chartered Accountants of Pakistan


Advanced accounting and financial reporting

A lump sum price of Rs.9.2 million for the total contract has been agreed between BL and school
network.
Cost and list prices of the goods are:

Item Price (Rs.) Cost (Rs.)

Card printing machines 800,000 400,000

Laminators 200,000

Plastic cards 12 5

BL does not sell printing machine without laminator. However, in order to get this order BL went
against its policy. There is another supplier of imported card printing machine of almost similar
specification. This supplier sells the machine at Rs.750,000.
In most recent customers’ surveys printing machine of BL has been given 7 out of 10 points as
against 9 out of 10 given to competitors’ imported machine. There is no supplier of laminator in
the market.
Required
Identify performance obligations and allocate the transaction price to the identified performance
obligations.

5.4 WAQAS LIMITED


Waqas Limited (WL) enters into a contract of construction of a reverse osmosis plant for the
manufacturing unit of Ali Chemical Limited (ACL) for Rs.20 million, for which WL estimated cost
is Rs.12 million. This included supply and installation of plant and related construction work. The
project is to be completed within 18 months. WL measures performance on the basis of cost
incurred.
At the end of seventh month ACL and WL agreed to modify the contract by adding construction
of an additional water reservoir at a price of Rs.2.5 million, which will supply drinking water to a
sister concern of ACL. The additional cost is estimated as Rs.1.8 million by WL. At the end of
seventh month WL incurred 4.2 million on the project.
At the end of tenth month ACL and WL agreed to modify the contract by increasing the size of
water reservoir that was included in the original design of the project. ACL and WL agreed to an
additional consideration of Rs.1 million, for which WL will incur an additional cost of Rs.1 million.
At the end of seventh month WL incurred Rs. 7.2 million on the plant project and Rs. 0.72 million
on additional reservoir.
At the end of sixteenth month ACL and WL agreed to modify the contract by adding pumping and
piping facility from plant to the manufacturing unit of ACL for a consideration of Rs.3 million. This
facility was part of the project, but at the inception this contract was awarded to another
contractor, which was terminated by ACL. The cost to be incurred by WL was estimated as
Rs.2.8 million. At the end of sixteenth month WL incurred Rs.11.7 million on the plant project and
Rs.1.35 million on additional reservoir.
Required
Advise how these transactions should be recognized in the books of Waqas Limited.

© Emile Woolf International 18 The Institute of Chartered Accountants of Pakistan


Questions

CHAPTER 6: IAS 16: PROPERTY, PLANT AND EQUIPMENT

6.1 FAM
Fam had the following tangible fixed assets at 31 December 2015.
Cost Depreciation NBV
Rs.000 Rs.000 Rs.000
Land 500 – 500
Buildings 400 80 320
Plant and machinery 1,613 458 1,155
Fixtures and fittings 390 140 250
Assets under construction 91 – 91
——— —— ———
2,994 678 2,316
════ ════ ════
In the year ended 31 December 2016 the following transactions occur.
(1) Further costs of Rs. 53,000 are incurred on buildings being constructed by the company. A
building costing Rs. 100,000 is completed during the year.
(2) A deposit of Rs. 20,000 is paid for a new computer system which is undelivered at the year
end.
(3) Additions to plant are Rs. 154,000.
(4) Additions to fixtures, excluding the deposit on the new computer system, are Rs. 40,000.
(5) The following assets are sold.
Cost Depreciation Proceeds
brought forward
Rs.000 Rs.000 Rs.000
Plant 277 195 86
Fixtures 41 31 2

(6) Land and buildings were revalued at 1 January 2016 to Rs. 1,500,000, of which land is
worth Rs. 900,000. The revaluation was performed by Jackson & Co, Chartered
Surveyors, on the basis of existing use value on the open market.
(7) The useful economic life of the buildings is unchanged. The buildings were purchased ten
years before the revaluation.
(8) Depreciation is provided on all assets in use at the year-end at the following rates.
Buildings 2% per annum straight line
Plant 20% per annum straight line
Fixtures 25% per annum reducing balance
Required
Show the disclosure under IAS 16 in relation to fixed assets in the notes to the published
accounts for the year ended 31 December 2016.

6.2 GUJRAT CONSTRUCTION LIMITED


Gujrat Construction Limited (GCL) accounts for non-current assets using the cost model except
for land and buildings for which it has adopted the revaluation model. GCL makes an annual
transfer between the revaluation surplus and retained earnings each year to reflect the realisation
of the revaluation reserve.

© Emile Woolf International 19 The Institute of Chartered Accountants of Pakistan


Advanced accounting and financial reporting

The company’s financial statements include the following balances for property, plant and
equipment for the year ended 31 December 2015:
Cost/valuation Accumulated depreciation
Rs. Rs.
Land and buildings 1,500,000 315,000
Plant and machinery 1,276,500 879,300
Office equipment 356,400 210,400
Additional information
(1) Land is included in the above at a valuation of Rs. 850,000.
(2) Items of property, plant and equipment are depreciated on a straight-line basis as follows:
Buildings – over 50 years
Plant and machinery – at a rate of 25% per annum on cost
Office equipment – at a rate of 20% per annum on cost
Land is not depreciated.
(2) On 30 June 2016 GCL purchased plant at a cost of Rs. 135,000 and office equipment for
Rs. 36,500. On 30 September 2016 it sold, for a profit, machinery which had cost Rs.
104,000 on 1 January 2014 and was classified as held for sale on 30 June 2016. For the
first time GCL utilised spare capacity in its workshops and started to self-construct a
specialised turning machine for its own use. This machine was almost complete by 31
December 2016. Costs incurred on the machine to 31 December 2016 amounted to Rs.
29,200.
(3) During December 2016 GCL's the foundations of one of the company's warehouses were
found to be insufficient to support some of the machinery that had been housed there,
although this machinery had subsequently been moved to another site. This workshop had
cost Rs. 150,000 on 1 January 2012 and had been revalued to Rs. 210,000 on 31
December 2013. It is now estimated that its fair value is only Rs. 100,000 and that costs to
sell would be Rs. 5,000. Its value in use has been estimated at Rs. 90,000.
(4) On 1 November 2016 the directors decided to sell an item of machinery. At that date a
buyer had been identified and contracts were on the point of being exchanged, at an
agreed price of Rs. 11,000. Selling costs were expected to be Rs. 1,500. This machine
had cost Rs. 50,000 on 1 May 2014.
Required
Prepare the note showing the movements on property, plant and equipment, including
accumulated depreciation, which would be included in the financial statements of GCL for the
year ended 31 December 2016.

© Emile Woolf International 20 The Institute of Chartered Accountants of Pakistan


Questions

CHAPTER 7– NON-CURRENT ASSETS: SUNDRY STANDARDS

7.1 SPIN INDUSTRIES LIMITED


On September 1, 2015, Spin Industries Limited (SIL) started construction of its new office
building and completed it on May 31, 2016. The payments made to the contractor were as
follows:

Date of Payment Rs.

September 1, 2015 10,000,000

December 1, 2015 15,000,000

February 1, 2016 12,000,000

June 1, 2016 9,000,000

In addition to the above payments, SIL paid a fee of Rs. 8 million on September 1, 2015 for
obtaining a permit allowing the construction of the building.
The project was financed through the following sources:
(i) On August 1, 2015 a medium term loan of Rs. 25 million was obtained specifically for the
construction of the building. The loan carried mark up of 12% per annum payable semi-
annually. A commitment fee @ 0.5% of the amount of loan was charged by the bank.
Surplus funds were invested in savings account @ 8% per annum. On February 1, 2016
SIL paid the six monthly interest plus Rs. 5 million towards the principal.
(ii) Existing running finance facilities of SIL
 Running finance facility of Rs. 28 million from Bank A carrying mark up of 13%
payable annually. The average outstanding balance during the period of
construction was Rs. 25 million.
 Running finance facility of Rs. 25 million from Bank B. The mark up accrued during
the period of construction was Rs. 3 million and the average running finance
balance during that period was Rs. 20 million.
Required
Calculate the amount of borrowing costs to be capitalised on June 30, 2016 in accordance with
the requirements of International Accounting Standards. (Borrowing cost calculations should
be based on number of months).

7.2 QURESHI STEEL LIMITED


On July 1, 2015, Qureshi Steel Limited (QSL) signed an agreement with Pak Construction
Limited for construction of a factory building at a cost of Rs. 100 million. It was agreed that
the factory would be ready for use from January 1, 2017. The terms of payments were agreed as
under:
(i) 10% advance payment would be made on signing of the agreement. The advance paid
would be adjusted at 10% of the quarterly progress bills.
(ii) 5% retention money would also be deducted from the progress bills. Retention money will
be refunded one year after completion of the factory building.
(iii) Progress bills will be raised on last day of each quarter and settled on 15th of the next
month.

© Emile Woolf International 21 The Institute of Chartered Accountants of Pakistan


Advanced accounting and financial reporting

The under mentioned progress bills were received and settled by QSL as per the agreement:

Invoice date Amount (Rs.)


September 30, 2015 30 million
December 31, 2015 20 million
March 31, 2016 10 million
June 30, 2016 15 million

On April 30, 2016 an invoice of Rs. 1.5 million was raised by the contractor for damages sustained
at the site, on account of rains. After negotiations, QSL finally agreed to make additional
payment of Rs. 1.0 million to compensate the contractor. The amount was paid on May 15, 2016.
It is expected that 75% of the payment would be recovered from the insurance company.
The cost of the project has been financed through the following sources:
(i) Issue of right shares amounting to Rs. 15 million, on September 1, 2015. The company
has been following a policy of paying dividend of 20% for the past many years.
(ii) Bank loan of Rs. 25 million obtained on December 1, 2015. The loan carries a markup of
13% per annum. The principal is repayable in 5 half yearly equal instalments of Rs. 5
million each along with the interest, commencing from May 31, 2016. Loan processing
charges of Rs.0.5 million were deducted by the bank at the time of disbursement of loan.
Surplus funds, when available, were invested in short term deposits at 8% per annum.
(iii) Cash withdrawals from the existing running finance facility provided by a bank.
Average running finance balance for the year was Rs. 60 million. Markup charged by the
bank for the year was Rs. 9 million.
Required
Compute cost of capital work in progress for the factory building as of June 30, 2016 in
accordance with the requirements of relevant IFRSs.
(Borrowing costs calculations should be based on number of months)

7.3 IMRAN LIMITED


On January 1, 2016, Imran Limited started the construction of its new factory. The
construction period is approximately 15 months and the cost is estimated at Rs. 80 million.
The work has been divided into 5 phases and payment to contractor shall be made on
completion of each phase.
In the year the company had the following sources of finance available.
(i) Rights i s s u e o f shares amounting to Rs. 15 million on January 1, 2016. The
company usually pays a dividend of 10% each year.
(ii) Bank loan of Rs. 32 million carrying a mark-up of 13% was raised on March 1, 2016.
(This loan was outstanding for 306 days in the year).
(iii) On August 1, 2016, Rs. 10 million were borrowed from the bank. Interest thereon, is
payable at the rate of 11%. (This loan was outstanding for 153 days in the year).
Investment income on temporary investment of the borrowings amounted to Rs.0.5 million.
The details of bills submitted by the contractor, during the year are as follows:

Particulars Date of payment Rs.


st
On completion of 1 phase March 1, 2016 20,000,000
nd
On completion of 2 phase April 1, 2016 18,000,000
rd
On completion of 3 phase October 1, 2016 16,000,000
th
On completion of 4 phase Payment not yet made 17,000,000

© Emile Woolf International 22 The Institute of Chartered Accountants of Pakistan


Questions

On June 1, 2016, the Building Control Authority issued instructions for stoppage of work on
account of certain discrepancies in the completion plan. The company filed a petition in the
Court and the matter was decided in the company’s favour on July 31, 2016. Work
recommenced after a delay of 61 days.
The following periods may be relevant:

Period Days
March 1 to December 31 306
April 1 to December 31 275
August 1 to December 31 153
October 1 to December 31 92

Required
a) Assuming that the loans were taken specifically for the project, calculate the amount of
borrowing costs that s h o u l d be capitalised i n t h e p e r i o d e n d i n g December 31,
2016 in accordance with the requirements of IAS 23 Borrowing Costs.
b) Assuming that the loans constituted general finance, calculate the amount of borrowing
costs that s h o u l d be capitalised i n t h e p e r i o d e n d i n g December 31, 2016 in
accordance with the requirements of IAS 23 Borrowing Costs.

7.4 KATIE
During the year ended 30 June Year 2, Katie received three grants, the details of which are set
out below.
(1) On 1 September, a grant of Rs. 40,000 from local government. This grant was in respect of
training costs of Rs. 70,000 which Katie had incurred.
(2) On 1 November Katie bought a machine for Rs. 350,000. A grant of Rs. 100,000 was
received from central government in respect of this purchase. The machine, which has a
residual value of Rs. 50,000, is depreciated on a straight-line basis over its useful life of
five years.
(3) On 1 June a grant of Rs. 100,000 from local government. This grant was in respect of
relocation costs that Katie had incurred moving part of its business from outside the local
area. The grant is repayable in full unless Katie recruits ten employees locally by the end
of Year 2. Katie is finding it difficult to recruit as the local skill base does not match the
needs of this part of the business.
Required
Show how the above transactions should be reflected in the financial statements of Katie for the
year ended 30 June Year 2. Where any accounting standards allow a choice you should show all
possible options.

7.5 ALNUS LIMITED AND BUTEA LIMITED


Alnus Limited owns 80% of Butea Limited
On 1 December 2016 Alnus Ltd obtained a Rs. 200,000 government grant, representing 50% of
the cost of a depreciating asset which was acquired on 1 October 2016 for a total cost of Rs.
400,000. The asset has a four-year useful life with no residual value, and depreciation of Rs.
75,000 has been charged in the year to 30 June 2017. The draft financial statements show the
Rs. 200,000 grant as income in the year.
During the year ended 30 June 2017, Butea Ltd commenced redevelopment of one of its holiday
parks, which was in a state of disrepair and not in use. Discussions with architects started on 1
October 2016, and work started on successfully obtaining planning permission on 1 November
2016. The first expenditure relating to the redevelopment was incurred on 1 January 2017. The

© Emile Woolf International 23 The Institute of Chartered Accountants of Pakistan


Advanced accounting and financial reporting

construction work was completed on 30 June 2017, and this was funded out of Butea Ltd’s
existing loan finance.
The following loan finance was in place during the year:
 Rs. 1.8 million of loan finance paying 5% pa interest
 Rs. 1.2 million of loan finance paying 8.5% pa interest
Neither loan was taken out to finance a specific purpose or building. The total cost of construction
work was Rs. 800,000 and the directors capitalised Rs. 51,000 of finance costs (being nine
months of interest on construction costs of Rs. 800,000 at 8.5% pa).
Required
Explain the correct IFRS accounting treatment for the above transactions and explain how
Butea’s transaction would be reflected in the consolidated financial statements.

7.6 VICTORIA
Victoria owns several properties and has a year end of 31 December. Wherever possible,
Victoria carries investment properties under the fair value model.
Property 1 was acquired on 1 January Year 1. It had a cost of Rs. 1 million, comprising Rs.
500,000 for land and Rs. 500,000 for buildings. The buildings have a useful life of 40 years.
Victoria uses this property as its head office.
Property 2 was acquired many years ago for Rs. 1.5 million for its investment potential. On 31
December Year 7 it had a fair value of Rs. 2.3 million. By 31 December Year 8 its fair value had
risen to Rs. 2.7 million. This property has a useful life of 40 years.
Property 3 was acquired on 30 June Year 2 for Rs. 2 million for its investment potential. The
directors believe that the fair value of this property was Rs. 3 million on 31 December Year 7 and
Rs. 3.5 million on 31 December Year 8. However, due to the specialised nature of this property,
these figures cannot be corroborated. This property has a useful life of 50 years.
Required
(a) For each of the above properties briefly state how it would be treated in the financial
statements of Victoria for the year ended 31 December Year 8, identifying any impact on
profit or loss.
(b) Produce an analysis of property, plant and equipment for Victoria for the year ended 31
December Year 8, showing each of the above properties separately.

© Emile Woolf International 24 The Institute of Chartered Accountants of Pakistan


Questions

CHAPTER 8 – IAS 38: INTANGIBLE ASSETS

8.1 BROOKLYN
Brooklyn is a bio-technology company performing research for pharmaceutical companies. The
finance director has contacted your financial consulting company to arrange a meeting to discuss
issues relevant to the preparation of the financial statements for the year to 30th June 2016. Your
initial telephone conversation has provided the necessary background information.
st st
1 On 1 August 2015 Brooklyn began investigating a new bio-process. On 1 September
2016, the new process was widely supported by the scientific community and the feasibility
project was approved. A grant was then obtained relating to future work. Several
pharmaceutical companies have expressed an interest in buying the ‘know how’ when the
project completes in June 2017. The nominal ledger account set up for the project shows
st th
that the expenditure incurred between 1 August 2015 and 30 June 2016 was Rs.
300,000 per month.
2 In August 2016, an employee lodged a legal claim against the company for damage to his
st
health as a result of working for the company for the two years through to 31 March 2015
when he had to retire due to ill health. He has argued that his health deteriorated as a
result of the stress from his position in the organisation. Brooklyn has denied the claim and
has appointed an employment lawyer to assist with contesting the case. The lawyer has
advised that there is a 25% chance that the claim will be rejected, 50% chance that the
damages will be Rs. 600,000 and 25% chance of Rs. 1 million. The company has an
insurance policy that will pay 10% of any damages to the company. The lawyer has said
th
that the case could take until 30 June 2019 to resolve. The present value of the estimated
damages discounted at 8% is Rs. 476,280 and Rs. 793,800 respectively.
3 Brooklyn owns several buildings, which include an administrative office in the centre of
London. The company has revalued these on a regular basis every five years and the next
th
valuation is due on 30 June 2018. Property prices have increased since the last review
and particularly for the London premises. The cost of engaging a professionally qualified
valuer is very expensive and so to reduce costs the finance director is proposing that the
property manager, who is a professionally qualified valuer, should value the London
property and that the increase in value should be included in the financial statements. The
finance director is of the opinion that the property prices may fall next year.
Required
Prepare notes for your meeting with the finance director which explain and justify the accounting
treatment of these issues, preparing calculations where appropriate and identifying matters on
which your require further information.

8.2 RAISIN INTERNATIONAL


(a) Discuss the criteria that should be used while recognizing intangible assets arising from
research and development work.
(b) Raisin International (RI) is planning to expand its line of products. The related information
for the year ended 31 December 2016 is as follows:
(i) Research and development of a new product commenced on 1 January 2016. On 1
October 2016, the recognition criteria for capitalization of an internally generated
intangible asset were met. It is estimated that the product would have a useful
life of 7 years. Details of expenditures incurred are as follows:
Rs. m
Research work 4.50
Development work 9.00
Training of production staff 0.50
Cost of trial run 0.80
Total costs 14.80

© Emile Woolf International 25 The Institute of Chartered Accountants of Pakistan


Advanced accounting and financial reporting

(ii) The right to manufacture a well-established product under a patent for a period of
five years was purchased on 1 March 2016 for Rs. 17 million. The patent has an
expected remaining useful life of 10 years. RI has the option to renew the patent for
a further period of five years for a sum of Rs. 12 million.
(iii) RI has acquired a brand at a cost of Rs. 2 million. The cost was incurred in the
month of June 2016. The life of the brand is expected to be 10 years. Currently,
there is no active market for this brand. However, RI is planning to launch an
aggressive marketing campaign in February 2017.
(iv) In September 2015, RI developed a new production process and capitalised it as an
intangible asset at Rs. 7 million. The new process is expected to have an indefinite
useful life. During 2016, RI incurred further development expenditure of Rs. 3
million on the new process which meets the recognition criteria for capitalization of
an intangible asset.
Required
In the light of International Financial Reporting Standards, explain how each of the above
transaction should be accounted for in the financial statements of Raisin International
for the year ended 31 December 2016.

8.3 OXTAIL LTD


During the year ended 31 December 2016 Oxtail Ltd entered into the following transactions
relating to intangible assets. Oxtail Ltd uses the cost model to measure intangible assets.
(1) Research costs of Rs. 70,000 were incurred from 1 January 2016 to 31 March 2016 on the
early development of a new product. A project review was undertaken on 1 April 2016 and
on that date Oxtail Ltd assessed that the development of the new product would be
economically viable. Subsequently, further costs of Rs. 120,000 were incurred to complete
the development phase of the project. This included Rs. 15,000 on promotional
advertising. Production of the new product started on 1 January 2017.
(2) On 1 April 2016 Oxtail Ltd acquired some technical know-how which will completely
change the way its manufacturing process operates.
The following costs have been incurred:
Rs.
Original cost of technical know-how 180,000
Legal costs incurred as part of acquisition 4,000
Manufacturing supervisors’ time to install new process 3,200
Staff training incurred in operating new process 13,000
Testing new manufacturing process 4,800
205,000

The new manufacturing process was available for use on 1 July 2016. It was believed that the
new process would be of benefit for the next four years after which it would be replaced.
Although operation of the new manufacturing process during its first six months went well, a
breakthrough in the development of improved technology by a competitor led to an impairment
review being carried out by Oxtail Ltd. At 31 December 2016 the fair value (less costs to sell) of
the technical know-how was assessed as being Rs. 152,000 compared with the present value of
the estimated future cash flows expected to be generated by the technology of Rs. 157,000.
Required
(a) Explain the required IFRS accounting treatment of the two issues above, preparing all
relevant calculations.
(b) Prepare an extract from Oxtail Ltd's statement of financial position as at 31 December
2016 and a summary of the related costs that would be recognised in profit or loss for the
year ended 31 December 2016 in respect of intangible assets.

© Emile Woolf International 26 The Institute of Chartered Accountants of Pakistan


Questions

8.4 SKY LIMITED


During the year 2016, SKY Limited developed two inter-linked websites in house. One of them
is for external users and provides information about the company’s products, operations and
financials. It can also be used for electronic order processing and accepting payments through
credit cards. The second website is for internal use like intra-net, providing and sharing
company’s policies, customer details, employees’ information, etc.
Both the websites were launched on September 30, 2016 and are now fully operational. The
company has received a few online orders which it believes will increase over time. On the other
hand, use of internal website has resulted in minor reduction in costs of communication and certain
other administrative costs. The management is optimistic that its utility will increase significantly.
However, it is not in a position to estimate the amount of economic inflows that this website can
generate.
During the year ended December 31, 2016, the company incurred the following expenditure in the
development of websites:
(i) An amount of Rs. 0.3 million was incurred on undertaking a feasibility study and defining
hardware/software specifications for the websites.
(ii) Rs. 4 million was incurred on the development of internal website while an
expenditure of Rs. 11 million has been made on development of external website.
The expenditure on external website includes an amount of Rs. 6 million paid for linking it
with the credit card clearing facilities and installation of security tools.
(iii) The company acquired two dedicated servers and one backup server costing Rs. 3 million
in total. Operating software for the server was acquired for Rs. 2.0 million whereas
software related to data processing and front-end development cost Rs. 3 million. The
management is of the view that these costs would not have been incurred if the website
project had not been initiated.
(iv) With effect from October 1, 2016 the company has signed a one year contract for
website maintenance at a cost of Rs. 2.0 million.
(v) Two IT personnel were trained to operate the websites, at a cost of Rs. 0.2 million.
(vi) Rs. 0.4 million were incurred on the promotion of its external website. The company
believes that this advertising will boost the company’s online sales.
Required
Comment on the accounting treatment of each of the above mentioned costs in the light of relevant
International Accounting Standards.

8.5 COMFORT SHOES LIMITED


In 2010 the management of Comfort Shoes Limited planned to acquire an international
trademark to boost its sales and enter into the international market. In this respect, the
management carried out a market survey and analysed the information obtained to initiate the
process. The relevant information is as follows:
(i) The cost incurred on the survey and related activities during the year 2010 amounted to
Rs. 1 million.
(ii) An agreement was finalised and the company acquired the trademark effective January 1,
2011. According to the agreement Rs. 5 million were paid on signing of the agreement
and Comfort Shoes was required to pay 1% of sale proceeds of the related products on
yearly basis. The analysis carried out at that time indicated that the trademark would have
an indefinite useful life.
(iii) The company has developed many new models under this trademark and successfully
marketed them in the country as well as in international markets. However, in 2015 the
company faced unexpected competition and had to discontinue the exports. It was
estimated that due to discontinuation of exports, net cash inflows for the foreseeable

© Emile Woolf International 27 The Institute of Chartered Accountants of Pakistan


Advanced accounting and financial reporting

future, would reduce by 30%. As a result the management was of the view that as of
December 31, 2015 the carrying value of the trademark had reduced to 90%.
(iv) Due to continuous inflation and flooding of markets with very low priced shoes, it was
decided in December 2016 that use of the trademark would be discontinued with effect
from January 1, 2018.
Required
(a) Explain how the above transactions should have been accounted for in the years 2010 to
2011 according to International Financial Reporting Standards (IFRSs).
(b) Prepare a note to the financial statements for the year ended December 31, 2016 in
accordance with the requirements of IFRSs. Show comparative figures.

© Emile Woolf International 28 The Institute of Chartered Accountants of Pakistan


Questions

CHAPTER 9 – IAS 36: IMPAIRMENT OF ASSETS

9.1 CHARLOTTE
Charlotte Ltd is a company with a 31 December year-end.
The following is relevant to three tangible non-current assets held by Charlotte.

Machine 1
This was purchased on 1 January Year 1 for Rs. 420,000. It had an estimated residual value of
Rs. 50,000 and a useful life of ten years and was being depreciated on a straight-line basis.
On 1 January Year 6 Charlotte revalued this machine to Rs. 275,000 and reassessed its total
useful life as fifteen years with no residual value.
On 1 January Year 7 an impairment review showed machine 1’s recoverable amount to be Rs.
100,000 and its remaining useful life to be five years.

Machine 2
This was purchased on 1 January Year 1 for Rs. 500,000. It had an estimated residual value of
Rs. 60,000 and a useful life of ten years and was being depreciated on a straight-line basis.
On 1 January Year 7 this machine was classified as held for sale, at which time its fair value was
estimated at Rs. 200,000 and costs to sell at Rs. 5,000. On 31 March Year 7 the machine was
sold for Rs. 210,000.

Machine 3
This was purchased on 1 January Year 1 for Rs. 600,000. In Year 1 depreciation of Rs. 30,000
was charged. On 1 January Year 2 this machine was revalued to Rs. 800,000 and its remaining
useful life assessed as eight years.
On 1 January Year 7 this machine was classified as held for sale, at which time, its fair value was
estimated at Rs. 550,000 and costs to sell at Rs. 5,000.
On 31 March Year 7 the machine was sold for Rs. 550,000.
Tax is at the rate of 30%.

Required
Show the effect of the above on profit or loss and revaluation reserve of Charlotte in Year 7.

9.2 ABA LIMITED


Aba Limited conducts its activities from two properties, a head office in the city centre and a
property in the countryside where staff training is conducted. Both properties were acquired on 1
April 2013 and had estimated lives of 25 years with no residual value. The company has a policy
of carrying its land and buildings at current values. However, until recently property prices had
not changed for some years. On 1 October 2015 the properties were revalued by a firm of
surveyors. Details of this and the original costs are:
Land Buildings
Rs. Rs.
Head office – cost 1 April 2013 500,000 1,200,000
– revalued 1 October 2015 700,000 1,350,000
Training premises – cost 1 April 2013 300,000 900,000
– revalued 1 October 2015 350,000 600,000
The fall in the value of the training premises is due mainly to damage done by the use of heavy
equipment during training. The surveyors have also reported that the expected life of the training
property in its current use will only be a further 10 years from the date of valuation. The estimated
life of the head office remained unaltered.

© Emile Woolf International 29 The Institute of Chartered Accountants of Pakistan


Advanced accounting and financial reporting

Note: Aba Limited treats its land and its buildings as separate assets. Depreciation is based on
the straight-line method from the date of purchase or subsequent revaluation.
Required
Prepare extracts of the financial statements of Aba Limited in respect of the above properties for
the year to 31 March 2016.

9.3 HUSSAIN ASSOCIATES LTD


The assistant financial controller of the Hussain Associates Ltd group has identified the matters
below which she believes may indicate impairment of one or more assets:
(a) Hussain Associates Ltd owns and operates an item of plant that cost Rs. 640,000 and had
accumulated depreciation of Rs. 400,000 at 1 October 2015. It is being depreciated at
12½% on cost.
On 1 April 2016 (exactly half way through the year) the plant was damaged when a factory
vehicle collided into it. Due to the unavailability of replacement parts, it is not possible to
repair the plant, but it still operates, albeit at a reduced capacity. It is also expected that as
a result of the damage the remaining life of the plant from the date of the damage will be
only two years.
Based on its reduced capacity, the estimated present value of the plant in use is Rs.
150,000. The plant has a current disposal value of Rs. 20,000 (which will be nil in two
years’ time), but Hussain Associates Ltd has been offered a trade-in value of Rs. 180,000
against a replacement machine which has a cost of Rs. 1 million (there would be no
disposal costs for the replaced plant). Hussain Associates Ltd is reluctant to replace the
plant as it is worried about the long-term demand for the product produced by the plant.
The trade-in value is only available if the plant is replaced.
Required
Prepare extracts from the statement of financial position and statement of profit or loss of
Hussain Associates Ltd in respect of the plant for the year ended 30 September 2016.
Your answer should explain how you arrived at your figures.
(b) On 1 April 2015 Hussain Associates Ltd acquired 100% of the share capital of Sparkle
Limited, whose only activity is the extraction and sale of spa water. Sparkle Limited had
been profitable since its acquisition, but bad publicity resulting from several consumers
becoming ill due to a contamination of the spa water supply in April 2016 has led to
unexpected losses in the last six months. The carrying amounts of Sparkle Limited’s
assets at 30 September 2016 are:

Rs.000
Brand (Sparkle Spring – see below) 7,000
Land containing spa 12,000
Purifying and bottling plant 8,000
Inventories 5,000
32,000

The source of the contamination was found and it has now ceased.
The company originally sold the bottled water under the brand name of ‘Sparkle Spring’,
but because of the contamination it has re-branded its bottled water as ‘Refresh’. After a
large advertising campaign, sales are now starting to recover and are approaching
previous levels. The value of the brand in the balance sheet is the depreciated amount of
the original brand name of ‘Sparkle Spring’.
The directors have acknowledged that Rs. 1.5 million will have to be spent in the first three
months of the next accounting period to upgrade the purifying and bottling plant.

© Emile Woolf International 30 The Institute of Chartered Accountants of Pakistan


Questions

Inventories contain some old ‘Sparkle Spring’ bottled water at a cost of Rs. 2 million; the
remaining inventories are labelled with the new brand ‘Refresh’. Samples of all the bottled
water have been tested by the health authority and have been passed as fit to sell. The old
bottled water will have to be relabelled at a cost of Rs. 250,000, but is then expected to be
sold at the normal selling price of (normal) cost plus 50%.
Based on the estimated future cash flows, the directors have estimated that the value in
use of Sparkle Limited at 30 September 2016, calculated according to the guidance in IAS
36, is Rs. 20 million. There is no reliable estimate of the fair value less costs to sell of
Sparkle Limited.
Required
Calculate the amounts at which the assets of Sparkle Limited should appear in the
consolidated statement of financial position of Hussain Associates Ltd at 30 September
2016. Your answer should explain how you arrived at your figures.

9.4 IMPS
A division of IMPS has the following non-current assets, which are stated at their carrying values
at 31 December Year 4:
Rs. m Rs. m

Goodwill 70

Property, plant and equipment:


Land and buildings 320
Plant and machinery 110
430
500
Because these assets are used to produce a specific product, it is possible to identify the cash
flows arising from their use. The management of IMPS believes that the value of these assets
may have become impaired, because a major competitor has developed a superior version of the
same product and, as a result, sales are expected to fall.
The following additional information is relevant:
Forecast cash inflows arising from the use of the assets are as follows:
Year ended 31 December:
Rs. m
Year 5 185
Year 6 160
Year 7 130
(i) The directors are of the opinion that the market would expect a pre-tax return of 12% on an
investment in an entity that manufactures a product of this type.
(ii) The land and buildings are carried at valuation. The surplus relating to the revaluation of
the land and buildings that remains in the revaluation reserve at 31 December Year 4 is
Rs. 65 million. All other non-current assets are carried at historical cost.
(iii) The goodwill does not have a market value. It is estimated that the land and buildings
could be sold for Rs. 270 million and the plant and machinery could be sold for Rs. 50
million, net of direct selling costs.
Required
(a) Calculate the impairment loss that will be recognised in the accounts of IMPS.
(b) Explain how this loss will be treated in the financial statements for the year ended 31
December Year 4.

© Emile Woolf International 31 The Institute of Chartered Accountants of Pakistan


Advanced accounting and financial reporting

CHAPTER 10 – IFRS 5: NON-CURRENT ASSETS HELD FOR SALE AND


DISCONTINUED OPERATIONS
10.1 SAUL
Saul operates its business through a number of divisions. It has a year end of 31 December. Set
out below are extracts from the draft financial statements of Saul for the year ended 31
December Year 1.
Statement of profit or loss for the year ended 31 December Year 1
Rs.000
Revenue 3,900
Cost of sales (2,500)
––––––
Gross profit 1,400
Distribution costs (300)
Administrative expenses (800)
––––––
Profit before tax 300
Income tax expense (90)
––––––
Profit for the period 210
––––––
Statement of financial position at 31 December Year 1
Assets Rs.000 Rs.000
Non-current assets
Property, plant and equipment 1,900
Intangible assets 40
–––––– 1,940
Current assets
Inventories 350
Trade and other receivables 190
Cash 90
–––––– 630
u ––––––
Total assets 2,570
––––––
Equity and liabilities
Equity
Share capital 600
Retained earnings 1,700
–––––– 2,300
Current liabilities
Trade and other payables 195
Current tax payable 75
–––––– 270
––––––
Total equity and liabilities 2,570
––––––
On 30 November Year 1 Saul made the decision to close Division A, which is located in a
different part of the country and covers a separate major line of business. This decision was
immediately announced to the press and to the workforce and, by the end of Year 1, a buyer had
been found.
The directors of Saul have calculated the following.
15% of the entity’s income and expenses for the year was attributable to Division A.
No tax is attributable to Division A.
Property, plant and equipment of Rs. 510,000 and payables of Rs. 10,000 in the above statement
of financial position relate to Division A. The fair value minus costs to sell of the property, plant
and equipment is Rs. 450,000.

© Emile Woolf International 32 The Institute of Chartered Accountants of Pakistan


Questions

Required
Redraft the above financial statements to meet the provisions of IFRS 5:Non-current assets held
for sale and discontinued operations. Work to the nearest Rs.000.

10.2 SHAHID HOLDINGS


(a) State the definition of both non-current assets held for sale and discontinued operations
and explain the usefulness of information for discontinued operations.
Shahid Holdings is in the process of preparing its financial statements for the year ended
31 October 2016. The company’s main activity is in the travel industry mainly selling
package holidays (flights and accommodation) to the general public through the Internet
and retail travel agencies. During the current year the number of holidays sold by travel
agencies declined dramatically and the directors decided at a board meeting on 15
October 2016 to cease marketing holidays through its chain of travel agents and sell off
the related high-street premises. Immediately after the meeting the travel agencies’ staff
and suppliers were notified of the situation and an announcement was made in the press.
The directors wish to show the travel agencies’ results as a discontinued operation in the
financial statements to 31 October 2016. Due to the declining business of the travel
agents, on 1 August 2016 (three months before the year end) Shahid Holdings expanded
its Internet operations to offer car hire facilities to purchasers of its Internet holidays.
The following are Shahid Holdings’s summarised statement of profit or loss results – years
ended:
31 October 2016 31 October 2015
Travel
Internet Car hire Total Total
agencies
Rs.’000 Rs.’000 Rs.’000 Rs.’000 Rs.’000
Revenue 23,000 14,000 2,000 39,000 40,000
Cost of sales (18,000) (16,500) (1,500) (36,000) (32,000)
    
Gross profit/(loss) 5,000 (2,500) 500 3,000 8,000
Operating expenses (1,000) (1,500) (100) (2,600) (2,000)
    
Profit/(loss) before tax 4,000 (4,000) 400 400 6,000
    
The results for the travel agencies for the year ended 31 October 2015 were: revenue Rs.
18 million, cost of sales Rs. 15 million and operating expenses of Rs. 1·5 million.
Required
(b) Discuss whether the directors’ wish to show the travel agencies’ results as a
discontinued operation is justifiable.
(c) Assuming the closure of the travel agencies is a discontinued operation, prepare the
(summarised) statement of profit or loss of Shahid Holdings for the year ended 31
October 2016 together with its comparatives.
Note: Shahid Holdings discloses the analysis of its discontinued operations on the
face of its statement of profit or loss.

10.3 PRIMA
Prima is a listed company with a year end of 31 December. It operates two businesses, the first
is the rental of luxury yachts and the second is a chain of luxury holiday villas in Europe. The
directors have requested your advice on the following matters.

Holiday villas
Prima’s policy is to carry the holiday villas at their re-valued amount, which, based on the most
recent valuation in 20X0, was Rs. 20m (historical cost was Rs. 10m). Prima is unsure how
frequently a revaluation of such properties is required and so has instructed a surveyor to provide
an up-to-date valuation as at 31 December Year 4. This valuation has provided the following
information:

© Emile Woolf International 33 The Institute of Chartered Accountants of Pakistan


Advanced accounting and financial reporting

Rs. million
Replacement cost 17
Value in use 28
Open market value 25
One of the villas has received very few bookings over the past two years and so a decision was
reached to exclude it from the Year 5 brochure. It is currently up for sale. The villa has a carrying
value of Rs. 1.25m. Its value in use is only Rs.0.85m and its expected market value is Rs. 1m,
before expected agents and solicitor’s fees of Rs. 50,000. The directors are unsure as to the
accounting treatment of this villa. A number of potential buyers have expressed an interest in the
property, and it is hoped that a deal will be negotiated in the first few months of Year 5.
Prima’s accounting policy is to not charge depreciation on the villas. Its justification is that the
villas are maintained to a high standard and have useful lives of at least 50 years.

Head Office
Over the past two years, Prima has built its own head office. Construction began on 1 October
Year 2 and finished on 1 June Year 4, although minor modifications meant that the company did
not relocate until 1 September Year 4.
The site cost Rs. 1m and the costs of construction were a further Rs. 8m. Prima took out a two
year loan of Rs. 5m on 1 October Year 2, at an interest rate of 9% per annum, to help fund the
work. In order to encourage businesses to operate in areas of high unemployment, the
government offered a Rs. 1.5m grant towards the cost of construction. The terms of settlement
were that payment would only be made upon completion of the building once a government
inspection had taken place. This inspection had not taken place by the year end, but Prima is
confident that the grant will be received shortly after the year end.
The company intends to use the head office for the next 50 years and, as for the villas, does not
intend to depreciate the land or buildings.

Yachts
Prima has spent the past year designing a new range of luxury yachts. Work was completed on 1
April Year 4 at a cost of Rs. 20m. During the construction, the economy took a downturn and the
company now believes that the market value of the yachts is only Rs. 17m, although the value in
use is estimated to be Rs. 18m. The engines of the yachts have a three year life, the interior has
a two year life, and the remainder should have a life of 15 years. The engine cost is believed to
represent 15% of the total cost of manufacture and the interior approximately 25%.

Required
Explain the accounting issues relating to the villas, head office and yachts, referring to relevant
IFRS guidance. Where possible, numerical information relating to the 31 December Year 4
financial statements should be provided.

© Emile Woolf International 34 The Institute of Chartered Accountants of Pakistan


Questions

CHAPTER 11 – IFRS 16: LEASES

11.1 X LTD
X Ltd is considering acquiring a machine. It has two options; cash purchase at a cost of Rs.
11,420,000 or a lease.
The terms of the lease are as follows:
(i) The lease period is for four years from 1 January 2016 with an annual rental of Rs.
4,000,000 payable on 31 December each year.
(ii) The lessee is required to pay all repairs, maintenance and other incidental costs.
(iii) The interest rate implicit in the lease is 15% p.a.
Note:
Estimated useful economic life span of the machine is four years.
Required
(a) Prepare a schedule of the allocation of the finance charges in the books of X Limited for
the entire lease period.
(b) Prepare an extract of the Statement of Financial Position of X Limited for the year ended
31 December 2016.

11.2 PROGRESS LTD


Progress Ltd. acquired a machine from Fine Rentals Ltd. on January 3, 2016 under a lease
agreement extending over three years.
The agreement required them to make an initial deposit of Rs. 1,280,000 to be followed by three
annual payments of Rs. 800,000 on 31 December each year starting from 2016.
The cash price of the machinery was Rs. 3,200,000 and Fine Rentals Ltd. added 12% interest
which was duly communicated to Progress Ltd.
The annuity method is used to allocate interest.
Required
(a) Compute the interest element and the capital portion of the annual repayments; and
(b) Show the journal entries that will record the transaction resulting from the lease
agreement.

11.3 MIRACLE TEXTILE LIMITED


On 1 July 2014, Miracle Textile Limited (MTL) acquired a machine on lease, from a bank.
Details of the lease are as follows:
(i) Cost of machine is Rs. 20 million.
(ii) The lease term and useful life is 4 years and 10 years respectively.
(iii) Instalment of Rs. 5.80 million is to be paid annually in advance on 1 July.
(iv) The interest rate implicit in the lease is 15.725879%.
(v) At the end of lease term, MTL has an option to purchase the machine on payment of
Rs. 2 million. The fair value of the machine at the end of lease term is expected to be
Rs. 3 million.
MTL depreciates the machine on the straight line method to a nil residual value.
Required
Prepare relevant extracts of the statement of financial position and related notes to the financial
statements for the year ended 30 June 2016 along with comparative figures. Ignore taxation.

© Emile Woolf International 35 The Institute of Chartered Accountants of Pakistan


Advanced accounting and financial reporting

11.4 ACACIA LTD


On 1 April 2015 Acacia Ltd entered into the following lease agreements. The terms of each lease
are as follows:
(1) Plant with a fair value of Rs. 275,000 was leased under an agreement which requires
Acacia Ltd to make annual payments of Rs. 78,250 on 1 April each year, commencing on
1 April 2015, for four years. After the four years Acacia Ltd has the option to continue to
lease the plant at a nominal rent for a further three years and is likely to do so as the asset
has an estimated useful life of six years. The present value of the lease payments is Rs.
272,850. Acacia Ltd is responsible for insuring and maintaining the plant during the period
of the lease.
(2) Office equipment with a fair value of Rs. 24,000 was leased under a non-cancellable
agreement which requires Acacia Ltd to make annual payments of Rs. 6,000 on 1 April
each year, commencing on 1 April 2015, for three years. The lessor remains responsible
for insuring and maintaining the equipment during the period of the lease. The equipment
has an estimated useful life of ten years. The present value of the lease payments is Rs.
16,415.
Acacia Ltd allocates finance charges on an actuarial basis. The interest rate implicit in both of the
leases is 10%.
Required
Prepare all relevant extracts from Acacia Ltd's financial statements for the year ended 31 March
2016 in respect of the above leases. The only notes to the financial statements required are
those in respect of lease liabilities or commitments.

11.5 SHOAIB LEASING LIMITED


Shoaib Leasing Limited (the lessor) has entered into a three year agreement with Sarfaraz
Limited (the lessee) to lease a machine with an expected useful life of 4 years. The cost of
machine is Rs. 2,100,000.
The following information relating to lease transaction is available:
(i) Date of commencement of lease is July 1, 2016.
(ii) The lease contains a purchase bargain option at Rs. 100,000. At the end of the lease term,
the value of the machine will be Rs. 300,000.
(iii) Lease instalments of Rs. 860,000 are payable annually, in arrears, on June 30.
(iv) The implicit interest rate is 12.9972%.
Required
(a) Prepare the journal entries for the years ending June 30, 2017, 2018 and 2019 in the
books of lessor. Ignore tax.
(b) Produce extracts from the statement of financial position including relevant notes as at
June 30, 2017 to show how the transactions carried out in 2017 would be reflected in
the financial statements of the lessor.
(Disclosure of accounting policy is not required.)

11.6 AKBAR LTD


Akbar Ltd. (AL) prepares financial statements on 31 March each year. On
1 April Year 4, AL sold a machine to another company, Shahwez Ltd. (SL), for Rs.850,000 and
then leased it back under a ten year arrangement. AL had purchased the machine exactly ten
years previously for Rs.500,000 and had charged total depreciation of Rs.60,000 on the machine
up to the date of disposal. Assume that the transfer of machine by the seller-lessee satisfies the
requirements of IFRS 15.
Details of the sale and leaseback arrangement are as follows:
Consideration received from SL Rs.850,000

© Emile Woolf International 36 The Institute of Chartered Accountants of Pakistan


Questions

Fair value at date of disposal Rs.550,000


Lease rentals (payable at the end of each year) is Rs.100,000 and interest rate implicit in the
lease is 10% p.a
Required
How AL should reflect in its books of accounts:
a) Right-of-use retained by AL
b) Gain / loss on rights transferred

11.7 ALI LIMITED


Ali Limited entered into a sale and leaseback arrangement with a bank on
1 April 2015. The arrangement involved the sale at fair value of plant and machinery to the bank
for Rs.1,440,000.
This amount has been credited to Ali Limited’s operating income. The carrying amount of the
plant and machinery was Rs.840,000 and its remaining useful life was five years at 1 April 2015.
No depreciation has been charged in respect of this plant and machinery for the year ended 31
March 2016.
Under the terms of the lease, Ali Limited is to pay five annual payments at
31 March each year, of Rs.360,000 (in arrears). The first payment has been made and has been
debited to operating costs. The interest rate implicit in the lease is 8%. The transfer of asset does
not satisfy the requirements of IFRS 15.
Required
Explain how the above transaction should be accounted for, with all relevant calculations, in the
financial statements for the year ended 31 March 2016.

11.8 MOAZZAM TEXTILE MILLS LIMITED


Moazzam Textile Mills Limited (MTML) is facing severe financial difficulties. To improve the cash
flows, the management has decided to sell and lease back three power generators of the
company under three different sale and lease back arrangements which were signed on August
15, 2016. At the same time, MTML enters into a contract with the buyer-lessor for the right to use
the generators for 5 years, with annual payments of Rs.1,000,000 each for Generator A and
Generator B and Rs.1,500,000 for Generator C, payable at the end of each year. The interest
rate implicit in the lease is 4.5%, The related information as on
August 15, 2016 is given below:

Cost Carrying Fair Value Value in Amount of


Value Use
Financing
Rs.000 Rs.000 Rs.000 Rs.000 Rs.000
Generator A 10,000 7,500 6,000 6,500 6,000
Generator B 12,000 6,000 5,000 5,000 6,000
Generator C 10,000 7,000 10,000 12,000 10,000

Required
Prepare the accounting entries that should be recorded by the company on August 15, 2016 in
respect of the above transactions.
Note: Cost of making sale is negligible. Ignore tax and deferred tax implications, if any.

© Emile Woolf International 37 The Institute of Chartered Accountants of Pakistan


Advanced accounting and financial reporting

11.9 MODIFICATION THAT DECREASES THE SCOPE OF THE LEASE (IFRS 16,
ILLUSTRATIVE EXAMPLE 17)
Lessee enters into a 10-year lease for 5,000 square metres of office space. The annual lease
payments are CU 50,000 payable at the end of each year. The interest rate implicit in the lease
cannot be readily determined. Lessee’s incremental borrowing rate at the commencement date is
6 per cent per annum. At the beginning of Year 6, Lessee and Lessor agree to amend the
original lease to reduce the space to only 2,500 square metres of the original space starting from
the end of the first quarter of Year 6. The annual fixed lease payments (from Year 6 to Year 10)
are CU30,000. Lessee’s incremental borrowing rate at the beginning of Year 6 is 5 per cent per
annum.
Required
How the lessee should reflect in its books of accounts:
a) Right-of-use retained b) Lease liability

11.10 MODIFICATION THAT BOTH INCREASES AND DECREASES THE SCOPE OF THE LEASE
(IFRS 16, ILLUSTRATIVE EXAMPLE 18)
Lessee enters into a 10-year lease for 2,000 square metres of office space. The annual lease
payments are CU100,000 payable at the end of each year. The interest rate implicit in the lease
cannot be readily determined. Lessee’s incremental borrowing rate at the commencement date is
6 per cent per annum. At the beginning of Year 6, Lessee and Lessor agree to amend the
original lease to;
a) include an additional 1,500 square metres of space in the same building starting from the
beginning of Year 6 and
b) reduce the lease term from 10 years to eight years. The annual fixed payment for the 3,500
square metres is CU150,000 payable at the end of each year (from Year 6 to Year 8).
Lessee’s incremental borrowing rate at the beginning of Year 6 is 7 per cent per annum.
Required
How the lessee should account for;
a) Pre-modification right-of-use and lease liability
b) At the effective date of modification
c) Decrease in the lease term
d) Increase in the leased space

11.11 SUBLEASE CLASSIFIED AS A FINANCE LEASE (IFRS 16, ILLUSTRATIVE EXAMPLE 20)
Head lease—An intermediate lessor enters into a five-year lease for 5,000 square metres of
office space (the head lease) with Entity A (the head lessor).
Sublease—At the beginning of Year 3, the intermediate lessor subleases the 5,000 square
metres of office space for the remaining three years of the head lease to a sublessee.
Required
How this transaction is accounted for in the books of intermediate lessor.

11.12 SUBLEASE CLASSIFIED AS AN OPERATING LEASE (IFRS 16, ILLUSTRATIVE


EXAMPLE 21)
Head lease—An intermediate lessor enters into a five-year lease for 5,000 square metres of
office space (the head lease) with Entity A (the head lessor).
Sublease—At commencement of the head lease, the intermediate lessor subleases the 5,000
square metres of office space for two years to a sublessee.
Required
How this transaction is accounted for in the books of intermediate lessor

© Emile Woolf International 38 The Institute of Chartered Accountants of Pakistan


Questions

CHAPTER 12 – IAS 37: PROVISIONS CONTINGENT LIABILITIES AND


CONTINGENT ASSETS

12.1 ROWSLEY
Rowsley is a diverse group with many subsidiaries. The group is proud of its reputation as a
‘caring’ organisation and has adopted various ethical policies towards its employees and the
wider community in which it operates. As part of its Annual Report, the group publishes details of
its environmental policies, which include setting performance targets for activities such as
recycling, controlling emissions of noxious substances and limiting use of non-renewable
resources.
The finance director is reviewing the accounting treatment of various items prior to finalising the
accounts for the year ended 31 March 20X4. All items are material in the context of the accounts
as a whole. The accounts are due to be approved by the directors on 30 June 20X4.
Closure of factory
On 15 February 20X4, the board of Rowsley decided to close down a large factory in Derbytown.
The board is trying to draw up a plan to manage the effects of the reorganisation, and it is
envisaged that production will be transferred to other factories. The factory will be closed on 31
August 20X4, but at 31 March 20X4 this decision had not yet been announced to the employees
or to any other interested parties. Costs of the reorganisation have been estimated at Rs. 45
million
Relocation of subsidiary
During December 20X3, one of the subsidiary companies moved from Buckington to Sundertown
in order to take advantage of government development grants. Its main premises in Buckington
are held under an operating lease, which runs until 31 March 20X9. Annual rentals under the
lease are Rs. 10 million. The company is unable to cancel the lease, but it has let some of the
premises to a charitable organisation at a nominal rent. The company is attempting to rent the
remainder of the premises at a commercial rent, but the directors have been advised that the
chances of achieving this are less than 50%.
Legal claim
During the year to 31 March 20X4, a customer started legal proceedings against the group,
claiming that one of the food products that it manufactures had caused several members of his
family to become seriously ill. The group’s lawyers have advised that this action will probably not
succeed.
Environmental impact of overseas subsidiary
The group has an overseas subsidiary that is involved in mining precious metals. These activities
cause significant damage to the environment, including deforestation. The company expects to
abandon the mine in eight years’ time. The mine is situated in a country where there is no
environmental legislation obliging companies to rectify environmental damage and it is very
unlikely that any such legislation will be enacted within the next eight years. It has been
estimated that the cost of cleaning the site and re-planting the trees will be Rs. 25 million if the
re-planting was successful at the first attempt, but it will probably be necessary to make a further
attempt, which will increase the cost by a further Rs. 5 million.
Required
Explain how each of the items above should be treated in the consolidated financial statements
for the year ended 31 March 20X4

© Emile Woolf International 39 The Institute of Chartered Accountants of Pakistan


Advanced accounting and financial reporting

12.2 MULTAN PETROCHEM LTD


Multan Petrochem Limited (MPL) operates in the oil extraction and refining business and is
preparing its draft financial statements for the year ended 31 December 2016. The following
information has been collected for the preparation of the provisions and contingencies notes.
(1) A new site was acquired on 1 January 2015 and is being used as the site for a new oil
refinery. Initial preparation work was undertaken at the site at the start of 2015 and the oil
refinery was completed and ready for use on 31 December 2015. The new refinery was
expected to have a useful life of 25 years. MPL has a well-publicised policy that it will
reinstate any environmental damage caused by its activities. The present value of the
estimated cost of reinstating the environment is Rs. 1,300,000 for damage caused during
the initial preparation work. This amount is based on a discount rate of 8%.
(2) An explosion at one of MPL’s oil extraction plants on 1 July 2016 has led to a number of
personal injury claims being made by employees who were injured during the explosion.
Five claims have been made to date but if these claims are successful, it is likely that a
further three employees who were also injured will make a claim. MPL’s lawyers estimate
that it is probable that the claims will succeed and that the estimated average cost of each
payout will be Rs. 150,000. The lawyers have recommended that MPL settles the claims
out of court as quickly as possible at their estimated amount for all eight employees injured
to avoid any adverse publicity.
An additional two claims have been made by employees for the stress, rather than injury,
that the explosion has caused them. If these claims were to succeed the lawyers have
estimated that the likely payout would be around Rs. 10,000 per employee. However, the
lawyers have stated that they believe it to be unlikely that these employees will win such a
case.
MPL made an insurance claim to try to recover the personal injury costs that it is probable
that it will incur. The claim is now in its advanced stages and the insurance company has
agreed to meet the cost of the claims in full. The insurance company will refund MPL once
the claims have been settled.
(3) The future of MPL’s business operations is in doubt following the explosion at the oil
extraction plant. The national press criticised MPL for the way that it handled the problem.
To address this, on 1 October 2016 MPL paid Rs. 12,000 to a risk assessment specialist
who has recommended introducing a new disaster recovery plan at an estimated cost of
Rs. 500,000.
(4) MPL entered into an operating lease in the previous period for some office space.
However, the company’s plans changed and the office space was no longer required. At 1
January 2016 a correctly calculated provision had been made for the future outstanding
rentals of Rs. 80,000 for the remaining five years. This was based on a discount rate of
8%. The rent paid during the period was Rs. 15,000. In addition, MPP has signed a sub-
lease to rent out the space for the first six months of next year for total rental income of Rs.
6,000. No other tenants are expected to be found for the office space.
Required
(a) Prepare the provisions and contingencies notes for inclusion in the financial statements of
MPP for the year ended 31 December 2016.
(b) List the amounts that should be recognised in the statement of profit or loss for the year
ended 31 December 2016.

12.3 VIOLET POWER LIMITED


Violet Power Limited is running a coal based power project in Pakistan. The Company has built
its plant in an area which contains large reserves of coal. The company has signed a 20 years
agreement for sale of power to the Government. The period of the agreement covers a
significant portion of the useful life of the plant. The company is liable to restore the site by
dismantling and removing the plant and associated facilities on the expiry of the agreement.

© Emile Woolf International 40 The Institute of Chartered Accountants of Pakistan


Questions

Following relevant information is available:


(i) The plant commenced its production on July 1, 2015. It is the policy of the company to
measure the related assets using the cost model;
(ii) Initial cost of plant was Rs. 6,570 million including erection, installation and borrowing
costs but does not include any decommissioning cost;
(iii) Residual value of the plant is estimated at Rs. 320 million;
(iv) Initial estimate of amount required for dismantling of plant, at the time of installation of plant
was Rs. 780 million. However, such estimate was reviewed as of June 30, 2016 and was
revised to Rs. 1,021 million;
(v) The Company follows straight line method of depreciation; and
(vi) Real risk-free interest rate prevailing in the market was 8% per annum when initial
estimates of decommissioning costs were made. However, at the end of the year such rate
has dropped to 6% per annum.
Required
Work out the carrying value of plant and decommissioning liability as of June 30, 2016.

© Emile Woolf International 41 The Institute of Chartered Accountants of Pakistan


Advanced accounting and financial reporting

CHAPTER 13: IAS 19: EMPLOYEE BENEFITS

13.1 LABURNUM LIMITED


Laburnum Limited operates a defined benefit pension plan for its employees. At 1 January 2016
the fair value of the pension plan assets was Rs. 2,600,000 and the present value of the plan
liabilities was Rs. 2,900,000.
The actuary estimates that the current and past service costs for the year ended 31 December
2016 is Rs. 450,000 and Rs. 90,000 respectively. The past service cost is caused by an increase
in pension benefits. The plan liabilities at 1 January and 31 December 2016 correctly reflect the
impact of this increase.
The interest cost on the plan liabilities is estimated at 8% and the expected return on plan assets
at 5%.
The pension plan paid Rs. 240,000 to retired members in the year to 31 December 2016.
Laburnum Limited paid Rs. 730,000 in contributions to the pension plan and this included Rs.
90,000 in respect of past service costs.
At 31 December 2016 the fair value of the pension plan assets is Rs. 3,400,000 and the present
value of the plan liabilities is Rs. 3,500,000.
In accordance with the amendment to IAS 19 Employee Benefits, Laburnum Limited recognises
actuarial gains and losses in other comprehensive income in the period in which they occur.
Required
Calculate the actuarial gains or losses on pension plan assets and liabilities that will be included
in other comprehensive income for the year ended 31 December 2016. (Round all figures to the
nearest Rs.000).

13.2 JABEL LIMITED


Jabel Limited operates a defined benefit pension plan for its employees. At 1 July 2015 the fair
value of the pension plan assets was Rs. 1,200,000 and the present value of the plan liabilities
was Rs. 1,400,000. The interest cost on the plan liabilities was estimated at 7% and the expected
return on plan assets at 4%.
The actuary estimates that the current service cost for the year ended 30 June 2016 is Rs.
300,000. Jabel Limited made contributions into the pension plan of Rs. 400,000 in the year. The
pension plan paid Rs. 220,000 to retired members in the year to 30 June 2016.
At 30 June 2016 the fair value of the pension plan assets was Rs. 1,400,000 and the present
value of the plan liabilities was Rs. 1,600,000.
In accordance with the amendment to IAS 19 Employee Benefits, Jabel Limited recognises
actuarial gains and losses in other comprehensive income in the period in which they occur.
Required
Calculate the net expense that will be included in Jabel Limited’s profit or loss AND the amounts
that would be included in other comprehensive income in respect of actuarial gains or losses for
the year ended 30 June 2016. (Round all workings to the nearest Rs.000)

13.3 KAGHZI LIMITED


Kaghzi Limited operates a defined benefit pension plan for its employees. At 1 January 2016 the
fair value of the pension plan assets was Rs. 1,400,000 and the present value of the pension
plan liabilities was Rs. 1,700,000.
The actuary estimates that the service cost for the year to 31 December 2016 is Rs. 320,000.
The interest cost on the plan liabilities is estimated at 7% and the expected return on plan assets
at 4% for the year to 31 December 2016. The pension plan paid Rs. 170,000 to retired members
and Kaghzi Limited paid Rs. 580,000 in contributions to the pension plan for the year to 31
December 2016.

© Emile Woolf International 42 The Institute of Chartered Accountants of Pakistan


Questions

At 31 December 2016 the fair value of the pension plan assets is Rs. 2,100,000 and the present
value of the pension plan liabilities is Rs. 2,400,000. In accordance with the amendment to IAS
19 Employee Benefits, Kaghzi Limited recognises actuarial gains and losses within other
comprehensive income in the period in which they occur.
Required
Calculate the actuarial gains or losses on pension plan assets and liabilities that will be included
in Kaghzi Limited’s other comprehensive income for the year ended 31 December 2016. (Round
all figures to the nearest Rs.000).

13.4 LASURA LTD


Lasura Ltd operates a defined benefit pension plan for its employees. At 1 July 2015 the fair
value of the pension plan assets was Rs. 2,200,000 and the present value of the pension plan
liabilities was Rs. 2,400,000. The interest cost on the pension plan liabilities was estimated at 8%
and the expected return on pension plan assets at 5%.
The actuary estimates that the current service cost for the year ended 30 June 2016 is Rs.
500,000. Lasura Ltd made contributions into the pension plan of Rs. 300,000 and the pension
plan paid Rs. 450,000 to retired members in the year to 30 June 2016. At 30 June 2016 the fair
value of the pension plan assets was Rs. 2,300,000 and the present value of the pension plan
liabilities was Rs. 2,700,000.
Actuarial gains and losses are included within the other comprehensive income of Lasura Ltd as
incurred.
Required
(i) Calculate the net expense that will be included in Lasura Ltd’s profit or loss for the year
ended 30 June 2016, in accordance with IAS 19 Employee benefits.
(ii) Calculate the amount that will be included in Lasura Ltd’s other comprehensive income for
the year ended 30 June 2016, in accordance with IAS 19 Employee benefits.

13.5 UNIVERSAL SOLUTIONS


(a) Explain the following as used in IAS 19 Employee Benefits:
(i) The term ‘defined benefit pension plan’
(ii) The basis to be adopted in measuring scheme assets
(iii) The basis to be adopted in measuring scheme liabilities
(iv) Actuarial gains and losses.
(b) Universal Solutions operates a defined benefit pension scheme on behalf of its employees.
The company conducts an annual review of funding in conjunction with their actuaries who
have supplied the following information:
At 31 Dec At 31 Dec
Year 3 Year 4
Rs. Rs.
Present value of pension fund obligations 1,200 1,300
Market value of pension fund assets 1,000 1,100
Information relevant to the actuarial valuation:
Discount rate used to determine pension fund liabilities 5%
Current service cost Rs. 100
Contributions to the pension fund Rs. 140
Benefits paid out amounted to Rs. 95

© Emile Woolf International 43 The Institute of Chartered Accountants of Pakistan


Advanced accounting and financial reporting

Required
(i) Show the figures that would appear on the face of the statement of financial
performance as at 31 December Year 3 and Year 4.
(ii) Construct a journal to explain the movement on the defined benefit net asset (or net
liability) during the year ended 31 December Year 4

13.6 DHA INTERIORS LTD


DHA Interiors Ltd operates two pension plans.
Pension Plan 1
The terms of the plan are as follows:
(i) employees contribute 6% of their salaries to the plan
(ii) DHA Interiors Ltd contributes, currently, the same amount to the plan for the benefit of the
employees
(iii) On retirement, employees are guaranteed a pension which is based upon the number of
years’ service with the company and their final salary.
(iv) This plan was closed to new entrants from 31 October 2015, but which was open to future
service accrual for the employees already in the scheme.
The following details relate to the plan in the year to 31 October 2016:
Rs. m
Present value of obligation at 1 November 2015 200
Present value of obligation at 31 October 2016 240
Fair value of plan assets at 1 November 2015 190
Fair value of plan assets at 31 October 2016 225
Current service cost 20
Pension benefits paid 19
Total contributions paid to the scheme for year to 31 October 2016 17
Actuarial gains and losses are recognised in the ‘statement of other comprehensive income
(OCI)’.
Pension Plan 2
Under the terms of the plan, DHA Interiors Ltd does not guarantee any return on the contributions
paid into the fund. The company’s legal and constructive obligation is limited to the amount that is
contributed to the fund. The following details relate to this scheme:
Rs. m
Fair value of plan assets at 31 October 2016 21
Contributions paid by company for year to 31 October 2016 10
Contributions paid by employees for year to 31 October 2016 10

The discount rates for the two plans are:


1 November 2015 31 October 2016
Discount rate 5% 6%
Required
(a) Explain the nature of and differences between a defined contribution plan and a defined
benefit plan with specific reference to the company’s two schemes.
(b) Show the accounting treatment for the two DHA Interiors Ltd pension plans for the year
ended 31 October 2016 under IAS 19 ‘Employee Benefits’.

© Emile Woolf International 44 The Institute of Chartered Accountants of Pakistan


Questions

CHAPTER 14: IFRS 2: SHARE BASED PAYMENTS

14.1 TOSHACK LTD


Toshack Ltd has granted 50 share appreciation rights to each of its 1000 employees on 1
January 2013. The rights are due to vest on 31 December 2016, with payment being made on 31
December 2017. Assume that 75% of the awards vest. Shares prices were:
Rs.
01/01/2013 22
31/12/2013 27
31/12/2016 31
31/12/2017 28
Required
In accordance with IFRS 2, Share Based Payment;
(i) What liability would be recorded on 31 December 2016 for the share appreciation rights?
(ii) How would the settlement of the transaction be accounted for on 31 December 2017.

14.2 IFRS 2
(a) IFRS 2 requires an entity to recognise share-based payment transactions in its financial
statements. These include transactions with the employees or other parties where they are
to be settled in cash, other assets or equity instruments of the entity.
The IFRS identifies three types of share-based payment transaction and sets out the
measurement principles and specific requirements for each.
Required
(i) Suggest why there was a need for a standard in this area.
(ii) Identify and briefly explain the three types of share based payments recognised by
IFRS 2.
(b) A client of your firm, a listed company with a 31 December year end, contacts you for
advice on a proposed share option scheme for its employees.
On 1 January Year 5, the client granted 100 options to each of its 500 employees. The
grant is conditional upon the employee working for the client over the next three years. At
the grant date, it is estimated that the fair value of each option is Rs. 15.
Calculate the expense in profit or loss for each year of the vesting period:
(i) assuming that the client’s expectations throughout the vesting period are that all
options will vest; and alternatively
(ii) assuming that the client’s best estimates of the proportion of options that will vest
are as follows:
Estimate at 31 December Year 5 85%
Estimate at 31 December Year 6 88%
With 44,300 options actually vesting at 31 December Year 7.

14.3 SAVAGE LTD


Savage Ltd granted share options to its 300 employees on 1 October 2014. Each employee will
receive 1,000 share options provided they continue to work for Savage Ltd for the following three
years from the grant date. The fair value of the options at the grant date was Rs. 11 each. In the
year ended 30 September 2015, 10 employees left and another 30 were expected to leave over
the next two years. For the year ended 30 September 2016, 20 employees left and another 15
are expected to leave in the year to 30 September 2017.

© Emile Woolf International 45 The Institute of Chartered Accountants of Pakistan


Advanced accounting and financial reporting

Required
Discuss the accounting treatment to be adopted for the share options and calculate the amount
to be recognised in the statement of profit or loss in respect of these options for the year ended
30 September 2016. Prepare appropriate accounting entries.

14.4 YORATH LTD


Yorath Ltd granted share options to its 600 employees on 1 October 2013. Each employee will
receive 500 share options provided they continue to work for Yorath Ltd for four years from the
grant date. The fair value of each option at the grant date was Rs. 148.
The actual and expected staff movement over the 4 years to 30 September 2017 is given below:
2014 20 employees left and another 50 were expected to leave over the next three years.
2015 A further 25 employees left and another 40 were expected to leave over the next two years.
2016 A further 15 employees left and another 20 were expected to leave the following year.
2017 No actual figures are available to date.
The sales director of Yorath Ltd has stated in the board minutes that he disagrees with the
treatment of the share options. No cash has been paid out to employees, therefore he fails to
understand why an expense is being charged against profits.
Required
(a) Calculate the charge to the statement of profit or loss for the year ended 30 September
2016 for Yorath Ltd in respect of the share options and prepare the journal entry to record
this.
(b) Explain the principles of recognition and measurement for share-based payments as set
out in IFRS 2 Share-based Payments so as to address the concerns of the sales director.

14.5 QUALTECH LTD


(a) Qualtech Ltd granted share options to its 300 employees on 1 January 2015. Each
employee will receive 1,000 share options provided they continue to work for Qualtech Ltd
for 3 years from the grant date. The fair value of each option at the grant date was Rs.
122.
The actual and expected staff movement over the 3 years to 31 December 2017 is
provided below:
2015: 25 employees left and another 40 were expected to leave over the next two years.
2016: A further 15 employees left and another 20 were expected to leave the following
year.
Required
(i) Calculate the charge to Qualtech Ltd’s statement of profit or loss for the year ended
31 December 2016 in respect of the share options and prepare the journal entry to
record this.
(ii) Explain how the recognition and measurement of a share-based payment would
differ if it was to be settled in cash rather than in equity, in accordance with IFRS 2
Share-based Payments.

14.6 BRIDGE LTD


Bridge Ltd granted 1,000 share options to each of its 300 employees on 1 January 2015, with the
condition that they continue to work for Bridge Ltd for 4 years from the grant date. The fair value
of each option at the grant date was Rs. 50.

© Emile Woolf International 46 The Institute of Chartered Accountants of Pakistan


Questions

20 employees left in the year to 31 December 2015 and at that date another 65 were expected to
leave over the next three years. 23 employees left in the year to 31 December 2016 and at that
date another 44 were expected to leave over the next two years.
Required
(i) Calculate the charge to Bridge Ltd’s statement of profit or loss for the year ended 31
December 2016 in respect of the share options and prepare the journal entry to record
this.
(ii) Explain why, in accordance with IFRS 2 Share-based Payment, share options, such as
those granted by Bridge Ltd, generate a charge to the statement of profit or loss despite no
cash transaction having occurred.

14.7 CAPSTAN LTD


Capstan Ltd granted 1,000 share appreciation rights (SARs), to its 300 employees on 1 January
2015. To be eligible, employees must remain employed for 3 years from the date of issue and the
rights must be exercised in January 2014, with settlement due in cash.
In the year to 31 December 2015, 32 staff left and a further 35 were expected to leave over the
following two years.
In the year to 31 December 2016, 28 staff left and a further 10 were expected to leave in the
following year.
The fair value of each SAR was Rs. 80 at 31 December 2015 and Rs. 120 at 31 December 2016.
Required
Prepare the accounting entry to record the expense associated with the SARs, for the year to 31
December 2016, in accordance with IFRS 2 Share-based Payments.

14.8 NEWTOWN LTD


Newtown Ltd granted 1,000 share appreciation rights (SARs) to each of its 500 employees on 1
July 2014. To be eligible for the rights, employees must remain employed by Newtown Ltd for 3
years from the date of grant. The rights must be exercised in July 2017, with settlement due in
cash.
In the year to 30 June 2015, 42 employees left and a further 75 were expected to leave over the
following two years.
In the year to 30 June 2016, 28 employees left and a further 25 were expected to leave in the
following year.
The fair value of each SAR was Rs. 90 at 30 June 2015 and Rs. 110 at 30 June 2016.
Required
(i) Prepare the journal entry to record the expense associated with the SARs for the year
ended 30 June 2016, in accordance with IFRS 2 Share-based payment.
(ii) Explain, in accordance with IFRS 2 Share-based payment, how the recognition and
measurement of a share-based payment would differ, if it was to be settled in equity rather
than cash.

14.9 SINDH TRANSIT LTD


Sindh Transit Ltd granted share options to all of its 400 employees on 1 January 2015. Each
employee will receive 1,000 share options provided they continue to be employed by Sindh
Transit Ltd for four years from the grant date. The fair value of an option at the grant date was
Rs. 220.
On the same date Sindh Transit Ltd granted 500 share appreciation rights to each of its
employees. To be eligible, employees again have to be employed by Sindh Transit Ltd for four
years from the grant date.

© Emile Woolf International 47 The Institute of Chartered Accountants of Pakistan


Advanced accounting and financial reporting

The rights are exercisable in the two-month period from 1 January 2019 and will be settled in
cash. The fair value of each share appreciation right was Rs. 120 at 31 December 2015 and Rs.
140 at 31 December 2016.
The actual and expected future staff movements as at 31 December 2015 and 31 December
2016 are provided below.
2015: 15 left and another 55 were expected to leave over the next three years.
2016: a further 22 left and another 36 were expected to leave over the next two years.
Required
(a) Prepare, in accordance with IFRS 2 Share-based Payment, the accounting entries
required in the financial statements of Sindh Transit Ltd for the year to 31 December 2016
in respect of the two financial instruments identified above.
(b) Explain the main principle of recognition set out by IFRS 2 Share-based Payment for share
based payments AND why the treatment of the two financial instruments identified above
will differ in the statement of financial position.

© Emile Woolf International 48 The Institute of Chartered Accountants of Pakistan


Questions

CHAPTER 15: FINANCIAL INSTRUMENTS: RECOGNITION AND MEASUREMENT

15.1 AJI PANCA LTD


On 1 January Year 1 Aji Panca Ltd has the following capital and reserves.
Equity Rs.
Share capital (Rs. 1 ordinary shares) 1,000,000
Share premium 200,000
Retained earnings 5,670,300
––––––––––
6,870,300
––––––––––
During Year 1 the following transactions took place.
1 January An issue of Rs. 100,000 8% Rs. 1 redeemable preference shares at a
premium of 60%. Issue costs are Rs. 2,237. Redemption is at 100% premium
on 31 December Year 5. The effective rate of interest is 9.5%.
31 March An issue of 300,000 ordinary shares at a price of Rs. 1.30 per share. Issue
costs, net of tax benefit, were Rs. 20,000
30 June A 1 for 4 bonus issue of ordinary shares.
Profit for the year, before accounting for the above, was Rs. 508,500. The dividends on the
redeemable preference shares have been charged to retained earnings.
Required
Set out capital and reserves and liabilities resulting from the above on 31 December Year 1.

15.2 PASSILA LTD


On 1 July 2016, Passila Ltd, issued 20,000 8% debentures at Rs. 97.50. The security is
redeemable in five years’ time. The interest on the debentures is payable bi-annually on 30 June
and 31 December.
On 31 December 2016, the Company’s year-end date, the debentures were quoted on the
Karachi Stock Exchange for Rs. 96.00. The company accountant has suggested each of the
following as possible valuation basis for reporting the debentures liability on the statement of
financial position as at 31 December 2016:
(i) Face value of the debentures.
(ii) Face value of the debenture plus interest payment for five years.
(III) Market value on the statement of financial position as at the year end.
Required
(a) Determine the face value of the debentures and the proceeds accruing to the company.

(b) Determine the amount and explain the nature of the differences between the face value
and the market value of the debentures on 1 July, 2016.
(c) Distinguish between nominal and effective rate of interest.
(d) Determine the nominal interest payable on the debentures for the year ended 31
December 2016.
(e) State arguments for or against each of the suggested alternatives for reporting the
debentures liability on the statement of financial position as at 31 December 2016.

© Emile Woolf International 49 The Institute of Chartered Accountants of Pakistan


Advanced accounting and financial reporting

15.3 FINANCIAL INSTRUMENTS


(a) Explain the IFRS 9 rules on the measurement of financial assets and financial liabilities.
(b) Explain how the following should be accounted for in accordance with IAS 32 and IFRS 9
in the financial statements to 31 December Year 3.
(i) A 3% bond was purchased on 1 January Year 3 for Rs. 250,000. The nominal value
is Rs. 300,000 and redemption will be at par on 31 December Year 6. The coupon is
received annually in arrears. The effective interest rate on the bond is 9.7%. The
company intends to hold the bond until its maturity. The market value of the bond at
31 December Year 3 is Rs. 275,000.
(ii) An investment was made in the equity shares of XYZ. 3,000 shares were purchased
(a 1% stake) at a cost of Rs. 10 per share on 1 April Year 1. A transaction fee of Rs.
300 was charged on the purchase. The entity intended to sell the shares within three
months but the market value of the investment continued to rise and so the
company decided not to sell in the near term. The market value of the shares over
the three years has been as follows:
Rs.000
31 December Year 1 32
31 December Year 2 34
31 December Year 3 35
(iii) The company issued a convertible bond at par on 31 December Year 3, raising Rs.
500,000. The coupon on the bond is 4%. The rate on an equivalent redeemable
bond is 7%. The bond can be redeemed at par on 31 December Year 6 or converted
into equity shares at a rate of five shares per Rs. 100. The bond has not been
classed as fair value through profit and loss.

15.4 ESPANOLA LTD


(a) Espanola Ltd acquired an investment in a debt instrument on 1 January 2016 at its par
value of Rs. 3 million. Transaction costs relating to the acquisition were Rs. 200,000. The
investment earns a fixed annual return of 6%, which is received in arrears. The principal
amount will be repaid to Espanola Ltd in 4 years’ time at a premium of Rs. 400,000. The
investment has been correctly classified as held to maturity. The investment has an
effective interest rate of approximately 7.05%.
Required
(i) Explain how this financial instrument will be initially recorded AND subsequently
measured in the financial statements of Espanola Ltd, in accordance with IAS 39
Financial Instruments: Recognition and Measurement.
(ii) Calculate the amounts that would be included in Espanola Ltd’s financial statements
for the year to31 December 2016 in respect of this financial instrument.
(b) Espanola Ltd acquired 100,000 shares in X Ltd on 25 October 2016 for Rs. 3 per share.
The investment resulted in Espanola Ltd holding 5% of the equity shares of X Ltd. The
related transaction costs were Rs. 12,000. X Ltd’s shares were trading at Rs. 3.40 on 31
December 2016. The investment has been classified as held for trading.
Required
Prepare the journal entries to record the initial AND subsequent measurement of this
financial instrument in the financial statements of Espanola Ltd for the year to 31
December 2016.

© Emile Woolf International 50 The Institute of Chartered Accountants of Pakistan


Questions

15.5 SANDIA LTD


Sandia Ltd acquired 40,000 shares in another entity, Y Ltd, in March 2016 for Rs. 2.68 per share.
The investment was classified as available for sale on initial recognition. The shares were trading
at Rs. 2.96 per share on 31 July 2016. Commission of 5% of the value of the transaction is
payable on all purchases and disposals of shares.
Required
(i) Prepare the journal entries to record the initial recognition of this financial asset and its
subsequent measurement at 31 July 2016 in accordance with IAS 39 Financial
Instruments: Recognition and Measurement.
The directors of Sandia Ltd are concerned about the value of Sandia Ltd’s investment in Y
Ltd and in an attempt to hedge against the risk of a fall in its value, are considering
acquiring a derivative contract. The directors wish to use hedge accounting in accordance
with IAS 39.
Required
(ii) Discuss how both the available for sale investment and any associated derivative contract
would be subsequently accounted for, assuming that the criteria for hedge accounting
were met, in accordance with IAS 39.

15.6 GEO ALLOYS LTD


GEO Alloys Ltd made an investment in a debt instrument on 1 July 2014 at its nominal value of
Rs. 4,000,000. The instrument carries a fixed coupon interest rate of 7%, which is receivable
annually in arrears. The instrument will be redeemed for Rs. 4,530,000 on 30 June 2018.
Transaction costs associated with the investment were Rs. 200,000 and were paid on 1 July
2014. The effective interest rate applicable to this instrument has been calculated at
approximately 8.4%. GEO Alloys Ltd intends to hold this investment until its redemption date.
Required
(a) (i) Explain how this investment should be classified and prepare the journal entry to
initially record it in accordance with relevant accounting standards.
(ii) Calculate the carrying value of the investment to be included in GEO Alloys Ltd’s
statement of financial position as at 30 June 2016, in accordance with IAS 39
Financial instruments: recognition and measurement.
GEO Alloys Ltd’s main business risk is the price of raw materials. As a manufacturer of jewellery,
its profits can be significantly affected by the price of precious metals. Therefore, in order to
minimise the risk of future price increases adversely affecting its future profits, GEO Alloys Ltd
entered into a forward contract on 1 May 2016, at nil cost, to purchase 100,000 units of metal A
at Rs. 10,500 per unit on 1 August 2016.
At 30 June 2016, the forward rate for purchasing 100,000 units of metal A was Rs. 10,100 per
unit. GEO Alloys Ltd adopts hedge accounting where permitted by IAS 39 Financial instruments:
recognition and measurement.
Required
(b) Explain how this forward contract should be accounted for by GEO Alloys Ltd in its
financial statements for the year ended 30 June 2016, in accordance with IAS 39 Financial
instruments: recognition and measurement.

15.7 CASCABEL LTD


Cascabel Ltd entered into a forward contract on 31 July 2016 to purchase A$.2 million at a
contracted rate of Rs. 1: A$0.64 on 31 October 2016. The contract cost was nil. Cascabel Ltd
prepares its financial statements to 31 August 2016. At 31 August 2016 an equivalent contract
for the purchase of A$2 million could be acquired at a rate of Rs. 1: A$0.70.
Required
(a) Explain how this financial instrument should be classified and prepare the journal entry
required for its measurement as at 31 August 2016.

© Emile Woolf International 51 The Institute of Chartered Accountants of Pakistan


Advanced accounting and financial reporting

(b) Assume now that the instrument described above was designated as a hedging instrument
in a cash flow hedge, and that the hedge was 100% effective.
Explain how the gain or loss on the instrument for the year ended 31 August 2016 should
now be recorded and why different treatment is necessary.

15.8 FAIR VALUE HEDGE ACCOUNTING


th
X Ltd holds an inventory of tin. At 30 September 2015 X Ltd decided to hedge this position with
a 6 month forward sale of 30 tonnes of the same grade of tin.

Relevant information
Tin inventory Forward
(Rs.) contract (Rs.)
Carrying amount of inventory at 30 Sept 2015, at
1,000,000 N/A
the lower of cost and NRV
Fair values at 30 Sept 2015 1,300,000 NIL
Fair values at 31 Dec 2015 1,200,000 95,000
Fair values at 31 March 2016, when the tin is sold
1,150,000 142,000
and the contract is closed
Required
(a) Prepare journals for the year ended 31 December 2015
(b) Prepare the journals that are necessary at 31 March 2016

15.9 CASH FLOW HEDGE ACCOUNTING


th
At 30 November 2015, the company decided to acquire a new drilling machine from a foreign
supplier. The machine is essential, and there is absolutely no likelihood that the purchase will be
delayed.
The price of the machine is A$400,000, payable upon delivery which is anticipated to be 28
February 2016.
th
Spot rate a 30 November 2015 is Rs.0.70 = A$1.
In order to hedge the exchange rate risk, the company enters into a forward foreign exchange
contract to buy A$400,000 forward 3 months, at a rate of 0.70 (Rs. = A$1).
At 31 December 2015, the forward rate in the market for 2 month delivery is 0.75.
The machine was duly delivered on 28 February, and the exchange rate ruling on that day of
payment was 0.80. The forward contract was closed out. It is the company’s accounting policy to
take any deferred gains/losses on a cash flow hedge of the acquisition of a non-financial asset,
against the cost of that asset (a basis adjustment).
For both situations, ignore the effect of time value of money and transaction costs.
Required
(a) Prepare journals for the year ended 31 December 2015
(b) Prepare the journals that are necessary at 28 February 2016)
(Note: A$ is a fictional currency used for the purposes of this example)

15.10 WATERS LTD


Waters Ltd acquired the following financial assets and liabilities in 2016.
1 On 1 September, Waters acquired 2,000 Rs. 100 nominal units of 7% treasury stock 2022
for Rs. 104.10 per unit. The gross redemption yield at the date of purchase was 6.30%.
Waters does not intend to hold the treasury stock until maturity, as the cash may be
required in the meantime. Interest is paid annually in arrears.

© Emile Woolf International 52 The Institute of Chartered Accountants of Pakistan


Questions

2 Waters buys and sells goods in Constantia, a country whose currency is the Prif (PR). On
3 December Waters enters into a futures contract to sell PR500,000 on 30 April 2017 at an
agreed price of PR1.98/Rs. 1. This contract is not part of a designated hedge. The cost of
entering into the contract was Rs. 750.
3 On 5 February Waters acquired 250,000 ordinary shares in Gilmour Ltd at Rs. 4.85 per
share incurring Rs. 35,000 attributable transaction costs.
4 On 1 July Waters sells goods to Mason for Rs. 500,000 on interest free credit payable 30
June 2017. The imputed rate of interest is 11%.
5 On 30 April Waters acquired 1,000 Rs. 100 nominal units of 8.5% treasury stock 2018 at
Rs. 107.10 per unit. The gross redemption yield is 5.9%. Waters intends to hold the
investment to maturity. Interest is paid annually in arrears.
6 On 26 December Waters purchased Rs. 25,000 of quoted company loan notes. This asset
has been designated as being held for short-term trading purposes.
7 On 24 December Waters sold 10,000 shares 'short' in Wright Ltd for Rs. 3.60 each, hoping
that the share price would fall so that it could clear its position by buying the shares in
January 2017 at a lower price.
On 31 December 2016, the values are as follows:
1 Rs. 100 nominal units of 7% treasury stock 2022 are trading at Rs. 98.07 per unit at 31
December 2016. The gross redemption yield at that date is 7.3%.
2 The futures rate for a Prif contract with a delivery date of 30 April 2017 is PR1.99/Rs. 1.
3 The shares in Gilmour are now trading at Rs. 5.20 – Rs. 5.25 per share, having an
average of Rs. 5.05 during the year. Disposal costs would be 2% of the sale proceeds.
4 Amounts receivable from Mason remain outstanding at the reporting date. The imputed
interest rate for current sales is 12%.
5 The 8.5% treasury stock 2018 is now trading at Rs. 101.50 per unit and the gross
redemption yield is currently quoted at 7.48%.
6 The loan notes are now worth Rs. 25,500 due to the market being more confident that the
interest will be paid in full and on time.
7 Shares in Wright Ltd are now trading at Rs. 3.30 each.
Required
Explain and calculate the impact of the above transactions on the financial statements of Waters
Ltd for the year ended 31 December 2016.

15.11 ARIF INDUSTRIES LIMITED


Arif Industries Limited (AIL) owns and operates a textile mill with spinning and weaving units.
Due to recurring losses, AIL disposed of the weaving unit for an amount of Rs. 100 million on
July 1, 2015 and invested the proceeds in Pakistan Investment Bonds (PIBs).
Details of investment in PIBs are as follows:
(i) The PIBs were purchased through a commercial bank at face value. The bank initially
charged premium and investment handling charges of Rs. 4,641,483. At the time of
purchase, AIL’s intention was to liquidate the investment after four years and utilize the
realized amount for expansion of its spinning business. The bank has agreed to
repurchase the PIBs on June 30, 2019, at their face value.
(ii) The markup on PIBs is 15% for the initial two years and 20% for the remaining two years.
The effective yield on investment at the time of purchase was 15.50%.
Required
(a) Prepare an amortisation table showing the amortised cost and interest income over the life of
the loan asset.

© Emile Woolf International 53 The Institute of Chartered Accountants of Pakistan


Advanced accounting and financial reporting

Following on from the facts in part (a), suppose that on June 30, 2017 AIL realized that the bank
was in financial difficulties. Further investigation led the company to believe that it would only
receive 75% of the expected future cash flows (both mark-up and capital).
(b) Prepare an amortisation table showing the amortised cost and interest income over the life of
the loan after taking account of any necessary impairment loss to the loan asset.
Following on from the facts in part (a) and ignoring those on part (b), suppose that on June 30,
2017 AIL decided to defer the expansion plan by one year. The bank agreed to extend the
holding period accordingly and pay 20% interest in year 5 but reduced the repurchase price by
2%.
Required
(c) Prepare an amortisation table showing the amortised cost and interest income over the life of
the loan after taking account of any necessary adjustment to the carrying amount of the loan
asset.

15.12 QASMI INVESTMENT LIMITED


On 1 January 2009 Qasmi Investment Limited (QIL) purchased 1 million 12% Term Finance
Certificates (TFCs) issued by Taj Super Stores (TSS), which operates a chain of five Super
Stores. The terms of the issue are as under:
The TFCs have a face value of Rs. 100 each and were issued at a discount of 5%. These are
redeemable at a premium of 20% after five years.
Interest on the TFCs is payable annually in arrears on 31 December each year.
Effective interest rate calculated on the above basis is 16.426% per annum.
Due to a property dispute, TSS had to temporarily discontinue operations of two stores in 2010.
Consequently, TSS was unable to pay interest due on 31 December 2010 and 31 December
2011.
At the time of finalization of accounts for the year ended 31 December 2010, QIL was quite
hopeful of recovery of the interest and therefore, no impairment was recorded. However, in 2011,
after a thorough review of the whole situation, QIL’s management concluded that it would be able
to recover the face value of the TFCs along with the premium on the due date i.e. 31 December
2013, but the interest for the years 2010 to 2013 would not be received. Accordingly, QIL
recorded impairment in the value of the TFCs on 31 December 2011.
In 2012, TSS reached an out of court settlement of the property dispute and the stores became
operational. Subsequently, QIL and TSS agreed upon a revised payment schedule according to
which the present value of the agreed future cash flows on 31 December 2012 is estimated at
Rs. 115 million.
Required
Prepare journal entries in the books of QIL for the years ended 31 December 2011 and 2012.
Show all the relevant computations.

© Emile Woolf International 54 The Institute of Chartered Accountants of Pakistan


Questions

CHAPTER 16: FINANCIAL INSTRUMENTS: PRESENTATION AND DISCLOSURE

16.1 SERRANO LTD


On 1 October 2015 Serrano Ltd issued Rs. 10 million 6% convertible loan stock on the following
terms:
The issue price was at par.
The loan stock is convertible into the company’s equity shares at the option of the stockholders
four years after the date of its issue (30 September 2019) on the basis of 20 shares for each Rs.
100 of loan stock. Alternatively it will be redeemed at par.
Ancho Services had advised that if Serrano Ltd had issued similar loan stock without the
conversion rights, then it would have had to pay an interest (coupon) rate of 10% on the loan
stock. This is because the terms of conversion to equity shares are favourable.
Ancho Services further advised that because it is almost certain that the loan stock holders will
exercise their right to convert to equity shares, the loan stock has the substance of equity and
can be included as such on the statement of financial position. This has the added advantage of
improving/reducing the company’s gearing (debt/equity) in comparison to what would be the case
with the issue of ‘straight’ loan stock.
The present value of Rs. 1 receivable at the end of each year, based on discount rates of 6% and
10% can be taken as:
Required
In relation to the 6% convertible loan stock, calculate the finance cost to be shown in the
statement of profit or loss and the extracts from the statement of financial position for the year to
30 September 2016; and comment on Ancho Services’ advice.

16.2 POBLANO LTD


Poblano Ltd issued Rs. 10 million of 4% convertible loan notes on 1 October 2015, on which
interest is paid annually in arrears on 30 September. The loan notes are convertible into equity
shares of Poblano Ltd on 30 September 2018 at the rate of 20 shares in Poblano Ltd for every
Rs. 100 of notes. Alternatively the notes can be redeemed on that date for cash at par, at the
option of the note holder.
If Poblano Ltd had issued straight loan notes, redeemable at par after 3 years, it would have had
to pay interest at the rate of 7% in order to persuade investors to subscribe for them.
Required
Show how the convertible loan notes would be accounted for in the financial statements of
Poblano Ltd for the year to 30 September 2016.

16.3 PIQUIN LTD


(a) Piquin Ltd issued 10 million 5% convertible Rs. 1 bonds 2021 on 1 January 2016. The
proceeds of Rs. 10 million were credited to non-current liabilities and debited to bank. The
5% interest paid has been charged to finance costs in the year to 31 December 2016.
The market rate of interest for a similar bond with a five year term but no conversion terms
is 7%.
Required
Explain AND demonstrate how this convertible instrument would be initially measured in
accordance with IAS 32 Financial Instruments: Presentation AND subsequently measured
in accordance with IAS 39 Financial Instruments: Recognition and Measurement in the
financial statements for the year ended 31 December 2016.

© Emile Woolf International 55 The Institute of Chartered Accountants of Pakistan


Advanced accounting and financial reporting

(b) The directors of Piquin Ltd want to avoid increasing the gearing of the entity. They plan to
issue 5 million 6% cumulative redeemable Rs. 1 preference shares in 2017.
Required
Explain how the preference shares would be classified in accordance with IAS 32 Financial
Instruments: Presentation, AND the impact that this issue will have on the gearing of
Piquin Ltd.

16.4 AJI LTD


Aji Ltd issued Rs. 6 million 7% convertible bonds on 1 January 2016 at par. The bonds are
redeemable at par on 31 December 2019 or convertible at that date on the basis of two Rs. 1
ordinary shares for every nominal Rs. 10 of bonds.
At the date of issue the prevailing market rate for similar debt without conversion rights was 9%.
The interest due was paid on 31 December 2016 and recorded within finance costs during the
year.
Required
(a) Explain how convertible instruments are initially recognised, in accordance with IAS 32
Financial Instruments: Presentation, and prepare the journal entry to record the issue of
the bonds by Aji Ltd.
(b) (i) Explain how the bonds will be subsequently measured, in accordance with IFRS 9
Financial Instruments: Recognition and Measurement, and prepare the journal entry
to record the subsequent measurement of the bonds in the financial statements of
Aji Ltd for the year to 31 December 2016.
(ii) Prepare extracts that illustrate how the bonds will be presented in the statement of
financial position of Aji Ltd as at 31 December 2016.

16.5 CHILTEPIN LTD


Chiltepin Ltd issued Rs. 4 million 5% convertible bonds on 1 October 2015 for Rs. 3.9 million.
The bonds have a four year term and are redeemable at par. At the time the bonds were issued
the prevailing market rate for similar debt without conversion rights was 7%. The effective interest
rate associated with the bonds is 7% and the liability is measured, in accordance with IAS 39
Financial Instruments: recognition and measurement, at amortised cost. The interest due was
paid and recorded within finance costs during the year.
Required
Prepare the accounting entries to record the issue of the convertible bonds and to record the
adjustment required in respect of the interest expense on the bonds for the year ended 30
September 2016.

16.6 HABENERO LTD


Habenero Ltd issued 6 million 5% cumulative Rs. 1 preference shares on 1 January 2016 and 2
million Rs. 1 ordinary shares on 1 May 2016.
Required
(a) Explain, with reference to the principles of IAS 32 Financial Instruments: Presentation how
both of these instruments would be classified AND how their associated dividends would
be recorded in the financial statements of Habenero Ltd for the year ended 31 December
2016.
Habenero Ltd acquired 500,000 shares in X on 1 November 2016 for Rs. 2.80 per share and
classified this investment as held for trading. Habenero Ltd paid 0.5% commission on the value
of the transaction to its broker. X’s shares were trading at Rs. 3.42 on 31 December 2016.
Required
Prepare the journal entries to record:
(i) the initial measurement of the investment at 1 November 2016; and
(ii) the subsequent measurement of this investment in the financial statements of
Habenero Ltd at 31 December 2016.

© Emile Woolf International 56 The Institute of Chartered Accountants of Pakistan


Questions

CHAPTER 18 – IAS 12: INCOME TAXES

18.1 SHAKIR INDUSTRIES


Given below is the statement of profit or loss and other comprehensive income of Shakir
Industries for the year ended December 31, 2016:
2016
Rs. m
Sales 143.00
Cost of goods sold (96.60)
Gross profit 46.40
Operating expenses (28.70)
Operating profit 17.70
Other income 3.40
Profit before interest and tax 21.10
Financial charges (5.30)
Profit before tax 15.80
Following information is available:
(i) Operating expenses include an amount of Rs.0.7 million paid as penalty to SECP on non-
compliance of certain requirements of the Companies Act, 2017.
(ii) During the year, the company made a provision of Rs. 2.4 million for gratuity. The
actual payment on account of gratuity to outgoing members was Rs. 1.6 million.
(iii) Lease payments made during the year amounted to Rs.0.65 million which include
financial charges of Rs.0.15 million. As at December 31, 2016, obligations against assets
subject to finance lease stood at Rs. 1.2 million. The movement in assets held under
finance lease is as follows:
Rs. m
Opening balance – 01/01/2016 2.50
Depreciation for the year (0.7)
Closing balance – 31/12/2016 1.80
(iv) The details of owned fixed assets are as follows:

Accounting Tax
Rs. m Rs. m
Opening balance – 01/01/2016 12.50 10.20
Purchased during the year 5.3 5.3
Depreciation for the year (1.10) (1.65)
Closing balance – 31/12/2016 16.70 13.85

(v) Capital work-in-progress as on December 31, 2016 include financial charges of Rs. 2.3
million which have been capitalised in accordance with IAS-23 “Borrowing Costs”.
However, the entire financial charges are admissible, under the Income Tax Ordinance,
2002.
(vi) Deferred tax liability and provision for gratuity as at January 1, 2016 was Rs.0.55
million and Rs.0.7 million respectively.
(vii) Applicable income tax rate is 35%.
Required
Based on the available information, compute the current and deferred tax expenses for the
year ended December 31, 2016.

© Emile Woolf International 57 The Institute of Chartered Accountants of Pakistan


Advanced accounting and financial reporting

18.2 DWAYNE LTD (PART 1)


The following information has been obtained in order to allow completion of Dwayne Ltd’s
deferred tax balances as at 31st December 2015.
Statement of financial position at 31st December 2015 - Extracts
Carrying amount Tax base
Rs. 000 Rs. 000
Assets
Land & buildings 45,500 17,500
Plant and equipment 68,000 26,000
Cost of investment in Larry 750 750
Investments 72,000 65,000
Dividend receivable 150 -

Liabilities
Long-term debt 20,500 21,000
Trade payables 9,500 9,500
Defined benefit liability 1,000
st
Deferred tax liability (31 December 2014) 13,500
(i) Dwayne revalues its land and buildings on an annual basis. It has no investment
properties. The fair value of land and buildings was Rs. 60 million at 31st December
2015.The 2015 revaluation has not yet been accounted for in Dwayne’s financial
statements. The pre-tax revaluation surplus as at 31st December 2014 stood at Rs. 24m.
(ii) The balance on the investments line relates to a portfolio of equity holdings. Some of these
are categorised as fair value through profit or loss and the balance as available-for-sale.
The fair value loss on AFS investments was Rs. 1m during 2015. This loss is considered to
be temporary in nature. The entire portfolio of equity holdings was acquired during 2015.
(iii) Tax relief on the defined benefit expense is given on a cash basis.
(iv) Dividend income is not taxed in the jurisdiction in which Dwayne operates.
(v) Dwayne borrowed Rs. 21m just before the year end and incurred transaction costs of
500k. Transaction costs are allowable in full in the year in which a loan is raised.
(vi) The tax rate changed from 30% to 28% in the current year.
Required
(a) Prepare a schedule of temporary differences and resultant deferred tax for Dwayne.
(b) Prepare a note showing the movement on the consolidated deferred tax balance for the
year ending 31st December 2015.
(c) Prepare a journal showing the movement on the deferred taxation account showing the
entries due to rate changes and temporary differences arising during the period.

18.3 DWAYNE LTD (PART 2)


The investment in Dwayne’s statement of financial position is the cost of 80% of Larry. The date
of this acquisition was 31st December 2015.
The following statement of net assets relates to Larry on 31st December 2015.
Fair value Carrying amount Tax base
Rs000 Rs000 Rs000
Buildings 600 400 300
Plant and equipment 56 46 25
Inventory 152 162 144
Trade receivables 120 120 120
Defined benefit liability (100) (150) –
Current liabilities (50) (50) (50)
778 528 539

© Emile Woolf International 58 The Institute of Chartered Accountants of Pakistan


Questions

Required
(a) Prepare a schedule of temporary differences and resultant deferred tax for Larry from the
point of view of the group.
(b) Combine the deferred tax figures to obtain the group deferred tax balance.
(c) Prepare a note showing the movement on the consolidated deferred tax balance for the
year ending 31st December 2015.
(d) Calculate the goodwill arising on acquisition of Larry.

18.4 COHORT
Cohort is a private limited company and has two 100% owned subsidiaries, Legion and Air, both
themselves private limited companies. Cohort acquired Air on 1 January 20X2 for Rs. 5 million
when the fair value of the net assets was Rs. 4 million, and the tax base of the net assets was
Rs. 3.5 million. The acquisition of Air and Legion was part of a business strategy whereby Cohort
would build up the value of the group over a three-year period and then list its share capital on
the Stock Exchange.
(a) The following details relate to the acquisition of Air, which manufactures electronic goods:
(i) Part of the purchase price has been allocated to intangible assets because it relates
to the acquisition of a database of key customers of Air. The recognition and
measurement criteria for an intangible asset under IFRS 3 Business Combinations
and IAS 38 Intangible Assets do not appear to have been met but the directors feel
that the intangible asset of Rs. 500,000 will be allowed for tax purposes and have
computed the tax provision accordingly. However, the tax authorities could possibly
challenge this opinion.
(ii) Air has sold goods worth Rs. 3 million to Cohort since acquisition and made a profit
of Rs. 1 million on the transaction. The inventory of these goods recorded in
Cohort’s statement of financial position at the year ending 31May 20X2 was Rs. 1.8
million.
(iii) The retained earnings of Air at acquisition were Rs. 2 million. The directors of
Cohort have decided that, during the three years leading up to the date that they
intend to list the shares of the company, they will realise earnings through future
dividend payments from the subsidiary amounting to Rs. 500,000 per year. Tax is
payable on any remittance of dividends and no dividends have been declared for the
current year.
(b) Legion was acquired on 1 June 20X1 and is a company which undertakes various projects
ranging from debt factoring to investing in property and commodities. The following details
relate to Legion for the year ending 31 May 20X2:
(i) Legion has a portfolio of readily marketable government securities which are held as
current assets. These investments are stated at market value in the statement of
financial position with any gain or loss taken to profit or loss. These gains and losses
are taxed when the investments are sold. Currently the accumulated unrealised
gains are Rs. 4 million.
(ii) Legion has calculated that it requires a general allowance of Rs. 2 million against its
total loan portfolio. Tax relief is available when the specific loan is written off.
Management feel that this part of the business will expand and thus the amount of
the general provision will increase.
(iii) When Cohort acquired Legion it had unused tax losses brought forward. At 1 June
20X1, it appeared that Legion would have sufficient taxable profit to realise the
deferred tax asset created by these losses but subsequent events have proven that
the future taxable profit will not be sufficient to realise all of the unused tax loss.
Impairment of goodwill is not allowed as a deduction in determining taxable profit.
Required
Write a note suitable for presentation to the partner of an accounting firm setting out the
deferred tax implications of the above information for the Cohort Group of companies.

© Emile Woolf International 59 The Institute of Chartered Accountants of Pakistan


Advanced accounting and financial reporting

18.5 MODEL TOWN GROUP


The following statement of financial position relates to Model Town Group, a public limited
company at 30 June 2016:
Rs.000
Assets:
Non-current assets:
Property, plant, and equipment 10,000
Goodwill 6,000
Other intangible assets 5,000
Financial assets (cost) 9,000
30,000
Trade receivables 7,000
Other receivables 4,600
Cash and cash equivalents 6,700
18,300
Total assets 48,300
Equity and liabilities
Share capital 9,000
Other reserves 4,500
Retained earnings 9,130
Total equity 22,630
Non-current liabilities
Long term borrowings 10,000
Deferred tax liability 3,600
Employee benefit liability 4,000
Total non-current liabilities 17,600
Current tax liability 3,070
Trade and other payables 5,000
Total current liabilities 8,070
Total liabilities 25,670
Total equity and liabilities 48,300

The following information is relevant to the above statement of financial position:

(i) The financial assets are investments in equity. Model Town has made an irrevocable
election to recognise gains and losses on these assets in other comprehensive income.
However, they are shown in the above statement of financial position at their cost on 1 July
2015. The market value of the assets is Rs. 10.5 million on 30 June 2016. Taxation is
payable on the sale of the assets.
(ii) The stated interest rate for the long term borrowing is 8 per cent. The loan of Rs. 10 million
represents a convertible bond which has a liability component of Rs. 9.6 million and an
equity component of Rs.0.4 million. The bond was issued on 30 June 2016.
(iii) The tax bases of the assets and liabilities are the same as their carrying amounts in the
statement of financial position at 30 June 2016 except for the following:
(a) Rs.000
Property, plant, and equipment 2,400
Trade receivables 7,500
Other receivables 5,000
Employee benefits 5,000
(b) Other intangible assets were development costs which were all allowed for tax
purposes when the cost was incurred in 2015.

© Emile Woolf International 60 The Institute of Chartered Accountants of Pakistan


Questions

(c) Trade and other payables include an accrual for compensation to be paid to
employees. This amounts to Rs. 1 million and is allowed for taxation when paid.

(iv) Goodwill is not allowable for tax purposes in this jurisdiction.


(v) Assume taxation is payable at 30%.

Required
Calculate the provision for deferred tax at 30 June 2016 after any necessary adjustments to the
financial statements showing how the provision for deferred taxation would be dealt with in the
financial statements.
(Assume that any adjustments do not affect current tax. You should briefly discuss the
adjustments required to calculate the provision for deferred tax).

© Emile Woolf International 61 The Institute of Chartered Accountants of Pakistan


Advanced accounting and financial reporting

CHAPTER 19: BUSINESS COMBINATIONS AND CONSOLIDATION

19.1 HELLO
On 1 January 2015, Hello acquired 60% of the ordinary share capital of Solong for Rs. 110,000.
At that date Solong had a retained earnings balance of Rs. 60,000.
The following statements of financial position have been prepared as at 31 December 2016.
Hello Solong
Rs. Rs.
Assets
Non-current assets
Property, plant and equipment 225,000 175,000
Investments in Solong 110,000

Current assets 271,000 157,000


———– ———–
606,000 332,000
———– ———–
Equity and liabilities
Capital and reserves
Share capital 100,000 100,000
Retained earnings 275,000 90,000
———– ———–
375,000 190,000
Current liabilities 231,000 142,000
———– ———–
606,000 332,000
———– ———–
The fair value of Solong’s net assets at the date of acquisition was determined to be Rs. 170,000.
The difference between the book value and the fair value of the new assets at the date of
acquisition was due to an item of plant which had a useful life of 10 years from the date of
acquisition.
Required
Prepare the consolidated statement of financial position of Hello and its subsidiary as at 31
December 2016.

19.2 HASAN LIMITED


On 1 April 2015, Hasan Limited acquired 90% of the equity shares in Shakeel Limited. On the
same day Hasan Limited accepted a 10% loan note from Shakeel Limited for Rs. 200,000 which
was repayable at Rs. 40,000 per annum (on 31 March each year) over the next five years.
Shakeel Limited’s retained earnings at the date of acquisition were Rs. 2,200,000.
Statements of financial position as at 31 March 2016
Hasan Limited Shakeel Limited
Rs.000 Rs.000 Rs.000 Rs.000
Non-current assets
Property, plant and equipment 2,120 1,990
Intangible – software – 1,800
Investments – equity in Shakeel Limited 4,110 –
Investments – 10% loan note Shakeel 200 –
Limited
Investments – others 65 210
6,495 4,000

© Emile Woolf International 62 The Institute of Chartered Accountants of Pakistan


Questions

Hasan Limited Shakeel Limited


Rs.000 Rs.000 Rs.000 Rs.000
Current assets
Inventories 719 560
Trade receivables 524 328
Shakeel Limited current account 75 –
Cash 20
1,338 888
Total assets 7,833 4,888

Equity and liabilities:


Capital and reserves
Equity shares of Rs. 1 each 2,000 1,500
Share premium 2,000 500
Retained earnings 2,900 1,955
6,900 3,955
Non-current liabilities
10% Loan note from Hasan Limited – 160
Government grant 230 40
230 200
Current liabilities
Trade payables 475 472
Hasan Limited current account – 60
Income taxes payable 228 174
Operating overdraft – 27
703 733
Total equity and liabilities 7,833 4,888

The following information is relevant:


(i) Included in Shakeel Limited’s property at the date of acquisition was a leasehold property
recorded at its depreciated historical cost of Rs. 400,000. The leasehold had been sub-let
for its remaining life of only four years at an annual rental of Rs. 80,000 payable in
advance on 1 April each year. The directors of Hasan Limited are of the opinion that the
fair value of this leasehold is best reflected by the present value of its future cash flows. An
appropriate cost of capital for the group is 10% per annum.
The present value of a Rs. 1 annuity received at the end of each year where interest rates
are 10% can be taken as:
3 year annuity Rs. 2.50
4 year annuity Rs. 3.20
(ii) The software of Shakeel Limited represents the depreciated cost of the development of an
integrated business accounting package. It was completed at a capitalised cost of Rs.
2,400,000 and went on sale on 1 April 2014. Shakeel Limited’s directors are depreciating
the software on a straight-line basis over an eight-year life (i.e. Rs. 300,000 per annum).
However, the directors of Hasan Limited are of the opinion that a five-year life would be
more appropriate as sales of business software rarely exceed this period.
(iii) The inventory of Hasan Limited on 31 March 2016 contains goods at a transfer price of Rs.
25,000 that were supplied by Shakeel Limited who had marked them up with a profit of
25% on cost. Unrealised profits are adjusted for against the profit of the company that
made them.

© Emile Woolf International 63 The Institute of Chartered Accountants of Pakistan


Advanced accounting and financial reporting

(iv) On 31 March 2016 Shakeel Limited remitted to Hasan Limited a cash payment of Rs.
55,000. This was not received by Hasan Limited until early April. It was made up of an
annual repayment of the 10% loan note of Rs. 40,000 (the interest had already been paid)
and Rs. 15,000 of the current account balance.
(v) The accounting policy of Hasan Limited for non-controlling interests (NCI) in a subsidiary is
to value NCI at a proportionate share of the net assets.
(v) An impairment test at 31 March 2016 on the consolidated goodwill concluded that it should
be written down by Rs. 120,000. No other assets were impaired.
Required
Prepare the consolidated statement of financial position of Hasan Limited as at
31 March 2016.

19.3 FLAMSTEED LTD AND HALLEY LTD


The draft Statement of Financial Position of Flamsteed Ltd and Halley Ltd on 30 June 2016 were
as follows:
Statement of financial position as at 30 June 2016
Flamsteed Ltd Halley Ltd
Rs.’000 Rs.’000
Assets:
Non-current Assets:
Property, plant and equipment 100,000 80,000
40,000 ordinary shares in Halley at cost 60,000 -
160,000 80,000
Current assets:
Inventory 6,000 16,000
Owed by Flamsteed Ltd - 20,000
Receivables 32,000 14,000
Cash 4,000 -
42,000 50,000
Total assets 202,000 130,000
Equity and liabilities:
Equity (ordinary shares @ Rs. 1) 90,000 50,000
Revaluation surplus 24,000 10,000
Retained earnings 52,000 56,000
166,000 116,000
Current Liabilities:
Owed to Halley Ltd 16,000 -
Trade payables 20,000 14,000
36,000 14,000
Total equity and liabilities 202,000 130,000
Additional information:
(i) Flamsteed Ltd acquired its investment in Halley Ltd on 1 July 2015, when the retained
earnings of Halley Ltd stood at Rs. 12,000,000.
(ii) The agreed consideration was Rs. 60,000,000 at the date of acquisition and a further Rs.
20,000,000 on 1 July 2017, Flamsteed Ltd’s cost of capital is 7%.
(iii) Halley Ltd has an internally developed brand name – TOLX – which was valued at Rs.
10,000,000 at the date of acquisition.
(iv) There have been no changes in the capital or revaluation surplus of Halley Ltd since the
date its shares were purchased.

© Emile Woolf International 64 The Institute of Chartered Accountants of Pakistan


Questions

(v) At 30 June 2016, Halley had invoiced Flamsteed Ltd for goods to the value of Rs.
4,000,000 and Flamsteed Ltd had sent payment in full but this had not been received by
Halley Ltd.
(vi) There is no impairment of goodwill.
(vii) It is the group’s policy to value non-controlling interest at full fair value.
(viii) At the acquisition date, the non-controlling interest was valued at Rs. 18,000,000.
Required
(a) Define Impairment loss in accordance with IAS 36 on Impairment of Assets.
(b) Explain any THREE sources of external information which an entity may consider in
assessing whether there is any indication that an asset may be impaired.
(c) Prepare an extract of consolidated Statement of Financial position of Flamsteed Ltd for the
year ended 30 June 2016, showing the assets side only.

19.4 BRADLEY LTD


Bradley Ltd’s purchased 960 million shares in Bliss Ltd a year ago when Bliss had a credit
balance of Rs. 190million in retained earnings. The fair value of the non-controlling interest at
the date of acquisition was Rs. 330million. At the date of acquisition, the freehold land of Bliss
Ltd was valued at Rs. 140million in excess of its carrying value. The revaluation has not been
recorded in the accounts of Bliss.
The statements of financial position of Bradley Ltd and Bliss Ltd as at 31 December 2016 are as
follows:
Bradley Ltd Bliss Ltd
Rs. Rs. Rs. Rs.
million million million million
Non Current Assets
Land and building 630 556
Machinery and equipment 570 440
Investment in Bliss Ltd. 1,320 -
2,520 996
Current Assets
Inventories 714 504
Trade receivables 1,050 252
Cash/bank 316 2,080 60 816
4,600 1,812

Ordinary Shares at Rs. 1 each 3,000 1,200


Retained Earnings 1,160 424
Shareholders fund 4,160 1,624

Current Liabilities
Trade payables 440 188
4,600 1,812
Bliss Ltd owes Bradley Ltd Rs. 50million for goods purchased during the year. Inventory of Bliss
Ltd includes goods bought from Bradley Ltd at the price that includes a profit to Bradley Ltd of
Rs. 24million.
The management of Bradley Ltd wants the financial statements to be consolidated using the
acquisition method and wishes to know whether there is goodwill on acquisition of Bliss Ltd and
the amount involved.
Required
Prepare the consolidated statement of financial position as at 31 December 2016.

© Emile Woolf International 65 The Institute of Chartered Accountants of Pakistan


Advanced accounting and financial reporting

19.5 X LTD
The statements of financial position for X Ltd and Y Ltd as at 31 December 2016 are provided
below:
X Ltd Y Ltd
ASSETS Rs.000 Rs.000
Non-current assets
Property, plant and equipment 12,000 4,000
Available for sale investment (note 1) 4,000 -
Current assets 16,000 4,000
Inventories 2,200 800
Receivables 3,400 900
Cash and cash equivalents 800 300
6,400 2,000
Total assets 22,400 6,000
EQUITY AND LIABILITIES Equity
Share capital (Rs. 1 equity shares) 10,000 1,000
Retained earnings 7,500 4,000
Other reserves 200 -
Total equity 17,700 5,000
Non-current liabilities
Long term borrowings 2,700 -
Current liabilities 2,000 1,000
Total liabilities 4,700 1,000
Total equity and liabilities 22,400 6,000
Additional information:
1. X Ltd acquired a 75% investment in Y Ltd on 1 May 2016 for Rs. 3,800,000. The
investment has been classified as available-for-sale in the books of X Ltd. The gain on its
subsequent measurement as at 31 December 2016 has been recorded within other
reserves in X Ltd’s individual financial statements. At the date of acquisition Y Ltd had
retained earnings of Rs. 3,200,000.
2. It is the group policy to value non-controlling interest at fair value at the date of acquisition.
The fair value of the non-controlling interest at 1 May 2016 was Rs. 1,600,000.
3. As at 1 May 2016 the fair value of the net assets acquired was the same as the book value
with the following exceptions:
The fair value of property, plant and equipment was Rs. 800,000 higher than the book
value. These assets were assessed to have an estimated useful life of 16 years from the
date of acquisition. A full year’s depreciation is charged in the year of acquisition and none
in the year of sale.
The fair value of inventories was estimated to be Rs. 200,000 higher than the book value.
All of these inventories were sold by 31 December 2016.
On acquisition X Ltd identified an intangible asset that Y Ltd developed internally but which
met the recognition criteria of IAS 38 Intangible Assets. This intangible asset is expected
to generate economic benefit from the date of acquisition until 31 December 2017 and was
valued at Rs. 150,000 at the date of acquisition.
A contingent liability, which had a fair value of Rs. 210,000 at the date of acquisition, had a
fair value of Rs. 84,000 at 31 December 2016.

© Emile Woolf International 66 The Institute of Chartered Accountants of Pakistan


Questions

4. An impairment review was conducted at 31 December 2016 and it was decided that the
goodwill on the acquisition of Y Ltd was impaired by 20%.
5. X Ltd sold goods to Y Ltd for Rs. 300,000. Half of these goods remained in inventories at
31 December 2016. X Ltd makes 20% margin on all sales.
6. No dividends were paid by either entity in the year ended 31 December 2016.
Required
Prepare the consolidated statement of financial position as at 31 December 2016 for the X Ltd
Group.

19.6 KHAN LIMITED


On January 1, 2010, Khan Limited (KL) acquired 375 million ordinary shares and 40 million
preference shares in Gul Limited (GL) whose general reserve and retained earnings on
the date of acquisition, stood at Rs. 200 million and Rs. 1,000 million respectively.
The following balances were extracted from the records of KL and its subsidiary on
December 31, 2016:
KL GL
Dr Cr Dr Cr
Rs. m Rs. m Rs. m Rs. m
Ordinary share capital (Rs. 10 each) - 6,800 5,000
12% Preference share capital (Rs. 10 each) - - - 1,000
General reserve - 1,750 - 500
Retained earnings - 2,000 - 1,200
Loan from KL at 15% rate of interest - - - 2,000
14% Term Finance Certificates (TFCs) (Rs. 100 - 2,250 - -
each)
Accounts payable - 445 - 190
Dividend payable – preference shares - - - 60
Dividend payable – ordinary shares - 750 - 300
Property, plant and equipment - at cost 16,250 - 25,000 -
Property, plant and equipment - acc. - 9,750 - 17,000
depreciation
Investment in ordinary shares of GL 5,500 - - -
Investment in preference shares of GL 400 - - -
Loan to GL at 15% rate of interest 2,000 - - -
Investment in KL's TFCs - - 1,500 -
(purchased at par value)
Profit before tax, interest and dividend - 2,865 - 1,550
Dividend income - 273 - -
Interest income - 300 - 210
Dividend receivable 249 - - -
Current assets 1,069 - 1,316 -
Interest on TFCs 315 - - -
Interest on loan from KL - - 300 -
Taxation 650 - 474 -
Preference dividend - - 120 -
Ordinary dividend – interim 750 - 300 -
27,183 27,183 29,010 29,010

© Emile Woolf International 67 The Institute of Chartered Accountants of Pakistan


Advanced accounting and financial reporting

Following relevant information is available:


(i) At the date of acquisition, the fair value of buildings, included in property, plant and
equipment of GL was assessed at Rs. 1,000 million above its carrying value. All other
identifiable assets and liabilities were considered to be fairly valued. GL provides for
depreciation on buildings at 10% per annum on the straight line basis.
(ii) GL purchased the TFCs in KL on January 1, 2016.
(iii) The non-controlling interests are measured at their proportionate share of the GL’s
identifiable net assets.
(iv) There is no impairment in the value of goodwill since its acquisition.
(v) There are no components of other comprehensive income.
Required
Prepare the following in accordance with the requirements of International Financial Reporting
Standards:
(a) Consolidated statement of financial position as at December 31, 2016.
(b) Consolidated statement of comprehensive income for the year ended December 31,
2016.
(c) Consolidated statement of retained earnings for the year ended December 31, 2016.
Note:
Ignore deferred tax and corresponding figures.
Notes to the above statements are not required. However, show workings wherever it is
necessary.

© Emile Woolf International 68 The Institute of Chartered Accountants of Pakistan


Questions

CHAPTER 20: CONSOLIDATED STATEMENTS OF PROFIT OR LOSS AND OTHER


COMPREHENSIVE INCOME

20.1 MILLARD LTD


The profit and loss account of Millard Ltd and its subsidiary Fillmore Limited for the year ended
31 December 2016 are as follows:
Millard Fillmore
Ltd Ltd
Rs.’000 Rs.’000
Revenue 312,500 125.000
Cost of Sales (125,000) (50,000)
Gross Profit 187,500 75,000
Distribution Cost (25,000) (10,000)
Administrative Expenses (20,000) (8,000)
Operating Profit 142,500 57,000
Investment Income 7,950 -
Debenture Interest (47,500) (15,000)
Profit on ordinary activities before taxation 102,950 42,000
Taxation on ordinary activities (35,000) (17,500)
Profit on ordinary activities after taxation 67,950 24,500
Dividends:
Preference (13,750) (4,375)
Ordinary (20,000) (5,250)
Retained Profits 34,200 14,875
Retained Profits: 1/1/2016 66,750 19,500
Retained Profits: 31/12/2016 100,950 34,375
Additional information:
(1) Included in the revenue of Fillmore Limited is Rs. 12.5million in respect of sales to Millard
Ltd, giving Fillmore Limited a profit of 25% on cost. These are sales of components that
Fillmore Limited has been supplying to Millard Ltd on a regular basis for a number of
years. The amounts included in the inventories of Millard Ltd in respect of goods
purchased from Fillmore Limited at the beginning and end of the year were as follows:
Inventories of components in Millard Ltd’s books
Date Rs.’000
31/12/2016 2,000
31/12/2015 1,500
(2) Some years ago, Millard Ltd bought 50 million ordinary shares in Fillmore Limited at a cost
of Rs. 67million. On the same date, Millard Ltd bought 25% of the debentures of Fillmore
Limited at par.

At the date of Millard Ltd’s investment in Fillmore Limited, the statement of financial
position of Fillmore limited showed:
Rs.’000
Ordinary share capital 62,500
Preference share capital 43,750
Profit and loss account 12,500
118,750

© Emile Woolf International 69 The Institute of Chartered Accountants of Pakistan


Advanced accounting and financial reporting

The goodwill acquired by Millard Ltd in Fillmore Limited had been written off fully in December
2016 as a result of impairment losses.
Required
Prepare the consolidated profit and loss account of Millard Ltd for the year. Assume that
investment income is dealt with by Millard Ltd on an accrual basis.

20.2 SHERLOCK LTD


The following draft financial statements relate to Sherlock Ltd and its subsidiaries.
Draft statements of profit or loss and other comprehensive income for the year ended 31 December
2016.
Sherlock Mycroft Katie
Ltd Ltd Ltd
Rs. m Rs. m Rs. m
Revenue 400 115 70
Cost of sales (312) (65) (36)
Gross profit 88 50 34
Other income 21 7 2
Administrative costs (15) (9) (12)
Other expenses (35) (19) (8)
Operating profit 59 29 16
Finance costs (5) (6) (4)
Finance income 6 5 8
Profit before tax 60 28 20
Income tax expense (19) (9) (5)
Profit for the year 41 19 15
Other comprehensive income – revaluation
surplus 10
Total comprehensive income for year 51 19 15

The following information is relevant to the preparation of the group statement of profit or loss and other
comprehensive income:
1. On 1 January 2015, Sherlock Ltd acquired 60% of the equity interests of Mycroft Ltd. The
purchase consideration comprised cash of Rs. 80 million.
The fair value of the identifiable net assets acquired was Rs. 110 million at that date. The
excess of the fair value of the identifiable net assets at acquisition is due to non-
depreciable land.
Sherlock Ltd measures the non-controlling interest at acquisition at its fair value. The fair
value of the non-controlling interest (NCI) in Mycroft Ltd was Rs. 45 million on 1 January
2015.
Goodwill has been impairment tested annually and as at 31 December 2016 had reduced
in value by 20%. At 31 December 2016, goodwill was estimated to have a value of Rs. 2
million above its original value.
2. Sherlock Ltd acquired 60% of Katie Ltd on 30 June 2016. There has been no impairment
of goodwill since the date of acquisition.
3. Sherlock Ltd sold inventory to Mycroft Ltd for Rs. 12 million making a loss of Rs. 2 million
on the transaction. The sale was at fair value and Mycroft Ltd still holds half of the
inventory at the year end.

© Emile Woolf International 70 The Institute of Chartered Accountants of Pakistan


Questions

4. The following information relates to Sherlock Ltd’s pension scheme:


Rs. m
Plan assets at 1 January 2016 48
Defined benefit obligation at 1 January 2016 50
Service cost for year ended 31 December 2016 4
Discount rate at 1 January 2016 10%
Re-measurement loss in year ended 31 December 2016 2
Past service cost 1 January 2016 3
The pension costs have not yet been accounted for.
5. On 1 January 2015, Sherlock Ltd purchased plant for Rs. 12 million and this is being
depreciated using the straight line basis over 10 years with a zero residual value.
Sherlock Ltd measures plant of this type using the revaluation model. At 31 December
2015, the asset was revalued to Rs. 13 million but at 31 December 2016, the value of the
asset had fallen to Rs. 7 million. The effect of the revaluation at 31 December 2016 had
not yet been accounted for but depreciation for the year has been charged.
6. On 1 January 2015, Sherlock Ltd made an award of 8,000 share Katie Ltds to each of its
seven directors. The condition attached to the award is that the directors must remain
employed by Sherlock Ltd for three years.
The fair value of each Katie Ltd at the grant date was Rs. 100 and the fair value of each
Katie Ltd at 31 December 2016 was Rs. 110. At 31 December 2015, it was estimated that
three directors would leave before the end of four years.
The estimate of directors who were going to leave was revised to one director at 31
December 2016.
The share Katie Ltd expense for the year has not been included in profit or loss for the
current year and no directors had left by 31 December 2016.
7. A loss on an effective cash flow hedge of Mycroft Ltd of Rs. 3 million has been included in
the subsidiary’s finance costs.
8. Any expense adjustments should be made in other expenses.
Required
Prepare a consolidated statement of profit or loss and other comprehensive income for the year
ended 31 December 2016 for the Sherlock Ltd Group. (Ignore the deferred tax consequences of
the above events)

20.3 FAISAL LIMITED


Following is the summarised trial balance of Faisal Limited (FL) and its subsidiaries, Saqib Limited
(SL) and Ayaz Industries Limited (AIL) for the year ended December 31, 2016:
FL SL AIL
Rs. in million

Cash and bank balances 4,920 660 2,700


Accounts receivable 6,240 2,460 6,580
Closing inventory 14,460 4,200 5,680
Investment in SL (at cost) 9,000 - -
Investment in AIL (at cost) 10,500 - -
Other investments 11,100 - -
Property, plant and equipment 22,500 3,480 5,940
Dividend paid 600
Cost of sales 49,200 18,000 21,000
Operating expenses 3,600 2,100 5,400
131,520 30,900 47,900

© Emile Woolf International 71 The Institute of Chartered Accountants of Pakistan


Advanced accounting and financial reporting

FL SL AIL
Rs. in million
Accumulated depreciation 5,760 420 1,260
Ordinary share capital (Rs. 10 each) 30,000 12,000 6,000
Retained earnings – opening 33,780 - 5,400
Sales 57,600 16,500 33,800
Accounts payable 2,760 1,980 1,440
Gain on sale of non-current assets 540 - -
Dividend income 1,080 - -
131,520 30,900 47,900
Following additional information is available:
(i) SL was incorporated on February 1, 2016. 75% of the shares were acquired by FL at par
value on the same date.
(ii) FL acquired 80% of AIL on January 1, 2016
(iii) The following inter-company sales were made during the year 2016:

Included in Amount
Sales buyer’s closing receivable/payable Gross profit %
inventories at year enzd on sales
Rs. in million
FL to AIL 2,400 900 - 20
SL to AIL 1,800 600 800 10
AIL to FL 3,600 1,200 - 30

(iv) The gain on sale of non-current assets includes a sale of an item of property, plant and
machinery by FL to SL. This transaction occurred on July 1, 2016. SL. Details of the
transaction are as follows:
Rs. in million
Sales value 144
Less: Cost of plant and machineries 150
Accumulated depreciation (60)
Carrying amount at date of sale 90
Gain on sale of plant 54
The plant and machinery was purchased originally by FL on July 1, 2014, and was being
depreciated on the straight line method over a period of five years. SL computed
depreciation thereon using the same method based on the remaining useful life as at the
date of the transfer.
(v) FL billed Rs. 100 million to each subsidiary for management services provided during
the year 2016 and credited it to operating expenses. The invoices were paid on December
15, 2016.
(iv) Details of cash dividend are as follows:
Dividend
Date of declaration Date of payment %
FL Nov 25, 2016 Jan 5, 2017 20
AIL Oct 15, 2016 Nov 20, 2016 10
Required
Prepare the consolidated statement of financial position and the consolidated statement
of profit and loss of FL and its subsidiaries for the year ended December 31, 2016. Ignore
tax and corresponding figures.

© Emile Woolf International 72 The Institute of Chartered Accountants of Pakistan


Questions

20.4 GOLDEN LIMITED


Golden Limited (GL) is a listed company and has held shares in two companies, Yellow
Limited (YL) and Black Limited (BL), since July 1, 2014. The details of acquisition of shares in
these companies are as follows:
(A) GL acquired 18 million shares in YL at par, when YL’s reserves were Rs. 24 million.
The acquisition was made by issuing four shares in GL for every five shares in YL. The
market price of GL’s shares at July 1, 2014 was Rs. 20 per share. A fair value exercise
was carried out for YL’s assets and liabilities at the time of its acquisition with the following
results:
Book Value Fair Value
Rupees in million
Land 170 192
Machines 25 45
Investments 3 6

The remaining life of machine on acquisition was 5 years. The fair values of the assets have
not been accounted for in YL’s financial statements.
(B) 6 million shares in BL were acquired for Rs. 12 per share in cash. At the date of
acquisition, the reserves of BL stood at Rs. 40 million.
The summarized statements of profit or loss of the three companies for the year ended
June 30, 2016 are as follows:
GL YL BL
Rupees in million
Sales 875 350 200
Cost of sales (567) (206) (244)
Gross profit / (loss) 308 144 (44)
Selling expenses (33) (11) (15)
Administrative expenses (63) (40) (16)
Interest expenses (30) (22) (15)
Other income 65 - -
Profit/(loss) before tax 247 71 (90)
Income tax (73) (15) 8
Profit/(loss) for the period 174 56 (82)

The following relevant information is available:


(i) The share capital and reserves as at July 1, 2015 were as follows:
GL YL BL
Rupees in million
Ordinary share capital of Rs. 10 each 600 200 150
Reserves 652 213 108
The share capitals of all companies have remained unchanged since their incorporation.
(ii) During the year, GL sold goods amounting to Rs. 40 million to YL. The sales were made
at a mark-up of 25% on cost. 30% of these goods were still in the inventories of YL at
June 30, 2016.
(iii) GL manufactures a component used by BL. During the year, GL sold these components
amounting to Rs. 20 million to BL. Transfers are made at cost plus 15%. BL held Rs.
11.5 million of these components in inventories at June 30, 2016.

© Emile Woolf International 73 The Institute of Chartered Accountants of Pakistan


Advanced accounting and financial reporting

(iv) All assets are depreciated on straight line method.


(v) Other income includes dividend received from YL on April 15, 2016.
(vi) During the year, YL paid 20% cash dividend to its ordinary shareholders.
(vii) An impairment test was carried out on June 30, 2016 for the goodwill of YL and
investments in BL, appearing in the consolidated financial statements. The test indicated
that:
- goodwill of YL was impaired by 20%;
- due to recent losses, the fair value of investment in BL has been reduced to
Rs. 40 million.
No such impairment was required in previous years.
Required
Prepare, in a format suitable for inclusion in the annual report, a consolidated statement of
profit or loss for the year ended June 30, 2016.

© Emile Woolf International 74 The Institute of Chartered Accountants of Pakistan


Questions

CHAPTER 21: ASSOCIATES AND JOINT VENTURES

21.1 JOINT ARRANGEMENTS


(a) State and explain the TWO types of joint arrangement identified in IFRS 11
(b) A joint operator is expected to recognise and account for certain elements in relation to the
joint operations. State FIVE elements to be recognised.
(c) State TWO characteristics of a joint arrangement.

21.2 HELIUM
The draft statements of financial position as at 31 December 2016 of three companies are set out
below.
Helium Sulphur Arsenic
Rs.000 Rs.000 Rs.000
Assets
Non-current assets
Property, plant and equipment 400 100 160
Investments:
- shares in Sulphur (60%) 75 – –
- shares in Arsenic (30%) 30 – –

Current assets 445 160 80


—— —— ——
950 260 240
—— —— ——
Equity and liabilities
Share capital 100 30 60
Retained earnings 650 180 100
Non-current loans 200 50 80
—— —— ——
950 260 240
—— —— ——
The reserves of Sulphur and Arsenic when the investments were acquired were Rs. 70,000 and
Rs. 30,000 respectively
Required
Prepare the consolidated statement of financial position as at 31 December 2016.

21.3 HAMACHI LTD


Hamachi Ltd acquired 90% of Saba Ltd’s Rs. 1 ordinary shares on 1 April 2014 paying Rs. 3.00
per share. The balance on Saba Ltd’s retained earnings at this date was Rs. 800,000. On 1
October 2015, Hamachi Ltd acquired 30% of Anogo Ltd’s Rs. 1 ordinary shares for Rs. 3.50 per
share. The statements of financial position of the three companies at 31 March 2016 are shown
below:
Hamachi Ltd Saba Ltd Anogo Ltd
Rs.000 Rs.000 Rs.000 Rs.000 Rs.000 Rs.000
Non-current assets
Property, plant and 8,050 3,600 1,650
equipment
Investments 4,000 910 nil
12,050 4,510 1,650
Current assets
Inventory 830 340 250

© Emile Woolf International 75 The Institute of Chartered Accountants of Pakistan


Advanced accounting and financial reporting

Hamachi Ltd Saba Ltd Anogo Ltd


Rs.000 Rs.000 Rs.000 Rs.000 Rs.000 Rs.000
Accounts receivable 520 290 350
Bank 240 nil 100
1,590 630 700
Total assets 13,640 5,140 2,350

Equity and liabilities


Equity:
Ordinary shares of Rs. 1 5,000 1,200 600
each
Reserves:
Retained earnings b/f 6,000 1,400 800
Profit year to 31 March 2016 1,500 900 600
7,500 2,300 1,400
12,500 3,500 2,000
Non-current liabilities
10% Loan notes 500 240 nil
Current liabilities
Accounts payable 420 960 200
Taxation 220 250 150
Overdraft nil 190 nil
640 1,400 350
Total equity and liabilities 13,640 5,140 2,350
The following information is relevant
(i) Fair value adjustments
On 1 April 2014 Saba Ltd owned an investment property that had a fair value of Rs.
120,000 in excess of its carrying value (book value). The value of this property has not
changed since acquisition. This property is included within investments in the balance
sheet.
Just prior to its acquisition, Saba Ltd was successful in applying for a six-year licence to
dispose of hazardous waste. The licence was granted by the government at no cost,
however Hamachi Ltd estimated that the licence was worth Rs. 180,000 at the date of
acquisition.
(ii) In January 2016 Hamachi Ltd sold goods to Anogo Ltd for Rs. 65,000. These were
transferred at a mark-up of 30% on cost. Two thirds of these goods were still in the
inventory of Anogo Ltd at 31 March 2016.
(iii) To facilitate the consolidation procedures the group insists that all inter company current
account balances are settled prior to the year-end. However a cheque for Rs. 40,000 from
Saba Ltd to Hamachi Ltd was not received until early April 2016. Inter company balances
are included in accounts receivable and payable as appropriate.
(iv) Anogo Ltd is to be treated as an associated company of Hamachi Ltd.
(v) An impairment test at 31 March 2016 on the consolidated goodwill of Saba Ltd concluded
that it should be written down by Rs. 468,000. No other assets were impaired.
Required
(a) Prepare the consolidated statement of financial position of Hamachi Ltd as at 31 March
2016.
(b) Discuss the matters to consider in determining whether an investment in another company
constitutes associated company status.

© Emile Woolf International 76 The Institute of Chartered Accountants of Pakistan


Questions

21.4 HIDE
Hide holds 80% of the ordinary share capital of Seek (acquired on 1 February 2016) and 30% of
the ordinary share capital of Arrive (acquired on 1 July 2015).
Hide had no other investments.
The draft statements of profit or loss for the year ended 30 June 2016, are set out below.
Hide Seek Arrive
Rs.000 Rs.000 Rs.000
Revenue 12,614 6,160 8,640
Operating expenses (11,318) (5,524) (7,614)
Dividends receivable 150 – –
——— ——– ——–
1,446 636 1,026
Income tax (621) (275) (432)
——— ——– ——–
Profit after taxation 825 361 594
——— ——– ——–
Included in the inventory of Seek at 30 June 2016 was Rs. 50,000 for goods purchased from
Hide in May 2016 which the latter company had invoiced at cost plus 25%. These were the only
goods sold by Hide to Seek but it did make sales of Rs. 180,000 to Arrive during the year. None
of these goods remained in Arrive’s inventory at the year end.
Required
Prepare a consolidated statement of profit or loss for Hide for the year ended 30 June 2016.

21.5 HARK, SPARK AND ARK


Hark acquired the following non-current investments on 1 April 2015:
(1) 4 million equity shares in Spark, by means of an exchange of one share in Handel for
every one share in Spark, plus Rs. 6.05 million in cash. The professional fees associated
with the acquisition amounted to Rs. 1 million. The market price of shares in Hark at the
date of the acquisition was Rs. 9 per share. The market price of Spark shares just before
the acquisition was Rs. 7. The cash part of the consideration is deferred and will not be
paid until two years after the acquisition.
(2) 25% of the equity shares in Ark, at a cost of Rs. 6 per share. The money to make this
payment was obtained by issuing one million new shares in Hark at Rs. 9 per share.
None of these transactions has yet been recorded in the summary statements of financial
position that are shown below.
The summarised draft statements of financial position of the three companies at 31 March 2016
are as follows.
Statement of financial position Hark Spark Ark
Rs. Rs. Rs.
million million million
Assets
Non-current assets
Property, plant and equipment 60.0 31.0 16.0
Other equity investments 0.8 nil nil
60.8 31.0 16.0
Current assets 18.2 8.0 9.0
Total assets 79.0 39.0 25.0
Equity and liabilities
Equity shares of Rs. 1 each 16.0 5.0 6.0
Share premium 2.0 4.0 4.0
Retained earnings: at 1 April 2015 36.0 16.0 8.0
- for year ended 31 March 2016 8.0 3.0 2.0
62.0 28.0 20.0

© Emile Woolf International 77 The Institute of Chartered Accountants of Pakistan


Advanced accounting and financial reporting

Statement of financial position Hark Spark Ark


Rs. Rs. Rs.
million million million
Non-current liabilities
6% loan notes 10.0 - -
7% loan notes - 6.0 3.0
Current liabilities 7.0 5.0 2.0
Total equity and liabilities 79.0 39.0 25.0
The following information is relevant:
(1) Hark has chosen to value the non-controlling interest in Spark using the fair value method
permitted by IFRS 3 (revised). The fair value of the non-controlling interests at the
acquisition date is estimated to be the market value of the shares before the acquisition.
(2) At the date of acquisition of Spark, the fair values of its assets were equal to their carrying
amounts.
(3) The cost of capital of Hark is 10% per year.
(4) During the year ended 31 March 2016, Spark sold goods to Hark for Rs. 3.6 million, at a
mark-up of 50% on cost. Hark had 75% of these goods in its inventory at 31 March 2016.
(5) There were no intra-group receivables and payables at 31 March 2016.
(6) On 1 April 2015, Hark sold a group of machines to Spark at their agreed fair value of Rs. 3
million. At the time of the sale, the carrying amount of the machines was Rs. 2 million. The
estimated remaining useful life of the plant at the date of the sale was four years. Plant and
machinery is depreciated to a residual value of nil using straight-line depreciation and at 1
April 2015 the machines had an estimated remaining life of five years.
(7) “Other equity investments” are included in the summary statement of financial position of
Hark at their fair value on 1April 2015. Their fair value at 31 March 2016 is Rs.0.65 million.
(8) Impairment tests were carried out on 31 March 2016. These show that there is no
impairment of the value of the investment in Ark or in the consolidated goodwill.
(9) No dividends were paid during the year by any of the three companies.
Required
Prepare the consolidated statement of financial position for Hark as at 31 March 2016.

21.6 P, S AND A
The statements of financial position of three entities P, S and A are shown below, as at 31
December Year 5. However, the statement of financial position of P records its investment in
Entity A incorrectly.
P S A
Rs. Rs. Rs.
Non-current assets
Property, plant and equipment 450,000 240,000 460,000
Investment in S at cost 320,000 - -
Investment in A at cost 140,000 - -
––––––––– ––––––––– –––––––––
910,000 240,000 460,000
Current assets
Inventory 70,000 90,000 70,000
Current account with P - 60,000 -
Current account with A 20,000 - -
Other current assets 110,000 130,000 40,000
––––––––– ––––––––– –––––––––
Total assets 1,110,000 520,000 570,000
––––––––– ––––––––– –––––––––

© Emile Woolf International 78 The Institute of Chartered Accountants of Pakistan


Questions

P S A
Rs. Rs. Rs.
Equity and reserves
Equity shares of Rs. 1 100,000 200,000 100,000
Share premium 160,000 80,000 120,000
Accumulated profits 650,000 140,000 250,000
––––––––– ––––––––– –––––––––
910,000 420,000 470,000
Long-term liabilities 40,000 20,000 30,000
Current liabilities
Current account with P - - 20,000
Current account with S 60,000 - -
Other current liabilities 100,000 80,000 50,000
––––––––– ––––––––– –––––––––
1,110,000 520,000 570,000
––––––––– ––––––––– –––––––––
Additional information
P bought 150,000 shares in S several years ago when the fair value of the net assets of S was
Rs. 340,000.
P bought 30,000 shares in A several years ago when A’s accumulated profits were Rs. 150,000.
There has been no change in the issued share capital or share premium of either S or A since P
acquired its shares in them.
There has been impairment of Rs. 20,000 in the goodwill relating to the investment in S, but no
impairment in the value of the investment in A.
At 31 December Year 5, A holds inventory purchased during the year from P which is valued at
Rs. 16,000 and P holds inventory purchased from S which is valued at Rs. 40,000. Sales from P
to A and from S to P are priced at a mark-up of one-third on cost.
None of the entities has paid a dividend during the year.
P uses the partial goodwill method to account for goodwill and no goodwill is attributed to the
non-controlling interests in S.
Required
Prepare the consolidated statement of financial position of the P group as at 31 December Year
5.

21.7 H LTD GROUP


The statements of comprehensive income for H Ltd, S Ltd and A Ltd for the year ended 31 May
2016 are shown below:
H Ltd S Ltd A Ltd
Rs.000 Rs.000 Rs.000
Revenue 6,000 3,000 1,000
Cost of sales (4,800) (2,400) (800)
Gross profit 1,200 600 200
Distribution costs (64) (32) (10)
Administrative expenses (336) (168) (52)
Finance costs (30) (15) (5)
Profit before tax 770 385 133
Income tax expense (204) (102) (33)
PROFIT FOR THE YEAR 566 283 100

© Emile Woolf International 79 The Institute of Chartered Accountants of Pakistan


Advanced accounting and financial reporting

H Ltd S Ltd A Ltd


Rs.000 Rs.000 Rs.000
Other comprehensive income:
Revaluation of property 200 100 30
Tax effect of revaluation (42) (21) (6)
Other comprehensive income for the
year, net of tax 158 79 24
TOTAL COMPREHENSIVE INCOME FOR
THE YEAR 724 362 124
Additional information:
1. H Ltd operates a defined benefit pension plan for its employees. At the year end, there is
an actuarial loss of Rs. 52,000 on the pension plan liabilities and an actuarial gain of Rs.
40,000 on pension plan assets. These amounts are not reflected in the above statements.
In accordance with the amendment to IAS 19 Employee Benefits, H Ltd recognises
actuarial gains and losses from the defined benefit plan in other comprehensive income in
the period that they occur.
2. H Ltd holds a 15% investment in XY which is designated as available for sale. The fair
value of this investment at 31 May 2016 was Rs. 106,000. The investment is currently
recorded in the financial statements at Rs. 92,000.
3. H Ltd owns 80% of the ordinary share capital of S Ltd and exercises control over its
operating and financial policies. H Ltd owns 30% of the ordinary share capital of A Ltd and
exerts significant influence over its operating and financial policies.
Required
Prepare the consolidated statement of profit or loss and other comprehensive income for the H
Ltd Group, taking account of the information provided in the notes above. Ignore any further
taxation effects of notes 1 and 2.

© Emile Woolf International 80 The Institute of Chartered Accountants of Pakistan


Questions

CHAPTER 22: BUSINESS COMBINATIONS ACHIEVED IN STAGES

22.1 STEP ACQUISITION


On 1 January Year 1, H purchased 25% of the equity of AS for Rs. 80 million. H then acquired an
additional 40% of the equity of AS for Rs. 160 million on 30 June Year 1. At this date it was
estimated that the fair value of the original 25% shareholding in S was Rs. 95 million.
During the year S did not issue any new shares or make any distribution to its shareholders.
The carrying value of the net assets of AS were as follows:
Rs. million
At 1 January Year 1 260
At 30 June Year 1 300
H measures non-controlling interest at acquisition at fair value. This was estimated to be Rs.
120m.
The financial year of H ends on 30 June.
Required
For the consolidated financial statements of H for the year to 30 June Year 1, state:
(i) the total gain or profit attributable to the investment in AS for the year
(ii) total amount of goodwill arising with the acquisition
(iii) the amount of goodwill attributable to the NCI.

22.2 A LTD
The statements of financial position for A Ltd and B Ltd as at 30 September 2016 are provided
below:
A Ltd B Ltd
Rs.000 Rs.000
ASSETS
Non-current assets
Property, plant and equipment 22,000 5,000
Available for sale investment (note 1) 4,000 -
Current assets 26,000 5,000
Inventories 6,200 800
Receivables 6,600 1,900
Cash and cash equivalents 1,200 300
14,000 3,000
Total assets 40,000 8,000
EQUITY AND LIABILITIES Equity
Share capital (Rs. 1 equity shares) 20,000 1,000
Retained earnings 7,500 5,000
Other components of equity 500
Total equity 28,000 6,000
Non-current liabilities
5% Bonds 2019 (note 2) 3,900 -
Current liabilities 8,100 2,000
Total liabilities 12,000 2,000
Total equity and liabilities 40,000 8,000

© Emile Woolf International 81 The Institute of Chartered Accountants of Pakistan


Advanced accounting and financial reporting

Additional information:
1. A Ltd acquired a 15% investment in B Ltd on 1 May 2010 for Rs. 600,000. The investment
was classified as available for sale and the gains earned on it have been recorded within
other reserves in A Ltd’s individual financial statements. The fair value of the 15%
investment at 1 April 2016 was Rs. 800,000.
On 1 April 2016, A Ltd acquired an additional 60% of the equity share capital of B Ltd at a
cost of Rs. 2,900,000. In its own financial statements, A Ltd has kept its investment in B
Ltd as an available for sale asset recorded at its fair value of Rs. 4,000,000 as at 30
September 2016.
2. A Ltd issued 4 million Rs. 1 5% redeemable bonds on 1 October 2011 at par. The
associated costs of issue were Rs. 100,000 and the net proceeds of Rs. 3.9 million have
been recorded within non-current liabilities. The bonds are redeemable at Rs. 4.5 million
on 30 September 2019 and the effective interest rate associated with them is
approximately 8.5%. The interest on the bonds is payable annually in arrears and the
amount due has been paid in the year to 30 September 2016 and charged to the
statement of profit or loss.
3. An impairment review was conducted at the year end and it was decided that the goodwill
on the acquisition of B Ltd was impaired by 10%.
4. It is the group policy to value non-controlling interest at fair value at the date of acquisition.
The fair value of the non-controlling interest at 1 April 2016 was Rs. 1.25 million.
5. The profit for the year of B Ltd was Rs. 3 million, and profits are assumed to accrue evenly
throughout the year.
6. B Ltd sold goods to A Ltd for Rs. 400,000. Half of these goods remained in inventories at
30 September 2016. B Ltd makes 20% margin on all sales.
7. No dividends were paid by either entity in the year to 30 September 2016.
Required
(a) Explain how the investment in B Ltd should be accounted for in the consolidated financial
statements of A Ltd, following the acquisition of the additional 60% shareholding.
(b) Prepare the consolidated statement of financial position as at 30 September 2016 for the A
Ltd Group.

22.3 X LTD GROUP


Extracts from the financial statements of X Ltd, Y Ltd and Z Ltd are presented below.
Statements of comprehensive income for the year X Ltd Y Ltd Z Ltd
ended 31 December 2016
Rs.000 Rs.000 Rs.000

Revenue 1,200 290 150


Cost of sales (810) (110) (80)
Gross profit 390 180 70
Operating expenses (100) (40) (20)
290 140 50
Investment income 50 -
Finance costs (45) (10) (5)
Profit before tax 295 130 45
Income tax expense (80) (30) (15)
Profit for the year 215 100 30
Other comprehensive income:
Revaluation of property, net of tax 60 20 10
Other comprehensive income for the year,
net of tax 60 20 10
Total comprehensive income 275 120 40

© Emile Woolf International 82 The Institute of Chartered Accountants of Pakistan


Questions

Statements of changes in equity for the year X Ltd Y Ltd Z Ltd


ended 31 December 2016
Rs.000 Rs.000 Rs.000

Equity at 1 January 2016 1,700 840 500


Total comprehensive income for the year 275 120 40
Dividends (100) (50) -
Equity at 31 December 2016 1,875 910 540
Additional information
1 X Ltd acquired 80% of the ordinary share capital of Y Ltd for Rs. 620,000 on 1 January
2010 when the retained reserves of Y Ltd were Rs. 420,000. Y Ltd has 200,000 Rs. 1
ordinary shares in issue and there have been no share issues since the acquisition date.
The group policy is to measure the non- controlling interest at fair value at the date of
acquisition. The fair value of the non-controlling interest at 1 January 2010 was Rs.
180,000.
2 On 1 January 2010, the fair value of Y Ltd’s net assets was the same as their book value
with the exception of depreciable property, the fair value of which was Rs. 60,000 higher
than its book value. The property had a remaining useful life of 15 years at the date of
acquisition. Depreciation on property is charged to cost of sales.
3 Goodwill on the acquisition of Y Ltd was impaired for the first time by 25% in the year to 31
December 2015. An impairment review conducted at 31 December 2016 indicated a
further impairment of 10% of the remaining carrying value of goodwill. Impairment losses
on goodwill are charged to group operating expenses.
4 X Ltd acquired 40% of the ordinary share capital of Z Ltd on 1 July 2011, when the equity
was Rs. 435,000.
Required
(a) Prepare for the X Ltd Group for the year ended 31 December 2016:
(i) a consolidated statement of profit or loss and other comprehensive income; and
(ii) a consolidated statement of changes in equity.
X Ltd purchased a further 10% of the ordinary share capital of Y Ltd on 1 January 2017 for Rs.
120,000.
Required
(b) (i) Explain how the acquisition of this additional investment will be accounted for in the
consolidated financial statements of the X Ltd group for the year to 31 December
2017.
(ii) Calculate the debit or credit that will be made to the consolidated retained reserves
of the X Ltd group for the year to 31 December 2017 in respect of this additional
10% share purchase.
X Ltd purchased a further 20% of the ordinary share capital of Z Ltd on 1 January 2017.
Required
(c) Explain how the acquisition of the additional investment in Z Ltd will be accounted for in
the consolidated financial statements of the X Ltd group for the year to 31 December 2017.

© Emile Woolf International 83 The Institute of Chartered Accountants of Pakistan


Advanced accounting and financial reporting

22.4 PLAIN LTD


The following statements of financial position are as at 31 March 2016:
Plain Stripes Spots
Rs. m Rs. m Rs. m
Assets
Tangible non-current assets 1,280 440 280
Investment in Stripes 413
Investment in Spots 60
Held to maturity investment 54
Current assets 477 190 130
Total assets 2,284 630 410
Equity and liabilities
Share capital of Rs. 1 800 240 200
Share premium 150 20 30
Revaluation reserve 90
Retained earnings 390 210 94
Total equity 1,430 470 324
Non-current liabilities 640 30 16
Current liabilities 214 130 70
Total equity and liabilities 2,284 630 410

Plain acquired the following shareholdings in Stripes and Spots.


Holding Fair value of Purchase
Date of acquisition
acquired net assets consideration
Rs. m Rs. m

Stripes 1 April 2013 30% 325 120


1 April 2015 50% 460 260

Spots 1 April 2015 25% 200 60


You are also provided with the following information which will be relevant to the consolidated
financial statements of Plain.
(1) None of the companies have issued any additional share capital since 1 April 2013.
(2) The financial statements of Plain have not yet been adjusted for the gain or loss arising on
gaining control of Stripes.
(3) At 1 April 2013, the carrying value of the net assets of Stripes was the same as their fair
value, Rs. 325 million.
(4) Plain wishes to use the full fair value method of accounting for the acquisition of Stripes,
and at 1 April 2015 the estimated value of goodwill attributable to non-controlling interests
was Rs. 3 million. The estimated fair value of the initial investment in 30% of the shares of
Stripes was Rs. 150 million at 31 March 2015.
(5) Included in the tangible non-current assets of Stripes is land valued at cost which on 1
April 2015 had a fair value of Rs. 25 million in excess of its carrying value. There has been
no subsequent significant change in that value.
(6) At 1 April 2014 the fair value of Spots’s land was Rs. 16 million in excess of its carrying
value. There has been no subsequent significant change in that value.
(7) Goodwill arising on acquisition is tested for impairment at each year end. At 31 March
2016 an impairment loss of Rs. 15 million was recognised for Stripes .

© Emile Woolf International 84 The Institute of Chartered Accountants of Pakistan


Questions

(8) There has been no impairment of the investment in Spots.


(9) During the year the directors of Plain decided to form a defined benefit pension scheme for
its employees. The company contributed cash to it of Rs. 250 million but the only
accounting entry for this has been to include it in receivables at 31 March 2016.
At 31 March 2016 the following details relate to the pension scheme:
Rs. m
Present value of obligation 317
Fair value of plan assets 302
Current service cost 276
Interest cost on pension scheme liabilities 41
Expected return on pension scheme assets 26
In the consolidated financial statements the directors wish to recognise any actuarial gain
or loss immediately.
(10) The ‘held to maturity’ investment in Plain’s financial statements is a zero coupon loan to an
unrelated third party. No interest has yet been recognised on this amount. The debt is
repayable in five years at Rs. 74 million. (Recognise interest on a straight line basis).
Required
Prepare the consolidated statement of financial position of the Plain group as at 31 March 2016.

22.5 MANGO LTD


The draft statements of financial position of Mango Ltd and its subsidiary at 30 November 2016
are as follows:

Mango Ltd Plum Ltd


Rs. m Rs. m
Assets:
Non-current assets
Property, plant and equipment 3,295 2,000
Investments in subsidiary 1,675
4,970 2,000
Current assets 1,685 861
Total assets 6,655 2,861
Equity and liabilities:
Share capital 850 1,020
Retained earnings 3,340 980
Other components of equity 250 80
Total equity 4,440 2,080
Non-current liabilities 1,895 675
Current liabilities 320 106
Total liabilities 2,215 781
Total equity and liabilities 6,655 2,861

The following information is relevant to the preparation of the group financial statements:
1. On 1 December 2013, Mango Ltd acquired 30% of the ordinary shares of Plum Ltd for a
cash consideration of Rs. 600 million. The fair value of Plum Ltd’s identifiable net assets
was Rs. 1,840 million at this date. The 30% holding gave Mango Ltd significant influence
over Plum Ltd and Mango Ltd accounted for the investment as an associate up to 1
December 2015. Mango Ltd’s share of Plum Ltd’s undistributed profit amounted to Rs. 90
million and its share of a revaluation gain amounted to Rs. 10 million.

© Emile Woolf International 85 The Institute of Chartered Accountants of Pakistan


Advanced accounting and financial reporting

On 1 December 2015, Mango Ltd acquired a further 40% of the ordinary shares of Plum
Ltd for a cash consideration of Rs. 975 million and gained control of the company. The
cash consideration has been added to the equity accounted balance for Plum Ltd at 1
December 2015 to give the carrying amount at 30 November 2016.
At 1 December 2015, the fair value of the equity interest in Plum Ltd held by Mango Ltd
before the business combination was Rs. 705 million.
At 1 December 2015, the fair value of Plum Ltd’s identifiable net assets was Rs. 2,250
million.
The retained earnings and other components of equity of Plum Ltd at 1 December 2015
were Rs. 900 million and Rs. 70 million respectively. It is group policy to measure the non-
controlling interest at fair value. The fair value of the non-controlling interest of 30% was
assessed as Rs. 620 million
2. At the time of the business combination with Plum Ltd, Mango Ltd has included in the fair
value of Plum Ltd’s identifiable net assets, an unrecognised contingent liability of Rs. 6
million in respect of a warranty claim in progress against Plum Ltd. In March 2016, there
was a revision of the estimate of the liability to Rs. 5 million. The amount has met the
criteria to be recognised as a provision in current liabilities in the financial statements of
Plum Ltd and the revision of the estimate is deemed to be a measurement period
adjustment.
3. The fair value of Plum Ltd’s identifiable net assets (Rs. 2,250 million) included an amount
of Rs. 200 million being the estimate of the fair value of buildings with the remainder
relating to non-depreciable land.
Mango Ltd had commissioned an independent valuation of the buildings of Plum Ltd which
was not complete at 1 December 2015 and therefore not considered in the fair value of the
identifiable net assets at the acquisition date. The valuations were received on 1 April 2016
and resulted in a decrease of Rs. 40 million in the fair value of property, plant and
equipment at the date of acquisition. The buildings have a remaining useful life of 20 years
at 1 December 2015. Buildings are depreciated on the straight-line basis and it is group
policy to leave revaluation gains on disposal in equity.
The decrease in the fair value of the buildings does not affect the fair value of the non-
controlling interest at acquisition and has not been entered into the financial statements of
Plum Ltd.
All goodwill arising on acquisitions has been impairment tested with no impairment being
required.
Required
Prepare the group consolidated statement of financial position of Mango Ltd as at 30
November 2016.

© Emile Woolf International 86 The Institute of Chartered Accountants of Pakistan


Questions

CHAPTER 23: COMPLEX GROUPS

23.1 PARVEZ LTD

Statements of profit or loss for Parvez Ltd, Saad Ltd and Vazir Ltd for the year ended 31
December 2016 are as follows:

Parvez Ltd Saad Ltd Vazir Ltd


Rs. 000 Rs. 000 Rs. 000
Revenues 45,600 24,700 22,800
Cost of sales (18,050) (5,463) (5,320)
Gross profit 27,550 19,237 17,480
Distribution costs (3,325) (2,137) (1,900)
Administrative expenses (3,475) (950) (1,900)
Operating profit 20,750 16,150 13,680
Interest paid (325)
Profit before tax 20,425 16,150 13,680
Tax (8,300) (5,390) (4,241)
Profit after tax 12,125 10,760 9,439

Statements of financial position as at 31 December 2016 are as follows:

Parvez Ltd Saad Ltd Vazir Ltd


Rs. 000 Rs. 000 Rs. 000
Property, plant and equipment 35,483 24,273 13,063
Investments
Shares in Saad Ltd 6,650
Shares in Vazir Ltd 3,800
Current assets 1,568 9,025 8,883
43,701 37,098 21,946
Equity and liabilities
Ordinary Rs. 1 shares 8,000 3,000 2,000
Retained earnings 22,638 24,075 19,898
30,638 27,075 21,898
Current liabilities 13,063 10,023 48
43,701 37,098 21,946

The following information is available relating to Parvez Ltd, Saad Ltd and Vazir Ltd:
(1) On 1 January 2010 Parvez Ltd acquired 2,700,000 Rs. 1 ordinary shares in Saad Ltd for
Rs. 6,650,000 at which date there was a credit balance of retained earnings of Saad Ltd of
Rs. 1,425,000. No shares have been issued by Saad Ltd since Parvez Ltd acquired its
interest.
(2) On 1 January 2010 Saad Ltd acquired 1,600,000 Rs. 1 ordinary shares in Vazir Ltd for Rs.
3,800,000 at which date there was a credit balance of retained earnings of Vazir Ltd of Rs.
950,000. No shares have been issued by Vazir Ltd since Saad Ltd acquired its interest.

© Emile Woolf International 87 The Institute of Chartered Accountants of Pakistan


Advanced accounting and financial reporting

(3) During 2016, Vazir Ltd had made inter-company sales to Saad Ltd of Rs. 480,000 making
a profit of 25% on cost and Rs. 75,000 of these goods were in inventory at 31 December
2016.
(4) During 2016, Saad Ltd had made inter-company sales to Parvez Ltd of Rs. 260,000
1
making a profit of 33 3 % on cost and Rs. 60,000 of these goods were in inventory at 31
December 2016.
(5) On 1 November 2016 Parvez Ltd sold warehouse equipment to Saad Ltd for Rs. 240,000
from inventory. Saad Ltd has included this equipment in its non-current assets. The
equipment had been purchased on credit by Parvez Ltd for Rs. 200,000 in October 2016
and this amount is included in its current liabilities as at 31 December 2016.
(6) Saad Ltd charges depreciation on its warehouse equipment at 20% on cost. It is company
policy to charge a full year’s depreciation in the year of acquisition to be included in the
cost of sales.
Required
(a) Prepare a consolidated statement of profit or loss for the Parvez Ltd Group for the year
ended 31 December 2016.
(b) Prepare statement of financial position as at that date.

23.2 HASAN, RIAZ AND SIDDIQ

The summarised balances extracted from the accounting records of Hasan (H) Ltd, Riaz (R) Ltd
and Siddiq (S) Ltd at 31 December 2016 are given below:

H Ltd R Ltd S Ltd

Rs. Rs. Rs.

Property, plant and equipment 1,102,500 271,950 122,550

Investments at cost

75% holding in shares of Riaz Ltd 367,500

40% holding in shares of Siddiq Ltd 49,000

20% holding in shares of Siddiq Ltd 24,500

Inventories 526,610 163,290 85,700

Receivables 241,920 129,680 29,750

Cash and bank balances 88,200 4,725 8,105

2,375,730 594,145 246,105

Share capital 1,750,000 420,000 175,000

Other reserves 350,000 70,000

Accumulated profits/(losses) 180,250 17,500 (17,500)

Payables 95,480 86,645 88,605

2,375,730 594,145 246,105

© Emile Woolf International 88 The Institute of Chartered Accountants of Pakistan


Questions

Further information:
(1) Hasan Ltd purchased its interest in Riaz Ltd and Siddiq Ltd in December 2013 at which
date Siddiq Ltd had accumulated losses of Rs. 35,000, and Riaz Ltd had accumulated
profits of Rs. 35,000.
(2) On 30 December 2016 Hasan Ltd despatched and invoiced goods for Rs. 12,500 to Riaz
Ltd which were not recorded by the latter until 3 January 2017. A mark-up of 25% is
added by Hasan Ltd to arrive at selling price. Riaz Ltd already had goods in inventories
which had been invoiced to them by Hasan Ltd at Rs. 10,400.
(3) Siddiq Ltd had accumulated losses of Rs. 52,500 when Riaz Ltd purchased 35,000 shares
in 2012.
(4) Hasan Ltd received a remittance of Rs. 8,000 on 2 January 2017 which had been sent by
Riaz Ltd on 29 December 2016.
(5) Included in Hasan’s receivables was a balance of Rs. 25,500 owed by Riaz Ltd.
(6) Neither Riaz Ltd nor Siddiq Ltd had any other reserves when their shares were purchased
by Hasan Ltd and Riaz Ltd.
(7) Payables of Riaz Ltd included an amount of Rs. 5,000 due to Hasan Ltd.
Required
Prepare the consolidated statement of financial position of Hasan Ltd and its subsidiaries at 31
December 2016.

© Emile Woolf International 89 The Institute of Chartered Accountants of Pakistan


Advanced accounting and financial reporting

CHAPTER 24: DISPOSAL OF SUBSIDIARIES

24.1 PATCHE LTD


Patche Ltd owns 85% of the ordinary share capital of Somers Ltd for many years. The shares
were bought for Rs. 765 million and Somers Ltd’s reserves at the time of purchase amounted Rs.
60million.
On 31 March 2016, Patche Ltd. sold 120 million of Somers Ltd shares for Rs. 480 million. The
only entry made in respect of this transaction was the receipt of the cash, which was credited to
the “investment in subsidiary” account. No dividend was paid by either entity during the period.
The summarised financial statements of the companies are as follows:
Statements of profit or loss and other comprehensive income for the year ended to 30 June
2016.
Patche Ltd Somers Ltd
Rs.’m Rs.‘m
Profit before tax 390 180
Income tax expenses (120) (60)
Profit for the year 270 120
Other comprehensive income that will not be
reclassified to profit or loss net of tax 60 30
330 150
STATEMENTS OF FINANCIAL POSITION AS AT 30 JUNE 2016
Non-current assets:
Property, plant & equipment 1,605 534
Investment in Somers Ltd 285 -
Current assets:
Inventories 960 570
Trade receivables 750 525
Cash and bank 240 267
3840 1896
Equity interest
Share capital: Rs. 1 ordinary shares 1,500 600
Reserves 930 510
2,430 1,110
Current liabilities:
Trade payables 885 513
Income tax 240 180
Provisions 285 93
3,840 1,896
There was no impairment loss in the group during the year.
Assume that the gain as calculated in the parent’s separate financial statement will be subject to
companies’ income tax at a rate of 30%, and that profit and other comprehensive income accrue
evenly throughout the year.
Patche Ltd. group policy is to measure non-controlling interest at fair value at the date of
acquisition.
The fair value of the non-controlling interest in Somers Ltd. was Rs. 135million at the date of
acquisition.
Required
Prepare the following
(a) Statement of profit or loss and comprehensive income and statement of changes in equity
of Patche Ltd for the year ended 30 June 2016.
(b) Consolidated statement of profit or loss and comprehensive income of Patche Ltd. for the
same period.
(c) Consolidated statement of financial position as at 30 June 2016.

© Emile Woolf International 90 The Institute of Chartered Accountants of Pakistan


Questions

24.2 DISPOSAL
At 31 December Year 1, Hoo owned 90% of the shares in Spool. At this date the carrying amount
of the net assets of Spool in the consolidated financial statements of the Hoo Group was Rs. 800
million. None of the assets of Spool are re-valued.
On 1 January Year 2, Hoo sold 80% of the equity of Spool for Rs. 960 million in cash.
The remaining shares in Spool held by Hoo are estimated to have a fair value of Rs. 100 million.
Required
Explain how the disposal of the shares in Spool should be accounted for in the consolidated
financial statements of the Hoo Group.

24.3 PART DISPOSAL


On 1 January Year 2, P acquired 80% of the equity of S for Rs. 620 million in cash. On 30 June
Year 2 it sold 10% of the equity in S for Rs. 94 million. S did not issue any shares or make any
distribution to its shareholders in the year to 31 December Year 2. P uses the partial goodwill
method to account for the acquisition of S and no goodwill is attributed to the non-controlling
interest.
The net assets of S were as follows, at carrying value:
Rs. million
At 1 January Year 2 700
At 31 December Year 2 900
At 31 December Year 2, P carries out an impairment review and decides that the goodwill in its
investment in S has been impaired by Rs. 8 million.
Required
Explain how the disposal of the shares in S should be accounted for.

24.4 THE A GROUP


The summarised statements of financial position of A and its two subsidiaries B and C at 31
December Year 3 are shown below:
Summarised statements of financial position at 31 December Year 3
A B C
Rs.000 Rs.000 Rs.000
Investment in subsidiaries:
B 1,164
C 1,120
Other net assets 2,516 1,260 1,400
4,800 1,260 1,400
Ordinary share capital
(Rs. 1 shares) 1,500 500 400
Accumulated profits 3,300 760 1,000
4,800 1,260 1,400
The summarised statement of profit or loss for A and B for the year ended 31 December Year 4
are as follows:
A B
Rs.000 Rs.000
Profit before tax 1,200 250
Taxation (360) (60)
–––––– ––––––
Profit after tax 840 190
Dividends paid (50) (20)
–––––– ––––––
Retained profit for year 790 170
Retained profit at start of year 3,300 760
–––––– ––––––
Retained profits at end of year 4,090 930
–––––– ––––––

© Emile Woolf International 91 The Institute of Chartered Accountants of Pakistan


Advanced accounting and financial reporting

Additional information:
(i) A acquired 80% of the ordinary share capital of B on 1 January Year 0 when the reserves
of B were Rs. 420,000.
(ii) A acquired 90% of the ordinary share capital of C on 1January Year 1 when the reserves
of C were Rs. 320,000.
(iii) On 1 January Year 4, A disposed of 350,000 shares in C for Rs. 1,925,000. This
transaction has not yet been accounted for by A. The remaining investment in shares of C
at this date had a fair value of Rs. 44,000.
(iv) There were no changes in the issued share capital of the subsidiaries since acquisition by
A.
(v) None of the companies re-value any of their non-current assets.
(vi) The A Group uses the partial goodwill method of accounting for acquisitions and no
goodwill is attributed to non-controlling interests. There has been no impairment of
goodwill.
Required
Prepare A’s consolidated statement of profit or loss and show the movement on consolidated
equity reserves for the year to 31 December Year 4 and a consolidated statement of financial
position as at that date.

24.5 BARTLETT LTD


Many years ago Bartlett Ltd bought 80% of the ordinary shares of Lymon Ltd for Rs. 175,000.
On 1 July 2016 Bartlett sold all of these shares and used the proceeds (Rs. 212,000) to purchase
65% of the ordinary shares of Zeigler Ltd on the same date.
Statements of profit or loss for all three companies for the year ended 31 December 2016 were
as follows.
Bartlett Ltd Lymon Ltd Zeigler Ltd
Rs. Rs. Rs.
Revenue 1,926,500 521,600 792,400
Cost of sales 1,207,200) (386,200) (405,900)
Gross profit 719,300 135,400 386,500
Distribution costs (207,500) (79,200) (198,200)
Administrative expenses (192,600) (26,100) (107,100)
Profit before tax 319,200 30,100 81,200
Taxation (110,000) (9,500) (27,500)
Profit after tax 209,200 20,600 53,700
No entries have been made in Bartlett Ltd’s statement of profit or loss relating to the sale of
Lymon Ltd. Lymon’s net assets were Rs. 140,000 at the 1st January 2016.
Goodwill arising on the acquisition of Lymon Ltd was Rs. 25,400.
Required
Prepare the consolidated statement of profit or loss for Bartlett Ltd for the year ended 31
December 2016.

© Emile Woolf International 92 The Institute of Chartered Accountants of Pakistan


Questions

CHAPTER 26: FOREIGN CURRENCY

26.1 DND LIMITED


DND Limited is a listed company, having its operations within Pakistan. During the year ended
December 31, 2016, the company contracted to purchase plants and machineries from a US
Company. The terms and conditions thereof , are given below:
(i) Total cost of contract = US$ 100,000.
(ii) Payment to be made in accordance with the following schedule:

Payment Dates Amount Payable


On signing the contract July 01, 2016 US$ 20,000
On shipment* September 30, 2016 US$ 50,000
After installation and test run January 31, 2017 US$ 30,000

*(risk and rewards of ownership are transferred on shipment)

The contract went through in accordance with the schedule and the company made all the payments
on time. The following exchange rates are available:

Dates Exchange Rates

July 1, 2016 US$ 1 = Rs. 60.50

September 30, 2016 US$ 1 = Rs. 61.00

December 31, 2016 US$ 1 = Rs. 61.20

January 31, 2017 US$ 1 = Rs. 61.50


Required
Prepare journals to show how the above contract should be accounted for under IAS 21.

26.2 STARLIGHT LIMITED


In December 2014, Lahore Holdings Ltd acquired an 80% interest in a Qatari investment
company, Starlight Limited. Starlight Limited has the Qatari Rial (QR) as its functional currency.
The acquisition cost was Rs. 2,500,000 and the revenue reserves balance of Starlight Limited
stood at QR 49,300,000 at the acquisition date.
The following financial information was extracted from the books for the year ended 31
December 2016.
Starlight Limited
QR’000
Turnover 344,880
Cost of sales (249,710)
Gross profit 95,170
Expenses (29,490)
Profit before tax 65,680
Taxation (17,325)
Profit after tax 48,355
Interim dividend (16,300)
Retained profit for the year 32,055

© Emile Woolf International 93 The Institute of Chartered Accountants of Pakistan


Advanced accounting and financial reporting

Extract of statement of financial position at 31 December, 2016


QR’000
Share capital 20,250
Revenue reserve 103,200
Liabilities 34,480
Note
Exchange rate (QR to one rupee)
December 31, 2014 30
December 31, 2015 31
December 31, 2016 33
Average rate for 2016 32

Required
(a) Prepare the translated profit and loss account of Starlight Limited.
(b) Calculate the goodwill on consolidation and the non-controlling interest that would appear
in the consolidated statement of profit or loss.

26.3 PERCEPT LTD


Percept Ltd acquired 70% of the share capital of Trint on 1 January 2016, a company based in
Japan for 6,900,000 Yen. The retained earnings on this date was 4,500,000 Yen. The fair value
of the identifiable net assets of the company was 12,375,000 Yen and the net asset relates to
items of property plant and equipment. Similarly, the fair value of the non-controlling Interest
(NCI) in Trint Ltd as at 1 January, 2016 was 6,250,000 Yen. It is the policy of Percept Ltd to use
the “full goodwill method” in the preparation of the group’s financial statements. Trint Ltd’s profit
for the year ended 31 December 2016 was 2,000,000 Yen. The acquisition on 1 January 2016
was done in Japan when the following exchange rates were in force:
Yen to rupees
01/01/2016 6
31/12/2016 5
The average rate for the year ended 31 December 2016 was 5.5 Yen to Rs. 1.
Trint Ltd: Statement of Financial Position as at 31 December 2016.
Yen (000)
Assets:
Non-current assets 9,500
Financial assets 1,250
10,750
Current assets 8,250
Total assets 19,000

Equity and liabilities


Share capital 5,000
Retained earnings 7,500
12,500
Non-current liabilities 4,000
Current liabilities 2,500
6,500
Total equity and liabilities 19,000

© Emile Woolf International 94 The Institute of Chartered Accountants of Pakistan


Questions

Required
(a) Translate the statement of financial position of Trint Ltd. as at 31 December 2016
(b) Calculate the goodwill arising on acquisition of Trint and any gain/loss arising on
retranslation of the goodwill as at 31 December 2016
(c) Calculate the exchange difference arising from the translation of Trint Ltd’s net assets.

26.4 ORLANDO
Orlando is an entity whose functional currency is the US dollar. It prepares its financial
statements to 30 June each year. The following transactions take place on 21 May Year 4 when
the spot exchange rate was $1 = €0.8.
Goods were sold to Koln, a customer in Germany, for €96,000.
A specialised piece of machinery was bought from Frankfurt, a German supplier. The invoice for
the machinery is for €1,000,000.
The company receives €96,000 from Koln on 12 June Year 4.
At 31 June Year 4 it still owns the machinery purchased from Frankfurt. No depreciation has
been charged on the asset for the current period to 30 June Year 4.
The liability for the machine is settled on 31 July Year 4.
Relevant $/€ exchange rates are:
12 June Year 4 $1 = €0.9
30 June Year 4 $1 = €0.7
31 July Year 4 $1 = €0.8
Required
Show the effect on profit or loss of these transactions for:
(a) the year to 30 June Year 4
(b) the year to 30 June Year 5

26.5 MANCASTER AND STOCKPOT


Part A
Required
(1) Define and explain the following terms as used in IAS 21 The effects of changes in
foreign exchange rates.
(a) Functional currency
(b) Presentation currency
(2) Outline the factors to be considered when determining the functional currency of an
overseas subsidiary.
Part B
The statements of financial position of Manchester and its subsidiary Stockpot at 31 March Year
4 and their statement of profit or loss for year ended on that date are set out below:
Statements of financial position at 31 March Year 4
Mancaster Stockpot
$000 $000 €000 €000
Non-current assets:
Property, plant and equipment 20,000 30,000
Investments (notes 1 and 2) 5,500 -
25,500 30,000

© Emile Woolf International 95 The Institute of Chartered Accountants of Pakistan


Advanced accounting and financial reporting

Statements of financial position at 31 March Year 4


Mancaster Stockpot
$000 $000 €000 €000
Current assets:
Inventories 10,000 18,000
Trade receivables 10,000 15,000
20,000 33,000
Total assets 45,500 63,000
Capital and reserves:
Share capital ($1/€1 Shares) 9,000 15,000
Accumulated profits 12,500 10,000
21,500 25,000
Non-current liabilities (note 4) 10,000 20,000
Current liabilities
Trade payables 7,900 10,400
Bank overdraft 6,100 7,600
14,000 18,000
Total equity and liabilities 45,500 63,000

Statement of profit or loss – year ended 31 March Year 4


Mancaster Stockpot
$000 €000
Revenue 50,000 60,000
Cost of sales (notes 2 and 5) (25,000) (30,000)
Gross profit 25,000 30,000
Other operating expenses (15,000) (16,000)
Operating profit 10,000 14,000
Investment income (note 3) 1,500 -
Interest payable (1,000) (2,000)
Profit before tax 10,500 12,000
Tax (3,600) (4,200)
Profit after tax 6,900 7,800

Statement of changes in equity – year ended 31 March Year 4


Mancaster Stockpot
Share Reserves Share Reserves
capital capital
$000 $000 €000 €000
Balance at 1 April Year 3 9,000 9,500 15,000 6,600
Profit for the period 6,900 7,800
Dividends paid (3,900) (4,400)
Balance at 31 March Year 4 9,000 12,500 15,000 10,000

You are provided with the following additional information:


(1) Investments represent the acquisition of 11.25 million shares in Stockpot on 31 March Year
0. The retained profits of Stockpot on this date stood at €5 million. Any goodwill arising on
the acquisition is to be treated as a foreign currency asset. Stockpot operates as a
reasonably autonomous entity on a day-to-day basis although Mancaster does control the
long-term strategy of Stockpot.

© Emile Woolf International 96 The Institute of Chartered Accountants of Pakistan


Questions

(2) Exchange rates have been as follows:


Date € to $1
31 March Year 0 3.0
31 March Year 3 2.4
31 March Year 4 2.2
Average for Year 4 2.3
(3) Investment income represents dividends received from Stockpot.
(4) The non-current liabilities represent long-term borrowings.
Required
(a) Translate the statement of financial position of Stockpot into the presentation currency of
dollars and prepare the consolidated statement of financial position of the group at 31
March Year 4.
(b) Translate the statement of profit or loss of Stockpot into dollars and prepare the
consolidated statement of profit or loss of the group for the year ended 31 March Year 4.

26.6 A, B AND C
Extracts from the financial statements of A, its subsidiary, B and its associate, C for the year to 30
September 2016 are presented below:
Summarised statement of profit or loss and A B C
other comprehensive income
Rs.000 A$000 Rs.000
Revenue 4,600 2,200 1,600
Cost of sales and operating expenses (3,700) (1,600) (1,100)
Profit before tax 900 600 500
Income tax (200) (150) (100)
Profit for the year 700 450 400
Other comprehensive income:
Revaluation of property, plant and equipment 200 120 70
Total other comprehensive income 200 120 70
Total comprehensive income 900 570 470

Statement of financial position A B C


Rs.000 A$000 C$000
Assets
Non-current assets
Property, plant and equipment 7,000 4,000 2,000
Investment in B 5,200
Investment in C 900 _
13,100 4,000 2,000
Current assets 3,000 2,000 1,000
Total assets 16,100 6,000 3,000
Equity and liabilities
Share capital 2,000 1,000 1,000
Reserves 12,100 3,500 1,500
14,100 4,500 2,500
Current liabilities 2,000 1,500 500
Total equity and liabilities 16,100 6,000 3,000

© Emile Woolf International 97 The Institute of Chartered Accountants of Pakistan


Advanced accounting and financial reporting

Additional information:
1. The functional currency of both A and C is the Rs. and the functional currency of B is the
A$.
2. A acquired 80% of B on 1 October 2013 for Rs. 5,200,000 when the reserves of B were
A$1,800,000. The investment is held at cost in the individual financial statements of A.
3. A acquired 40% of C on 1 October 2011 for Rs. 900,000 when the reserves of C were Rs.
700,000. The investment is held at cost in the individual financial statements of A.
4. No impairment to either investment has occurred to date.
5. The group policy is to value the non-controlling interest at fair value at the date of
acquisition. The fair value of the non-controlling interest of B at 1 October 2013 was
A$600,000.
6. Relevant exchange rates are as follows:

1 October 2013 Rs./A$0.5000

30 September 2015 Rs./A$0.7100

30 September 2016 Rs./A$0.6300

Average rate for year ended 30 September 2016 Rs./A$0.6500

Required
Prepare the consolidated statement of profit or loss and other comprehensive income for the A
Group for the year ended 30 September 2016 and the consolidated statement of financial
position as at that date.

26.7 OMEGA LIMITED


Omega Limited (OL) is incorporated and listed in Pakistan. On 1 May 2012, it acquired 20,000
ordinary shares (2% shareholding) in Al-Wadi Limited (AWL), a Dubai based company at a cost
of AED 240,000 which was equivalent to Rs. 6,000,000. The face value of the shares is AED 10
each. OL intends to hold the shares to avail benefits of regular dividends and capital gains.
On 1 June 2013, AWL was acquired by Hilal Limited (HL), which issued three shares in HL in
exchange for every four shares held in AWL.
Other relevant information is as under:
AWL HL
Final dividend received on 31 March 2013:
Cash 15% -
Bonus shares 10% -
Final cash dividend received on 10 April 2014 - 20%
Fair value per share as at: 31 December 2012 AED 13.00 -
1 June 2013 AED 14.00 AED 18.00
31 December 2013 - AED 19.50
Exchange rates on various dates were as follows:
31-Dec-2012 31-Mar-2013 1-Jun-2013 31-Dec-2013 10-Apr-2014
1 AED Rs. 25.00 Rs. 26.50 Rs. 28.00 Rs. 28.70 Rs. 28.20
Required
Determine the amounts (duly classified under appropriate heads) that would be included in
OL’s statement of comprehensive income for the year ended 31 December 2013 in respect of the
above investment.

© Emile Woolf International 98 The Institute of Chartered Accountants of Pakistan


Questions

26.8 PARENT COMPANY LIMITED


Parent Company Limited (PCL) is a listed company and owns 80% and 75% equity in LS Limited
and FS Limited respectively. FS is registered and operates in a foreign country and its functional
currency is CU. Summarised statements of financial position as at 30 June 2014 and other
information relating to the group companies are as under:

PCL LS FS
Rs. in million CU in million
Assets
Property, plant and equipment 4,200 3,500 250
Investments in LS and FS 6,500 - -
Current assets 3,500 4,000 450
14,200 7,500 700
Equity and liabilities
Share capital (Rs. 10/CU 10
each 6,000 1,800 120
Retained earnings 3,500 900 280
Current liabilities 4,700 4,800
14,200 7,500 700
Profit after tax for the year ended
30 June 2014 700 400 30
Final dividend for the year ended
30 June 2013: 12% - 15%
Cash (paid on 1 January 2014) 10% 20% -
700 400 30

The following information is also available:

i.
At the acquisition
No. of date
Investment
Company shares Cost Fair
date Retained
value of
acquired
earnings *
NCI
---- in million ----
LS 1-Jan-2012 120 Rs. 2,000 Rs. 250 Rs. 540
FS 1-Jul-2012 9 CU 300 CU 160 CU 90

*NCI stands for Non-controlling interest


ii. On the date of acquisition, fair value of the net assets of LS and FS were equal to their
book value. However, a contingent liability of Rs. 25 million was disclosed in the financial
statements of LS. PCL's legal adviser had at that time estimated that LS would be liable
to pay Rs. 6 million to settle the claim and it was finally settled at the same amount in
May 2014.

© Emile Woolf International 99 The Institute of Chartered Accountants of Pakistan


Advanced accounting and financial reporting

iii. No further shares have been issued by LS and FS since their acquisitions, except for the
bonus issue as mentioned above.
iv. An impairment test carried out on 30 June 2014 revealed that goodwill of FS is impaired
by CU 10 million.
v. PCL values non-controlling interest on the date of acquisition at fair value.
vi. The exchange rates in terms of Rs. per CU, were as follows:

1-Jul-2012 30-Jun-2013 1-Jan-2014 30-Jun-2014 Average for 2013-14

Rs. 15.00 Rs. 16.80 Rs. 16.90 Rs. 17.30 Rs. 17.00

vii. The break-up of exchange reserve in the consolidated financial statements for the year
ended 30 June 2013 is as follows:

Relating to goodwill Rs. 148.50 million

Relating to translation of foreign operations Rs. 463.05 million

Required:
In accordance with the requirements of the International Financial Reporting Standards, prepare:
(a) Consolidated statement of financial position as at 30 June 2014; and
(b) Consolidated statement of other comprehensive income for the year ended 30 June
2014. (Ignore taxation)

© Emile Woolf International 100 The Institute of Chartered Accountants of Pakistan


Questions

CHAPTER 27: IAS 7: STATEMENTS OF CASH FLOWS

27.1 EVERNEW LTD


The following relates to the financial statements of Evernew Ltd.
(a) Consolidated statement of profit or loss for the year ended 31 December 2016
Rs.000
Operating profit 144,000
Interest expenses (10,080)
Profit after interest 133,920
Profit from disposal of subsidiary 5,040
Profit before taxation 138,960
Taxation (46,800)
Profit after taxation 92,160

Profit for the year attributable to:


Owners of the parent 84,960
Non-controlling interest 7,200
Retained profit c/f 92,160
(b) Consolidated statement of financial position as at 31 December, 2016
2016 2015
Assets Rs.’000 Rs.’000 Rs.’000 Rs.’000
Non-current assets 369,720 360,000
Current assets:
Inventory 180,000 165,600
Trade receivables 151,200 136,800
Cash in hand 63,360 14,400
394,560 316,800
Total assets 764,280 676,800
Equity and liabilities
Ordinary share capital 144,000 144,000
Accumulated profits 317,520 232,560
461,520 376,560
Non-controlling interest 36,360 41,400
10% debenture 68,400 90,000

Current liabilities
Trade creditors 108,000 93,600
Taxation 46,800 39,240
Bank overdraft 43,200 36,000
Net current assets 198,000 168,840
Total equity and liabilities 764,280 676,800
(c) The following additional information is relevant:
(i) Depreciation for the year in the consolidated profit and loss account was Rs.
72,720,000. Non-current assets were not disposed by the group except those made
during disposal of the investment in the shares of Pastit Limited.

© Emile Woolf International 101 The Institute of Chartered Accountants of Pakistan


Advanced accounting and financial reporting

(ii) Evernew Ltd sold its investment in Pastit Limited in July 2016. The entire 80%
shareholding in the subsidiary was sold for Rs. 39.6million. Information about the
disposal is as follows:
Rs.’000 Rs.’000
Inventories 14,400
Receivables 18,000
Non-current assets 28,800
Trade creditors (10,800)
Taxation (2,160)
Bank overdraft (1,440)
Debenture stock (3,600) (18,000)
43,200
Non-controlling interest (8,640)
34,560

The investment was acquired many years ago for Rs. 13.68million when the net assets of
Pastit Limited were Rs. 14.4million. Goodwill had been fully written off before due to
impairment.
Required
Prepare the Evernew Ltd group consolidated cash flow statement for the year ended 31
December, 2016.

27.2 BELLA
The financial statements of Bella include the following:
Statements of financial position as at 31 March Year 6
Year 6 Year 5
Rs.000 Rs.000 Rs.000 Rs.000
Assets
Non-current assets
Property, plant and equipment 12,900 8,000
Intangible assets 800 300
13,700 8,300
Current assets
Inventories 280 100
Trade and other receivables 1,290 1,350
Cash 55 45
1,625 1,495
Total assets 15,325 9,795
Equity and liabilities
Capital and reserves
Issued capital (Rs. 1 ordinary shares) 1,900 1,100
Share premium 95 30
Accumulated profits 11,407 7,540
13,402 8,670
Non-current liabilities
Long-term loans 600 500
600 500

© Emile Woolf International 102 The Institute of Chartered Accountants of Pakistan


Questions

Year 6 Year 5
Rs.000 Rs.000 Rs.000 Rs.000
Current liabilities
Bank overdraft (repayable on demand) 313 -
Trade and other payables 430 275
Interest payable 40 25
Current tax payable 540 325
1,323 625
Total equity and liabilities 15,325 9,795
Statement of profit or loss for the year ended 31 March Year 6 (extract).
Rs.000
Operating profit 4,677
Interest payable (60)
Profit before tax 4,617
Tax expense (400)
Profit for the period 4,217
The following occurred during the year.
(1) Dividends of Rs. 350,000 were paid.
(2) New plant was purchased for Rs. 6 million.
(3) Old plant which had a net book value of Rs. 800,000 was sold for Rs. 700,000.
(4) Shares were issued for cash during the period.
Required
Prepare a statement of cash flows for the year ended 31 March Year 6 using the indirect method.

27.3 BISHOP GROUP


You are provided with the information set out below relating to a group of companies.
Consolidated statement of profit or loss for Bishop Group for the year ended 31
December 20X2
20X2 20X1
Rs.000 Rs.000
Revenue 19,308 18,173
Cost of sales (4,315) (4,620)
––––––––– –––––––––
Gross profit 14,993 13,553
Distribution costs (6,439) (6,126)
Administrative expenses (5,705) (6,719)
––––––––– –––––––––
Profit before tax and finance costs (note 1) 2,849 708
Finance income 90 75
Finance costs (note 2) (350) (230)
––––––––– –––––––––
Profit before taxation 2,589 553
Income tax expense (note 3) (800) (125)
Profit for the year 1,789 428
––––––––– –––––––––
Attributable to:
Equity holders of the parent 1,369 318
Non-controlling interest 420 110
––––––––– –––––––––
1,789 428
––––––––– –––––––––

© Emile Woolf International 103 The Institute of Chartered Accountants of Pakistan


Advanced accounting and financial reporting

Summary statement of changes in equity for the year to


31 December 20X2
Foreign
Issued Share currency Retained
capital premium translation earnings Total NCI Equity
B/fwd 1 Jan 7,500 77 2,100 4,905 14,582 2,500 17,082
Issue of shares 3,500 324 3,824 3,824
Profit for year 1,369 1,369 420 1,789
Dividends (600) (600) (295) (895)
Exchange gain 700 700 175 875
–––––– –––––– –––––– –––––– –––––– –––––– ––––––
C/fwd 31 Dec 11,000 401 2,800 5,674 19,875 2,800 22,675
–––––– –––––– –––––– –––––– –––––– –––––– ––––––
Notes: NCI = non-controlling interest
Exchange gain = exchange gain on translation of subsidiary

Consolidated statement of financial position as at 31 December 20X2


20X2 20X1
Rs.000 Rs.000
Non-current assets
Tangible assets 11,720 7,520
Investments 3,000 2,700
Current assets
Inventories 6,135 5,740
Receivables 5,720 4,380
Cash at bank and in hand 820 169
27,395 20,509
Equity and liabilities
Ordinary share capital 11,000 7,500
Share premium 401 77
Foreign currency translation 2,800 2,100
Retained earnings 5,674 4,905
Equity attributable to owners of parent 19,875 14,582
Non-controlling interest 2,800 2,500
Total equity 22,675 17,082
Current liabilities
Payables 1,420 1,760
Tax 700 167
Obligations under finance leases 110 50
Non- current liabilities
Loans 1,200 800
Obligations under finance leases 740 250
Provisions for liabilities and charges
Deferred tax 550 400
27,395 20,509
Notes to the accounts
(1) Operating profit is stated after charging Rs.000 Rs.000
Depreciation: Owned assets 960 840
Assets held under finance leases 240 120
1,200 960

© Emile Woolf International 104 The Institute of Chartered Accountants of Pakistan


Questions

(2) Finance costs Rs.000 Rs.000


Loan interest 120 80
Finance charge on finance leases 205 132
Exchange rate losses on long-term loans 25 18
350 230

(3) Taxation on profits on ordinary activities Rs.000 Rs.000


Tax on income at 30% 600 90
Deferred tax 150 35
Under provision in respect of previous years 50 -
800 125

(4) Foreign exchange differences Rs.000 Rs.000


Gains arising on re-translation 700 400
The exchange rate gain relates to the translation of an 80% owned overseas
subsidiary, Louise, under the closing rate method. The gain comprises:
Non-current assets 424
Inventories 117
Receivables 339
Cash 53
Trade payables (58)
875
Attributable to NCI (175)
Attributable to owners of parent company 700
(5) During the year non-current assets additions of Rs. 700,000 were acquired under
finance leases. Payments on finance leases are made in arrears. The net book
value of non-current assets disposed of was Rs. 720,000, with sale proceeds of
Rs. 810,000.

Required
(a) Prepare the group statement of cash flows of Bishop in accordance with IAS 7 together
with any required notes for the year ended 31 December 20X2.
(b) Explain why external users of financial statements benefit from receiving a statement of
cash flows.

27.4 THE GRAPE GROUP


The draft statements of financial position and statement of profit or loss of the Grape Group at 31
March Year 4 and 31 March Year 3 are as follows:
Notes Year 4 Year 3
Rs.000 Rs.000
Non-current assets
Intangible assets 24 -
Property, plant and equipment (1) 13,515 12,990
Investments – associated undertakings 1,966 1,920
15,505 14,910
Current assets
Inventory 11,657 10,530

© Emile Woolf International 105 The Institute of Chartered Accountants of Pakistan


Advanced accounting and financial reporting

Notes Year 4 Year 3


Rs.000 Rs.000
Receivables 7,209 6,936
Cash at bank and in hand 5,190 1,728
24,056 19,194
39,561 34,104
Capital and reserves
Share capital 8,100 7,425
Share premium 1,989 1,470
Accumulated profits 13,200 8,700
23,289 17,595
Non-current liabilities (2) 6,900 7,890
Current liabilities (3) 9,372 8,619
39,561 34,104

Statement of profit or loss: Year to 31 March Year 4 Year 3


Rs.000 Rs.000
Sales revenue 74,100 59,400
Cost of sales (54,138) (42,240)
Gross profit 19,962 17,160
Distribution costs (5,889) (4,869)
Administrative expenses (4,092) (3,384)
Operating profit 9,981 8,907
Income from interests in associates 139 144
Loss on sale of tangible non-current assets (18) -
Interest expense (552) (651)
Profit before tax 9,550 8,400
Income tax expense (2,950) (2,400)
Profit after tax 6,600 6,000

Notes
(1) Property, plant and equipment
Year 4 Year 3
Rs.000 Rs.000
Cost
At 1 April 20,598 19,416
Additions 1,875 2,022
Disposals (429) (840)
At 31 March 22,044 20,598
Depreciation
At 1 April 7,608 6,984
Charge for year 1,176 936
Disposals (255) (312)
At 31 March 8,529 7,608
Net book value 13,515 12,990

© Emile Woolf International 106 The Institute of Chartered Accountants of Pakistan


Questions

(2) Non-current liabilities


Year 4 Year 3
Rs.000 Rs.000
8% Loan notes 6,900 6,900
10% Unsecured loan notes - 990
6,900 7,890
(3) Current liabilities
Year 4 Year 3
Rs.000 Rs.000
Accounts payable 6,422 6,219
Income tax 2,950 2,400
9,372 8,619
You are provided with the following additional information:
Interest on the 8% loan note is payable half-yearly on 30 September and 31 March.
Income tax is payable nine months after the year-end.
Dividends of Rs. 2.1m and Rs. 2.4m were proposed for Year 3 and Year 4 respectively.
During the year, the Grape Group acquired 100% of the ordinary share capital of Pip. The
purchase was financed by Rs. 346,000 in cash and the issue of 54,000 ordinary shares of
Rs. 1 each. The ordinary shares had a market value of Rs. 152,000. The following figures
related to Pip at the date of acquisition:
Rs.000
Property, plant and equipment 315
Inventory 139
Receivables 85
Cash at bank and in hand 3
Payables (68)
474
Share capital 180
Reserves 294
474

(4) The share capital consists of Rs. 1 ordinary shares.


Required
Prepare a statement of cash flows and related notes for the Grape Group for the
year ended 31 March Year 4.

© Emile Woolf International 107 The Institute of Chartered Accountants of Pakistan


Advanced accounting and financial reporting

CHAPTER 28: IAS 33: EARNINGS PER SHARE

28.1 AIRCON LTD


Mr Hamad, currently owns 20 million shares in Aircon Ltd. He recently received the published
financial statements of Aircon Ltd for the year ended 31 March 2016. Mr Hamad is not sure how
the performance of the company during the year will affect the market value of the entity’s shares
but he is aware that the earnings per share statistics are often used by analysts in assessing the
performance of listed companies.
Extracts from these published financial statements and other relevant information are given
below.
Statement of profit or loss for the period ended 31 March 2016
2016 2015
Rs.’m Rs.’m
Revenue 18,000 15,300
Cost of sales (11,340) (9,180)
Gross profit 6,660 6,120
Operating expenses (3,420) (3,240)
Operating profit 3,240 2,880
Interest payable (540) (576)
Profit before tax 2,700 2,304
Taxation (846) (720)
Profit after tax 1,854 1,584
Statement of financial position as at 31 March 2016
2016 2015
Rs.’m Rs.’m Rs.’m Rs.’m
Intangible assets 5,400 -
Tangible assets 7,200 6,660
──── ────
12,600 6,660
Current Assets
Inventory 2,340 1,800
Receivables 2,700 2,160
Cash in bank 180 5,220 162 4,122
──── ──── ──── ────
17,820 10,782
──── ────
Capital and Reserves
Share Capital 2,700 900
Share Premium 4,860 900
Retained Earnings 1,620 1,206
──── ────
9,180 3,006
Current Liabilities
Trade Payables 3,060 2,160
Taxation 900 756
Bank Overdraft 1,080 1,260
──── ────
5,040 4,176
──── ────
14,220 7,182
15% Loan Note 3,600 3,600
──── ────
17,820 10,782
──── ────

© Emile Woolf International 108 The Institute of Chartered Accountants of Pakistan


Questions

The following information is also relevant:


(i) The share capital of the company comprises Rs. 1 equity shares only.
(ii) On 1 October 2015, the company made a rights issue to existing shareholders of two new
shares for every one share held at a price of Rs. 5.94 per share and paid issue cost of Rs.
180,000.
(iii) The market price of shares immediately before the rights issue was Rs. 6.30 per share.
(iv) No other changes took place in the equity capital of Aircon Ltd in the year ended 31 March
2016.
Required
(a) Compute EPS for the year and the comparative figures that will be included in the
published financial statements of Aircon Ltd for the year ended 31 March 2016.
(b) Using the extracts you have been provided with, write a report to Mr Hamad identifying the
key factors which led to the change in the EPS of Aircon Ltd since the year ended 31
March 2016.
(c) Comment on the relevance of the EPS statistics to shareholders.

28.2 CACHET LTD


The statement of profit or loss for the year ended 31 December 2016 relates to Cachet Ltd.
Rs. Rs.
Profit Before Tax 121,900
Less: Taxation 52,900
69,000
Less: Transfer to general reserve 5,750
Dividends:
Preference shares 1,380
Ordinary shares 2,070
(92,00)
Retained profit 59,800
1 January 2016, the issued share capital of Cachet Ltd was 23,000 6% preference shares of Rs.
1 each and 20,700 ordinary shares of Rs. 1 each.
Required
Calculate the basic and diluted earnings per share for the year ended 31 December, 2016 under
the following circumstances:
(i) No change in the issued share capital.
(ii) The company made a bonus issue of one ordinary share for every four shares in issue at
30 September, 2016.
(iii) The company made a rights issue of shares on 1 October 2016 in the proportion of 1 for
every 5 shares held at a price of Rs. 1.20. The middle market price for the shares on the
last day of quotation cum rights was Rs. 1.80 per share.

28.3 MARY
On 1 January Year 5, Mary had 5 million ordinary shares in issue. The following transactions in
shares took place during the next year.
1 February A 1 for 5 bonus issue
1 April A 1 for 2 rights issue at Rs. 1 per share. The market price of the shares prior to
the rights issue was Rs. 4.
1 June An issue at full market price of 800,000 shares.

© Emile Woolf International 109 The Institute of Chartered Accountants of Pakistan


Advanced accounting and financial reporting

In Year 5 Mary made a profit before tax of Rs. 3,362,000. It paid ordinary dividends of Rs.
1,200,000 and preference dividends of Rs. 800,000. Tax was Rs. 600,500. The reported EPS for
Year 4 was Rs.0.32.
Required
Calculate the EPS for Year 5, and the adjusted EPS for Year 4 for comparative purposes.

28.4 MANDY
Mandy has had 5 million shares in issue for many years. Earnings for the year ended 31
December Year 4 were Rs. 2,579,000. Earnings for the year ended 31 December Year 3 were
Rs. 1,979,000. Tax is at the rate of 30%.
Outstanding share options on 500,000 shares have also existed for a number of years. These
can be exercised at a future date at a price of Rs. 3 per share. The average market price of
shares in Year 3 was Rs. 4 and in Year 4 was Rs. 5.
On 1 April Year 3 Mandy issued Rs. 1,000,000 convertible 7% bonds. These are convertible into
ordinary shares at the following rates.
On 31 December Year 6 30 shares for every Rs. 100 of bonds
On 31 December Year 7 25 shares for every Rs. 100 of bonds
On 31 December Year 8 20 shares for every Rs. 100 of bonds
Required
Calculate the diluted EPS for Year 4 and the comparative diluted EPS for Year 3.

28.5 AAZ LIMITED


The profit after tax earned by AAZ Limited during the year ended December 31, 2016 amounted to
Rs. 127.83 million. The weighted average number of shares outstanding during the year were 85.22
million.
Details of potential ordinary shares as at December 31, 2016 are as follows:
 The company had issued debentures which are convertible into 3 million ordinary
shares. The debenture holders can exercise the option on December 31, 2018. If the
debentures are not converted into ordinary shares they shall be redeemed on December
31, 2018. The interest on debentures for the year 2016 amounted to Rs. 7.5 million.
 Preference shares issued in 2013 are convertible into 4 million ordinary shares at the
option of the preference shareholders. The conversion option is exercisable on December
31, 2020. The dividend paid on preference shares during the year 2016 amounted to Rs.
2.45 million.
 The company has issued options carrying the right to acquire 1.5 million ordinary
shares of the company on or after December 31, 2016 at a strike price of Rs. 9.90 per
share. During the year 2016, the average market price of the shares was Rs. 11 per
share.
The company is subject to income tax at the rate of 30%.
Required
(a) Compute basic and diluted earnings per share.
(b) Prepare a note for inclusion in the company’s financial statements for the year ended
December 31, 2016 in accordance with the requirements of International Accounting
Standards.

© Emile Woolf International 110 The Institute of Chartered Accountants of Pakistan


Questions

28.6 ABC LIMITED


The following information pertains to ABC Limited, in respect of year ended March 31, 2016.

Rs. in ‘000
Consolidated profit for the year (including non-controlling interest) 15,000
Profit attributable to non-controlling interest 2,000
Dividend paid during the year to ordinary shareholders 4,000
Dividend paid on 10% Cumulative preference shares for the year 2015 2,000
Dividend paid on 10% Cumulative preference shares for the year 2016 2,000
Dividend declared on 12% Non-cumulative preference shares for the year
2016 2,400

(i) The company had 10 million ordinary shares at March 31, 2015.
(ii) The cumulative preference shares were issued at the time of inception of the company.
(iii) The 12% non-cumulative preference shares are convertible into ordinary shares, on or
before December 31, 2017 at a premium of Rs. 2 per share. The conversion rights are not
adjusted for subsequent bonus issues.
(iv) 0.50 million non-cumulative preference shares were converted into ordinary shares on July
1, 2015.
(v) The dividend declared on the non-cumulative preference shares, as referred above, was
paid in April 2016.
(vi) 1.20 million right shares of Rs. 10 each were issued at a premium of Rs. 1.50 per share on
October 1, 2015. The market price on the date of issue was Rs. 12.50 per share.
(vii) 20% bonus shares were issued on January 1, 2016.
(viii) Due to insufficient profit no dividend was declared during the year ended March 31,
2015.
(ix) The average market price for the year ended March 31, 2016 was Rs. 15 per share.
Required
Compute the basic and diluted earnings per share and prepare a note for inclusion in the
consolidated financial statements for the year ended March 31, 2016.

28.7 ALPHA LIMITED


Alpha Limited (AL), a listed company, acquired 80% equity in Zee Limited (ZL) on 1 July 2010.
The following information has been extracted from their draft financial statements:

AL ZL
----- Rs. in '000 -----
Balance as at 1 January 2013:
Share capital (Rs. 100 each) 80,000 35,000
12% Convertible bonds (Rs. 100 each) 30,000 -
Profit for the year ended 31 December 2013 (after tax) 60,000 25,000

Following information is also available:


(i) The bonds were issued at par on 1 January 2011 and are convertible at any time
before the redemption date of 31 December 2015, at the rate of five ordinary shares
for every four bonds.

© Emile Woolf International 111 The Institute of Chartered Accountants of Pakistan


Advanced accounting and financial reporting

(ii) Cost and fair value information of ZL’s investment property is as under:
31-Dec-2013 31-Dec-2012
-------- Rs. in '000 --------
Cost 65,000 60,000
Fair value 67,000 59,000

ZL uses cost model while the group policy is to use the fair value model to account
for investment property.
(iii) AL operates a defined benefit gratuity scheme for its employees. The actuary’s
report has been received after the preparation of draft financial statements and
provides the following information pertaining to the year ended 31 December 2013:
Rs. in '000
Actuarial losses 150
Current service costs 8,000
Net interest income 3,000

(iv) On 1 August 2013, under employees’ share option scheme, 60,000 shares were
issued by AL to its employees at Rs. 150 per share against the average market
price of Rs. 250 per share.
(v) Dividend details are as under:

AL ZL
2013 (Interim) 2012 (Final) 2013 (Interim) 2012 (Final)
Cash 18% 10% 12% 15%
Bonus shares - 20% - 16%
At the time of payment of dividend, income tax at 10% was deducted by AL and ZL.
(vi) Applicable tax rate for business income is 35%.

Required:
Extracts from the consolidated profit and loss account of Alpha Limited (including earnings per
share) for the year ended 31 December 2013 in accordance with the International Financial
Reporting Standards.
(Note: Comparative figures and information for notes to the financial statements are not required)

© Emile Woolf International 112 The Institute of Chartered Accountants of Pakistan


Questions

CHAPTER 29: ANALYSIS AND INTERPRETATION OF FINANCIAL STATEMENTS


29.1 ALPHA LIMITED AND OMEGA LIMITED
Alpha Limited and Omega Limited are in the same trade, but operate in different areas. Their
accounts for the year ended 31 December, 2016 are as follows:
Profit and loss account Alpha Limited Omega Limited
Rs.’000 Rs.’000 Rs.’000 Rs.’000
Sales 1,440 1,720
Less: Cost of sales 1,120 1,342
──── ────
Gross profit 320 378
Less: Overheads 220 300
──── ────
Profit before tax 100 78
Taxation 40 30
Dividends 20 24
──── ────
60 54
──── ────
Retained earnings 40 24
──── ────

Statement of financial position


Share capital of Rs. 1 each 600 200
Reserves 240 104
──── ────
840 304
8% Debentures - 120
840 424
──── ────
Represented by:
Non-current assets at cost 660 520
Less: Depreciation 200 160
──── ────
460 360
Current assets:
Inventory 280 172
Receivables 310 300
Cash 30 32
──── ────
620 504
──── ────
Current liabilities:
Taxation 40 30
Creditors 180 344
Bank overdraft - 42
Dividends 20 24
──── ────
240 440
──── ────
Net Current assets 380 64
──── ────
840 424
──── ────
Required
(a) Compute the following ratios for each of the companies:
(i) Current ratio
(ii) Acid test
(iii) Creditors ratio
(iv) Collection period or Receivables Ratio
(iv) Earnings per share
(b) Carry out comparative analysis of the companies based on the computed ratios in (a)
above.

© Emile Woolf International 113 The Institute of Chartered Accountants of Pakistan


Advanced accounting and financial reporting

29.2 COOK LIMITED


The following are the financial statements of Cook Limited for the years ended 31 December,
2015 and 2016.
Profit and loss accounts for the year ended 31 December 2016
2016 2015
Rs.’m Rs.’m
Sales 5,600 1,800
Cost of sales (3,360) (720)
Gross profit 2,240 1,080
Selling expenses (540) (300)
Bad debts (280) (36)
Depreciation (416) (116)
Interest (384) (24)
Net profit 620 604
Balance b/fwd 1,258 654
Balances c/fwd 1,878 1,258
Statements of financial position
2016 2015
Rs.’m Rs.’m Rs.’m Rs.’m
Non-current assets
Factory 882 900
Machinery 3,582 980
4,464 1,880
Current assets
Inventory 476 60
Receivables 1,166 166
Bank 24
1,642 250
Current liabilities
Creditors (350) (72)
Bank (22) --
(372) (72)

Net current assets 1,270 178


5,734 2,058
Loans (3,200) (200)
2,534 1,858

Share capital 656 600


Profit and loss 1,878 1,258
2,534 1,858

The Directors of Cook Limited appointed a new sales manager towards the end of 2016. This
manager devised a plan to increase sales and profit by means of a reduction in selling price and
extended credit terms to customers.

© Emile Woolf International 114 The Institute of Chartered Accountants of Pakistan


Questions

This involved considerable investment in new machinery early in 2016 to meet the increased
sales. All sales are on credit.
Required
(a) Explain whether the performance for the year ended 31 December 2016 and the financial
position at the date have improved as a result of the new policies adopted by the company.
You should support your answer with appropriate profitability and liquidity ratios.
(b) Calculate the amount of cash which would be released if the company could impose a
collection period of 45 days.

29.3 FITZROY LIMITED


The following ratios and an un-completed statement of financial position of Fitzroy Limited were
extracted from the working sheet of a financial consultant to the company.
RATIOS
1. Cash flow ratio = 0.6711:1
2. Current ratio = 2.5:1
3. Cash ratio = 0.2:1
4. Average collection period = 46.596 days
5. Gearing assets = 0.34545:1
6. Non-current assets = 70% of total assets
7. Inventory turnover = 5.74 times
1
8. Margin and mark-up = /5 and ¼ respectively
9. Creditors payment period = 27.06 days
Fitzroy Limited
Uncompleted statement of financial position as at 31 December, 2016
Capital Rs.’000 Non-current assets Rs.’000
Ordinary shares X Land & building 5,000
71/2 debenture X Furniture & fitting 1,450
RESERVES Motor vehicles 2,300
Profit & Loss X 8,750

Current liabilities Current assets


Creditors X Inventories 1,800
Dividend X Receivables X
Cash X
X X
You are given the following additional information relating to the financial statements.
1. Depreciation on all non-current assets for the year is Rs. 972,000
2. Dividend payable is 11.76% of ordinary shares
3. Company tax is estimated to be Rs. 400,000
4. Retained profit brought forward from the previous year is Rs. 800,000
Required
From the above information, you are required to complete the statement of financial position of
Fitzroy Limited as at 31 December, 2016 and prepare the statement of profit and loss for the
period then ended.

© Emile Woolf International 115 The Institute of Chartered Accountants of Pakistan


Advanced accounting and financial reporting

29.4 TRAVELWELL LTD


The recently produced financial statements of Travelwell Ltd for the year to 30 September 2016
are shown below, together with comparative figures for the previous year.

Statement of financial position as at 30 September 2016 2015


Rs.000 Rs.000
Non-current assets
Property, plant and equipment 252,000 105,000
Goodwill 12,000 nil
264,000 105,000
Current assets
Inventory 29,000 17,000
Trade receivables 17,000 9,000
Bank Nil 28,000
46,000 54,000
Total assets 310,000 159,000
Equity and liabilities
Equity shares of Rs. 1 each 100,000 100,000
Retained earnings 44,000 41,000
144,000 141,000

Non-current liabilities
7.5% loan notes 130,000 nil
Current liabilities
Trade payables 28,000 15,000
Bank overdraft 2,000 nil
Current tax payable 6,000 3,000
36,000 18,000
Total equity and liabilities 310,000 159,000

Statement of profit or loss for the year ended 2016 2015


30 September 2016
Rs.000 Rs.000
Revenue 300,000 210,000
Cost of sales (240,000) (175,000)
Gross profit 60,000 35,000
Operating expenses (26,250) (23,000)
Finance costs (9,750) nil
Profit before tax 24,000 12,000
Income tax expenses (at 25%) (6,000) (3,000)
Profit for the year 18,000 9,000

© Emile Woolf International 116 The Institute of Chartered Accountants of Pakistan


Questions

The following are extracts from the report of the Chief Executive Officer.
“Highlights of the performance of Travelwell Ltd for the year ended 30 September 2016 are:
(1) an increase of 43% in sales revenue
(2) an increase in the gross profit margin from 16.7% to 20%
(3) a 100% increase in profit for the year.
In response to the improvement in financial performance the board of directors paid a dividend of
Rs. 15m in September 2016, an increase of 25% on the previous year.”
The following information is also available.
On 1 October 2015, Travelwell Ltd purchased 100% of the net assets of Rondel Ltd for Rs. 130
million. Rondel Ltd was previously a privately-owned business entity. The contribution of this
purchase to the financial results of Travelwell Ltd for the year to 30 September 2016 was as
follows:

Rs.000

Revenue 90,000
Cost of sales (50,000)

Gross profit 40,000


Operating expenses (11,000)

Profit before tax 29,000

There were no disposals of non-current assets during the year.


The following ratios were calculated for the year ended 30 September 2015:

Return on year-end capital employed 8.5%


profit before interest and tax
(= /total assets less current liabilities)
Net asset turnover (net assets = capital employed) 1.5
Net profit margin (before tax) 5.7%
Current ratio 3.0
Closing inventory holding period (in days) 35
Trade receivables collection period (in days) 16
Trade payables payment period (in days and based on cost of sales) 31
Gearing ratio (debt/debt plus equity) nil

Required
(a) Calculate ratios for Travelwell Ltd for the year ended 30 September 2016 equivalent to
those shown above for the year to 30 September 2015. Show your workings.
(b) Assess the financial performance and financial position of Travelwell Ltd for the year
ended 30 September 2016, in comparison with the previous year referring to the
comments in the report of the Chief Executive Officer and you should also assess the
effect of the purchase of the net assets of Rondel Ltd.

© Emile Woolf International 117 The Institute of Chartered Accountants of Pakistan


Advanced accounting and financial reporting

29.5 SACHAL LIMITED


Sachal Limited (SL) is planning to acquire 100% shareholdings in Waris Limited (WL).
Before submission of financial proposal, SL is carrying out an analysis of WL’s financial and
operating performance. The CFO of SL has gathered the following information which is based on
the financial statements for the year ended December 31, 2016:

WL’s Industry Ratios


Description
Ratios High Low Average
Operating performance Ratios

Gross profit 29% 30% 20% 25%

Operating profit 11% 15% 10% 13%

Return on shareholders equity 9% 13% 7% 10%

Working capital ratios


Current ratio 1.54 : 1 2:1 1:1 1.5 : 1

Inventory turnover days 83 days 114 days 81 days 91 days

Receivables collection 93 days 95 days 60 days 74 days

Gearing ratios
Debt equity ratio 55 : 45 60 : 40 40 : 60 50 : 50

Interest cover 1.3 times 3 times 1.2 times 2 times

Investors ratios
Earnings per share Re. 0.9 Rs. 1.8 Re. 0.75 Rs. 1.2

Dividend per share Re. 0.2 Re. 0.9 Re. 0.25 Re. 0.6

Required
(a) Draft a report to the board of directors, on behalf of the CFO, analyzing the financial
performance of Waris Limited by evaluating each category of ratios in comparison with the
industry.
(b) List any four types of additional information which would have helped you in a better
analysis.

29.6 OPAL INDUSTRIES LIMITED


Opal Industries Limited (OIL) is a listed company. As at 30 June 2014 OIL has various
investments as detailed under:

At the acquisition date


Investment Equity Share capital Retained
Company date held Cost (Rs. 100 each) earnings
--------------- Rs. in million ---------------
AL 1-Jul-2012 30% 50 80 60
BL 31-Dec-2011 10% 8 70 40
GL 1-Jan-2014 65% 195 150 95

© Emile Woolf International 118 The Institute of Chartered Accountants of Pakistan


Questions

Information pertaining to profit and dividend of the investee companies is as follows:

Profit/(loss) for the year ended Final cash dividend for year ended
Company 2014 2013
Rs. in million 2014 2013
AL 30 28 20% 16%
BL (10) 14 - 18%
GL 55 50 30% 15%

BL is a listed company and fair value of its shares as at 30 June 2014 was Rs. 110 per share
(2013: Rs. 160). OIL classifies investment in BL as available for sale.
AL and GL are private companies and market value of their shares is not available.
GL is the first subsidiary of OIL, since its incorporation. Following information pertains to OIL:

2013 2012
Rs. in million
Share capital (Rs. 100 each) 2,875 2,500
Profit for the year 1,260 1,100
Closing retained earnings balance 850 465
Final dividend - cash 25% 20%
- bonus issue - 15%
OIL’s profit for the year ended 30 June 2014 prior to taking effects of the transactions of its
investee companies was Rs. 1,450 million and it has announced a final cash dividend of 30%.
Required:
Prepare following for inclusion in the first separate financial statements of OIL for the year ended
30 June 2014 as required by the International Financial Reporting Standards.
(a) Movement in retained earnings for inclusion in the statement of changes in equity; and
(b) Note on investments.
(Show comparative figures and ignore taxation)

© Emile Woolf International 119 The Institute of Chartered Accountants of Pakistan


Advanced accounting and financial reporting

CHAPTER 30: SUNDRY STANDARDS AND INTERPRETATIONS

30.1 GUJRANWALA FOODS LIMITED


Gujranwala Foods Limited has the following information in its records for the year ended 31
January 2017.

Chickpea Apricot Dates Onion


Rs.’000 Rs.’000 Rs.’000 Rs.’000
Sales 200 300 750 250
Purchases 75 150 300 80
Subsidies 40 80 60 60
Own consumption 45 40 75 20
Opening Inventory 40 40 70 -
Closing Inventory 60 110 300 60

The farm expenses for the period 1 February 2016 to 30 April 2017 are as follows:

Rs.
Casual labour 20,000
Regular workers 30,000
Land preparation and clearing costs 80,000
Hire of tractors 60,000

The farm’s non- current assets for the year ended 31 January 2017 were as follows:
Rs.
Farm’s irrigation at cost 800,000
Farm’s implement and equipment 400,000

Additional Information:

(i) Farm’s irrigation costs are to be written off over 10 years


(ii) Farm’s implement and equipment were purchased on 31 April 2016 and these are to be
depreciated at 20% per annum.
Required
(a) Prepare Gujranwala Foods Limited’s gross output and statement of profit or loss for the
year ended 31 January 2017.
(b) In accordance with IAS 41 on Agriculture, you are required to define the following terms:
(i) Biological assets
(ii) Biological transformation
(iii) Harvest

© Emile Woolf International 120 The Institute of Chartered Accountants of Pakistan


Questions

30.2 WAH AGRIPROD LTD


The following trial balance was extracted from Wah Agriprod Ltd’s books at 31 December 2016:
Rs.’000 Rs.’000
Revenue 855,000
Purchases 503,600
Distribution costs 31,950
Administrative expenses 104,400
Finance costs 1,020
Investment income 18,250
Leased property at cost 125,000
Plant and equipment at cost 98,800
Accumulated amortisation/depreciation at 1/1/2016:
Leased property 35,000
Plant and equipment 28,800
Financial instruments 40,500
Inventory at 1/1/2016 85,075
Trade receivables 72,400
Trade payables 62,550
Bank 5,175
Share premium 8,400
Equity shares (Rs. 1 per share) 115,000
Retained earnings at 1/1/2016 55,600
Investment in subsidiary 131,030
6% Convertible Loan Notes 10,000
1,193,775 1,193,775
The following notes are relevant:
(i) Revenue includes goods sold and dispatched from 15 December 2016, on a 30-day right
of return basis. Their selling price was Rs. 3.6million and they were sold at a gross profit
margin of 20%. Wah Agriprod Ltd is uncertain as to whether any of these goods will be
returned within the 30-day period.
(ii) The directors decided to revalue the leased property in line with recent increases in market
values. On 1 September 2016, an independent surveyor valued the leased property at Rs.
99million, which the directors have accepted. The leased property was amortised over an
original life of 25 years which has not changed. Wah Agriprod Ltd does not make a transfer
to retained earnings in respect of excess amortisation.
(iii) The plant and equipment is depreciated at 15% per annum using the reducing balance
method and all depreciation and amortisation are charged to cost of sales. No
depreciation or amortisation has yet been charged on any non-current asset for the year
ended 31 December 2016.
(iv) The financial instruments are investments in equities of public companies and had a fair
value of N39.7million on 31 December 2016. There were no purchases or disposals of
any of these investments during the year. Wah Agriprod Ltd has not made the election in
accordance with IFRS 9 on Financial Instruments. The company adopts this standard
when accounting for its financial assets.
(v) On 20 November 2016, Wah Agriprod Ltd’s share price stood at Rs. 2.20 per share. On
this date, a dividend that was calculated to give a dividend yield of 5% was paid by the
company. The dividend paid was included as part of administrative expenses figure
shown in the trial balance.
(vi) The inventory on Wah Agriprod Ltd’s premises at 31 December 2016, after stock taking
was valued at cost of Rs. 106million and a provision for income tax for the year then ended
of Rs. 86.75million is required.

© Emile Woolf International 121 The Institute of Chartered Accountants of Pakistan


Advanced accounting and financial reporting

(vii) During the year, the company issued ten million shares at a premium of 20%. The
conversion rate for the loan note is Rs. 100 loan notes for three ordinary shares. The
current market price per share is Rs. 2.54.
Required
(a) Prepare the statement of profit or loss and other comprehensive income for the year ended
31 December 2016.
(b) Prepare the statement of changes in equity for the same period.
(c) Explain the term financial assets and state the FOUR classes of financial assets identified
in IAS 39 and how each is measured.
(d) What are biological assets? State any THREE conditions to be met before a biological
asset or agricultural produce can be recognised in the books of accounts.

30.3 HELIOS GROUP


Helios Ltd is an agricultural production company.
Helios Ltd acquired 70% of the ordinary shares of Sol Ltd, an agricultural based company for Rs.
600 million on 1 January 2015, when the reserves of Sol Ltd were Rs. 300 million. At the date of
acquisition the fair value of the non-controlling interest in Sol Ltd was Rs. 160 million.
The fair values of the net assets of Sol Ltd are the same as their carrying values with the
exception of a plot of agricultural land. This land was carried by Sol Ltd at its cost of Rs. 300
million. It was estimated at a fair value of Rs. 360 million.
Statement of financial position of Helios Ltd and Sol Ltd as at 31December 2016

Helios Ltd Sol Ltd


Rs.’000 Rs.’000
Assets
Non-current assets:
Property, plant and equipment 600,000 450,000
Investments 800,000 -

Current assets:
Inventories 160,000 150,000
Trade & other receivables 120,000 280,000
Cash and cash equivalent 20,000 50,000
Total assets 1,700,000 930,000
Equity and liabilities:
Ordinary share capital 160,000 120,000
Share premium 40,000 20,000
Reserves 590,000 500,000
Non-current liabilities:
Loan notes 600,000 170,000
Current liabilities
Trade & other payables 310,000 120,000
Total equity & liabilities 1,700,000 930,000

© Emile Woolf International 122 The Institute of Chartered Accountants of Pakistan


Questions

Additional information:
Immediately after acquisition, the following agricultural products were procured and included in
property, plant and equipment and inventories of Sol Ltd as at 31 December 2016:

(i) Included in property, plant and equipment of Sol Ltd are: Rs.’000
Dairy livestock – immature 40,000
Dairy livestock – mature 50,000

(ii) Included in inventories of Sol Ltd is:


Cotton plants 20,000

Required
(a) Prepare the consolidated statement of financial position for Helios Ltd group as at 31
December 2016 as expected for an agricultural business.
(b) State how to measure agricultural products harvested by an entity in line with the
requirements of IAS 41 on Agriculture.

30.4 FASHION BLUE ENTERPRISES


Fashion Blue Enterprises carries out business of readymade garments through large shops in the
major cities of Pakistan.
Its inventory ledger account balance at December 31, 2017 under the perpetual inventory system
was Rs. 73,410,000. The physical count revealed that the cost of inventory on hand was Rs.
71,400,000 only. Its owner Mr. Kaizer expected a small inventory shortfall due to damage and
petty theft, but considered this shortfall to be excessive.
On January 5, 2018, Kaizer carried out an investigation and discovered the following:
(i) Goods costing Rs. 300,000 were invoiced to Ebrahim Limited for Rs. 425,000 on
December 29, 2017 on FOB basis. The goods were actually dispatched to the customer on
January 2, 2018.
(ii) Included in the physical count were goods worth Rs. 200,000 which were held on behalf of
a third party.
(iii) Goods costing Rs. 410,000 purchased on credit from Mustafa & Co. were received on
December 28, 2017 and included in the physical count. However, the purchase had not
been recorded.
(iv) On December 23, 2017 goods costing Rs. 400,000 were purchased on credit from Mubina
Supplies, Faisalabad. The purchase was recorded on December 27, 2017 i.e. when the
goods were lifted by the transport company appointed by Mr. Kaizar, from the warehouse
of Mubina Supplies. The goods arrived on January 3, 2018.
(v) List of inventory at a shop situated in Sialkot had been under cast by Rs. 90,000.
(vi) On December 25, 2017 goods costing Rs. 310,000 were sold on credit to Skims Industries
for Rs. 500,000. The goods were shipped on December 28, 2017 and were received by
the customer on January 2, 2018.
(vii) Goods costing Rs. 2,500,000 had been returned to Ali Garments on December 30, 2017. A
credit note was received from the supplier on January 5, 2018 and entered in the books in
January 2018. No payment had been made for the goods prior to their return.
(viii) Goods sold to a customer Mr. Saleem were recorded in inventory ledger account at the
sale price of Rs. 780,000. The goods were sold at cost plus 30%.
Required
(a) Reconcile the ledger balance with the physical record to determine the shortage (if any).
(b) Determine the value of inventory that should be recorded in the statements of financial
position.

© Emile Woolf International 123 The Institute of Chartered Accountants of Pakistan


Advanced accounting and financial reporting

(c) Prepare the adjusting entries that should be recorded in the books of Fashion Blue
Enterprises, in December 2017.

30.5 KHAN LIMITED


Khan Limited closes its accounts on June 30 each year. The company was unable to take
inventory of physical inventory until July 14, 2017 on which date the physical inventory was
valued at Rs. 185,000. The following details are available in respect of the period July 1 to July
14, 2017:
(i) Payments against purchases amounted to Rs. 48,000 and included:
 Rs. 5,000 in respect of goods received on June 28, 2017;
 Rs. 6,000 in respect of goods received on July 18, 2017;
 Rs. 2,000 in respect of goods received and returned to supplier on the same date
i.e. July 7, 2017.
(ii) Collection against sales amounted to Rs. 60,000 and included:
 Rs. 1,500 in respect of goods which left the warehouse on June 29, 2017;
 Rs. 2,800 in respect of goods which were not dispatched until July 15, 2017;
 Rs. 760 in respect of goods invoiced and dispatched on July 10, 2017 but returned
by the customers on July 12. These were included in the inventory taken on July 14,
2017.
(iii) The rate of gross profit is 25% of selling price.
(iv) Goods costing Rs. 6,000 were purchased on June 28 but remained unpaid till July 24,
2017.
(v) An invoice amounting to Rs. 10,000 was raised on July 9, 2017 but remained uncollected
till July 14, 2017.
(vi) An item costing Rs. 9,000 which had been purchased on June 25, 2017 was damaged on
July 4, 2017. It can be repaired at a cost of Rs. 1,000 and sold for Rs. 7,000 and has been
taken in inventory at its net realizable value.
(vii) An inspection of the inventory count sheets prepared on July 14, 2017 showed that a page
total of Rs. 5,000 had been carried to the summary as Rs. 6,000. In addition, the total of
another page had been undercast by Rs. 200.
(viii) Included in the physical count were goods costing Rs. 2,200 which were held on behalf of
a supplier.
Required
Determine the amount of inventory required to be disclosed in the financial statements as at June
30, 2017.

30.6 AFRIDI
Afridi does not keep perpetual records of inventory. At the end of each quarter, the value of
inventory is determined through physical inventory. However, the record of inventory taken on 31
March 2017 was destroyed in an accident and Afridi has extracted the following information for
the purpose of inventory valuation:
(i) Invoices entered in the purchase day book, during the quarter, totalled Rs. 138,560 of
which Rs. 28,000 related to the goods received on or before 31 December 2016. Invoices
entered in April 2017 relating to goods received in March 2017 amount to Rs. 37,000.
(ii) Sales invoiced to customers amounted to Rs. 151,073 of which Rs. 38,240 related to
goods dispatched on or before 31 December 2016. Goods dispatched to customers before
31 March 2017 but invoiced in April 2017 amounted to Rs. 25,421.

© Emile Woolf International 124 The Institute of Chartered Accountants of Pakistan


Questions

(iii) Credit notes of Rs. 12,800 had been issued to customers in respect of goods returned
during the period.
(iv) Purchases included Rs. 2,200 spent on acquisition of a ceiling fan for the shop.
(v) A sale invoice of Rs. 5,760 had been recorded twice in the sales day book.
(vi) Goods having sale value of Rs. 2,100 were given by way of charity.
(vii) Afridi normally sells goods at a margin of 20% on cost. However, he had allowed a special
discount of 10% on goods costing Rs. 6,000 which were sold on 15 February 2017.
(viii) On 31 December 2016, the inventory was valued at Rs. 140,525. However, while
reviewing these inventory sheets on 31 March 2017 the following discrepancies were
found:
(a) A page total of Rs. 15,059 had been carried to the summary as Rs. 25,059.
(b) 1,000 items costing Rs. 10 each had been valued at Rs. 0.50 each.
Required
Calculate the amount of inventory in hand as on 31 March 2017.

© Emile Woolf International 125 The Institute of Chartered Accountants of Pakistan


Advanced accounting and financial reporting

CHAPTER 31: IFRS 1: FIRST TIME ADOPTION OF IFRS

31.1 IFRS 1
A company which has always prepared its Financial Statements to 31 December each year,
prepared its first IFRS Financial Statements for the year ended 31 December 2016. These
statements show comparative figures for the year ended 31 December 2015.
Required
(a) Identify the first IFRS reporting period and state the date of transition to IFRS.
(b) Present the procedures which must be followed in order to prepare the financial
statements for the year ended 31 December 2016.
(c) Identify the reconciliations which the company must include in its financial statements for
the year ended 31 December 2016.
(d) State the contents of a typical statement of changes in equity.

© Emile Woolf International 126 The Institute of Chartered Accountants of Pakistan


Questions

CHAPTER 32 – SPECIALISED FINANCIAL STATEMENTS

32.1 IFRS FOR SMES


International Financial Reporting Standards (IFRS) are unnecessary impositions on developing
countries with preponderance of small and medium entities (SME). Some IFRS are
unnecessarily demanding and might be difficult for these SMEs to meet. Some of the information
produced by adopting IFRS are not necessary for users of SME Financial Statements.
Required
Discuss the following:
(a) The need to develop a set of IFRS specifically for SMEs.
(b) The need for the modification of existing IFRS to meet the needs of SMEs.
(c) How items not dealt with by an IFRS for SMEs should be treated.

32.2 AKMAL GENERAL INSURANCE LIMITED


Akmal General Insurance Limited (AGIL) is engaged in general insurance business. The
following information is available for the year ended December 31, 2016:
(i) During the year, AGIL earned direct and facultative premiums of Rs. 5,586,382
thousand against which it incurred reinsurance expense amounting to Rs. 2,076,499
thousand. Details of premium earned and reinsurance expenses are as follows:

Fire & Marine, Motor Misc.


Property Aviation
Damage &Transport

Rupees in thousand

Premiums 1,905,027 883,942 2,495,120 302,293

Reinsurance expense 1,520,962 300,605 4,671 250,261

(ii) The outstanding balance of unearned premium reserve and prepaid reinsurance
premium ceded were as follows:

Fire & Marine,


Property Aviation &
Damage Transport Motor Misc.

Rupees in thousand

Balances as of
December 31, 2016

Unearned premium reserve 1,014,552 174,780 1,053,094 152,911

Prepaid reinsurance premium


ceded 741,934 93,702 311 122,866
Balances as of
December 31, 2015

Unearned premium
reserve 844,425 159,844 1,191,933 133,424

Prepaid reinsurance premium


ceded 726,800 59,098 - 114,190

© Emile Woolf International 127 The Institute of Chartered Accountants of Pakistan


Advanced accounting and financial reporting

(iii) Premium received under the treaty arrangements (proportional) amounted to Rs. 167,108
thousand. The outstanding balance of unearned premiums reserve relating to treaty
arrangement as of December 31, 2016 was Rs. 56,128 thousand (2015: Rs. 61,303
thousand).
Required
Prepare the statement of premiums for the year ended December 31, 2016. Ignore the
corresponding figures.

32.3 MAHFOOZ GENERAL INSURANCE LIMITED


Mahfooz General Insurance Limited (MGIL) is a listed company. The information pertaining to
the business underwritten inside Pakistan for the year ended June 30, 2016 is as under:
Direct and facultative Treaty
Fire & Marine
property aviation & Accident
damage transport Motor & health Proportional
Rupees in million
Claims:
Total claims paid 900 450 1,150 250 13
Outstanding - Opening 600 400 900 300 10
Outstanding - Closing 500 450 750 150 12
Reinsurance and other
recoveries:
Total received 600 300 850 160 -
Outstanding - Opening 500 300 700 150 -
Outstanding - Closing 350 400 550 80 -
Required
Prepare a statement of claims for the year ended June 30, 2016 in accordance with the
Insurance Ordinance, 2000. Ignore the comparative figures.

32.4 DEE GENERAL INSURANCE LIMITED


Dee General Insurance Limited is a listed company. The following information relates to the
year ended 31 December 2016:
Direct and facultative Treaty
Marine
Fire and
aviation Miscellaneo
property Motor Proportional
and us
damage
transport
(Rs. In million)
Commissions:
Paid / payable 321.41 126.87 215.00 90.94 0.30
Deferred: opening 148.79 11.31 128.50 38.59 -
Deferred: closing 160.43 5.68 114.23 35.17 -
Receipts from
reinsurers 270.44 5.70 12.72 82.40 -
Net premium earned 907.75 768.70 2,745.64 948.48 0.70
During the year, management expenses (other than commission) amounted to Rs. 978
million. These expenses are allocated on the basis of net premium earned.
Required
Prepare a statement of expenses for inclusion in the financial statements for the year ended 31
December 2016. (Ignore comparative figures)

© Emile Woolf International 128 The Institute of Chartered Accountants of Pakistan


Questions

32.5 BANK LATEEF BANK LIMITED


Lateef Bank Limited (LBL) is listed on Karachi and Lahore Stock Exchanges and has 150
branches including 10 overseas branches. The LBL’s lending to financial institutions as of
September 30, 2016 comprised of the following:
(i) Call money lending at year end amounted to Rs. 850 million (2015: Rs. 1,200 million).
The markup on these unsecured lendings ranged between 15% to 17% (2015: 10% to
12%) and they matured on various dates, in October 2016.
(ii) Short term lending on account of repurchase agreement (reverse repo) amounted to Rs.
2,100 million (2015: Rs. 2,850 million). These carried markup ranging from 9.5% to
13.2% (2015: 8% to 10.5%) and matured on various dates, in October 2016. These
were secured against Market Treasury Bills of Rs. 1,650 million (2015: Rs. 1,850 million)
and Pakistan Investment Bonds of Rs. 450 million (2015: Rs. 1,000 million). The market
value of these securities held as collateral, on September 30, 2016, amounted to
Rs. 2,250 million (2015: Rs. 2,930 million).
The above amounts include lendings in foreign currencies amounting to Rs. 110
million (2015: Rs. 150 million).
Required
Prepare a note on lendings to financial institutions for inclusion in LBL’s financial
statements for the year ended September 30, 2016 giving appropriate disclosures in accordance
with the guidelines issued by State Bank of Pakistan.

32.6 SECURED BANK LIMITED


Secured Bank Limited (SBL) is listed on all the Stock Exchanges in Pakistan. The cost of
various types of Investments held by the bank as of December 31, 2016 are as follows:

2016 2015
Rupees in million
Market treasury bills 366 309
Pakistan investment bonds 69 61
Government of Pakistan bonds (USD/Euro) 26 30
Investments in associates 9 8
Fully paid-up ordinary shares – listed 6 5
Fully paid-up ordinary shares – unlisted 2 3
Corporate debt instruments – listed 19 30
Corporate debt instruments – unlisted 260 210
Investments of mutual funds 32 28
Overseas government securities 60 52
Other investments 19 29

Provision for diminution / impairment in the value of investments as at January 1, 2015


amounted to Rs. 28 million. Other information relevant to the provision is as under:

Impairment (reversal) / loss for the year (6) 2


Charge for the year 17 12
Amounts written off during the year 5 3
Required
Prepare a note on ‘investments by segments’ for inclusion in SBL’s financial statements for the
year ended December 31, 2016 giving appropriate disclosures in accordance with the
guidelines issued by the State Bank of Pakistan.

© Emile Woolf International 129 The Institute of Chartered Accountants of Pakistan


Advanced accounting and financial reporting

32.7 AL-AMIN BANK LIMITED


Al-Amin Bank Limited is listed on all the stock exchanges in Pakistan. At year end, the
total advances amounted to Rs 75,000 million which include non-performing advances of
Rs. 5,000 million. The break-up of the non-performing advances and the provisions there-against
is as under:
Other
assets
especially Sub-
mentioned Standard Doubtful Loss Total
Rs. in million
Advances 100 660 840 3,400 5,000
Provisions required and held 5 120 530 3,345 4,000
The sub-standard category includes advances of Rs. 260 million pertaining to overseas
operations of the bank. The required provision of Rs. 50 million has been made against such
advances.
During the year the movement in the specific provision was as under:
Rs. in million
Opening balance 3,320
Charge for the year 802
Reversals (90)
Amounts written off (50)
Exchange rate adjustment 18
Total 4,000

In addition to the above specific provisions, it is the bank’s policy to make additional
general provision based on the judgment of the bank. The opening balance for general provision
was Rs. 65 million. During the year, the bank made provisions of Rs. 25 million and Rs. 15 million
against consumer and agriculture advances respectively.
Required
Prepare relevant notes on non-performing advances and provisions for inclusion in the
financial statements of Al-Amin Bank Limited giving appropriate disclosure in accordance with the
guidelines issued by the State Bank of Pakistan.

32.8 BLUE-CHIP ASSET MANAGEMENT LIMITED


Blue-chip Asset Management Limited is in the process of finalizing the financial statements of
one of its open ended mutual funds. The following information is available from the fund’s
records;
Rs. in
000
Net assets - opening balance 350,050
Net income for the year 65,325
765,900 units issued during the year against 85,015
717,480 units redeemed during the year against 77,488
The par value of each unit is Rs. 100.
Required
Prepare the statement of movement in unit holders’ Fund for the year ended June 30, 2016.

© Emile Woolf International 130 The Institute of Chartered Accountants of Pakistan


Questions

32.9 A-ONE ASSET MANAGEMENT FUND LIMITED


Following information has been extracted from the records of A-One Asset Management
Fund Limited for the year ended March 31, 2016.

Rs. in
million
Net assets at the beginning of the year (900 million units) 27,000
100 million units issued during the year 3,500
95 million units redeemed during the year 3,277

Investments classified as ‘available for sale’


- Fair value at year end 1,800
- Carrying value at year end 1,200
- Net unrealized appreciation in fair value of investments at the
beginning of the year 480

Investments classified as ‘at fair value through profit or loss - held for
trading’
- Fair value at year end 2,500
- Carrying value at year end 2,200

Element of income and capital gains included in prices of units


issued/redeemed and transferred to income statement 173
- Capital gains 400
- Other net income for the year 3,000

Final distribution for the year ended March 31, 2016 of Rs. 5.00 per unit (2015: Rs. 4.00 per
unit) was announced on April 16, 2016.
Required
Prepare a statement of movement in unit holders' fund for the year ended March 31, 2016.

32.10 IAS 26
IAS 26: Accounting and Reporting by Retirement Benefit Plans and IAS 19: Employee Benefits
deal with employee benefits but there are differences between the two standards.
(a) Highlight the main differences between IAS 26 and IAS 19.
(b) What is a Defined Benefit Plan?
(c) What is a Defined Contribution Plan?
(d) Explain the meaning of the actuarial present value of promised retirement benefit.

32.11 SOGO LIMITED


SOGO Limited operates an approved funded gratuity scheme for all its employees.
Benefits under the scheme become vested after 5 years of service. No benefit is payable to an
employee if he leaves before 5 years of service. A total of 752 employees were eligible for the
benefits under the fund as of December 31, 2016.

© Emile Woolf International 131 The Institute of Chartered Accountants of Pakistan


Advanced accounting and financial reporting

Following is the trial balance of the Fund as of June 30, 2016:


Debit Credit
Amounts in Rupees
Cash at bank - current account 17,930,120
Receivable from SOGO Limited 1,147,150
Defence Savings Certificate 102,133,664
Term Finance Certificates 11,832,089
Term Deposits 6,414,058
Investment – SUN Limited 17,594,893
Investment – PEACE Company Limited 587,169
Investment - NIT Units 16,911,510
Due to outgoing members 4,301,017
Accrued expenses 3,822
Withholding tax payable 61,251
Members Fund 142,472,122
Profit on investments 23,389,251
Dividend income 2,696,399
Contribution for the year 10,623,106
Transferred / paid to outgoing members 12,432,973
Bank charges 3,342
Audit fee 10,000
Liabilities no more payable 3,450,000
186,996,968 186,996,968
Following are the details of investments and income thereon:
Balance as During the year 2016
at Profit / Profit /
Principal
July 01, Addition interest interest
realized
2015 accrued realized
Government
Securities
Defence Savings
-
Certificate 87,812,855 21,376,809 (1,600,000) (5,456,000)
Unlisted Securities
and deposits
Term Finance (12,873,06
19,943,656 5,000,000 1,655,223 (1,893,722)
Certificates 8)
Term Deposits 11,584,631 - 357,219 (5,300,000) (227,792)
Listed Securities
SUN Limited 8,220,957 9,373,936 - - -
PEACE Limited 587,169 - - - -
NIT Units 16,911,510 - - - -
The following gains/(losses) on restatement of investments at their fair values, have not been
accounted for:
Rupees
SUN Limited (784,518)
PEACE Limited 317,728
NIT Units 4,026,551
Required
Prepare the following in accordance with the requirements of International Accounting
Standards:
(a) Statement of net assets available for benefits along with the note on investments.
(b) Statement of changes in net assets available for benefits.

© Emile Woolf International 132 The Institute of Chartered Accountants of Pakistan


Questions

CHAPTER 36: ETHICAL ISSUES IN FINANCIAL REPORTING

36.1 ETHICAL ISSUES


Waheed is a chartered accountant, recently employed by AA Limited as deputy to the finance
director, Arif (also a chartered accountant). AA Limited is listed on the Lahore stock exchange.
On Waheed’s first day on the job he met with Arif who said ‘Look, keep it to yourself but I’m
having a second interview next week for a new job. The first thing that I need you to do is to
review the financial statements before the auditors arrive. I qualified a few years ago and am not
up to date on all of the little technicalities in IFRS. You should know these better than me and
you’ll know more about what the auditors might focus on. We must do our best to present the
financial statements in the most favourable light as the bonus paid to employees (including me)
depends on profit being more than 10% higher than last year’s and remember that you qualify
for this too. Keep this in mind when you carry out the review as we do not really want to find
anything. Do well at this and I might put in a good word for you when I leave as I’m sure you’ll be
a great replacement for me.”
Required
Explain the ethical issues inherent in the above conversation and what Waheed should do about
them.

36.2 SINDH INDUSTRIES LTD


Jafar has recently been appointed as financial controller to Sindh Industries Ltd. Until a month
ago, Sindh Industries had a finance director, who resigned suddenly, due to ill health. Since Jafar
joined the company, he has learned that his resignation was related to stress caused by a series
of disagreements with the managing director about the performance of the business. The
directors have not yet appointed a replacement.
It is now March 2017 and you have been asked to finalise the financial statements for the year
ended 31 December 2016. The draft statement of profit or loss extract and statement of financial
position are shown below:
Draft statement of profit or loss for the year ended 31 December 2016
Rs.000
Profit before tax 2,500
Draft statement of financial position at 31 December 2016
Rs.000
Property, plant and equipment 12,000
Current assets 3,500
Total assets 15,500

Share capital 2,000


Retained earnings 6,000
Equity 8,000
Non-current liabilities 5,000
Current liabilities 2,500
Total equity and liabilities 15,500
During the year ended 31 December 2016 Sindh Industries entered into the following
transactions.
(1) Just before the year end Sindh Industries signed a contract to deliver consultancy services
for a period of 2 years at a fee of Rs. 500,000 per annum. The full amount of this fee has
been paid in advance and is non-refundable.
(2) Sindh Industries has constructed a new factory. The construction has been financed from
the pool of existing borrowings. Land at a cost of Rs. 1.8 million was acquired on 1
February 2016 and construction began on 1 June 2016. Construction was completed on

© Emile Woolf International 133 The Institute of Chartered Accountants of Pakistan


Advanced accounting and financial reporting

30 September 2016 at an additional cost of Rs. 2.7 million. Although the factory was
usable from that date, full production did not commence until 1 December 2016.
Throughout the year the company’s average borrowings were as follows:
Annual
interest
Amount rate
Rs. %
Bank overdraft 1,000,000 9.75
Bank loan 1,750,000 10
Debenture 2,500,000 8
An amount of Rs. 450,000 has been included in property, plant and equipment in respect
of borrowing costs relating to the construction of the factory. The useful life of the factory
has been estimated at 20 years. No depreciation has been charged for the year. The
reason for this is that the factory has only been in use for one month and that the
depreciation charge would be immaterial.
(3) A blast furnace with a carrying amount at 1 January 2016 of Rs. 3.5 million has been
depreciated in the draft financial statements on the basis of a remaining life of 20 years. In
December 2016 the directors carried out a review of the useful lives of various significant
items of plant and machinery, including the blast furnace and came to the conclusion that
the useful life of the furnace was 20 years at 31 December 2016. The reasoning behind
this judgement was that the lining of the furnace had been replaced in the last week of
December 20X6 at a cost of Rs. 1.4 million. Provided that the lining is replaced every five
years, the life of the furnace can be extended accordingly. You have found a report,
commissioned by the previous finance director and prepared by a firm of asset valuation
specialists, which assesses the remaining useful life of the main structure of the furnace at
1 January 2016 at 15 years and the lining of the furnace at 5 years. You have also found
evidence that the managing director has seen this report.
Jafar has had a conversation with the managing director who told him, “We need to make
the figures look as good as possible so I hope you’re not going to start being difficult. The
consultancy fee is non-refundable so there’s no reason why we can’t include it in full. I
think we should look at our depreciation policies. We’re writing off our assets over far too
short a period. As you know, we’re planning to go for a stock market listing in the near
future and being prudent and playing safe won’t help us do that. It won’t help your future
with this company either.”
Required
(a) Explain the required IFRS accounting treatment of these issues, preparing relevant
calculations where appropriate.
(b) Prepare a revised draft of the statement of profit or loss extract for the year ended
31 December 2016 and the statement of financial position at that date.
(c) Discuss the ethical issues arising from your review of the draft financial statements and the
actions that you should consider.

36.3 SOHAIB AND OMAR


Sohaib is student member of ICAP who is will sit his finals in the next few months.
One of his friends, Omar, has asked him for advice as he is planning to start up a business that
provides plant care services to firms occupying upmarket office accommodation.
Omar wants to assess the profitability and financial position of two similar businesses in order to
prepare his business plan.
These two businesses are called, Eden Ltd (Eden) and Gethsemene Ltd (Gethsemene), both
unlisted companies. Omar has asked Sohaib to write a report analysing the two companies' most
recent financial information.
Omar is short of money and because of their friendship Sohaib has agreed to undertake the work
for no fee. The Eden financial statements have been filed with the regulatory authorities.

© Emile Woolf International 134 The Institute of Chartered Accountants of Pakistan


Questions

However, the Gethsemene statements have not yet been filed, and the documents provided to
Sohaib by Omar are stamped 'Draft: strictly private and confidential'. Omar recently completed a
work placement at Gethsemene. When Sohaib asks him for further information, Omar replied “I
obtained these financial statements quite legitimately, and in any case, I am sure they will
become filed very soon. Anyway you are only a student member and I am not paying you so
what is the fuss'.
Required
Advise Sohaib about the ethical issues arising in respect of his work for Omar, referring, where
appropriate, to the ICAP's Code of Ethics, and explaining any action he should take.

36.4 ABBAS AND BASHIR


Nourish Limited is an unlisted company manufactures and transports food products to its
customers (supermarkets and stores) throughout Pakistan.
In June 2018 Nourish Limited was awarded a five-year contract to supply BigShop Limited, one
of the Pakistan's leading supermarket groups. The successful bid for the new contract has
involved Nourish Limited in substantial amounts of capital investment, funded principally by bank
borrowing.
Insight Ltd is a firm of multi-disciplinary consultants. Nourish Limited has engaged Insight Ltd to
provide advice on business planning and logistics. Bashir, a logistics specialist employed by
Insight Ltd, has been assigned as project leader.
Bashir has asked Abbas, a member of his team, to produce a preliminary report on Nourish
Limited's most recent financial statements, for the year ended 31 December 2016. Abbas is a
recently qualified chartered accountant.
BigShop Limited also uses the consulting services of Insight Ltd. Both Bashir and Abbas have
worked on consulting projects commissioned by BigShop Limited, and are likely to do so again in
the future.
Abbas is the only chartered accountant on his team (in fact, Insight Ltd employs few other
chartered accountants) and is concerned that there may be a potential conflict of interest
between his work on Nourish Limited and BigShop Limited projects. He is not sure whether such
conflicts are covered by the ICAP's ethical guidance.
Required
Advise Abbas about his concerns over the potential conflict of interest arising from his work for
Nourish Limited and BigShop Limited, referring where appropriate to the ICAP's Code of Ethics,
and explain any action he should take.

© Emile Woolf International 135 The Institute of Chartered Accountants of Pakistan


Advanced accounting and financial reporting

© Emile Woolf International 136 The Institute of Chartered Accountants of Pakistan


Certified Finance and Accounting Professional

B
Advanced accounting and financial reporting

SECTION
Answers
CHAPTER 1 – REGULATORY FRAMEWORK

1.1 GENERAL PURPOSE FINANCIAL STATEMENTS


(a) A general purpose financial statement is a statement that is intended to meet the needs of
users who are not in a position to demand information that are tailored to their needs.
Such information is useful to existing and potential investors.
(b) Reasons for the prevalence of IFRS in many countries
(i) Production of high quality financial statements
It involves the preparation of financial statements that have qualitative features, that
is, faithful representation, error free, neutrality, understandability etc.
(ii) Preparation of user-friendly financial statements
Production of financial statements that contains necessary information that will
assist users of financial statements to make crucial economic decisions.
(iii) Uniformity of financial statements prepared
Financial accounting is a language of business. This language must not be different
across countries of the world for it to assist international investors.
(iv) Access to international finances
With a General Purpose Financial Statements, it would be easier for multi-national
entities to have more access internationally.
(v) Enhancement of major economic decisions
High quality financial statements will assist users of the statements to make
informed and important financial decisions.
(vi) Comparability of financial statements
General purpose financial statements enhance comparability of financial information
among similar industries.
(vii) Globalisation and integration
With IFRS, Chartered Accountants become more mobile, since the standards are
the same across the countries.
(viii) Job Creation
Jobs are created across countries and a Chartered Accountant in Pakistan can
practice in other countries with convenience.

© Emile Woolf International 137 The Institute of Chartered Accountants of Pakistan


Advanced accounting and financial reporting

(ix) Others
Saving of time and money
Promotion of Regional trade
Easier accounting and auditing practices
(c) Arguments for and against financial reporting standards
Arguments for:
(i) They guide preparers and users of financial statements
(ii) Their use enhances objectivity and comparability of financial statement which would
in turn engender reliability
(iii) Their use helps to curtail or significantly narrow the divergence in the principles
adopted by preparers of financial statements.
(iv) Standards improve the uniformity of treatment of transactions in the financial
statements among companies thereby increasing the credibility and comparability of
financial statement.
(v) Standards compel organisations to disclose information which they may not want to
disclose had the standards not been in existence.
(vi) Standards reduce the number of choices in the methods used to prepare financial
statement thereby reducing the risk of creative accounting.
(vii) Foreign companies which are targets for takeovers or mergers can be more easily
evaluated.
Arguments against:
(i) The cost of setting up and maintaining a standard-setting apparatus is quite
significant and not all countries can afford it.
(ii) The standards cannot address all issues or transactions. There are some which are
unique and so rare/unusual that global standards are not and cannot be available for
them.
(iii) Low level of details or explanations.

© Emile Woolf International 138 The Institute of Chartered Accountants of Pakistan


Answers

CHAPTER 2 – ACCOUNTING AND REPORTING CONCEPTS

2.1 DEFINITIONS
An asset is:
 a resource controlled by the entity
 as a result of past events
 from which future economic benefits are expected to flow to the entity.
A liability is:
 a present obligation of the entity
 arising from past events
 the settlement of which is expected to result in an outflow of economic benefits.
Income includes both revenue and gains e.g. sales, fees, interest, dividend, royalty and rent.
Revenue is income arising in the course of the ordinary activities of the entity such as sales
revenue and income from investments.
Gains include, for example, the gain on disposal of a non-current asset. They might arise in the
normal course of business activities. They might be realised or unrealised. Unrealised gains
occur whenever an asset is revalued upwards, such as the upward revaluation of marketable
securities.
Expenses include:
 expenses arising in the normal course of activities, such as the cost of sales and other
operating costs, including depreciation of non-current assets.
 losses, including, for example, the loss on disposal of a non-current asset, and losses
arising from damage due to fire or flooding.
Appraisal of statement of financial position as all that is required
The statement of financial position does show the position of a business at a point in time (like a
snapshot), but by itself is insufficient to give a comprehensive view of performance and/or
adaptability.
The IASB Framework states that information on financial performance is provided by the
statement of profit or loss and other comprehensive income. This is because the statement of
financial position fails to give any account of transactions leading up to the statement of position.
It is the statement of profit or loss and other comprehensive income and the SOCIE that show the
performance of a business in a given period and reconcile the opening and closing statements of
financial position.
Information on financial adaptability is given primarily by the statement of cash flows. This is
because financial adaptability is the ability to take effective action to alter the amount and timing
of cash flows. Some information comes from the statement of financial position (e.g. the note
about future finance lease commitments) but the statement of financial position is by no means
‘all that is required’.
‘Statement of profit or loss is a superfluous statement’
Although income (revenue and gains) and expenses (and losses) would be reflected in an
increase/decrease in the assets and liabilities in the statement of financial position, the volume
and type of income generated would give a better indication of company performance. It is the
statement of profit or loss and other comprehensive income that provides such detailed
information; without it the performance of the business cannot be properly evaluated.

© Emile Woolf International 139 The Institute of Chartered Accountants of Pakistan


Advanced accounting and financial reporting

2.2 CONCEPTUAL FRAMEWORK


(a) (i) Conceptual framework is a constitution, a coherent system of interrelated objectives
and fundamentals that can lead to consistent standards and prescribes the nature,
functions and limits of financial accounting and financial statements.
(ii) It enables certain critical issues to be addressed.
(iii) It also facilitates the development of accounting standards and Generally Accepted
Accounting Practice (GAAP.),in accordance with the principles and underlying
assumptions of the concepts.
(iv) It also promotes consistency in the application of accounting principles and policies.
Without conceptual framework there will not be a common definition of the elements
of financial statements (asset, liabilities, income and expenses). For example, in
line with the framework for preparation and presentation of financial statements
gives a precise definition of assets and liabilities and no expenditure on these could
be recognized unless such expenditure meets the definition of the framework.
(b) Assets
Assets are resources controlled by an entity as a result of past events and from which
future economic benefits are expected to flow to the entity
Liabilities
Liabilities are present obligations of the entity arising from past events, the settlement of
which, are expected to result in an outflow from the entity of resources embodying
economic benefit.
(c) (i) Materiality
An item is considered material if its inclusion, omission or misstatement will have a
fundamental effect on the financial statements as a whole and affect the economic
decision of the user. The requirement of the framework, is that items which are
material in nature to an entity should be accorded separate recognition, presentation
and disclosure, while those that are immaterial (small and separate unimportant)
should be aggregated or added up.
(ii) Comparability
This implies that the financial statements of a given year should have relative figure
of the past period or periods. This helps to evaluate the performance of the entity
and trend analysis over time. It also helps to assess the comparability on consistent
application of accounting policies over time.
(iii) Faithful Representation
Financial information must faithfully represent what it purports to represent. Faithful
representation has three characteristics:
a) completeness i-e, all information necessary for a user to understand the
phenomenon being depicted, including all necessary description and
explanations;
b) Neutrality i-e. a neutral depiction is without bias in the selection or presentation
of financial information;
c) freedom from error but does not mean to be accurate in all respects
(d) (i) Lenders
The lenders are concerned with the ability of the company to pay the finance cost on
the borrowed fund and pay the loan when due. They are also interested in the
availability of assets to secure loans.
(ii) Suppliers
The suppliers are interested in information that indicates that their debts can be paid
by the entity and that the entity will continue as a going concern in order to ensure
continued patronage.

© Emile Woolf International 140 The Institute of Chartered Accountants of Pakistan


Answers

(iii) Customers
The customers are interested in information relating to the entity’s continued
existence, especially for those that depend on the entity to meet their daily needs.
(iv) Employees
The employees are concerned with their job security and the company’s ability to be
profitable, in order to guarantee the payment of their salaries in the future.
(v) Government and their agencies
Government and their agencies are interested in information relating to taxes,
regulations, resource allocation and evaluation of government policies on
businesses.

2.3 CARRIE
(a) (b)
Physical Financial Capital Maintenance
Capital
Maintenance
(i) Historical (ii) Constant
cost purchasing
accounting power
Profit for the year accounting
Rs. Rs. Rs.
Sales 1,400 1,400 1,400
Cost of sales (1,000) (1,000) (1,000)
Inflation adjustment
- Specific
(1,100 – 1,000) (100) - -
- General
(1,000  7%) - - (70)
––––– ––––– –––––
Profit 300 400 330
––––– ––––– –––––
Balance sheet as at
31 December Year 1
Cash at bank 1,400 1,400 1,400
––––– ––––– –––––
Share capital
(1,000 + 100)
(1,000 + 70) 1,100* 1,000 1,070*
Reserves 300 400 330
––––– ––––– –––––
1,400 1,400 1,400
––––– ––––– –––––
Tutorial note
Share capital at the year end is restated under the physical capital maintenance concept for an
increase in specific price changes and under CPP accounting for general price changes. This is
the other side of the entry to the inflation adjustments in the statement of profit or loss

© Emile Woolf International 141 The Institute of Chartered Accountants of Pakistan


Advanced accounting and financial reporting

CHAPTER 3 – PRESENTATION OF FINANCIAL STATEMENTS

3.1 CLIFTON PHARMA LIMITED


(a) Statement of profit or loss for the year ended 30 September 2016
Rs.in ‘000
Revenue 338,300
Cost of sales (working 1) (180,000)
Gross profit 158,300
Operating expenses (working 2) (36,600)
Investment income 2,000
Loan notes (working 3) 3,000
Leases (working 2) 1,700
Finance costs (4,700)
Profit before tax 119,000
Income tax expense (working 4) (21,000)
Profit for the period 98,000

(b) Statement of financial position as at 30 September 2016


Non-current assets Rs.in ‘000
Property, plant and equipment (working 5) 358,000
Investments 92,400
450,400
Current assets
Inventory 23,700
Trade receivables 76,400
Bank 12,100
112,200
Total assets 562,600
Equity and liabilities
Capital and reserves
Share capital 280,000
Share premium 20,000
Retained earnings (working 6) 117,300
417,300
Revaluation surplus 20,000
Non-current liabilities
3% loan notes (working 3) 51,500
Deferred tax (working 4) 23,000
Lease liability (working 2) 11,700
86,200
Current liabilities
Trade payables 14,100
Accrued lease finance costs (working 2) 1,700
Lease liability (working 2) 5,300
Income tax payable 18,000
39,100
Total equity and liabilities 562,600

© Emile Woolf International 142 The Institute of Chartered Accountants of Pakistan


Answers

Workings

(1) Cost of sales Rs.in ‘000


As given in the trial balance 134,000
Depreciation of plant and equipment: 20%  (197,000 – 47,000) 30,000
Depreciation of leased vehicles: 24,000/4 years 6,000
Amortisation of leasehold property: 250,000/25 years 10,000
180,000

(2) Operating expenses Rs.in ‘000


Rental costs given in the trial balance 8,600
Relating to lease (7,000)
Balance – operating expense 1,600
Other operating expenses (trial balance in question) 35,000
Total operating expenses 36,600

Lease Liability Rs.in ‘000


Fair value of leased assets 24,000
Less: First rental payment, paid in advance (7,000)
1 October 2015
Remaining obligation, 1 October 2015 17,000
Interest at 10% to 30 September 2016 (current) 1,700
Lease payment due 1 October 2016 7,000
Capital repayment due (balance, current) (5,300)
Remaining lease liability i-e. non-current 11,700

(3) Loan notes


The effective interest rate is 6%. Actual interest paid was Rs. 1,500,000 (in trial balance);
therefore the balancing Rs. 1,500,000 should be added to the loan notes obligation, to
make the total loan notes liability Rs. 50 million + Rs. 1,500,000 = Rs. 51.5 million.

(4) Taxation

Rs.in ‘000
Deferred tax liability b/f 20,000
Deferred tax: debit in the statement of profit or loss 3,000
Deferred tax liability c/f (92,000  25%) 23,000

Tax expense Rs.000


Income tax on profits for the year 18,000
Deferred tax movement 3,000
Tax charge in the statement of profit or loss 21,000

© Emile Woolf International 143 The Institute of Chartered Accountants of Pakistan


Advanced accounting and financial reporting

(5) Non-current assets and depreciation


Leasehold property Rs.in ‘000
Carrying value in the trial balance (250,000 – 40,000) 210,000
Amortisation charge for the year to 30 September 2016 (10,000)
200,000
Revalued amount 220,000
Transfer to revaluation surplus 20,000

The annual depreciation charges for plant and equipment and the leased vehicles are
shown in workings (1)
Cost or Accumulated Carrying
valuation depreciation amount
Rs.in ’000 Rs.in ’000 Rs.in ’000
Leasehold property 220,000 0 220,000
Plant and equipment 197,000 77,000 120,000
(non-leased)
Leased vehicles 24,000 6,000 18,000
441,000 83,000 358,000

(6) Retained profits Rs.in ‘000


At 1 October 2015 (trial balance) 19,300
Profit for the year 98,000
Retained profits at 30 September 2016 117,300

3.2 BSZ LIMITED


Statement of financial position as at June 30, 2016
Note Rs.in
million
ASSETS
Fixed Assets
Property, plant & equipment 1 576
Intangible assets 2 8
584

Long term advances – considered good 4

Current assets
Stocks in trade 90
Accounts receivable 3 57
Advances, deposits, prepayments and other
receivables 4 45
Cash at banks 5 29
221
809

© Emile Woolf International 144 The Institute of Chartered Accountants of Pakistan


Answers

Rs.in
million
EQUITY AND LIABILITIES
Share capital and reserves
Authorized share capital
50,000,000 shares of Rs. 10 each 500

Issued, subscribed and paid up capital


40,000,000 shares of Rs. 10 each 400
Unappropriated profit 65
465
Surplus on revaluation of fixed assets 120
Non-current liabilities
Deferred taxation 40
Current liabilities
Short term loan 85
Account and other payables 6 82
Provision for taxation 17
184
809

Rs.in
Notes million
1. Property, plant and equipment
Operating assets 556
Capital work in progress – building 20
576

1.1 Operating assets Rs.in million


Cost/revalued Freehold
amount land Building Machines Fixtures Total
As of July 01 2010 375.0 130.0 100.0 19.0 624.0
Additions - - - 8.0 8.0
Disposals - - (15.0) - (15.0)
As at June 30 2016 375.0 130.0 85.0 27.0 617.0

Accumulated
depreciation
As of July 01 2010 - 19.5 22.5 5.9 47.9
For the year - 6.5 18.1
(105 × 85) + 10% × 9.5
8
15 × /12)
(105 × 19) + 10% × 2.1
3
8 × /12)
Disposals - - (5.0) - (5.0)
As at June 30 2016 - 26.0 27.0 8.0 61.0
Carrying amount 375.0 104.0 58.0 19.0 556.0
Depreciation rate - 5% 10% 10%

© Emile Woolf International 145 The Institute of Chartered Accountants of Pakistan


Advanced accounting and financial reporting

1.2 Revaluation
During the year 2012, the first revaluation of freehold land was carried out. The valuation was
carried out under market value basis by an independent valuer, Mr. Dee, Chartered Civil Engineer
of M/s SSS Consultants (Pvt.) Ltd., Islamabad. It resulted in a surplus of Rs. 120 million over book
values which was credited to surplus on revaluation of fixed assets. Had there been no
revaluation, the value of freehold land would be Rs. 255 million.

1.3 Disposal of machine


Rs.in
million
Proceeds 13.0
Cost 15.0
Accumulated depreciation (5.0)
Carrying amount (10.0)
Profit on disposal 3.0

Note 2016
Rs.in
million
2. Intangible Assets
Cost of computer software/license 10.0
Accumulated Amortization as of July 1, 2010 1.0
Amortization for the year 1.0
Accumulated Amortization as of June 30, 2016 2.0
Carrying value as at June 30, 2016 8.0
Amortization rate 10%

3 - Accounts Receivable
Considered good
- Secured 30
- Unsecured 27
57
Considered doubtful 3
60
Less: Provision for bad debts 3.1 3
57
3.1 - Provision for bad debts
Balance as at July 1, 2010 3.4
Provision made during the year 1.0
Amount written off during the year (1.4)
Balance as at June 30, 2016 3.0
4 - Advances, Deposits, Prepayments and Other Receivables
Advances
- suppliers - considered good 12
- staffs 6
18
Deposits 11
Prepayments 4
Sales tax receivable 12
45

© Emile Woolf International 146 The Institute of Chartered Accountants of Pakistan


Answers

Note 2016
Rs.in
million
5 - Cash at banks
Cash at banks - current accounts 7
saving accounts 5.1 22
29
5.1: It carries interest / mark-up ranging from 3% to 7% per annum.
6 - Accounts and other payables
Accounts payable 75
Accrued liabilities 7
82

3.3 YASIR INDUSTRIES LIMITED


(a) Statement of financial position as at June 30, 2016
Rs.in
million
Assets
Non-current assets
Property, plant and equipment (W2) 351.00
Intangible assets (20 – 12) 8.00
359.00
Current assets
Inventories (W6) 64.50
Trade receivables (W5) 39.00
103.50
462.50
Equity and Liabilities
Equity
Issued, subscribed and paid up capital 120.00
Retained earnings (W4) 87.10
207.10

Revaluation surplus 41.25

Non-current liabilities
Redeemable preference shares 40.00
Debentures 80.00
Deferred taxation (W 10) 9.00
129.00
Current liabilities
Trade payables 30.40
Accrued expenses (W3) 25.00
Taxation 16.50
Bank overdraft 13.25
85.15
Total equity and liabilities 462.50

© Emile Woolf International 147 The Institute of Chartered Accountants of Pakistan


Advanced accounting and financial reporting

(b) Statement of profit or loss and other comprehensive income for the year
ended June 30, 2016
Rs.in
million
Sales revenue (W5) 445.40
Cost of sales (W7) (250.72)
Gross profit 194.68
Distribution costs (W8) (20.05)
Administrative expenses (W8) (40.38)
Financial charges (W9) (9.10)
125.15
Loss due to fraud (30.00)
Profit before tax 95.15
Income tax expense (W10) (19.50)
Profit for the year 75.65
Workings
(W1) Leasehold property
Annual depreciation before the revaluation (230 ÷ 40 years) = Rs. 5.75 million per
annum.
Depreciation this year has been charged incorrectly on cost (whereas it should have
been on the revalued amount).
This year’s charge must be added back
Rs.in
million
Dr Cr
Accumulated depreciation 5.75
Cost of sales (50%) 2.88
Administrative expenses (30%) 1.72
Distribution costs (20%) 1.15

Rs.in
million
Carrying amount at the 30 June (as per trial
balance)(230.00 – 40.25) 189.75
Add back depreciation incorrectly charged (see above) 5.75
Carrying amount of property at the start of the year 195.5

Revaluation surplus Rs.in


million
Revalued amount of leasehold property 238.00
Less: WDV of leasehold property at revaluation 195.50
Revaluation surplus arising in the year 42.50
Transfer to retained earnings in respect of incremental
depreciation (Rs. 7 million – Rs. 5.75 million) (1.25)
41.25

© Emile Woolf International 148 The Institute of Chartered Accountants of Pakistan


Answers

Depreciation of revalued property


Number of years depreciation by the year end: (40.25 ÷ 5.75) = 7 years.
Therefore, remaining useful life as at the year-end = 33 years
Revaluation was at the start of the year
Remaining useful life at the start of the year = 34 years
238
Depreciation charge based on the revalued amount ( /34 years) = Rs. 7 million

Rs. In
million
Dr Cr
Cost of sales (50%) 3.5
Administrative expenses (30%) 2.1
Distribution costs (20%) 1.4
Accumulated depreciation 7.00

(W2) Property, plant and equipment


Rs.in
million
Leasehold property (Rs. 238m – 7) 231
Machines (Rs. 168.6 – Rs. 48.6m) 120
351
(W3) Accrued expenses
Rs.in
million
As per trial balance 16.20
6
Accrued interest on debentures (Rs. 80m × 12% × /12) 4.80
Dividend on preference shares (Rs. 40m × 10%) 4.00
25.00
(W4) Retained earnings
Rs.in
million
Balance as per trial balance 10.20
Profit for the year 75.65
Transfer from revaluation surplus 1.25
87.10
(W5) Sales and receivables
Sales. Rec.
Rs. In
million
Given in the trial balance 472.40 66.00
Deduct revenue incorrectly recognised (sale or
return) (27.00) (27.00)
Adjusted sales and receivables 451.40 39.00

© Emile Woolf International 149 The Institute of Chartered Accountants of Pakistan


Advanced accounting and financial reporting

(W6) Closing inventory


Rs.in
million
Given in the question 42.00
Add back inventory held by customer on sale or return
( /120  27)
100
22.50
Adjusted closing inventory 64.50
(W7) Cost of sales
Rs.in
million
Opening inventory as of July 1, 2015 38.90
Purchases 175.70
Direct labour 61.00
Manufacturing overheads excluding incremental
depreciation 39.00
Less: Closing inventory (adjusted (W6)) (64.50)
Deduct depreciation incorrectly charged on cost (2.88)
Add depreciation charged on revalued amount 3.50
Cost of sales 250.72
(W8) Administrative expenses and distribution costs
Admin. Dist/
Rs.in
million
Given in the trial balance 40.00 19.80
Deduct depreciation incorrectly charged on cost (1.72) (1.15)
Add depreciation charged on revalued amount 2.10 1.40
Adjusted expenses 40.38 20.05
(W9) Financial charges
Rs.in
million
Balance as per trial balance 0.30
6
Accrued interest on debentures (Rs. 80m × 12% × /12) 4.80
Preference dividend for the year (Rs. 40m × 10%) 4.00
9.10
(W10) Taxation
Deferred taxation Rs.in
million
Balance b/f 6.00
Charge for the year (balancing figure) 3.00
Balance c/f (30%  Rs. 30 million temporary difference) 9.00

Tax expense Rs.in


million
Current tax 16.50
Deferred tax (see above) 3.00
19.50

© Emile Woolf International 150 The Institute of Chartered Accountants of Pakistan


Answers

3.4 FIGS PAKISTAN LIMITED


Statement of profit or loss and other comprehensive income
For the year ended 31 December 2016
Note Rs.in
million
Sales 1 44,758
Cost of sales 2 (26,203)

Gross profit 18,555


Distribution costs 3 (6,431)
Administrative expenses 4 (752)
Other operating expenses 5 (399)
Other operating income 6 30

Profit from operations 11,003


Finance costs 7 (166)

Profit before tax 10,837


Taxation 8 (2,532)

Profit after tax 8,305


Other comprehensive income -

Total comprehensive income for the year 8,305

Notes to the financial statements


For the year ended 31 December 2016

1 Sales Note Rs.in


million
Manufactured goods
Gross sales 56,528
Sales tax (10,201)

46,327
Imported goods
Gross sales 1,078
Sales tax (53)

1,025
Sales discounts (2,594)

44,758

© Emile Woolf International 151 The Institute of Chartered Accountants of Pakistan


Advanced accounting and financial reporting

2 Cost of sales
Rs.in
million
Raw material consumed (1,751 + 22,603 - 2,125) 22,229
Stores and spares consumed 180
Salaries, wages and benefits (2,367 × 55%) 2.1 1,302
Utilities (734 × 85%) 624
Depreciation and amortizations (1,287 × 70%) 901
Stationery and office expenses (230 × 25%) 58
Repairs and maintenance (315 × 85%) 268
25,562
Opening work in process 73
Closing work in process (125)
25,510
Opening finished goods (manufactured) 1,210
Closing finished goods (manufactured) (1,153)
25,567
Finished goods (imported)
Opening stock 44
Purchases 658
702
Closing stock (66)
636
26,203

2.1 Salaries, wages and benefits include Rs. 30 million (54 × 55%) and Rs. 24 million (44 ×
55%) in respect of defined contribution plan and defined benefit plan respectively.

3 Distribution costs Rs.in


million
Advertisement and sales promotion 4,040
Outward freight and handling 1,279
Salaries, wages and benefits (2,367 × 30%) 3.2 710
Utilities (734 × 5%) 37
Depreciation and amortization (1,287 × 20%) 257
Stationery and office expenses (230 × 40%) 92
Repairs and maintenance (315 × 5%) 16
6,431

© Emile Woolf International 152 The Institute of Chartered Accountants of Pakistan


Answers

3.1 Salaries, wages and benefits include Rs. 16 million (54 × 30%) and Rs. 13 million
(44×30%) in respect of defined contribution plan and defined benefit plan respectively.
Rs.in
4 Administrative expenses million
Salaries, wages and benefits (2,367 × 15%) 4.1 355
Utilities (734 × 10%) 73
Depreciation and amortization (1,287 × 10%) 129
Stationery and office expenses (230 × 35%) 80
Repairs and maintenance (315 × 10%) 31
Legal and professional charges 71
Auditor's remuneration 4.2 13
752

4.1 Salaries, wages and benefits include Rs. 8 million (54 × 15%) and Rs. 7 million
(44×15%) in respect of defined contribution plan and defined benefit plan
respectively.

4.2 Auditor's remuneration Rs.in


million
Audit fees 8
Taxation services 4
Out of pocket expenses 1
13
5 Other operating expenses
Donation 5.1 34
Worker's Profit Participation Fund 257
Worker Welfare Fund 98
Loss on disposal of property, plant and equipment 10
399
5.1 Donations
Donations include Rs. 5 million given to Dates Cancer Foundation (DCF). One of the
company’s directors, Mr. Peanut is a trustee of DCH.
Donations other than that mentioned above were not made to any donee in which a
director or his spouse had any interest at any time during the year.
6 Other operating income Rs.in
million
Income from financial assets
Dividend income 12
Return on savings account 2
Income from non-financial assets
Scrap sales 16
30

© Emile Woolf International 153 The Institute of Chartered Accountants of Pakistan


Advanced accounting and financial reporting

Rs.in
million
7 Finance costs
Short term borrowings 133
Exchange loss 22
Lease 11
166
8 Taxation
Current - for the year 1,440
Deferred (3,120 × 35%) 1,092
2,532

3.5 FAZAL LIMITED


The related parties comprise a subsidiary, an associated undertaking/an entity having significant
influence, director and key management personnel.
Aggregate transactions with related parties are as follows:
Entity Key
having
Subsidiary Associate Director Manageme
significant nt
influence Personnel
Rupees Rupees Rupees Rupees
Transactions
Sales 500,000,000
Sales discount 25,000,000
Sales return 5,500,000
Purchase of raw
5,000,000
material
Purchase of equipment 3,000,000
Purchase of machinery 14,000,000

Balances
Advances
At beginning of the 1,400,00
year 0
Repaid during the year 300,000
At the end of the year 1,100,00
0

(i) Sales discount represents a special discount which is not usually allowed to other
customers.
(ii) All transactions with related parties have been carried out on commercial terms and
conditions.

© Emile Woolf International 154 The Institute of Chartered Accountants of Pakistan


Answers

3.6 BABER LIMITED


(i) AK Associates will not be treated as related party merely on the ground that both
entities have a director in common.
However, if it can be proved that an entity has some influence on other entity; they will
be treated as related parties.
(ii) Provider of finance is not necessarily a related party. However, SS Bank has power to
appoint its nominee director in the Board and therefore enjoys significant influence; it
will be treated as related party.
(iii) Mr. Zee will not be treated as related party unless it can be proved that he has
significant influence over the CEO. Further, IAS-24 does not explicitly include ‘Brother’
in the definition of close family member of an individual.
(iv) A distributor with whom an entity transacts a significant volume of business will not be
treated as related party, merely by virtue of the resulting economic dependence.
(v) Since Mr. Tee is the key management personnel of the company, he will be treated as
related party.
(vi) A post-employment benefit plan for the benefits of the employee is treated as related
party.
(vii) A supplier with whom an entity transacts a significant volume of business cannot be
termed a related party, merely by virtue of the resulting economic dependence.
(viii) Ms. Vee will be treated as related party as she is a close family member of CEO, and
hence in a position to influence her husband.

3.7 GOLDEN LIMITED


Notes to the Financial Statements
For the year ended December 31, 2011

Platinum Limited is the parent company which holds majority shares of the company.

Related party transactions


The transaction with related parties are carried out in the ordinary course of business at
commercial rates except stated otherwise.

Associated Key Major


Parent
Under- Management Share-
Company
takings Personnel holders
Rupees in '000
Transactions:
Sales 18,000
Sales discount 1,500
Sale of property 10,000
Reimbursement of expenses on sale
of property 500
Interest free loans granted 2,000
Short term borrowings acquired 25,000
Interest on short term borrowings 1,500
Balances:
Accounts receivable 6,500 5,000
Loans to staff 1,800
Loans payable 25,000
Interest payable on the loan at
12% 1,500

© Emile Woolf International 155 The Institute of Chartered Accountants of Pakistan


Advanced accounting and financial reporting

20.1 Sales to related parties have been made at 20% mark-up as against GL's policy to sell
at a markup of 30%.
20.2 Administrative services are provided by the parent company free of cost as per the
agreement. Market value of these services is Rs. 350,000.
20.3 In respect of sale of property, a buyer is required to bear all costs incurred on transfer.
But in this case the company has reimbursed the costs to SL
20.4 The interest free loan has been granted to the executive director as per the terms of
employment.

3.8 METAL LIMITED


IN THE BOOKS OF METAL LIMITED
Transactions with Related Parties
Related parties comprises of the company’s subsidiaries. Transactions with related parties are
as follows:
2013 2012
Rupees
Subsidiaries
Sale of machine (at carrying amount plus 20%) - 19,200,000
Management fees income (Note 23.1) 12,000,000 -
Management fee receivable 1,000,000
Other receivables - Sale of machine - 19,200,000

No management fee was charged during the year ended 30 June 2012. Except for this, all
transactions have been carried out on arm’s length basis, as approved by the board of
directors of the company.

IN THE BOOKS OF COPPER LIMITED


Transactions with Related Parties
Related parties comprise of Metal Limited (parent company) and its subsidiaries. Transaction
with related parties can be summarized as follows:
2013 2012
Rupees
Parent Company
Purchase of machine - 19,200,000
Management fees (Note 23.1) 6,000,000 -
Management fee payable 500,000 -
Other payables - Sale of machine 19,200,000

23.1 No management fee was charged for the year ended 30 June 2012. Except for this, all
transactions have been carried out on arm’s length basis, as approved by the board of
directors of the company.

© Emile Woolf International 156 The Institute of Chartered Accountants of Pakistan


Answers

IN THE BOOKS OF ZINC LIMITED


23 – Transactions with Related Parties
Related parties comprise of Metal Limited (parent company) and its subsidiaries. Transaction
with related parties can be summarized as follows:
2013 2012
Rupees
Parent Company
Contract for factory extension project (Note 23.1) 15,000,000 -
Management fees (Note 23.2) 6,000,000 -
Management fee payable 500,000 -

23.1 The contract has been awarded to Iron Builders and Developers in which one of the
directors of the parent company is a partner.
23.2 No management fee was charged for the year ended 30 June 2012. Except for this, all
transactions have been carried out on arm’s length basis, as approved by the board of
directors of the company.

IN THE BOOKS OF STEEL LIMITED

Related parties comprise of Metal Limited (parent company) and its subsidiaries. However,
there was no related party transaction during the year.

3.9 ENGINA
Report to: The Board of Directors of Engina
From: XXXXXXXX
Date:
Subject: Related party transactions
Related party transactions
This report addresses the disclosure requirements of IAS 24 Related Party Disclosures with regard to
Engina. IAS 24 requires that all entities, listed or otherwise, provide disclosure of such transactions as
they may affect the assessments made by users of an entity’s operations, risks and opportunities.
It is understood that Engina is reluctant to disclose related party transactions because they are
believed to be both politically and culturally sensitive, however the following advice must be
followed in order to secure a listing/stock exchange registration.
IAS 24: Scope and purpose
IAS 24 does not provide any exclusion from its scope, and so disclosure must be made. Related
party transactions are a normal feature of business, but an entity’s ability to succeed in business
is often affected by the strength of its relationship with other entities and individuals. The results
of the entity may be affected if these relationships were to be terminated. For example, the ability
of an entity to trade in a particular country may only be possible because of the presence of its
subsidiary in that local market. Similarly, prices and terms of trade may be preferential because
of the strength of the relationship. Therefore IAS 24 requires knowledge of these transactions to
be provided to the reader of the financial statements.
The results of an entity may be affected even if the related party transactions do not occur. A
parent may cease trading with a business partner upon acquisition of a subsidiary that can
supply similar products.

© Emile Woolf International 157 The Institute of Chartered Accountants of Pakistan


Advanced accounting and financial reporting

Disclosure must be given irrespective of whether the transactions took place at an arm’s length
value, as such transactions may still be lost if the relationship is terminated. Hence the
knowledge of such transactions provides valuable information to investors and regulators.
Disclosure requirements
IAS 24 requires that, at a minimum, the following disclosures must be given:
 The amount of the transaction
 The amount of any outstanding balance and the terms, conditions and guarantees
attached
 Allowance for any irrecoverable debts or amounts written off in the period
 Disclosure that transactions were at an arm’s length value can only be given if this
information can be substantiated.
Disclosures relevant to Engina
The following outlines the related party disclosure requirements for the three transactions you
have specifically requested comment on. It is your responsibility to bring any further related party
transactions to our attention in order that they can also be incorporated into your financial
statement disclosures.
(a) Sale of goods to directors
The sale of goods and a company car to Mr Satay both constitute related party
transactions, due to Mr Satay’s position as a director of Engina. IAS 24 requires disclosure
of all related party transactions with key management personnel. However, accounting
standards only apply to material transactions. An item is considered material where
knowledge of that transaction might influence the decisions of a user of the financial
statements. Materiality is not just a matter of size, as small transactions with a director may
still be of relevance to an investor if the transaction is material to the director, despite not
being material to the entity.
In the situation described, the transactions amount to Rs. 600,000 of sales and the sale of
a company car for Rs. 45,000 (market value Rs. 80,000). In terms of value these
transactions appear not to be material to Engina and neither do they appear to be material
in value to Mr Satay. However, given the sensitive nature of transactions with directors,
and especially senior directors like Mr Satay, the transactions should be disclosed in the
financial statements in accordance with good corporate governance practice.
Significant contracts with directors, such as these with Mr Satay, may also require
disclosure by the local Stock Exchange.
(b) Hotel property
The sale of the hotel to the brother of Mr Soy, constitutes a related party transaction
because of Mr Soy’s status as Managing Director. The property seems to have been sold
at below market price and IAS 24 requires disclosure of any information surrounding a
transaction which will allow the reader to understand its impact on the financial statements.
The hotel had a carrying value of Rs. 5m, but given the fall in market values it should have
been written down to its recoverable amount in accordance with IAS 36 Impairment.
Recoverable amount is measured at the higher of value in use (Rs. 3.6m) and fair value
minus costs of sale (Rs. 4.3 - 0.2m). Hence the property should have been recorded in the
statement of financial position at Rs. 4.1m.
As the property was sold at Rs. 100,000 less than this impaired value, disclosure of this
fact should be made, together with any other information relevant to the reader, such as
the reason for the sale in light of the expected decline in prices in the future.
(c) Mr Satay
Mr Satay has investments in 100% of the equity of Car and 80% of the equity of Wheel. In
turn, Wheel owns 100% of Engina. Engina and Wheel are related because of their parent-
subsidiary relationship. In addition, because all three entities are under the common
control of Mr Satay, IAS 24 also considers Engina and Car to be related. Therefore, the
transactions between Engina and both Wheel and Car are related party transactions.

© Emile Woolf International 158 The Institute of Chartered Accountants of Pakistan


Answers

The transactions will need to be disclosed in the individual financial statements of all three
entities. In the group accounts, all intra-group transactions are cancelled on consolidation,
and so disclosure need not be made at this level.
Further disclosure requirements of director’s interests in the equity of Engina may be
necessary under local Companies Acts requirements and Stock Exchange rules.

3.10 SHAZAD INDUSTRIES LTD


(a) The purposes of segmental information are:
(i) to provide users of financial statements with sufficient details for them to be able to
appreciate the different rates of profitability, different opportunities for growth and
different degrees of risk that apply to an entity’s classes of business and various
geographical locations.
(ii) to appreciate more thoroughly the results and financial position of the entity by
permitting a better understanding of the entity’s past performance and thus a better
assessment of its future prospects.
(iii) to create awareness of the impact that changes in significant components of a
business may have on the business as a whole.
b) IFRS 8 defines an operating segment as a component of an entity:
 that engages in business activities from which it may earn revenues and incur
expenses (including revenues and expenses relating to transactions with other
components of the same entity).
 whose operating results are regularly reviewed by the entity’s chief operating
decision-maker to make decisions about resources to be allocated to the segment
and assess its performance.
 for which discrete financial information is available.
In order to identify the separate reportable segments, the following criteria should be
adopted:
(i) The reported revenue of the segment in Shazad Industries Ltd, including both
sales to external customers and inter-segment sales, is ten percent or more of
the combined revenue of its four operating segments.
(ii) The Assets of the segment in Shazad Industries Ltd are ten percent or more
of the combined assets of its four operating segments.
(iii) The reported profit or loss of the segment in Shazad Industries Ltd should be
ten percent or more of the greater, in absolute amount, of:
 the combined reported profit of all its operating segments that did not report a loss
and
 the combined reported loss of all operating segments that reported losses.
(c) IFRS 8 lays down some very broad and inclusive criteria for reporting segments. Unlike
earlier attempts to define segments in more quantitative terms, segments are defined
largely in terms of the breakdown and analysis used by management. This is, potentially,
a very powerful method of ensuring that preparers provide useful segmental information.
There will still be problems in deciding which segments to report, if only because
management may still attempt to reduce the amount of commercially sensitive information
that they produce.
The growing use of executive information systems and data management within
businesses makes it easier to generate reports on an ad hoc basis. It would be relatively
easy to provide management with a very basic set of internal reports and analyses and
leave the individual managers to prepare their own more detailed information using the
interrogation software provided by the system.

© Emile Woolf International 159 The Institute of Chartered Accountants of Pakistan


Advanced accounting and financial reporting

If such analyses become routine then they would be reportable under IFRS 8, but that
would be very difficult to check and audit.
There are problems in the measurement of segmental performance if the segments trade
with each other. Disclosure of details of inter-segment pricing policy is often considered to
be detrimental to the good of a company. There is little guidance on the policy for transfer
pricing.
Different internal reporting structures could lead to inconsistent and incompatible
segmental reports, even from companies in the same industry.

3.11 AZ
(a) (i) Usefulness of segmental data
Many entities carry out several classes of business and operate in a number of
countries across the world. Each of these businesses and geographical segments
carries with it different opportunities for growth, different rates of profit and varying
degrees of risk. Some business segments may be strongly influenced by the health
of the economy whereas other segments may be unaffected by recession. One
country may be experiencing growth; another country may be less stable because of
political events. Awareness of these cultural and environmental differences is
important to investors in order to allow them to fully understand the performance and
position of the entity over the past, its prospects for the future and the risks that it
faces.
IFRS 8 requires that segmental information should be provided to enable investors
to understand the impact that the different segments of a business may have on the
business as a whole. If the user of financial statements is only provided with figures
for the entity as a whole, this might hide the risks and problems or profits and
opportunities of the underlying business segments. The disaggregated financial
information provided by segmental reporting allows for analytical review on a
segment by segment basis which will provide greater understanding of the entity’s
position and performance and allow a better assessment of its future.
(ii) Analysing segments
IFRS 8 defines an operating segment as a component of an entity that engages in
business activities from which it may earn revenues and incur expenses, whose
operating results are reviewed regularly by the chief operating decision maker in the
entity and for which discrete financial information is available.
Not every part of a business is necessarily an operating segment or part of an
operating segment. Head office is an example, since head office does not usually
earn revenues. Generally an operating segment has a segment manager who is
directly accountable to and maintains regular contact with the chief operating
decision-maker, to discuss the performance of the segment.
IFRS 8 requires that entities should report information about each operating
segment that is identified and that exceeds certain quantitative thresholds for size of
revenue, operating profit or loss or assets. Financial information about operating
segments with similar characteristics can be aggregated.
IFRS 8 sets out the information about each reportable operating segment that
should be disclosed, including total assets, profit or loss, revenue from external
customers, revenue from sales to other segments, interest income and expense,
depreciation, material items of income or expense and tax. The amount reported for
each item should be the same measure that is reported for the segment to the chief
operating decision maker of the entity.
IFRS8 applies to quoted companies only.

© Emile Woolf International 160 The Institute of Chartered Accountants of Pakistan


Answers

(b) AZ segmental reporting issues


(i) Fleet of aircraft
The answer to (a) explains how operating segments should be identified. It seems
probable that information is provided separately for aircraft seat sales and for
holiday homes to the chief operating decision maker of AZ; therefore it seems
probable that these should be treated as reportable operating segments for the
purpose of IFRS 8, provided that they exceed the minimum threshold limits for size
(at least 10% of revenues, or profit or loss, or assets of the reporting entity).
(ii) Aircraft manufacturing plant
IFRS 8 requires that in measuring and reporting segment revenue from transactions
by an operating segment with other segments, inter-segment transfers should be
measured on the basis of the information provided to the chief operating decision
maker. In this case, the revenues from inter-segment sales should be based on the
prices actually charged.
IFRS 8 also requires that AZ must disclose the basis of accounting for transactions
between reportable segments.
(iii) Exceptional loss
Items which are unusual in size, nature or incidence and hence where knowledge of
their nature and amount is relevant to a reader’s understanding of performance are
required to be disclosed in the notes or on the face of the statement of profit or loss
in accordance with IAS 1.
In addition, IFRS 8 requires disclosure for each reportable segment of any material
items of income or expense that is disclosed in accordance with IAS1. If the
activities of the airline manufacturing plant are a reportable segment or part of a
reportable segment, the loss should therefore be reported within the segmental
information in the financial statements.
(iv) Discontinued operations
IFRS 8 does not deal with discontinued operations as this is the subject of IFRS 5
Non-current assets held for sale and discontinued operations. IFRS 5 requires
the disclosure of the impact of a discontinued operation, and has strict conditions
that must be met when identifying a discontinued operation.
To be classified as a discontinued operation, IFRS 5 requires that the component of
the entity has been sold in the year or is classified as ‘held for sale’. A decision by
the board and an announcement to the press alone is insufficient to be classed as
‘held for sale’. The criteria that must be met by the end of the reporting period are as
follows:
 The asset must be available for immediate sale in its present condition; and
 The sale must be highly probable. For this to be so the following sub-
conditions must also be met:
 The appropriate level of management must be committed to the plan to sell
 An active programme to locate a buyer must have been initiated
 The asset is being marketed at a reasonable price, and
 The sale should be expected to be completed within one year from the date of
classification.
Hence, unless marketing of the holiday business is in progress at a reasonable
price, and sale in the next 12 months is expected, the operation need not be
classified as discontinued. If however, all of the above criteria are met, then the
results of this operation must be disclosed as a single line on the face of the
statement of profit or loss together with supporting disclosures in the notes to the
accounts.

© Emile Woolf International 161 The Institute of Chartered Accountants of Pakistan


Advanced accounting and financial reporting

(v) 40% investment in Eurocat


IFRS 8 requires that information should be disclosed for each reportable segment of
the entity’s interest in the profit or loss of associates and joint ventures accounted
for by the equity method. In addition, if the information is provided to the chief
operating decision maker of the entity, for each reportable segment there should be
disclosure of the amount of investment in associates or joint ventures accounted for
under the equity method.
Therefore, AZ must disclose the results of the associate in its segmental
disclosures.

3.12 J-MART LIMITED


(a) Adjusting events:
Adjusting events are events that provide further evidence of conditions that existed at
the reporting date.
Examples of adjusting events include:
(i) The subsequent determination of the purchase price or of the proceeds of sale of
non-current assets purchased or sold before the year end.
(ii) The renegotiation of amounts owing by customers or the insolvency of a customer
(iii) Amounts received or receivable in respect of insurance or the insolvency of a
customer.
(iv) The settlement after the reporting date of a court case that confirms that the entity
had a present obligation at the reporting date.
(v) The receipt of the information after the reporting date indicating that an asset was
impaired at the reporting date.
(vi) The discovery of fraud or errors that show that the financial statements are
incorrect.
Non-adjusting events:
Non-adjusting events are indicative of conditions that arose subsequent to the reporting
date.
Examples of non-adjusting events might be:
(i) Losses of non-current assets or inventories as a result of a catastrophe such as
fire or flood
(ii) Closing a significant part of the trading activities if this was not begun before the
year end
(iii) The value of an investment falls between the reporting date and the accounts are
authorised
(iv) Announcement of dividend after year end.
(b) (i) The conditions attached to the sale give rise to a constructive obligation on the
reporting date.
A provision for the sales return should be recognised for 5% of June 2016 sales.
The related cost should also be reversed.
(ii) Since the law suit was already in progress at year-end and the amount of
compensation can also be estimated, it is an adjusting event.
A provision of Rs. 400,000 should be made.
(iii) There is no obligating event at the year-end either for the costs of fitting the
smoke detectors or for fines under the legislation.
No provision should be recognised in this regard.
(iv) The obligating event is the communication of decision to the customers and
employees, which gives rise to a constructive obligation from that date, because
it creates a valid expectation that the division will be closed.

© Emile Woolf International 162 The Institute of Chartered Accountants of Pakistan


Answers

Since no communication has yet been made, no provision is required in this


regard.
(v) The obligating event is the signing of the lease contract, which gives rise to a
legal obligation.
A provision is required for the unavoidable rent payments.
(vi) Since the declaration was announced after year-end, there is no past event and
no obligation at year-end and is therefore non-adjusting event.
Details of the dividend declaration must, however, be disclosed.

3.13 QALLAT INDUSTRIES LIMITED


(i) Provision must be made for estimated future claims by customers for goods already
sold.
The expected value i.e. Rs. 10 million ([Rs. 150m x 2%] + [Rs. 70m x 10%]) is the best
estimate of the provision.
(ii) Warehouse A: It is an onerous contract as the warehouse has been sublet at a loss of
Rs. 200,000 per month. QIT should therefore create a provision for the onerous contract
that arises on vacating the warehouse. This is calculated as the excess of unavoidable
costs of the contract over the economic benefits to be received from it. Therefore, QIL
should immediately provide for the amount of Rs. 13.2 million. [5.5 years x 12 month x
Rs. 200,000] in its financial statements i.e. for the year ended June 30, 2016.
Warehouse B: It is not an onerous contract because the warehouse has been sublet at
profit. Hence this would require no adjustment.
(iii) A provision is to be made by QIL.
(i) There is a present obligation (legal or constructive) as a result of a past event; i.e.
accident occurred on June 15, 2016.
(ii) It is probable that outflow of resources will be required to settle the obligation; and
(iii) A reliable estimate can be made of the amount of the obligation.
The amount of provision shall be Rs. 2.0 million i.e. the most probable amount as
determined by the lawyer.
(iv) A provision of Rs.0.4 million is required in relation to penalty for March 1 to June 30,
2016 because at the reporting date there is a present obligation in respect of a past
event.
The reimbursement of penalty amount from the vendor shall be recognised when and
only when it is virtually certain that reimbursement will be received if the entity settles
the obligation. The reimbursement should be treated as a separate asset in the
statement of financial position. However, in profit and loss statement, the expense
relating to a provision may be netted off with the amount recognised as recoverable, if
any.

3.14 SKYLINE LIMITED


(i) Although the debt owing by the customer existed at the reporting date, the customer’s
inability to pay did not exist at that point. This condition only arose in 10 February 2017
after the fire.
Thus, this is a non-adjusting event. However, if it is material for the financial statements,
the following disclosure should be made.
 Nature of the event
 An estimate of its financial effect
(ii) The amount withdrawn before year end i.e. Rs. 1.5 million is an adjusting event as
although it was discovered after year end it existed at the year end. However, since 60%
has been recovered subsequently, Rs.0.6 million would be provided.

© Emile Woolf International 163 The Institute of Chartered Accountants of Pakistan


Advanced accounting and financial reporting

The further withdrawal of Rs. 6.0 million is a non-adjusting event as it occurred after year
end. However, if the events are considered material the following disclosures should be
made:
 Nature of the event
 The gross amount of contingency
 The amount recovered subsequently
SL should not recognise the contingent gain until it is realised. However, if recovery of
(iii)
damages is probable and material to the financial statements, SL should disclose the
following facts in the financial statements:
 Brief description of the nature of the contingent asset
 An estimate of the financial effect.
SL should make a provision of the expected amount i.e. Rs. 1.2 million (Rs. 1.0 million x
(iv)
60% + Rs. 1.5 million x 40%) because
 it is a present obligation as a result of past event;
 it is probable that an outflow of resources embodying economic benefits will be
required to settle the obligations; and
 a reliable estimate can be made of the amount.
In addition, SL should disclose the following in the notes to the financial statements:
 Brief nature of the contingent liability
 The amount of contingency
 An indication of the uncertainties relating to the amount or timing of any outflow.

3.15 WALNUT LIMITED


(i) This is an adjusting event as it provides evidence of conditions that existed at the end of
the reporting period. The reasons for the competitor’s price reduction will not have
arisen overnight, but will normally have occurred over a period of time, may be due to
superior investment in technology.
An inventory write down of Rs. 2.5 million should be recognised and the amount
included as inventory on the Statement of Financial Position reduced to Rs. 12.5 million.
(ii) The provision should be recognised because the obligating event is the communication
of event by the Board of Directors to the public which creates a valid expectation that
the division will be closed.
However, the provision should only be recognised to the extent of redundancy costs.
IAS prohibits the recognition of future operating losses, staff training and profits on sale
of assets.
(iii) This is a non-adjusting event because the burglary and theft of consumable stores
occurred after reporting date. However, if the event is material, it should be disclosed in
the financial statements unless the loss is recoverable from the insurance company.
(iv) The drop in value of investment in shares is a non-adjusting event. Since the legislation
was announced after the reporting date, the event is not a past event. However, if the
amount is material, it should be disclosed in the financial statements.
(v) This is an adjusting event as it provides evidence of conditions that existed at the end of
the reporting period. The insolvency of a debtor and the inability to pay usually builds up
over a period of time and it can therefore be assumed that it was facing financial
difficulty at year-end.
A bad debts expense of Rs. 1.5 million should be recognised in SOCI.
(vi) It is a non-adjusting event because the declaration was announced after the year-end
and there was no obligation at year end. Details of the bonus shares declaration must,
however, be disclosed.

© Emile Woolf International 164 The Institute of Chartered Accountants of Pakistan


Answers

CHAPTER 4: IAS 8: ACCOUNTING POLICIES, CHANGES IN ACCOUNTING


ESTIMATES AND ERRORS

4.1 WONDER LIMITED


(a) Wonder Limited
Statement of financial position as at 30 June 2016 (extracts)
2015
2016 (Restated)
Rs. m Rs. m
Property, plant and equipment 178.50 111.50
Retained earnings 158.65 95.05
Deferred tax liability 41.85 21.45

Workings: Property, plant and equipment 2016 2015


Given in question 189.0 130.0
Deduct repair cost debited in error (20.0) (20.0)
Add back depreciation charged on this
(20  10%  21/12) 3.5
(20  10%  9/12) 1.5
(16.5) (18.5)
Incorporate review of useful life
Charged already (70/5 years) 14.0 -
Correct charge ((70  4/5) = 56/7 years 8.0 -
Add back 6.0 -
178.5 111.5

Workings: Deferred tax liability 2016 2015


Given in question 45.00 27.00
Adjustments to PP&E
Correction of error (16.50) (18.50)
Change of useful life 6.00 -
Net adjustment (Cr to PP&E) 10.50 18.50
Tax thereon (at 30%) (Dr DT liability) (3.15) (5.55)
41.85 21.45

Workings: Retained earnings 2016 2015


Given in question 166.00 108.00
Net adjustment
(Cr to PP&E therefore Dr retained earnings)) (10.50) (18.50)
Tax thereon (at 30%)
(Dr DT liability therefore Cr Retained earnings) 3.15 5.55
158.65 95.05

© Emile Woolf International 165 The Institute of Chartered Accountants of Pakistan


Advanced accounting and financial reporting

(b) Wonder Limited


Extracts of statement of profit or loss for the year ended 30 June 2016
2015
2016 (Restated)
Rs. m Rs. m
Profit before taxation 98.00 101.50
Taxation (34.40) (36.45)
Profit after taxation 63.60 65.05

Workings: Profit before tax 2016 2015


Given in question 90.0 120.0
Deduct repair cost capitalised in error (20.0)
Add back depreciation on this
(20  10%  12/12) 2.0
(20  10%  9/12) 1.5
2.0 (18.5)
Adjustment for re-estimate of useful life
Charged already (70/5 years) 14.0 -
Correct charge ((70  4/5) = 56/7 years ) 8.0 -
Add back 6.0 -
98.0 101.5

(c) Wonder Limited


Extracts of statement of changes in equity for the year ended 30 June 2016
Retained
earnings
Rs. m
Balance as on 1 July 2014 (108-78) 30.00
Profit for the year ended 30 June 2015 (78 - 12.95 (restated) 65.05
Balance as at 30 June 2015 (restated) 95.05
Profit for the year ended 30 June 2016 63.60
Balance as at 30 June 2016 158.65

(d) Wonder Limited


Notes to the financial statements for the year ended 30 June 2016
Correction of error: During the year ended 30 June 2014, a repair was
erroneously debited to machinery account. The effect of this error is as follows:
2015
Effect on the statement of profit or loss Rs. m
(Increase) / decrease in expenses or losses
Repairs and maintenance (20.00)
Depreciation (20  10%  9/12) 1.50

Tax expense (30%  (20  1.5)) 5.55


Decrease in profit for the year 1.4 (12.95)

© Emile Woolf International 166 The Institute of Chartered Accountants of Pakistan


Answers

Effect on the statement of financial position


Increase / (decrease) in assets
Property, plant and equipment (20 – 1.5) (18.50)

(Increase) / decrease in liabilities


Deferred tax liability (Rs. 18.5 × 30%) 5.55
Retained earnings (18.50 - 5.55) (12.95)

4.2 DUNCAN
Statement of changes in equity (extract) for the year ended December 31, 2016

Retained Retained
earnings earnings
2016 2015
Rs.000 Rs.000
Opening balance as reported 23,950 22,500
Change in accounting policy (W2) 450 400
––––––– –––––––
Re-stated balance 24,400 22,900
Profit after tax for the period (W1) 4,442 3,250
Dividends paid (2,500) (1,750)
––––––– –––––––
Closing balance 26,342 24,400
––––––– –––––––

Workings
(1) Revised profit
2016 2015
Rs.000 Rs.000
Per question 4,712 3,200
Add back: Expenditure for the year 600 500
Minus: Depreciation (870) (450)
–––––– ––––––
Revised profit 4,442 3,250
–––––– ––––––

(2) Prior period adjustment


The prior period adjustment is the reinstatement of the Rs. 400,000 asset on 1 January
2015 and the Rs. 450,000 asset at 1 January 2016. On 31 December 2016 the closing
balance above of Rs. 26,342,000 can be reconciled as the original Rs. 26,162,000 plus the
reinstatement of the remaining asset of Rs. 180,000.

© Emile Woolf International 167 The Institute of Chartered Accountants of Pakistan


Advanced accounting and financial reporting

4.3 MOHANI MANUFACTURING LIMITED


Statement of changes in equity
For the year ended December 31, 2016
Retained
Earnings
Rs.in million
Balance at December 31, 2014 as previously reported
(Rs. 89m – Rs. 21m) 68.00
Effect of change in accounting policy (Rs. 37m - Rs. 35.5m) (1.50)
Balance at December 31, 2014 – restated 66.50
Profit for the year ended December 31, 2015 - restated (W1) 39.70
Balance at December 31, 2015 – restated 106.20
Profit for the year ended December 31, 2016 (W2) 8.80

Balance at December 31, 2016 115.00

W1: Profit for the year ended December 31, 2015 (as restated) Rs.in million
Profit as previously reported 21.00
Incorrect recording of depreciation (Rs. 25 million – Rs. 10 million) 15.00

Reversal of FIFO method


Opening inventory 37.00
Closing inventory (42.30)
(5.30)
Application of weighted average method
Opening inventory (35.50)
Closing inventory 44.50
9.00

39.70

W2: Adjusted profit for year ended June 30, 2016 Rs.in million
Profit as per draft financial statements 15.00
Adjustment in Opening Inventory
FIFO 42.30
Weighted average (44.50)
(2.20)
Adjustment in Closing Inventory
FIFO (58.40)
Weighted average 54.40
(4.00)

Adjusted profit 8.80

© Emile Woolf International 168 The Institute of Chartered Accountants of Pakistan


Answers

CHAPTER 5: IFRS 15: REVENUE FROM CONTRACT WITH CUSTOMER

5.1 PARVEZ LIMITED


(1) Sale and repurchase agreement
The transaction is in the nature of sale and repurchase agreement therefore the economic
phenomenon of the transaction is that of a loan for which the goods have been given as
security. Therefore no contract of sale of goods or services is identified.
The difference between the sale price of Rs.18m and the repurchase price of Rs.19m
represents the interest on the loan for a period of four months.
To account for the transaction in accordance with its substance:
 The goods should remain in inventories of PL at the lower of cost and net realisable
value.
 No sale should be recorded.
 The amount once received from the bank should be treated as a current loan liability
of Rs.18m.
 Interest should be charged applying implicit rate to profit or loss for each reporting
period.
(2) Consignment inventories
There is a contract for sale of cars between Pervez Limited (PL) and dealer containing
confirmation of respective right and obligation, payment term, commercial substance and
probability of collection of price.
There is only one performance obligation, namely, the transfer of cars to the dealer.
As per contract, the transaction price would be list price on the date of sale to third parties
during the six month period. Thereafter, though not specifically mentioned, after the lapse
of fifteen days the list price applicable on sixteenth day would be the transaction price of
the unsold cars not returned.
Since there is only one performance obligation, the question of allocation of transaction
price does not arise till the time of sale to third parties.
PL will recognize revenue upon satisfaction of performance obligation. Performance
obligation would be satisfied once the dealer has sold any cars to third parties during the
six month period. Thereafter, if the dealer does not return the unsold cars within fifteen
days, the performance obligation would be considered as satisfied on sixteenth day.
On 31 March 2017 the vehicles should remain in inventories in PL books of accounts.

5.2 SACHAL LIMITED


International Financial Reporting Standard (IFRS 15) provides that the revenue is recognized:
(a) when the performance obligation is satisfied by the entity by transferring a promised
good or service (ie an asset) to the customer; and
(b) the asset is transferred when the customer obtains the control of that asset.
Based on this principle, the following is the considerations to be taken into account in determining
accounting for revenue:
a) Restaurant management software
There exists a contract for sale of Restaurant management software between SL and
customers containing confirmation of respective right and obligation, payment term,
commercial substance and price is collected in advance.

© Emile Woolf International 169 The Institute of Chartered Accountants of Pakistan


Advanced accounting and financial reporting

There are two performance obligations, namely:


 Explicit: delivery of software and
 Implicit: six month on-site support
As per contract, the transaction price is Rs.1.5 million for both performance obligations.
Based on stand-alone selling price approach, software will be priced as Rs.1.35 million
(i-e. 1.50 m – 0.15) and six month on-site support services will be priced as Rs.0.15
million (i-e. 0.30 million x 6/12).
PL will recognize revenue from sale of software upon delivery if SL can objectively
conclude that the software meets the requirements of the customer. The term of full
payment of transaction price in advance is a reasonable evidence of clarity of
specification between SL and customer. The agreed thirty days trial time will be
considered as a formality of the contract.
PL will recognize revenue from on-site support services over six months period on
straight-line basis.
b) Maintenance support for the standard software package
Such service is provided under a written contract that contains confirmation of respective
right and obligation, payment term, commercial substance. SL will assess the
collectability of the price if not received in advance.
The performance obligation is to provide maintenance and support services.
The price of the service is Rs.0.30 million for one year term.
Since there is only one performance obligation, the question of allocation of transaction
price does not arise.
PL will recognize revenue over one year period on straight-line basis, as in this case
input method is appropriate. The pattern of resources consumed by SL is evenly spread
over the period of contract.
c) Customized software
Such service is provided under a written contract that contains confirmation of respective
right and obligation, payment term, commercial substance. SL will assess the
collectability of the price.
The performance obligations are:
 Designing and development of customized software, and
 Maintenance and support services of the said software
The price of the service will be determined on the basis of terms of contract.
The price will be allocated between the two performance obligations. Price of
maintenance services for the first year is included in the total contract price. The
allocated price would be 10% of the contract price, which is the stand-alone price of the
said services.
Satisfaction of performance obligation:
Revenue from design and development - PL will recognize revenue from design and
development over time, because the software at every stage is expected to be customer
specific and would have no alternative use for SL. The terms of payment at different
stages of project also confirms that SL would have an enforceable right to receive
payment if the contract is terminated before completion. In this case output method would
be appropriate, as the resources applied on different stages vary. Therefore, the amount
of recognized revenue would correspond to the development stage of the software at the
end of reporting period.
Revenue from Maintenance and support services - PL will recognize revenue over one
year period on straight-line basis, as in this case, input method is appropriate. The
pattern of resources consumed by SL is evenly spread over the period of contract.

© Emile Woolf International 170 The Institute of Chartered Accountants of Pakistan


Answers

5.3 BRILLIANT LIMITED


Identification of performance obligations
There are three performance obligations:
1. Transfer of 15 Plastic card printing machines and its software
2. Transfer of 8 Laminators
3. Transfer of 100,000 plastic cards
Although the software is distinct from printing machine, but both are highly dependable to each
other and inter-related. In the context of this contract, these are providing a combined output to
PL. Therefore, software is not a separate performance obligation.
The total transaction price as per the contract is Rs.9.2 million.
On the basis of available information the stand-alone prices of each item will be estimated using
the following approaches:
Plastic card printing machines and its software:
In the absence of observable stand-alone price, we may use ‘adjusted market assessment’
approach. The competitor’s machine is sold at Rs.750,000 which is similar (not identical) to BL’s
machine. As per given information, we may use customers’ rating for adjustment of competitors’
price that worked out as follows:
Rupees
Competitors’ price 750,000
Adjusted price of BL machine (7/9*750,000) 583,000
Total price (15*583,000) 8,745,000
Laminators:
There is neither observable stand-alone price nor any comparable competitors’ product available
in the market in which BL operates. In this case, we may use ‘expected cost plus a margin
approach’. The estimated stand-alone price is worked out as follows:
Rupees
Expected cost to BL 200,000
Margin estimated (800,000 - 600,000)/600,000 = 33% 66,000
266,000
Total price (8*266,000) 2,128,000
Plastic cards:
Observable stand-alone price is available
Total price (100,000*12) 1,200,000

Total of stand-alone prices is:


Plastic card printing machines and its software 8,745,000
Laminators 2,128,000
Plastic cards 1,200,000
Total 12,073,000

Allocation of Rs.9.2 million (transaction price) will be based on relative stand-alone prices, as the
difference of Rs.2.873 million between stand-alone price and transaction price is not specific to
any performance obligation.

© Emile Woolf International 171 The Institute of Chartered Accountants of Pakistan


Advanced accounting and financial reporting

Rupees
Plastic card printing machines and its software 6,663,961
(9,200,000*8,745,000/12,073,000)
Laminators 1,621,602
(9,200,000*2,128,000/12,073,000)
Plastic cards 914, 437
(9,200,000*1,200,000/12,073,000)
Total 9,200,000

5.4 WAQAS LIMITED


The following is the available data of the original project:
Transaction price Rs.20 million
Cost of the project Rs.12 million
At the signing of the contract only one performance obligation is identified. Therefore, the
question of allocation the transaction price of Rs.20 million would not arise.
The revenue would be recognized over time because the installation and construction will be
done on the land of ACL and control of asset will be transferred progressively and will create right
of payment for WL. Amount of revenue recognized would correspond to the progress of the
project. The progress will be measured using input method, that is, cost incurred plus margin.

At the end of seventh month:


Additional Reservoir:
a) is distinct from original RO plant project
b) increased the price of the contract by Rs.2.5 million which reflected WL’s stand-alone price
of similar construction work. The following working explains it further:
Cost estimated Rs.1.8 million
Usual margin (8/12*100=67%) Rs.1.2 million
Normal price Rs.3.0 million
Agreed consideration Rs.2.5 million

The reduced price is reasonable due to less administrative resources is to be applied for
additional work.

The contract of additional reservoir will be treated as separate contract and its revenue will be
recognized separate from original contract. The revenue from this contract will be recognized
over time, as construction of reservoir will be done on the land of ACL and control of asset will be
transferred progressively and will create right of payment for WL.
At this stage the revenue from RO plant project will be recognized as follows:
Percentage of work completed
(4.2/12.0*100) 35%
Revenue to be recognized (35%*20) Rs.7.0 million
At the end of tenth month:
Increasing the size of reservoir will increase the scope of the contract, but it cannot be
considered as a distinct work already agreed. Increased contract price also does not reflect WL’s
stand-alone price of similar work because it is equal to the cost of work. Therefore, WL should
account for this modification as part of single performance obligation that is partially satisfied on
the date of modification. A cumulative catch-up adjustment will be done, which is worked out as
follows:

© Emile Woolf International 172 The Institute of Chartered Accountants of Pakistan


Answers

Original contract Modified


Contract price 20.0 21.0
Total contract cost 12.0 13.0
Cost incurred so far 7.2 7.2
% of completion 60% 55%
Cumulative Revenue recognition 12.0 11.55

Difference between the two amounts of cumulative revenue will be the adjustment to the revenue
account.

Revenue from additional reservoir


% of completion (0.72/1.8 * 100) 40%
Revenue to be recognized (40% * 2.5) Rs.1.0 million

At the end of sixteenth month:


Additional work of pumping and piping facility increased the scope of the contract. It is also
distinct from the RO plant project. However, the increased price of the contract does not reflect
WL’s stand-alone price of similar work because it provides nominal margin to WL. Therefore, this
contract cannot be accounted for as separate contract. This contract will terminate the existing
contract and create a new contract. There will be two performance obligations (a) Transfer of RO
plant; and (b) transfer of pumping and piping facility.
The price of new contract is worked out as follows:
% of completion of existing contract (11.70/13.0*100) 90%
Rupees
Revenue recognized (21.0 * 90%) 18.9m
Remaining promised consideration (21.0 – 18.9) 2.1m
Consideration of modification 3.0m
New contract price 5.1m

Allocation of new contract price on the basis of cost plus margin approach
Total estimated cost of new modified contract (13.0+2.8) 15.8m
Less: Already incurred cost 11.7m
Cost to be incurred 4.1m

Allocation
RO plant project (1.3/4.1*5.1) 1.62m
Pumping and piping facility (2.8/4.1*5.1) 3.48m

The revenue will be recognized over time.

Revenue from additional reservoir


% of completion (1.35/1.8 * 100) 75%
Revenue to be recognized (75% * Rs. 2.5m) 1.875m

© Emile Woolf International 173 The Institute of Chartered Accountants of Pakistan


Advanced accounting and financial reporting

CHAPTER 6 – IAS 16: PROPERTY, PLANT AND EQUIPMENT

6.1 FAM
Accounting policies
(a) Property, plant and equipment is stated at historical cost less depreciation, or at valuation.
(b) Depreciation is provided on all assets, except land, and is calculated to write down the cost
or valuation over the estimated useful life of the asset.
The principal rates are as follows.
Buildings 2% pa straight line
Plant and machinery 20% pa straight line
Fixtures and fittings 25% pa reducing balance

tools and equipment

account and assets


Fixed asset movements
Land and buildings

Fixtures, fittings,

in the course of
Payments on

construction
machinery
Plant and

Total
Cost/valuation Rs.000 Rs.000 Rs.000 Rs.000 Rs.000

Cost at 1 January 2016 900 1,613 390 91 2,994


Revaluation adjustment 600 – – – 600
Additions – 154 40 73 (W1) 267
Reclassifications 100 – – (100) –
Disposals – (277) (41) – (318)
——— ——— —— ——— ———
As at 31 December
2016 1,600 1,490 389 64 3,543
——— ——— —— ——— ———
Depreciation
At 1 January 2016 80 458 140 – 678
Revaluation adjustment (80) – – – (80)
Provisions for year (W2) 17 298 70 – 385
Disposals – (195) (31) – (226)
—— —— —— ——— ———
At 31 December 2016 17 561 179 – 757
—— —— —— ——— ———
Written down value 1,583 929 210 64 2,786
—— —— —— ——— ———
Land and buildings have been revalued during the year by Messrs Jackson & Co on the
basis of an existing use value on the open market.

© Emile Woolf International 174 The Institute of Chartered Accountants of Pakistan


Answers

The corresponding historical cost information is as follows.


Land and
buildings
Rs.000
Cost
Brought forward 900
Reclassification 100
———
Carried forward 1,000
———
Depreciation
Brought forward 80
Provided in year 10
———
Carried forward 90
———
Net book value 910
———
Workings
Rs.000
(1) Additions to assets under construction 53
Deposit on computer 20
——
73
——

Rs.000
600
(2) Depreciation on buildings 40 + (100  2%) 17
2% straight line depreciation is equivalent to a 50 year life.
The buildings are ten years old at valuation and therefore
have 40 years remaining.

Depreciation on plant (1,613 + 154 – 277)  20% 298

Depreciation on fixtures (390 + 40 – 41 – 140 + 31)  25% 70

6.2 GUJRAT CONSTRUCTION LIMITED


Property, plant and equipment note (extract) for the year ended 31 December 2016
construction
equipment
machinery
Plant and
buildings
Land and

Assets
Office

under

Total

Cost or valuation Rs Rs Rs Rs Rs
At 1 January 2016 1,500,000 1,276,500 356,400 - 3,132,900
Additions – 135,000 36,500 29,200 200,700
Classified as held
for sale – (50,000) – – (50,000)
Disposals – (104,000) – – (104,000)
At 31 December
2016 1,500,000 1,257,500 392,900 29,200 3,179,600

© Emile Woolf International 175 The Institute of Chartered Accountants of Pakistan


Advanced accounting and financial reporting

construction
equipment
machinery
Plant and
buildings
Land and

Assets
Office

under

Total
Depreciation
At 1 January 2016 315,000 879,300 210,400 – 1,404,700
Held for sale (W3) – (40,500) – – (40,500)
Disposals (W4) – (65,000) – – (65,000)
Impairment losses
(W1 and W3) 101,875 9,250 – – 111,125
Charge for year
(W2) 13,175 320,917 74,930 – 409,022
At 31 December
2016 430,050 1,103,967 285,330 – 1,819,347
Carrying amount
At 31 December
2016 1,069,950 153,533 107,570 29,200 1,360,253
At 1 January 2016 1,185,000 397,200 146,000 – 1,728,200

Workings
(W1) Impaired workshop
Rs.
Valuation on 31 December 2013 210,000
Depreciation to 31 December 2015 (210,000 ÷ 48  2) (8,750)
Depreciation to 31 December 2016 (210,000 ÷ 48) (4,375)
Carrying amount at 31 December 2016 196,875
Recoverable amount (100,000 – 5,000) (95,000)
Impairment 101,875
(W2) Depreciation charges for year
Land and buildings
Rs.
Impaired workshop (W1) 4,375
Other ((1,500,000 – 850,000 – 210,000 (W1)) ÷ 50) 8,800
13,175
Plant and machinery
Rs.
Depreciation on assets held for whole year
((1,276,500-50,000-104,000) × 25%) 280,625
Depreciation on asset disposed of (104,000 × 25% × 6/12) 13,000
Depreciation on additions (135,000 × 25% × 6/12) 16,875
Depreciation on asset classified as held for sale but not sold
(W3) 10,417
320,917
Office equipment
Rs.
Held for whole year (356,400 × 20%) 71,280
Additions (36,500 × 20% × 6/12) 3,650
74,930

© Emile Woolf International 176 The Institute of Chartered Accountants of Pakistan


Answers

(W3) Asset classified as held for sale


Rs.
Cost on 1 May 2014 50,000
Depreciation to 31 December 2015 (50,000 × 25% × (1 and
8/12)) (20,833)
Depreciation from 1 January 2016 to 1 November 2016 (50,000
× 25% × 10/12) (10,417)
Carrying amount at 1 November 2016 18,750
Fair value less costs to sell (11,000 – 1,500) (9,500)
Impairment loss on classification as held for sale (9,250)
Total to be eliminated on classification as held for sale (20,833 +
10,417 + 9,250) 40,500
(W4) Accumulated depreciation on disposal
Rs.
To 31 December 2015 (104,000 × 25% × 2) 52,000
For current year (W2) 13,000
65,000

© Emile Woolf International 177 The Institute of Chartered Accountants of Pakistan


Advanced accounting and financial reporting

CHAPTER 7 – NON-CURRENT ASSETS: SUNDRY STANDARDS

7.1 SPIN INDUSTRIES LIMITED


Rs.
Commitment fee 125,000
Actual borrowing costs of specific loan (W1) 2,050,000
General borrowing costs (W1) 1,175,283
Less: Investment income (W2) (137,500)
Interest costs to be capitalised 3,212,783

W1
Borrowing
Outstanding Outstanding
Rate of cost to be
amount Months outstanding month up to
interest capitalised
completion
Rs. Rs.
Specific loan
Utilised till first
repayment 25,000,000 1-Sep-15 31-Jan-16 5 12% 1,250,000
Utilised after
the first
repayment 20,000,000 1-Feb-16 31-May-16 4 12% 800,000
2,050,000
General
Borrowings (W4)
Utilised after
specific loan
exhausted on
nd
2 payment to
contractor
(W3) 8,125,000 1-Dec-15 31-May-16 6 12.08% 490,750
Principal
payment of
specific loan* 5,000,000 1-Feb-16 31-May-16 4 12.08% 201,333
3rd payment to 12.08%
contractor 12,000,000 1-Feb-16 31-May-16 4 483,200
4rd payment to 1-Jun-16 12.08%
contractor 9,000,000 31-May-16 0 -
1,175,283

W2: Investment income Rs.


Surplus fund available from 1-Sep-15 to 30-Nov-15
(Rs. 25m – Rs.0.125m – Rs. 8m – Rs. 10m) × 8% × 3/12 137,500

W3: Specific loan utilization


Commitment fee 125,000
Payment for obtaining permit 8,000,000
st
1 payment to contractor 10,000,000
nd
2 payment to contractor (balancing) 6,875,000
25,000,000
nd
2 payment to contractor (total) 15,000,000
Less: paid out of specific loan (as worked out above) 6,875,000
Paid from general borrowing 8,125,000

© Emile Woolf International 178 The Institute of Chartered Accountants of Pakistan


Answers

W4: Weighted average rate of borrowing

Weighted average
amount of loan Interest Rs.
Rs.
From Bank A 25,000,000 Rs. 25,000,000 × 13% × 9/12 = 2,437,500
From Bank B 20,000,000 3,000,000
45,000,000 5,437,500

Weighted average rate of borrowing (Rs. 5,437,500 / 45,000,000) 12.08%

7.2 QURESHI STEEL LIMITED


Capital work in progress – Factory building Rs.000
Progress invoices received from the contractor
(30,000+20,000+10,000+15,000) 75,000.00
(Rain damages paid would be chargeable to profit and loss account/ insurance claim)

Borrowing costs to be capitalised:


Loan processing charges 500.00
Interest on bank loan W1 1,841.67
Interest on running finance W2 2,730.00
Interest income from surplus loan amount W4 (395.00)
Capital work in progress – June 30, 2016 79,676.67

W1: Interest on bank loan:


Rs.000
Interest amount Outstanding Interest
From To Months loan amount at 13%
01-12-2015 31-05-2016 6 25,000 1,625.00
01-06-2016 30-06 -2016 1 20,000 216.67
1,841.67

W2: Interest on running finance


Rs.000
Payments from Interest
Payment Months at 15%
s Runnin
Right Bank outstandin per
Payments Invoice net of g g up to annum
issue loan
date Description amount deductions finance 30-6-10 (W3)
01-07-15 Advanced 10,00
payment 0 10,000 10,000 12.00 1,500
st
15-10-15 1
progress 30,00
bill 0 25,500 15,000 10,500 8.50 1,116
nd
15-01-16 2
progress 20,00
bill 0 17,000 17,000 - - -
rd
15-04-16 3
progress 10,00
bill 0 8,500 7,500 1,000 2.50 31
31-05-16 Loan
interest 1,625 1,625 1.00 20
31-05-16 Loan
instalment 5,000 5,000 1.00 63
15,000 *24,500 28,125 2,730

© Emile Woolf International 179 The Institute of Chartered Accountants of Pakistan


Advanced accounting and financial reporting

W2: Interest on running finance


Rs.000
Payments from Interest
Payment Months at 15%
s Runnin outstandin per
Right Bank
Payments Invoice net of g g up to annum
issue loan
date Description amount deductions finance 30-6-10 (W3)
*Loan amount of Rs. 25,000,000 less processing charges of Rs. 500,000
W3: Average rate of interest for running finance facility (9,000/60,000) 15%
W4: Interest income from surplus loan amounts:
Rs.000
Interest income Interest income
Surplus loan amounts
From To Months at 8%
01-12-15 15-01-13 1.5 24,500 (245)
16-01-16 15-04-13 3.0 7,500 (150)
(395)

7.3 IMRAN LIMITED


(a) Specific borrowings
Rs.
Borrowing costs incurred:
13% bank loan outstanding for 10 months
306
(Rs. 32 million x /365 x 13%) 3,487,562
11% bank loan outstanding for 5 months
153
(Rs. 10 million x /365 x 11%) 461,096
Borrowing costs 3,948,658
Less: Interest that relates to suspension
61
13% bank loan: (Rs. 32 million x /365 x 13%) 695,233
61
11% bank loan (Rs. 10 million x /365 x 11%) 183,836
(879,068)
3,069,590
Less: Investment income on temporary investment of the
borrowings (500,000)
2,569,590

(b) General borrowings


Phase 1 Phase 2 Phase 3
Building cost capitalised 20,000,000 18,000,000 16,000,000
Financed out of rights issue (15,000,000)
Financed from borrowing 5,000,000 18,000,000 16,000,000
Period to the year end
March 1 to December 31 306
April 1 to December 31 275
October 1 to December 31 92
Period of suspension (61) (61) 
Number of days for which
borrowing should be capitalised 245 214 92

© Emile Woolf International 180 The Institute of Chartered Accountants of Pakistan


Answers

Phase 1 Phase 2 Phase 3


Weighted average borrowing
rate (W3)  12.73%  12.73%  12.73%
Fraction of the year for which
the rate should be applied to
  
245 214 92
costs incurred /365 /365 /365
Capitalised borrowing 427,240 1,343,451 513,385
Total 2,284,076
Workings
W1: Average borrowings Rs. m
13% bank loan outstanding for 10 months
306
(Rs. 32 million x /365 days) 26,827,397
11% bank loan outstanding for 5 months
153
(Rs. 10 million x /365 days) 4,191,781
Average outstanding for the year 31,019,178
W2: Borrowing costs incurred (or from part a) Rs. m
13% bank loan outstanding for 10 months
306
(Rs. 32 million x /365 x 13%) 3,487,562
11% bank loan outstanding for 5 months
153
(Rs. 10 million x /365 x 11%) 461,096
Borrowing costs 3,948,658
W3: Weighted average rate
Borrowing costs
/ Average outstanding for the year =
3,948,658 (W2)
/31,019,178 (W1) = 12.73%
7.4 KATIE
Option 1 – Net grants off related expenditure
Statement of financial position as at 30 June Year 2 (extracts)
Rs.
Non-current assets
Property, plant and equipment 223,333
Current liabilities
Other current liabilities 100,000

Notes to the financial statements for the year ended 30 June Year 2 (extracts)
Property, plant and equipment Rs.
Cost (350,000 – 100,000) 250,000
Accumulated depreciation ((250,000 – 50,000) ÷ 5  8/12) (26,667)
––––––––

Carrying amount 223,333


––––––––

Included in statement of profit or loss for the year ended 30 June Year 2
Rs.
Depreciation charge 26,667
Training costs (70,000 – 40,000) 30,000

© Emile Woolf International 181 The Institute of Chartered Accountants of Pakistan


Advanced accounting and financial reporting

Option 2 – Show grants separately from related expenditure


Statement of financial position as at 30 June Year 2 (extracts)
Rs.
Non-current assets
Property, plant and equipment 310,000

Current liabilities
Other current liabilities 186,667

Notes to the financial statements for the year ended 30 June Year 2 (extracts)
Rs.
Property, plant and equipment
Cost 350,000
Accumulated depreciation ((350,000 – 50,000) ÷ 5  8/12) (40,000)
––––––––
Carrying amount 310,000
––––––––
Other current liabilities
Deferred income relating to government grants 86,667
(100,000 - (100,000 ÷ 5  8/12))
Government grant repayable 100,000
––––––––
186,667
––––––––
Included in statement of profit or loss for the year ended 30 June Year 2
Rs.
Depreciation charge 40,000
Training costs 70,000
Government grant received (40,000)
Release of deferred government grant (13,333)

Tutorial note
The Rs. 100,000 grant in (3) has conditions attached to it. In such a situation, IAS 20 states that
grants should not be recognised until there is reasonable assurance that the entity will comply
with any conditions attaching to the grant. Since Katie is struggling to recruit, and there is only
one month left for recruitment to meet these conditions, then it does not seem that there is
‘reasonable assurance’. Hence the grant should not be recognised as such, but should be held in
current liabilities, pending repayment.

7.5 ALNUS LTD AND BUTEA LIMITED


Government grant
The current accounting treatment is not correct. IAS 20 Accounting for Government Grants and
Disclosure of Government Assistance requires government grants to be recognised in profit or
loss over the periods in which the entity recognises as expenses the costs which the grants are
intended to compensate.
Government grants provided to assist in the acquisition of an asset should be presented in the
statement of financial position either:
1. By setting up the grant as deferred income in the statement of financial position, and
recognising it in profit or loss on a systematic basis over the useful life of the asset,
normally corresponding to the method of depreciation of the asset.
2. By deducting the grant in arriving at the carrying amount of the asset (i.e. netting off),
thereby reducing the depreciation charge. This method will make the entity less
comparable with a similar entity without government assistance.

© Emile Woolf International 182 The Institute of Chartered Accountants of Pakistan


Answers

For Alnus Ltd, therefore, the two methods will result in the following presentation in the statement
of financial position and statement of profit or loss:
Statement of financial position
Method 1 Method 2
Rs. Rs.
Asset (400 – 200) 400,000 200,000
less depreciation ((400/200)/4yrs × 9/12) (75,000) (37,500)
325,000 162,500
Liabilities
Current – deferred income (12/48 × 200) 50,000 –
Non-current – deferred income (27/48 × 200) 112,500 –
162,500 –
Statement of profit or loss
Depreciation (75,000) (37,500)
Deferred income (9/48 × 200) 37,500
The net effect on income and net assets will be the same under either method: grant income is
matched with the use of the asset. The grant recognised in the year is Rs. 37,500 (Rs. 200,000/4
years × 9/12
Capitalised borrowing costs
Under IAS 23 Borrowing Costs certain borrowing costs form part of the cost of a qualifying asset.
A qualifying asset is an asset that takes a substantial period of time to get ready for its intended
use or sale. This includes property, plant and equipment provided it is not ready for use. The
development of the holiday park is therefore a qualifying asset, as it was in disrepair and not in
use.
Borrowing costs are defined as interest and other costs that an entity incurs in connection with
the borrowing of funds. Only borrowing costs that are directly attributable to the acquisition,
construction or production of the qualifying asset should be capitalised – which are those
borrowing costs that would have been avoided if the expenditure on the qualifying asset had not
been made. If the construction is financed out of the general borrowing of the entity, then:
 The borrowing costs capitalised should be calculated by reference to the weighted average
cost of the borrowings.
 The weighted average calculation should exclude borrowings to finance a specific purpose
or building. In the case of Butea Ltd, although there are two loans, neither is for a specific
purpose, and so the weighted average cost of the loans should be used to determine the
borrowing cost rate for capitalisation purposes.
The correct calculation of the weighted average borrowing rate is therefore:
((Rs. 1.8m × 5%) + (Rs. 1.2m × 8.5%))/Rs. 3m = 6.4%
Capitalisation of borrowing costs should commence when the entity meets all three of the
following conditions:
1. It incurs expenditures for the asset (1 January 2017).
2. It incurs borrowing costs (already being incurred).
3. It undertakes activities that are necessary to prepare the asset for its intended use or sale.
Such activities include obtaining planning permission. (1 November 2016) January 2017 is the
date at which the last of the three conditions was met, so the period for capitalisation is six
months (1 January to 30 June 2017) rather than nine months.
The amount to be capitalised is therefore: Rs. 800,000 × 6.4% × 6/12 = Rs. 25,600, and an
adjustment to correct finance costs capitalised in error is needed of Rs. 25,400 (51,000 –
25,600).
As this is a correction in the financial statements of Butea Ltd, the Alnus Ltd group will need to
apportion this between the group (80%) and NCI (20%), so that group profit will fall by Rs. 20,320
(25,400 × 80%) and NCI by Rs. 5,080 (25,400 × 20%).

© Emile Woolf International 183 The Institute of Chartered Accountants of Pakistan


Advanced accounting and financial reporting

7.6 VICTORIA
(a) Treatment in the financial statements for the year ended 31 December Year 8 (IAS16)
Property 1
This is used by Victoria as its head office and therefore cannot be treated as an investment
property. It will be stated at cost minus accumulated depreciation in the statement of
financial position. The depreciation for the year will be charged in the statement of profit or
loss.
Property 2
This is held for its investment potential and should be treated as an investment property. It
will be carried at fair value, Victoria’s policy of choice for investment properties. It will be
revalued to fair value at each year end and any resultant gain or loss taken to the
statement of profit or loss (Rs. 400,000 gain in Year 8).
Property 3
This is held for its investment potential and should be treated as an investment property.
However, since its fair value cannot be arrived at reliably it will be held at cost minus
accumulated depreciation in the statement of financial position. The depreciation for the
year will be an expense in the statement of profit or loss.
This situation provides the exception to the rule whereby all investment properties must be
held under either the fair value model, or the cost model.
(b) Analysis of property, plant and equipment for the year ended 31 December Year 8
Other land Investment Investment
and buildings property held property held
(W1) at fair value at cost (W2) Total
Rs. Rs. Rs. Rs.
Cost/valuation
On 1 January Year 8 1,000,000 2,300,000 2,000,000 5,300,000
Revaluation - 400,000 - 400,000
––––––––– ––––––––– ––––––––– –––––––––
On 31 December Year 8 1,000,000 2,700,000 2,000,000 5,700,000
––––––––– ––––––––– ––––––––– –––––––––
Accumulated depreciation
On 1 January Year 8 87,500 - 220,000 307,500
Charge for the year (W1) 12,500 - 40,000 52,500
––––––––– ––––––––– ––––––––– –––––––––
On 31 December Year 8 100,000 - 260,000 360,000
––––––––– ––––––––– ––––––––– –––––––––
Carrying amount
On 31 December Year 7 912,500 2,300,000 1,780,000 4,992,500
––––––––– ––––––––– ––––––––– –––––––––
On 31 December Year 8 900,000 2,700,000 1,740,000 5,340,000
––––––––– ––––––––– ––––––––– –––––––––
Tutorial note
In practice, with a more complex property, plant and equipment table the investment
properties would be included within the land and buildings column with the required
disclosures being given separately in a note to the table.
Workings
(1) Depreciation on Property 1
Rs.
Brought forward (500,000 ÷ 40  7) 87,500
Year 8 (500,000 ÷ 40) 12,500
(2) Depreciation on Property 3
Rs.
Brought forward (2,000,000 ÷ 50  5.5) 220,000
Year 8 (2,000,000 ÷ 50) 40,000

© Emile Woolf International 184 The Institute of Chartered Accountants of Pakistan


Answers

CHAPTER 8 – IAS 38: INTANGIBLE ASSETS

8.1 BROOKLYN
1 Development expenditure
IAS 38 on intangibles requires that research and development be considered separately:
 research – which must be expensed as incurred
 development – which must be capitalised where certain criteria are met.
It must first be clarified how much of the Rs. 3 million incurred to date (10 months at Rs.
300,000) is simply research and how much is development. The development element will
only be capitalised where the IAS 38 criteria are met. The criteria are listed below together
with the extent to which they appear to be met.
 The project must be believed to be technically feasible. This appears to be so as the
feasibility has been acknowledged.
 There must be an intention to complete and use/sell the intangible. Completion is
scheduled for June 2017
 The entity must be able to use or sell the intangible. Interest has been expressed in
purchasing the knowhow on completion
 It must be considered that the asset will generate probable future benefits.
Confirmation is required from Brooklyn as to the extent of interest shown by the
pharmaceutical companies and whether this is of a sufficient level to generate
orders and to cover the deferred costs.
 Availability of adequate financial and technical resources must exist to complete the
project. The financial position of Brooklyn must be investigated. A grant is being
obtained to fund further work and the terms of the grant, together with any
conditions, must be discussed further.
 Able to identify and measure the expenditure incurred. A separate nominal ledger
account has been set up to track the expenditure.
If all of the above criteria are met, then the development element of the Rs. 3m incurred to
date must be capitalised as an intangible asset. Amortisation will not begin until
commercial production commences.
2 Provision
Although the claim was made after the reporting period, IAS 10 considers this to be an
adjusting event after the reporting period. The employment of the individual dates back to
20X2 and so the lawsuit constitutes a current obligation for the payment of damages as a
result of this past event (the employment).
The amount and the timing are not precisely known but the likelihood of payment of
damages by Brooklyn is probable and so a provision should be made for the estimated
amount of the liability, as advised by the lawyer. Disclosure, rather than provision, would
only be appropriate if the expected settlement was possible or remote, and the lawyer’s
view is that a payment is more likely than not.
It is not appropriate to calculate an expected value where there is only one event, instead
a provision should be made for the most likely outcome. The lawyer has various views on
the possible, but the most likely payout is Rs. 500,000 as this has a 50% probability. As
settlement of the provision is not anticipated until 2019, the provision should be discounted
back at 8% to give a liability of Rs. 476,280.
Provided that the payment from the insurance company is virtually certain, this should be
shown as an asset, also at its discounted value of Rs. 47,628, being 10% of the provision.
In both cases the discounting should be unwound over the coming three years through
profit or loss.

© Emile Woolf International 185 The Institute of Chartered Accountants of Pakistan


Advanced accounting and financial reporting

3 Revaluation
IAS 16 on Property, Plant and Equipment does not impose a frequency for updating
revaluations. It simply requires a revaluation where it is believed that the fair value of the
asset has materially changed. Hence, if in the past there have been material differences
between the carrying amount and fair value at the 5 yearly review then Brooklyn should
consider having more frequent valuations following on from this year’s valuation.
Revaluations should be regular and not timed simply when property prices are at a peak. It
is not acceptable for Brooklyn to defer its next revaluation while values are low. If property
prices do fall in 2017, then it may be necessary to perform an impairment test in
accordance with IAS 36 Impairment of assets.
If it is believed that an asset value has moved materially, then all assets in that class must
be revalued. Hence it is not sufficient for Brooklyn to just revalue the London property.
IAS 16 does not require the valuation to be performed by an external party, and so the use
of the property manager to conduct the valuations is acceptable. Notes to the financial
statements will disclose that he is not independent of the company.

8.2 RAISIN INTERNATIONAL


(a) Following are the criteria that should be used while recognizing intangible assets from
research and development work.
(i) No intangible asset arising from research shall be recognised.
(ii) An intangible arising from development shall be recognised if, and only if , an entity
can demonstrate all of the following:

 the technical feasibility of completing the intangible asset so that it will be


available for use or sale.

 its intention to complete the intangible asset and use or sell it.
 its ability to use or sell the intangible asset.
 how the intangible asset will generate probable future economic benefits.
Among other things, the entity can demonstrate the existence of a market for
the output of the intangible asset or the intangible asset itself or, if it is to be
used internally, the usefulness of the intangible asset.

 the availability of adequate technical, financial and other resources to complete


the development and to use or sell the intangible asset.

 its ability to measure reliably the expenditure attributable to the intangible asset
during its development.
(b) (i) Since the product met all the criteria for the development of the product, it should
be recognised as an intangible in the statement of financial position (SOFP) of the
company. However, RI should capitalise only the development work (i.e. Rs. 9
million) as intangible asset. IAS-38 does not allow capitalization of cost relating to
the research work, training of staff and cost of trial run.
Since the product has a useful life of 7 years, the amortization expense amounting
to Rs.0.32 million (Rs. 9 million × 3/12 ÷ 7 years) should be recorded in the
statement of profit or loss and other comprehensive income (SOCI).
(ii) This purchasing of right to manufacture should be recognised as an intangible in
the SOFP because:
 it is for an established product which would generate future economic benefits.
 cost of the patent can be measured reliably.
Since there is a finite life, the patent must be amortised over its useful life. The

© Emile Woolf International 186 The Institute of Chartered Accountants of Pakistan


Answers

useful life will be shorter of its actual life (i.e. 10 years) and its legal life (i.e. 5 years.
The amortization to be recorded in SOCI is Rs. 2.83 million (Rs. 17 million × 10/12
÷ 5).
(iii) The acquired brand should be recognised as an intangible in the SOFP because
acquisition price is a reliable measure of its value. The amortization to be recorded
in SOCI is Rs.0.12 million (Rs. 2 million ÷ 10 years x 7/12).
(iv) The carrying value of the intangible asset should be increased to Rs. 10 million in
the SOFP. Since there is an indefinite useful life of the intangible assets, it should
not be amortised. Instead, RI should test the intangible asset for impairment by
comparing its recoverable amount with its carrying amount. Impairment testing
should be done at least annually

8.3 OXTAIL LTD


(a) Explanation of required IFRS accounting treatment
(1) Research and development costs
In accordance with IAS 38 Intangible Assets the evolution of internally generated
assets (other than goodwill) should be split into the research phase and the
development phase. All expenditure that arises in the research phase should be
recognised as an expense when it is incurred. Hence, the initial Rs. 70,000 spent on
research should be written off as an expense in 2016 to profit or loss.
Development costs qualify for recognition as an intangible asset provided that the
entity can demonstrate that the strict criteria of IAS 38 are met:
 It is technically feasible to complete the asset.
 The entity intends (and is able to) to complete the asset and use or sell it.
 The asset will generate probable future economic benefits.
 Adequate technical, financial and other resources are available to allow
completion/use/sale of the asset.
 The development expenditure can be reliably measured.
Once these criteria are met the development expenditure should be capitalised. The
phrase in the question 'would be economically viable' and the fact that Oxtail Ltd
goes on to complete the asset and start production, are taken to mean that these
criteria were met. Hence, the further costs of Rs. 120,000 incurred after the project
review should be capitalised as an intangible asset, within non-current assets.
In general, under IAS 38, an intangible asset should only be recognised in the
financial statements if it can be measured reliably and future economic benefits can
be attributed to it. Hence, the promotional advertising costs of Rs. 15,000 should not
be capitalised as it is difficult to attribute specific future economic benefits to it as
Oxtail Ltd cannot know how successful the advertising per se has been.
Accordingly, IAS 38 states that expenditure on training, advertising and promotional
activities and relocation/reorganisation should be recognised as an expense when it
is incurred. Therefore only Rs. 105,000 of the Rs. 120,000 spent should be
capitalised. Amortisation of the Rs. 105,000 will commence on 1 January 2017,
when production of the new product commences.
(2) Technical know-how
According to IAS 38 an item is recognised as an intangible asset if it is identifiable,
its cost can be measured reliably, and it is probable that future economic befits can
be attributed to it.
The technical know-how is identifiable as it has been purchased. Costs appear to
have been reliably measured. Given that this know-how will 'completely change the
way (the entity’s) manufacturing process operates' it also appears to be probable
that future economic benefits can be attributed to it.

© Emile Woolf International 187 The Institute of Chartered Accountants of Pakistan


Advanced accounting and financial reporting

The costs which should be capitalised are those which can be directly attributed to
creating, producing or preparing the asset for its intended use. Therefore the
purchase cost of Rs. 180,000, legal costs of Rs. 4,000, supervisors’ time of Rs.
3,200 and testing costs of Rs. 4,800 (a total of Rs. 192,000) can all be capitalised.
The costs of staff training of Rs. 13,000 (see above) must be expensed as incurred.
Once an intangible asset has been recognised, it should be carried under the cost
model or the revaluation model. Oxtail Ltd use the cost model so this intangible will
be carried at cost less any accumulated amortisation and any accumulated
impairment losses.
An intangible asset with a finite useful life should be amortised over its expected
useful life. The know-how of Rs. 192,000 should therefore be amortised over the
four year 'life' of the new manufacturing process. Amortisation for the six months to
31 December 2016, will therefore be calculated as Rs. 24,000 (Rs. 192,000/4 x
6/12). This gives the know-how a carrying amount at the point that an impairment is
identified of Rs. 168,000 (Rs. 192,000 – Rs. 24,000).
According to IAS 36: Impairment of Assets, if there is an indication of impairment,
the asset’s carrying amount of Rs. 168,000 should be compared to its recoverable
amount. If the recoverable amount is lower than the carrying amount then an
impairment loss should be recognised in the statement of profit or loss for the
period.
The recoverable amount of an asset is defined by IAS 36 as the higher of the
asset’s fair value less costs to sell (here Rs. 152,000) and its value in use (here Rs.
157,000). So the recoverable amount of the know-how is Rs. 157,000 and the Rs.
168,000 should be written down to that amount – ie by Rs. 11,000 (Rs. 168,000 –
Rs. 157,000). The Rs. 11,000 should be recognised as an expense in profit or loss
for the current year.
(b) Summary of costs included in profit or loss for the year ended 31 December 2016
Rs.
Amortisation (see a) 24,000
Impairment of know-how (see a) 11,000
Research costs 70,000
Promotional advertising costs 15,000
Staff training costs 13,000
Statement of financial position as at 31 December 2016 (extract)
Rs.
Non-current assets:
Intangible assets (120,000 – 15,000 + 157,000 (OF)) 262,000

8.4 SKY LIMITED


(i) Cost incurred in the planning stage should be expensed out as research.
(ii) (a) Cost incurred on development of internal website should be charged off because
the benefits (if any) cannot be estimated reliably.
(b) Cost of External Website
 Cost incurred on development of external website including the cost of
linking it to credit card facilities should be capitalized because it can be
established that external revenue is generated directly with the use of such
website through external orders.
 However, a reasonable estimate of future revenues should be made for
impairment testing.

© Emile Woolf International 188 The Institute of Chartered Accountants of Pakistan


Answers

(iii) (a) Cost of purchase of servers plus cost of their operating software should be
capitalized as tangible assets in line with the requirements of IAS 16 and
depreciated according to their expected useful economic life.
(b) Cost of purchase of software licenses other than operating software should be
capitalized as intangible assets because economic benefit is accruing to the
company.
(iv) Cost of maintenance of websites is a recurring expenditure and should be expensed out.
(v) IAS-38 does not allow the capitalization of training costs. Therefore, these must be
expensed.
(vi) Cost of advertising should be expensed as and when incurred.

8.5 COMFORT SHOES LIMITED


(a) In accordance with the IAS transactions related to the trademark as given in the question
should be accounted for as explained below:
(i) As the costs and benefits of the trade mark cannot be measured reliably, and it was
not even decided at that time to buy the trademark, the cost of Rs. 1 million incurred
in 2010 to carry out the market survey should have been expensed in the year 2010.
(ii) In 2011, the rights to use the trademark for the company’s products have been
obtained and costs and benefits of the trademark were measured reliably. Therefore,
initially the trademark should have been accounted for as an intangible asset at a cost
of Rs. 5 million.
At that time the trademark was estimated to have an indefinite useful life as there was
an expectation that it will contribute to net cash inflows indefinitely. Therefore, the
trademark should not have been amortised.
However, the trademark should have been tested for impairment and the cost should
have been reduced, if required.
A trademark fee payable at 1% of annual sales should have been treated as a
periodical cost and charged to expense in the year of sales.
(b) Comfort Shoes Limited
Notes to the Financial Statements
For the year ended December 31, 2015
1 Intangible Assets – Trademark

2016 2015
Rupees in ‘000
Cost January 1 4,500 5,000
For the year - impairment - (500)
December 31 4,500 4,500
Amortization January 1 - -
For the year 2,250 -
December 31 2,250 -
Carrying amount December 31 2,250 4,500
% / useful life 50% / 2
years -

1.1 The amortisation expense for the year has been allocated to cost of sales.

© Emile Woolf International 189 The Institute of Chartered Accountants of Pakistan


Advanced accounting and financial reporting

CHAPTER 9 – IAS 36: IMPAIRMENT OF ASSETS

9.1 CHARLOTTE
Effect on Year 7 profit or loss

Rs.
Impairment loss
Machine 1 (W1) 122,300
Machine 2 (W2) 41,000
163,300
Depreciation charge
Machine 1: (100,000 ÷ 5) 20,000

Gain on disposal
Machine 2: (W2) 10,000
Machine 3: (210,000 - 195,000 (W2)) 245,000
255,000

Workings
(1) Machine 1

Rs.
Cost on 1 January Year 1 420,000
Depreciation to 1 January Year 6
5 years  ((420,000 – 50,000)/10 years)) (185,000)
Carrying amount on 1 January Year 6 235,000
Revalued to: 275,000
Revaluation gain before tax 40,000

In the year to 31 December Year 6 (on 1 January), the asset is revalued upwards by Rs.
40,000. Of this, Rs. 28,000 is taken to the revaluation reserve and Rs. 12,000 (Rs. 40,000
 30%) to deferred tax as a liability.

Dr (Rs.) Cr (Rs.)
Property, plant and equipment 145,000
Accumulated depreciation 185,000
Net effect on non-current assets 40,000
Revaluation surplus 28,000
Deferred tax liability 12,000

The total useful life of the asset was assessed as 15 years on 1 January Year 6. The asset
has already been owned for 5 years and depreciation in year 6 is based on the remaining
useful life of 10 years.
The company must also recognise incremental depreciation in accordance with section
235 of the Companies’ Act, 2017. An amount equal to the incremental depreciation net of
deferred taxation must be transferred to retained earnings through the statement of
changes in equity.

© Emile Woolf International 190 The Institute of Chartered Accountants of Pakistan


Answers

Dr (Rs.) Cr (Rs.)
Depreciation charge for the year
(275,000/10 years) 27,500
Accumulated depreciation 27,500
Revaluation surplus
(Rs. 28,000/10 years) 2,800
Retained earnings 2,800
Impairment loss:
Rs.
Carrying amount on 1 January Year 6 275,000
Depreciation to 1 January Year 7 (275,000 ÷ (15 – 5)) (27,500)
Carrying amount at 1 January Year 7 247,500
Recoverable amount (100,000)
Impairment loss 147,500

In the year to 31 December Year 7, the impairment loss is Rs. 147,500. Of this, Rs. 40,000
reverses the gain in the previous year. The revaluation reserve is reduced by Rs. 25,200
(Rs. 28,000 – Rs. 2,800). The remaining impairment loss of Rs. 122,300 is written off as a
loss in Year 7.
Also in the year to 31 December Year 7 the asset would be depreciated based on the
estimate of its remaining useful life of 5 years giving a charge of Rs. 20,000 (Rs. 100,000/
5 years).
(2) Machine 2
Rs.
Cost on 1 January Year 1 500,000
Depreciation to 1 January Year 7
6 years  ((500,000 – 60,000)/10 years)) (264,000)
Carrying amount on 1 January Year 7 236,000
Fair value minus cost to sell (200,000 – 5,000) (195,000)
Impairment loss 41,000

On 31 March Year 7 the machine is sold for Rs. 210,000 giving a gain on sale as follows:
Rs.
Proceeds 210,000
Selling costs (assumed to be as forecast) (5,000)
205,000
Carrying amount (195,000)
10,000
(3) Machine 3
Rs.
1 January Year 1 Cost 600,000
Depreciation to 1 January Year 2 (30,000)
Carrying amount on 1 January Year 2 570,000
Revalued to 800,000
Taken to revaluation reserve/deferred tax 230,000

© Emile Woolf International 191 The Institute of Chartered Accountants of Pakistan


Advanced accounting and financial reporting

The revaluation would have been accounted for as follows at 1 January Year 2
Dr (Rs.) Cr (Rs.)
Property, plant and equipment 200,000
Accumulated depreciation 30,000
Net effect on non-current assets 230,000
Revaluation surplus 161,000
Deferred tax liability 69,000
Depreciation and incremental depreciation would have been recognised in Year 2 to Year
6 inclusive as follows:
Dr (Rs.) Cr (Rs.)
Depreciation charge for the year
(800,000/8 years) 100,000
Accumulated depreciation 100,000

Revaluation surplus
(Rs. 161,000/8 years) 20,125
Retained earnings 20,125
This would result in balances for machine 3 and the revaluation surplus in respect of
machine 3 as follows:
Revaluation
Machine 3 surplus
Rs. Rs.
Carrying amount on1 January Year 2 800,000 230,000
Depreciation (5 years) (500,000)
Incremental depreciation (5 years) (100,625)
Balance at 1 January Year 7 300,000 129,375

Fair value on classification as held for sale 550,000


Costs to sell (5,000)
Fair value less costs to sell 545,000
Value at lower of carrying amount and fair
value less costs to sell: 300,000

On 31 March Year 7 the machine is sold for Rs. 550,000 giving a gain on sale as follows:
Rs.
Proceeds 550,000
Selling costs (assumed to be as forecast) (5,000)
545,000
Carrying amount (300,000)
245,000

The balance on the revalution reserve is transferred to retained earnings on the disposal of
the asset.
Dr (Rs.) Cr (Rs.)
Revaluation surplus 129,375
Retained earnings 129,375

© Emile Woolf International 192 The Institute of Chartered Accountants of Pakistan


Answers

9.2 ABA LIMITED


Aba Limited statement of profit or loss (extracts) – year to 31 March 2016
Note: workings in brackets are in Rs.000 Rs. Rs.
Depreciation: head office – 6 months to 1 October 2015
(1,200/25  6/12) 24,000
– 6 months to 31 March 2016
(1,350/22.5 (W1)  6/12) 30,000
–––––––
54,000
–––––––
Depreciation: training premises
– 6 months to 1 October 2016
(900/25  6/12) 18,000
– 6 months to 31 March 2016
(600/10  6/12) 30,000
––––––––
48,000
––––––––
Impairment loss (W2) 210,000
––––––––
258,000
––––––––
Statement of financial position (extracts) as at
31 March 2016 Rs. Rs.
Non-current assets
Land and buildings – head office (700 + 1,350 – 30) 2,020,000
– training premises (350 + 600 – 30) 920,000
––––––––
2,940,000
––––––––
Revaluation reserve
Head office land (700 – 500) 200,000
Building (1,350 – 1,080 (W1)) 270,000
Training premises land (350 – 300) 50,000
––––––––
520,000
Transfer to realised profit (270/22.5 (W1)  6/12
re depreciation of buildings) (6,000)
––––––––
514,000
––––––––
Workings
(W1) The date of the revaluation is two and a half years after acquisition. This means the
remaining life of the head office would be 22.5 years. The carrying value of the head
office building at the date of revaluation is Rs. 1,080,000 i.e. its cost less two and a half
years at Rs. 48,000 per annum (Rs. 1,200,000 – Rs. 120,000).
(W2) Impairment loss: the carrying value of training premises at date of revaluation is Rs.
810,000 i.e. its cost less two and a half years at Rs. 36,000 per annum (Rs. 900,000 – Rs.
90,000). It is revalued down to Rs. 600,000 giving a loss of Rs. 210,000. As the land and the
buildings are treated as separate assets the gain on the land cannot be used to offset the
loss on the buildings.

© Emile Woolf International 193 The Institute of Chartered Accountants of Pakistan


Advanced accounting and financial reporting

9.3 HUSSAIN ASSOCIATES LTD


(a) Impairment of plant
The plant had a carrying amount of Rs. 240,000 on 1 October 2015. The accident that may
have caused impairment occurred on 1 April 2016 and an impairment test would be done
at this date. The depreciation on the plant from 1 October 2015 to 1 April 2016 would be
Rs. 40,000 (640,000 x 121/2% x 6/12) giving a carrying amount of Rs. 200,000 at the date
of impairment. An impairment test requires the plant’s carrying amount to be compared
with its recoverable amount. The recoverable amount of the plant is the higher of its value
in use of Rs. 150,000 or its fair value less costs to sell. If Hussain Associates Ltd trades in
the plant it would receive Rs. 180,000 by way of a part exchange, but this is conditional on
buying new plant which Hussain Associates Ltd. is reluctant to do. A more realistic amount
of the fair value of the plant is its current disposal value of only Rs. 20,000. Thus the
recoverable amount would be its value in use of Rs. 150,000 giving an impairment loss of
Rs. 50,000 (Rs. 200,000 – Rs. 150,000).
The remaining effect on income would be that a depreciation charge for the last six months
of the year would be required. As the damage has reduced the remaining life to only two
years (from the date of the impairment) the remaining depreciation would be Rs. 37,500
(Rs. 150,000/ 2 years  6/12).Thus extracts from the financial statements for the year
ended 30 September 2016 would be:

Statement of financial position


Non-current assets Rs.
Plant (150,000 – 37,500) 112,500

Statement of profit or loss


Plant depreciation (40,000 + 37,500) 77,500
Plant impairment loss 50,000

(b) Purchase of Sparkle

There are a number of issues relating to the carrying amount of the assets of Sparkle
Limited that have to be considered. It appears the value of the brand is based on the
original purchase of the ‘Sparkle Spring’ brand. The company no longer uses this brand
name; it has been renamed ‘Refresh’. Thus it would appear the purchased brand of
‘Sparkle Spring’ is now worthless. Sparkle Limited cannot transfer the value of the old
brand to the new brand, because this would be the recognition of an internally developed
intangible asset and the brand of ‘Refresh’ does not appear to meet the recognition criteria
in IAS 38. Thus prior to the allocation of the impairment loss the value of the brand should
be written off as it no longer exists.
The inventories are valued at cost and contain Rs. 2 million worth of old bottled water
(Sparkle Spring) that can be sold, but will have to be relabelled at a cost of Rs. 250,000.
However, as the expected selling price of these bottles will be Rs. 3 million (Rs. 2 million 
150%), their net realisable value is Rs. 2,750,000. Thus it is correct to carry them at cost
i.e. they are not impaired. The future expenditure on the plant is a matter for the following
year’s financial statements.
Applying this, the revised carrying amount of the net assets of Sparkle Limited’s cash-
generating unit (CGU) would be Rs. 25 million (Rs. 32 million – Rs. 7 million re the brand).
The CGU has a recoverable amount of Rs. 20 million, thus there is an impairment loss of
Rs. 5 million. This would be applied first to goodwill (of which there is none) then to the
remaining assets pro rata. However under IAS2 the inventories should not be reduced as
their net realisable value is in excess of their cost. This would give revised carrying
amounts at 30 September 2016 of:

© Emile Woolf International 194 The Institute of Chartered Accountants of Pakistan


Answers

Rs.000
Brand nil
Land containing spa: 12,000 – [(12,000/20,000)  5,000] 9,000
Purifying and bottling plant:
8,000 – [(8,000/20,000)  5,000] 6,000
Inventories 5,000
20,000

9.4 IMPS
(a) Impairment loss

Rs. m
Carrying value 500
Recoverable amount (385)
Impairment loss 115

Recoverable amount is value in use (Working 1) as this is higher than the fair value less
costs of disposal (Working 2).
Workings
(1) Value in use:
Forecast cash flows discounted at 12%:
Rs. m
Year 1 (185 × 0.893) 165.2
Year 2 (160 × 0.797) 127.5
Year 3 (130 × 0.712) 92.6
Total 385.3

(2) The fair value less costs of disposal:

Rs. m
Goodwill 0
Freehold 270
Freehold land and buildings 50
320

(b) Treatment of impairment loss


IAS 36 requires the impairment loss to be allocated to the various non-current assets in the
following order: firstly, goodwill, secondly, to other assets, either pro-rata or on another
more appropriate basis.
Before Impairment After
impairment loss (W1) impairment
Rs. m Rs. m Rs. m
Goodwill 70 (70) -
Land and buildings 320 (33) 287
Plant and machinery 110 (12) 98
500 (115) 385

© Emile Woolf International 195 The Institute of Chartered Accountants of Pakistan


Advanced accounting and financial reporting

Because the land and buildings have been re-valued, the impairment is treated as a
revaluation decrease until the carrying amount of the asset reaches its depreciated
historical cost. The revaluation reserve relating to the asset is Rs. 65 million and so is
adequate to cover the full impairment of Rs. 33m. The impairment must be separately
disclosed and the notes to the accounts must specify by class of asset the impairment
recognised directly to equity.
The impairment loss on the goodwill and plant (Rs. 82 million) must be recognised in profit
or loss for the year. The notes to the accounts must specify the line item in which the
impairment loss has been included.
Where the impairment write-down is material, information must also be provided as to the
events and circumstances that led to the loss, the nature of the assets affected, the
segment to which the asset belongs, that recoverable amount was based on value in use
and the discount rate used to calculate this.
Workings
Loss on the various non-current assets
After the impairment loss has been recognised on the goodwill there is still 115 - 70 = 45
loss to be allocated to the other noncurrent assets, on a pro-rata basis.

Loss on land and buildings:


320
x 45  33
320  110

Loss on plant and machinery:


 110
x 45  12
320  110



© Emile Woolf International 196 The Institute of Chartered Accountants of Pakistan


Answers

CHAPTER 10 – IFRS 5: NON-CURRENT ASSETS HELD FOR SALE AND


DISCONTINUED OPERATIONS

10.1 SAUL
Statement of profit or loss for the year ended 31 December Year 1
Rs.000
Continuing operations
Revenue 3,315
Cost of sales (2,125)
––––––
Gross profit 1,190
Distribution costs (255)
Administrative expenses (680)
––––––
Profit before tax 255
Income tax expense (90)
––––––
Profit for the period from continuing operations 165
Discontinued operations
Loss for the period from discontinued operations (W) (15)
––––––
Profit for the period 150
––––––
Statement of financial position as at 31 December Year 1
Rs.000 Rs.000
Assets
Non-current assets
Property, plant and equipment (1,900 – 510) 1,390
Intangible assets 40
––––––
1,430
Current assets
Inventories 350
Trade and other receivables 190
Cash 90
––––––
630
––––––
2,060
Non-current assets classified as held for sale 450
––––––
Total assets 2,510
––––––
Equity and liabilities
Equity
Share capital 600
Retained earnings (1,700 – 60) 1,640
––––––
2,240
Current liabilities
Trade and other payables (195 – 10) 185
Current tax payable 75
Liabilities classified as held for sale 10
––––––
270
––––––
Total equity and liabilities 2,510
––––––

© Emile Woolf International 197 The Institute of Chartered Accountants of Pakistan


Advanced accounting and financial reporting

Tutorial note
Division A is classified as discontinued in Year 1 because, although it has not been sold during
the period it meets the IFRS 5 criteria for classification as ‘held for sale’.
Working: Discontinued operation

Continuing Discontinued
operations
operations Total
Rs.000 Rs.000 Rs.000
Revenue 3,315 585 3,900
Cost of sales (2,125) (375) (2,500)
–––––– –––– ––––––
Gross profit 1,190 210 1,400
Distribution costs (255) (45) (300)
Administrative expenses (680) (120) (800)
Impairment loss (510 – 450) - (60) (60)
–––––– –––– ––––––
Profit before tax 255 (15) 240
Income tax expense (90) - (90)
–––––– –––– ––––––
Profit/(loss) for the period 165 (15) 150
–––––– –––– ––––––

10.2 SHAHID HOLDINGS


(a) IFRS 5 Non-current assets held for sale and discontinued operations defines non-current
assets held for sale as those assets (or a group of assets) whose carrying amounts will be
recovered principally through a sale transaction rather than through continuing use. A
discontinued operation is a component of an entity that has either been disposed of, or is
classified as ‘held for sale’ and:
(i) represents a separate major line of business or geographical area of operations
(ii) is part of a single co-ordinated plan to dispose of such, or
(iii) is a subsidiary acquired exclusively for sale.
IFRS 5 says that a ‘component of an entity’ must have operations and cash flows that can
be clearly distinguished from the rest of the entity and will in all probability have been a
cash-generating unit (or group of such units) whilst held for use. This definition also means
that a discontinued operation will also fall to be treated as a ‘disposal group’ as defined in
IFRS 5. A disposal group is a group of assets (possibly with associated liabilities) that it is
intended will be disposed of in a single transaction by sale or otherwise (closure or
abandonment). Assets held for disposal (but not those being abandoned) must be
presented separately (at the lower of cost or fair value less costs to sell) from other assets
and included as current assets (rather than as non-current assets) and any associated
liabilities must be separately presented under liabilities. The results of a discontinued
operation should be disclosed separately as a single figure (as a minimum) on the face of
the statement of profit or loss with more detailed figures disclosed either also on the face
of the statement of profit or loss or in the notes.
The intention of this requirement is to improve the usefulness of the financial statements
by improving the predictive value of the (historical) statement of profit or loss. Clearly the
results from discontinued operations should have little impact on future operating results.
Thus users can focus on the continuing activities in any assessment of future income and
profit.

© Emile Woolf International 198 The Institute of Chartered Accountants of Pakistan


Answers

(b) The timing of the board meeting and consequent actions and notifications is within the
accounting period ended 31 October 2016. The notification of staff, suppliers and the press
seems to indicate that the sale will be highly probable and the directors are committed to a
plan to sell the assets and are actively locating a buyer. From the financial and other
information given in the question it appears that the travel agencies’ operations and cash
flows can be clearly distinguished from its other operations. The assets of the travel
agencies appear to meet the definition of non-current assets held for sale; however the
main difficulty is whether their sale and closure also represent a discontinued operation.
The main issue is with the wording of ‘a separate major line of business’ in part (i) of the
above definition of a discontinued operation. The company is still operating in the holiday
business, but only through Internet selling. The selling of holidays through the Internet
compared with through high-street travel agencies requires very different assets, staff
knowledge and training and has a different cost structure. It could therefore be argued that
although the company is still selling holidays the travel agencies do represent a separate
line of business. If this is the case, it seems the announced closure of the travel agencies
appears to meet the definition of a discontinued operation.
(c) Shahid Holdings statement of profit or loss year ended 31 October:

2016 2015
Rs.’000 Rs.’000
Continuing operations
Revenue 25,000 22,000
Cost of sales (19,500) (17,000)
 
Gross profit 5,500 5,000
Operating expenses (1,100) (500)
 
Profit/(loss) from continuing operations 4,400 4,500
Discontinued operations
Profit/(loss) from discontinued operations (4,000) 1,500
 
Profit for the period 400 6,000
 
Analysis of discontinued operations
Revenue 14,000 18,000
Cost of sales (16,500) (15,000)
 
Gross profit/(loss) (2,500) 3,000
Operating expenses (1,500) (1,500)
 
Profit/(loss) from discontinued operations (4,000) 1,500
 

Note: other presentations may be acceptable.

© Emile Woolf International 199 The Institute of Chartered Accountants of Pakistan


Advanced accounting and financial reporting

10.3 PRIMA
Holiday villas
IAS 16 allows property, plant and equipment to be re-valued or left at historical cost. Revaluation
should be based on the fair value (the open market value in an arm’s length transaction).
Revaluation is not required every year, but must be conducted when it is believed that the fair
value differs materially from the carrying value.
The method of accounting for the villa that is to be sold is covered by IFRS 5 which requires that
where, at the end of a reporting period, an asset is held for sale it should be reclassified, re-
measured and no longer depreciated. An asset is only classified as held for sale where the
following conditions are all met:
 The asset is available for sale in its present condition.
 The sale is believed to be highly probable:
 Appropriate level of management is committed to the sale;
 There is an active programme underway to find a buyer;
 The asset is marketed at a realistic price.
 Completion of sale expected within 12 months of classification.
From the limited information provided it appears that these conditions have been met and
therefore, under the rules of IFRS 5, the villa should be re-measured to the lower of its carrying
value and its fair value minus costs to sell.
Therefore, the villas should be valued at 31 December Year 4 as follows:
Fair Carrying
value value
Rs. Rs.
All villas 25.00 20.00
Property held for sale (1.00) (1.25)
Properties to be retained 24.00 18.75

The villas to be retained should be re-valued to Rs. 24m, resulting in an increase in the
revaluation reserve of Rs. 5.25m (24-18.75).
The villa to be sold should be written down from its carrying value to its fair value minus costs to
sell of Rs.0.95m (Rs. 1m – 50,000). This impairment of Rs. 300,000 (1.25m – 0.95m) will be
charged against the revaluation reserve for this asset. If there is insufficient revaluation reserve,
then the write down must be charged to profit or loss.
The villa held for sale must be re-classified from ‘Non-current assets’ to ‘Current assets’ as a
separate line item.
Depreciation should not be charged when an asset has been classified as held for sale.
However, the other villas should be depreciated. IAS 16 states that expenditure on repairs and
maintenance does not remove the need to depreciate an asset. The villas have a finite useful life
and therefore must be depreciated. If the residual value of these assets is greater than the
carrying value then the depreciation charge will be zero. It is not acceptable therefore to have a
policy of non-depreciation on such assets, and a prior year adjustment should be made to correct
the error if the error is material.
Head office
The head office should be recorded under property, plant and equipment at cost. IAS 23 (revised
2009) requires that borrowing costs should be capitalised as part of the cost of an asset if they
are directly attributable to the acquisition, construction or production of a ‘qualifying asset’. A
qualifying asset is an asset that necessarily takes a long period of time to get ready for its
intended use or sale.

© Emile Woolf International 200 The Institute of Chartered Accountants of Pakistan


Answers

In this situation the company is therefore required to capitalise the borrowing costs as part of the
asset cost. Capitalisation must cease when the asset is substantially complete. Construction
finished on 31 May Year 4 and, although minor modifications continued for a further three
months, the standard states that minor modifications indicate that the asset is substantially
complete.
Cost at 30 June Year 4 Rs.000 Rs.000
Land 1,000
Building: Construction cost 8,000
Interest 9% × 5million × (20/12) years
(1 October Year 2 to 1 June Year 4) 750
8,750
Total 9,750

Prima is to receive a government grant. IAS 20 requires that the grant be recognised when there
is reasonable assurance that the entity will meet any conditions and receive the grant. As the
grant has not been received, a receivable will be recorded under current assets. The credit can
be treated in one of two ways:
Option 1: Record as deferred income and release to profit or loss over the useful life of the asset
Option 2: Deduct the grant from the carrying amount of the asset.
If the second option is taken, the asset will be carried at Rs. 8.25m rather than at Rs. 9.75m. The
effect on profit or loss will be the same in both cases.
Land should not normally be depreciated, because land has an indefinite useful life in most
situations. However, as buildings have a limited useful life, a residual value must be allocated to
the building and the depreciable amount must then be written off over the 50 year useful life.
Depreciation will be charged in Year 4 for the four months from 1 September to 31 December.
The estimates of residual value and useful life must be revised each year and the depreciation
amended prospectively.
Yachts
It is important to note that the yachts are held for rental purposes, so they are non-current assets,
not inventory.
The yachts cost Rs. 20m to build, but the recoverable amount on completion (higher of value in
use and net selling price) is only Rs. 18m, and so the assets must be initially recognised at their
recoverable amount. The impairment write down of Rs. 2m will be charged to profit or loss in
Year 4 in accordance with IAS 36.
Recove
Cost rable
amount
Rs. m Rs. m
Engines (15%) 3 2.7
Interior (25%) 5 4.5
Remainder (60%) 12 10.8
20 18
IAS 16 requires that each part of the asset that has a cost that is significant in relation to the total
cost must be depreciated separately. Therefore, in the first year the depreciation charge will be
as follows:
Rs. m
Engines Rs. 2.7m × 1/3 × 9/12 = 0.675
Interior Rs. 4.5m × 1/2 × 9/12 = 1.688
Remainder Rs. 10.8m × 1/5 × 9/12 = 1.620
Charge to profit or loss in Year 4 3.983

© Emile Woolf International 201 The Institute of Chartered Accountants of Pakistan


Advanced accounting and financial reporting

CHAPTER 11 – IFRS 16: LEASES

11.1 X LTD
(a)
A B C D E
Period Opening Fin. Charge Rentals Closing Balance
Balance at 15% of B (B – (D - C)
Rs.’000 Rs.’000 Rs.’000 Rs.’000
2016 11,420 1,713 4,000 9,133
2017 9,133 1,370 4,000 6,503
2018 6,503 975 4,000 3,478
2019 3,478 522 4,000
──── ────
4,580 16,000
──── ────
(b)
Statement of Financial Position (Extract) as at 31 December 2016
Rs.’000
Non-Current assets
(Rs.11,420,000 – Rs.2,855,00) 8,565
Non-Current Liabilities
(Obligation under lease) 6,503
Current Liabilities
Obligation under lease
(Rs.9,133,000 – Rs.6,503,000) 2,630

Note: Annual Depreciation


11 , 420 ,000
= = Rs.2,855,000
4

11.2 PROGRESS LTD


(a) Annuity method

Year 1 Year 2 Year 3


Rs. Rs. Rs.
Cash flow 3,200,000 - -
Outstanding - 1,920,000 1,350,400
Capital repayment 1,280,000 569,600 637,952
Balance 1,920,000 1,350,400 712,448
Interest @ 12% of balance 230,400 162,048 85,494
Capital repayment 569,600 637,952 714,506
800,000 800,000 800,000

© Emile Woolf International 202 The Institute of Chartered Accountants of Pakistan


Answers

(b) Journal entries


Dr Cr
Rs. Rs.
2016
Jan. 3 Right of use - Plant and machinery 3,200,000
Fine Rentals Limited 3,200,000
Initial recognition of machine
Jan. 3 Fine Rentals Limited 1,280,000
Bank 1,280,000
Payment of initial deposit under lease
Dec. 31 Fine Rentals Limited 569,600
Interest expense 230,400
Bank 800,000
Apportionment of annual installment
between Principal repayment and
interest
Dec. 31 Profit and Loss Account 230,400
Interest Expense 230,400
Write-off of FL interest expense to
Profit and loss account
2017
Dec. 31 Fine Rentals Ltd 637,952
Interest expense 162,048
Bank 800,000
Apportionment of annual installment
for the year between Principal
repayment and interest
Dec. 31 Profit and Loss Account 162,048
Interest Expense 162,048
Write-off of FL interest expense to
Profit and loss account
2018
Dec. 31 Fine Rentals Limited 714,506
Interest expense 85,494
Bank 800,000
Apportionment of annual installment
for the year between Principal
repayment and interest
Dec. 31 Profit and Loss Account 85,494
Interest Expense 85,494
Write-off of FL interest expense to
Profit and loss account

© Emile Woolf International 203 The Institute of Chartered Accountants of Pakistan


Advanced accounting and financial reporting

11.3 MIRACLE TEXTILE LIMITED


Miracle Textile Limited
Statement of financial position (extracts) as at 30 June 2016
Note 2016 2015
ASSETS Rs. Rs.
Non-current assets
Right of use - Machinery 4 16,000,000 18,000,000

LIABILITIES
Non-current liabilities
Obligation under lease 9 6,505,219 10,633,074

Current liabilities
Current portion of obligation 9 4,127,856 3,566,925

Miracle Textile Limited


Notes to the financial statements (extracts) for the year ended 30 June 2016
4- Property, plant and equipment 2016 2015

Right of Use Assets


Cost
Opening balance 20,000,000 -
Addition during the year - 20,000,000
20,000,000 20,000,000
Accumulated depreciation
Opening balance (2,000,000) -
Depreciation for the year (2,000,000) (2,000,000)
(4,000,000) (2,000,000)
Balance as at 30 June 16,000,000 18,000,000

9- Obligations under lease (W1)


30-Jun-16 30-Jun-15
Financial Financial
Lease charges Present Lease charges for Present
payment for future Value payment future Value
periods periods
Rs. Rs. Rs. Rs. Rs. Rs.
Not later
than one
year 5,800,000 - 5,800,000 5,800,000 - 5,800,000
Later
than one
year but
not later
than five
years 7,800,000 1,294,781 6,505,219 13,600,000 2,966,925 10,633,075
Later
than five
years - - - - - -
13,600,000 1,294,781 12,305,219 19,400,000 2,966,925 16,433,075

© Emile Woolf International 204 The Institute of Chartered Accountants of Pakistan


Answers

9.1 The Company has entered into a lease agreement with a bank in respect of a machine.
The lease liability bears interest at the rate of 15.725879% per annum. The company has
the option to purchase the machine by paying an amount of Rs.2 million at the end of the
lease term. The lease rentals are payable in annual instalments ending in June 2016.
There are no financial restrictions in the lease agreement.
W1: Lease Schedule
Payment Opening Principal Interest @ Closing
Instalment
date principal repayment 15.73% principal
01-Jul-14 20,000,000 5,800,000 5,800,000 - 14,200,000
01-Jul-15 14,200,000 5,800,000 3,566,925 2,233,075 10,633,075
01-Jul-16 10,633,075 5,800,000 4,127,856 1,672,144 6,505,219
01-Jul-17 6,505,219 5,800,000 4,776,997 1,023,003 1,728,222
30-Jun-18 1,728,222 2,000,000 1,728,222 271,778 -
20,000,000 5,200,000

11.4 ACACIA LTD


Relevant extracts
Statements of profit or loss for the year ended 31 March 2016 (extracts)
Rs.

Depreciation (272,850 ÷ 6) 45,475

Lease payments 6,000*

Finance costs (W) 19,460

* Considering low value item as described in IFRS16

Statement of financial position as at 31 March 2016 (extracts)

Rs.

Non-current assets

Right of use(272,850 – 45,475) 227,375

Non-current liabilities

Lease liabilities (Note 1) 135,810

Current liabilities

Lease liabilities (Note 1) 78,250

Statement of cash flows for the year ended 31 March 2016 (extracts)

Cash flows from financing activities

Payment of lease liabilities (78,250)

Notes to the financial statements (extracts)

© Emile Woolf International 205 The Institute of Chartered Accountants of Pakistan


Advanced accounting and financial reporting

(1) Analysis of lease liabilities


Gross basis
Rs.
Lease liabilities include the following:
Gross payments due within
One year 78,250
Two to five years (2 × 78,250) 156,500
234,750
Less: Finance charges allocated to future periods
((78,250 × 4) – 272,850 – 19,460) (20,690)
214,060
(Alternatively) Net basis
Rs.
Lease liabilities include the following:
Amounts due within
One year 78,250
Two to five years 135,810
214,060
WORKING:
Lease of plant
Year to 31 Interest @
March B/f Payment Capital 10% C/f
Rs. Rs. Rs. Rs. Rs.
2016 272,850 (78,250) 194,600 19,460 214,060
2017 214,060 (78,250) 135,810

11.5 SHOAIB LEASING LIMITED


(a) Entries in the books of Lessor
Date Particulars Dr. Cr.
1-Jul-16 Lease payments receivable (W1) 2,680,000
Machine 2,100,000
Unearned finance income (W1) 580,000
30-Jun-17 Bank 860,000
Lease payments receivable 860,000
30-Jun-17 Unearned finance income 272,941
Finance income (W2) 272,941
30-Jun-18 Bank 860,000
Lease payments receivable 860,000
30-Jun-18 Unearned finance income 196,640
Finance income (W2) 196,640
30-Jun-19 Bank 960,000
Lease payments receivable 960,000
30-Jun-19 Unearned finance income 110,419
Finance income (W2) 110,419

© Emile Woolf International 206 The Institute of Chartered Accountants of Pakistan


Answers

W1: Total finance income Rs.


Total future lease payments (Rs.860,000 x 3) 2,580,000
Add: Purchase bargain option 100,000
Gross investment in finance lease 2,680,000
Less: Cost of assets 2,100,000
Total finance income 580,000

W2: Amortization schedule


Principal Principal
Instalment Interest Principal
Date Opening Closing
Rs.
30-Jun-17 2,100,000 860,000 272,941 587,059 1,512,941
30-Jun-18 1,512,941 860,000 196,640 663,360 849,581
30-Jun-19 849,581 960,000 110,419 849,581 nil
580,000 2,100,000

(b) Shoaib Leasing Limited


Extracts from the statement of financial position as at June 30, 2017
Note 2017
Rs.
Non-current Assets
Net investment in leases 10 849,578
Current Assets
Current portion of net Investment in leases 663,360
10 Net investment in leases
Lease payments receivables 10.1 1,720,000
Add: Residual value of leased assets (part
of LP) 100,000
Gross Investments in leases 1,820,000
Less: Unearned lease income (307,062)
Net investment in leases 10.2 1,512,938
Less: Current portion of net investment in leases (663,360)
849,578
10.1 Lease payments
Less than one year 860,000
More than one year and less than 5 years 960,000
1,820,000

10.2 Net investment in leases


Less than one year 663,360
More than one year and less than 5 years 849,578
1,512,938

© Emile Woolf International 207 The Institute of Chartered Accountants of Pakistan


Advanced accounting and financial reporting

11.6 AKBAR LTD.

a) Right-of-use retained by AL
Financing
Since the consideration (Rs.850,000) exceeds the fair value (Rs.550,000) of the machine,
the agreement contains a financing transaction.
AL initially recognises a right-of-use asset as the proportion of the carrying amount that
reflects the right of use retained. The proportion is calculated by dividing the present value
of the lease payment by fair value
=> 440,000 CV ÷ 550,000 FV × 314,457 (W-1) = Rs.251,565
W-1
Fair value of Rs.614,456 less the part of the lease payments that is just a repayment of the
financing granted to the seller-lessee (Rs.300,000) = Rs.314,456

b) Gain / loss on rights transferred


Gain (refer below) = Rs.47,109
Rs.
Consideration received 850,000
Less: Financial liability
Financing 300,000
PV of lease liability 314,456
(A) 235,544
Less: Carrying value of machine transferred
Total carrying value 440,000
Less: Right-of-use asset 251,565
(B) 188,435
Gain on rights transferred (A-B) 47,109
Accounting Entry by Akbar Ltd.
Dr. Cr.
Rs. Rs.
Cash 850,000
Right-of-use asset 251,565
Machine 440,000
Financial liability 614,456
Gain on rights transferred to lessor 47,109

11.7 ALI LIMITED


Since transfer of an asset does not satisfy the requirements of IFRS 15 therefore Ali Ltd. treats
the transaction as a financing arrangement.
The sale proceeds have been incorrectly credited to operating income, and the operating costs
have been incorrectly debited with the lease payment. Both amounts should be reversed.
Therefore, Ali Ltd. is required to adjust its books by passing the following accounting entries:

Dr. Cr.
Rs. Rs.
Operating income 1,440,000
Financial liability 1,440,000

© Emile Woolf International 208 The Institute of Chartered Accountants of Pakistan


Answers

Operating expense of Rs.360,000 booked erroneously is rectified by reversing it and debiting:


 Interest expense of Rs.115,200 (i-e. Rs.1,440,000 x 8%)
 Financial liability of Rs.244,800 (i-e. principal portion)
Rs.360,000
The accounting entry would be:
Dr. Cr.
Rs. Rs.
Interest expense 115,200
Financial liability 244,800
Operating Expense 360,000
The remaining liability of Rs.1,195,200 should be shown as Rs.931,200 non-current and
Rs.264,000 as current.

11.8 MOAZZAM TEXTILE MILLS LIMITED


Generator A
The ratio between the carrying value (Rs.7,500,000) and fair value
(Rs.6,000,000) will determine the value of right-of-use as against PV of lease payments.
Lease liability
The PV of lease payments is computed by the following formula:
PV = R[1-(1+i)^-n]/i
R = Yearly payment; i = rate per annum; n = number of years
PV = 1,000,000x[1-(1+4.5%)^-5}/4.5%
PV = Rs.4,389,977
Right-of-use
ROU = CV/FV*PV
ROU => 7,500,000/6,000,000*4,389,977 = Rs.5,487,471
Loss on sale
Loss (refer working) = Rs.402,506
Working
Consideration received 6,000,000
Less: PV of lease liability (4,389,977)
Less: Carrying value of machine transferred
Total carrying value 7,500,000
Less: Right-of-use asset (5,487,471) (2,012,529)
Loss on sale = 402,506
Particulars Debit Credit
Rs. Rs.
Cash / Bank 6,000,000
Right-of-use 5,487,471
Loss on sale 402,506
Generator – Carrying value 7,500,000
Lease Liability 4,389,977

© Emile Woolf International 209 The Institute of Chartered Accountants of Pakistan


Advanced accounting and financial reporting

Generator B
Financing transaction
Since the consideration received (Rs.6,000,000) exceeds the fair value (Rs.5,000,000) of the
power generator, the agreement contains a financing transaction.

Sale and lease back


The ratio between the carrying value (Rs.6,000,000) and fair value (Rs.5,000,000) will determine
the value of right-of-use as against PV of lease payments.
Lease liability
The PV of lease payments is computed by the following formula:
PV = R[1-(1+i)^-n]/i
R = Yearly payment; i = rate per annum; n = number of years
PV = 1,000,000x[1-(1+4.5%)^-5}/4.5%
= Rs.4,389,977
Less: Financing = Rs.1,000,000
PV = Rs.3,389,977
Right-of-use
ROU = CV/FV*PV
ROU = 6,000,000/5,000,000*3,389,977
ROU = Rs.4,067,972
Loss on sale
Loss (refer W1) = Rs.322,005
W1
Consideration received 6,000,000
Less:
PV of lease liability (3,389,977)
Financing (1,000,000)
1,610,023
Less: Carrying value of machine transferred
Total carrying value 6,000,000
Less: Right-of-use asset (4,067,972) 1,932,028
Loss = Rs.322,005
Particulars Debit Credit
Rs. Rs.
Cash / Bank 6,000,000
Right-of-use 4,067,972
Loss 322,005
Generator – Carrying value 6,000,000
Lease Liability 4,389,977
Generator C
The ratio between the carrying value (Rs.7,000,000) and fair value (Rs.10,000,000) will
determine the value of right-of-use as against PV of lease payments.

© Emile Woolf International 210 The Institute of Chartered Accountants of Pakistan


Answers

Lease liability
The PV of lease payments is computed by the following formula:
PV = R[1-(1+i)^-n]/i
R = Yearly payment; i = rate per annum; n = number of years
PV = 1,500,000x[1-(1+4.5%)^-5}/4.5%
PV = Rs.6,584,965
Right-of-use
ROU = CV/FV*PV
ROU => 7,000,000/10,000,000*6,584,965 = Rs.4,609,475
Gain on sale
Gain (refer W2) = Rs.1,024,510
W2
Consideration received 10,000,000
Less: PV of lease liability 6,584,965
Less: Carrying value of machine transferred
Total carrying value 7,000,000
Less: Right-of-use asset (4,609,475) 2,309,525
Gain = Rs.1,024,510
Particulars Debit Credit
Rs. Rs.
Cash / Bank 10,000,000
Right-of-use 4,609,475
Generator – Carrying value 7,000,000
Lease Liability 6,584,965
Gain on sale 1,024,510

11.9 MODIFICATION THAT DECREASES THE SCOPE OF THE LEASE (IFRS 16,
ILLUSTRATIVE EXAMPLE 17)
At the effective date of the modification (at the beginning of Year 6), Lessee remeasures the
lease liability based on:
(a) a five-year remaining lease term,
(b) annual payments of CU30,000 and
(c) Lessee’s incremental borrowing rate of 5 per cent per annum. This equals CU129,884.
Lessee determines the proportionate decrease in the carrying amount of the right-of-use asset
on the basis of the remaining right-of-use asset (ie 2,500 square metres corresponding to 50
per cent of the original right-of-use asset).
50 per cent of the pre-modification right-of-use asset (CU184,002) is CU92,001. Fifty per cent
of the pre-modification lease liability (CU210,618) is CU105,309. Consequently, Lessee
reduces the carrying amount of the right-of-use asset by CU92,001 and the carrying amount of
the lease liability by CU105,309.
Lessee recognises the difference between the decrease in the lease liability and the decrease
in the right-of-use asset (CU105,309 – CU92,001 = CU13,308) as a gain in profit or loss at the
effective date of the modification (at the beginning of Year 6).
Lessee recognises the difference between the remaining lease liability of CU105,309 and the
modified lease liability of CU129,884 (which equals CU24,575) as an adjustment to the right-
of-use asset reflecting the change in the consideration paid for the lease and the revised
discount rate.

© Emile Woolf International 211 The Institute of Chartered Accountants of Pakistan


Advanced accounting and financial reporting

11.10 MODIFICATION THAT BOTH INCREASES AND DECREASES THE SCOPE OF THE LEASE
(IFRS 16, ILLUSTRATIVE EXAMPLE 18)
a) The consideration for the increase in scope of 1,500 square metres of space is not
commensurate with the stand-alone price for that increase adjusted to reflect the
circumstances of the contract. Consequently, Lessee does not account for the increase in
scope that adds the right to use an additional 1,500 square metres of space as a separate
lease.
The pre-modification right-of-use asset and the pre-modification lease liability in relation to
the lease are as follows.
Lease liability Right-of-use asset
Beginning 6% Lease Ending Beginning Deprecia- Ending
balance interest payment balance balance tion balance
expense charge
Year CU CU CU CU CU CU CU
1 736,009 44,160 (100,000) 680,169 736,009 (73,601) 662,408
2 680,169 40,810 (100,000) 620,979 662,408 (73,601) 588,807
3 620,979 37,259 (100,000) 558,238 588,807 (73,601) 515,206
4 558,238 33,494 (100,000) 491,732 515,206 (73,601) 441,605
5 491,732 29,504 (100,000) 421,236 441,605 (73,601) 368,004
6 421,236 368,004

b) At the effective date of the modification (at the beginning of Year 6), Lessee remeasures
the lease liability on the basis of: (a) a three-year remaining lease term, (b) annual
payments of CU150,000 and (c) Lessee’s incremental borrowing rate of 7 per cent per
annum. The modified liability equals CU393,647, of which (a) CU131,216 relates to the
increase of CU50,000 in the annual lease payments from Year 6 to Year 8 and (b)
CU262,431 relates to the remaining three annual lease payments of CU100,000 from Year
6 to Year 8.
c) Decrease in the lease term
At the effective date of the modification (at the beginning of Year 6), the pre-modification
right-of-use asset is CU368,004. Lessee determines the proportionate decrease in the
carrying amount of the right-of-use asset based on the remaining right-of-use asset for the
original 2,000 square metres of office space (ie a remaining three-year lease term rather
than the original five-year lease term). The remaining right-of-use asset for the original
2,000 square metres of office space is CU220,802 (ie CU368,004 ÷ 5 × 3 years).
At the effective date of the modification (at the beginning of Year 6), the pre-modification
lease liability is CU421,236. The remaining lease liability for the original 2,000 square
metres of office space is CU267,301 (ie present value of three annual lease payments of
CU100,000, discounted at the original discount rate of 6 per cent per annum).
Consequently, Lessee reduces the carrying amount of the right-of-use asset by
CU147,202 (CU368,004 – CU220,802), and the carrying amount of the lease liability by
CU153,935 (CU421,236 – CU267,301). Lessee recognises the difference between the
decrease in the lease liability and the decrease in the right-of-use asset (CU153,935 –
CU147,202 = CU6,733) as a gain in profit or loss at the effective date of the modification
(at the beginning of Year 6).
Lease liability CU153,935
Right-of-use asset CU147,202
Gain CU6,733
At the effective date of the modification (at the beginning of Year 6), Lessee recognises the
effect of the remeasurement of the remaining lease liability reflecting the revised discount
rate of 7 per cent per annum, which is CU4,870 (CU267,301 – CU262,431), as an
adjustment to the right-of-use asset.

© Emile Woolf International 212 The Institute of Chartered Accountants of Pakistan


Answers

Lease liability CU4,870


Right-of-use asset CU4,870
d) Increase in the leased space
At the commencement date of the lease for the additional 1,500 square metres of space
(at the beginning of Year 6), Lessee recognises the increase in the lease liability related to
the increase in scope of CU131,216 (ie present value of three annual lease payments of
CU50,000, discounted at the revised interest rate of 7 per cent per annum) as an
adjustment to the right-of-use asset.
Right-of-use asset CU131,216
Lease liability CU131,216

The modified right-of-use asset and the modified lease liability in relation to the modified
lease are as follows.
Lease liability Right-of-use asset
Beginning 7% Lease Ending Beginning Deprecia- Ending
balance interest payment balance balance tion balance
expense charge
Year CU CU CU CU CU CU CU
6 393,647 27,556 (150,000) 271,203 347,148 (115,716) 231,432
7 271,203 18,984 (150,000) 140,187 231,432 (115,716) 115,716
8 140,187 9,813 (150,000) - 115,716 (115,716) -

11.11 SUBLEASE CLASSIFIED AS A FINANCE LEASE (IFRS 16, ILLUSTRATIVE EXAMPLE 20)
The intermediate lessor classifies the sublease by reference to the right-of-use asset arising
from the head lease. The intermediate lessor classifies the sublease as a finance lease,
having considered the requirements in paragraphs 61–66 of IFRS 16.
When the intermediate lessor enters into the sublease, the intermediate lessor:
i. derecognises the right-of-use asset relating to the head lease that it transfers to the
sublessee and recognises the net investment in the sublease;
ii. recognises any difference between the right-of-use asset and the net investment in the
sublease in profit or loss; and
iii. retains the lease liability relating to the head lease in its statement of financial position, which
represents the lease payments owed to the head lessor.
During the term of the sublease, the intermediate lessor recognises both finance income on
the sublease and interest expense on the head lease.

11.12 SUBLEASE CLASSIFIED AS AN OPERATING LEASE (IFRS 16, ILLUSTRATIVE


EXAMPLE 21)
The intermediate lessor classifies the sublease by reference to the right-of-use asset arising
from the head lease. The intermediate lessor classifies the sublease as an operating lease,
having considered the requirements in paragraphs 61–66 of IFRS 16.
When the intermediate lessor enters into the sublease, the intermediate lessor retains the
lease liability and the right-of-use asset relating to the head lease in its statement of financial
position.
During the term of the sublease, the intermediate lessor:
(a) recognises a depreciation charge for the right-of-use asset and interest on the lease liability;
and
(b) recognises lease income from the sublease.

© Emile Woolf International 213 The Institute of Chartered Accountants of Pakistan


Advanced accounting and financial reporting

CHAPTER 12 – IAS 37: PROVISIONS, CONTINGENT LIABILITIES AND


CONTINGENT ASSETS

12.1 ROWSLEY
Introduction
All four scenarios relate to the rules of IAS 37 Provisions, contingent liabilities and
contingent assets. In each scenario, the key issue is whether or not a provision should be
recognised.
Under IAS 37, a provision should only be recognised when three conditions are met:
 there is a present obligation as a result of a past event; and
 it is probable that a transfer of economic benefits will be required to settle the obligation;
and
 a reliable estimate can be made of the amount of the obligation.
Factory closure
As the factory closure changes the way in which the business is conducted (it involves the
relocation of business activities from one part of the country to another) it appears to fall within
the IAS 37 definition of a restructuring.
The key issue here is whether the group has an obligation at the end of the reporting period to
incur expenditure in connection with the restructuring. There is clearly no legal obligation, but
there may be a constructive obligation. IAS 37 states that a constructive obligation only exists if
the group has created valid expectations in other parties such as employees, customers and
suppliers that the restructuring will actually be carried out. As the group is still in the process of
drawing up a formal plan for the restructuring and no announcements have been made to any of
the parties affected, there cannot be an obligation to restructure. A board decision alone is not
sufficient. Therefore no provision should be made.
If the group starts to implement the restructuring or makes announcements to those affected after
the end of the reporting period but before the accounts are approved by the directors it may be
necessary to disclose the details in the financial statements as a non-adjusting post event after
the reporting period in accordance with IAS 10. This will be the case if the restructuring is of such
importance that non-disclosure would affect the ability of the users of the financial statements to
reach a proper understanding of the group’s financial position.
Operating lease
The lease contract appears to be an ‘onerous contract’ as defined by IAS 37 as the unavoidable
costs of meeting the obligations under it exceed the economic benefits expected to be received
from it.
Because the enterprise has signed the lease contract there is a clear legal obligation and the
enterprise will have to transfer economic benefits (pay the lease rentals) in settlement. Therefore,
the group should recognise a provision for the net present value of the remaining lease
payments.
In principle, a corresponding asset may be recognised in relation to the future rentals expected to
be received, if these receipts are virtually certain. The current arrangement with the charity
generates only nominal rental income and so the asset is unlikely to be material enough to
warrant recognition. The chances of renting the premises at a commercial rent are less than 50%
and so no further potential rent receivable may be taken into account as the outcome is not
virtually certain and so recognition would not be prudent.
The financial statements should disclose the carrying amount of the onerous lease provision at
the end of the reporting period, a description of the nature of the obligation and the expected
timing of the lease payments. Disclosure should also be made of the contingent assets where the
amount of any expected rentals receivable from sub-letting are material and the likelihood is
believed probable.

© Emile Woolf International 214 The Institute of Chartered Accountants of Pakistan


Answers

Legal proceedings
It is unlikely that the group has a present obligation to compensate the customer; therefore no
provision should be recognised. However, there is a contingent liability. Unless the possibility of a
transfer of economic benefits is remote, the financial statements should disclose a brief
description of the nature of the contingent liability, an estimate of its financial effect and an
indication of the uncertainties relating to the amount or timing of any outflow.
Environmental damage
It is clear that there is no legal obligation to rectify the damage. However, through its published
policies, the group has created expectations on the part of those affected that it will take action to
do so. There is, therefore, a constructive obligation to rectify the damage and a transfer of
economic benefits is probable.
The group must recognise a provision for the best estimate of the cost. As the most likely
outcome is that more than one attempt at re-planting will be needed, the full amount of Rs. 30
million should be provided. The expenditure will take place sometime in the future, and so the
provision should be discounted at a pre-tax rate that reflects current market assessments of the
time value of money and the risks specific to the liability.
The financial statements should disclose the carrying amount at the end of the reporting period, a
description of the nature of the obligation and the expected timing of the expenditure. The
financial statements should also give an indication of the uncertainties about the amount and
timing of the expenditure.

12.2 MULTAN PETROCHEM LTD


(a) Provisions and contingencies
Environmental Legal claims Onerous Total
damage lease
At 1 Jan 2016 1,300,000 – 80,000 1,380,000
Unwinding of the
discount (8%) 104,000 6,400 110,400
Utilised in the year – – (15,000) (15,000)
Charge/(credit) to
statement of profit or
loss – 1,200,000 (6,000) 1,194,000
At 31 Dec 2016 (W) 1,404,000 1,200,000 65,400 2,669,400
Environmental damage
The provision in respect of the environmental damage relates to restoration of land
following the initial ground work undertaken to set up a new oil refinery. The company has
an advertised policy that it will restore all environmental damage caused by its business
operations. The provision is based on the estimated cost of reinstating the environmental
damage caused and is not likely to be paid until 2040.
Legal claims
During the year an explosion at one of the company’s oil extraction plants caused a
number of employees to suffer injury. This provision is to cover personal injury claims
made by the individuals concerned. The provision is based on lawyers’ best estimate of
the likely amount at which the claims can reasonably be settled. It is hoped that the claims
will be settled in the next financial year. It is expected that the full amount of these claims
will be reimbursed by an insurance company following their payment.
Onerous lease
The company has an ongoing lease obligation in respect of office space that is not being
utilised by the company. The outstanding lease liability at the year-end was Rs. 65,000
and the lease has another four years to run. MPP has found a tenant for the office space
on a six-month short lease and this will reduce the outstanding obligation by Rs. 6,000 in
2017.

© Emile Woolf International 215 The Institute of Chartered Accountants of Pakistan


Advanced accounting and financial reporting

Contingent liability
Following the explosion at the oil extraction plant a number of employees have made
claims against the company for undue stress. Based on lawyers’ advice the company do
not believe that it is probable that a court case against the company will be brought. If such
a case was to be heard the estimated payout in total is Rs. 20,000.
Workings
Personal injury claims: 8 × 150,000 = 1,200,000
Onerous lease: (80,000 – 15,000) – 6,000 = 59,000
(b) Summary of amounts included in income statement for year ended 31 December
2016
Operating costs: Rs.
Movement in provision 394,000
Consultancy fees 12,000
Depreciation on oil refinery environmental damage (1,300,000 ÷ 52,000
25yrs)
Borrowing costs
Unwinding of the discount 110,400
Other operating income:
Insurance reimbursement 400,000

12.3 VIOLET POWER LIMITED


Rs. in
million
Assets carrying value as at June 30, 2016 (Asset)
Cost (Given) 6,570
Decommissioning liability on July 1, 2015
20
(780 / (1+0.08) ) 167
Depreciation for the year (321) Working 1
Adjustment for revision in provision for decommissioning cost 157 Working 2
6,573

Decommissioning liability on June 30, 2016 (1,021 /


19
(1+0.06) ) 337

Working 1: Depreciation for the year (P&L)


Cost 6,570
Decommissioning liability on July 1, 2015 167
Residual value (320)
6,417
Depreciation (6,417 / 20) 321

Working 2: Increase in decommissioning liability during the


year ended June 30, 2016
Decommissioning liability on June 30, 2016 337
Less: Decommissioning liability on July 1, 2015 (167)
Less: Unwinding of interest for the year (167 x 8%) (13)
157

© Emile Woolf International 216 The Institute of Chartered Accountants of Pakistan


Answers

CHAPTER 13: IAS 19: EMPLOYEE BENEFITS

13.1 LABURNUM LIMITED


Calculation of the actuarial gain/losses in year to 31 December 2016
FV of PV of
plan plan
assets liabilities
Rs.000 Rs.000
Opening balance 2,600 2,900
Service cost 450
Interest cost (8% x Rs. 2,900,000) 232
Expected return (8% x Rs. 2,600,000) 208
Past service cost 90
Benefits paid (240) (240)
Contributions 730
3,298 3,432
Actuarial gain on assets 102 -
Actuarial loss on liabilities - 68
Closing balance 3,400 3,500

13.2 JABEL LIMITED


Statement of profit or loss expense Rs.000
Service cost 300
Interest cost (7% x Rs. 1,400,000) 98
Expected return (7% x Rs. 1,200,000) (84)
Net expense 314
The net expense in profit or loss will be Rs. 314,000.
Actuarial gains and losses FV of plan PV of plan
assets liabilities
Rs.000 Rs.000
Opening balance 1,200 1,400
Service cost 300
Interest cost (7% x Rs. 1,400,000) 98
Expected return (7% x Rs. 1,200,000) 84
Benefits paid (220) (220)
Contributions 400
1,464 1,578
Actuarial loss on plan assets (64) -
Actuarial loss on plan liabilities - 22
Closing balance 1,400 1,600

Within other comprehensive income there will be an actuarial loss on plan assets of Rs. 64,000
and an actuarial loss on plan liabilities of Rs. 22,000.

© Emile Woolf International 217 The Institute of Chartered Accountants of Pakistan


Advanced accounting and financial reporting

13.3 KAGHZI LIMITED


Calculation of the actuarial gains/losses in the year to 31 December 2016
FV of plan PV of plan
assets liabilities
Rs.000 Rs.000
Opening balance 1,400 1,700
Service cost 320
Interest cost (7% x Rs. 1,700,000) 119
Expected return (7% x Rs. 1,400,000) 98
Benefits paid (170) (170)
Contributions 580
1,908 1,969
Actuarial gain on plan assets 192 -
Actuarial loss on plan liabilities - 431
Closing balance 2,100 2,400

13.4 LASURA LTD


(i) Statement of profit or loss expense
Rs.000
Service cost 500
Interest cost (8% x Rs. 2,400,000) 192
Expected return (8% x Rs. 2,200,000) (176)
Net expense in profit or loss 516

(ii) Other comprehensive income


Rs.000
Actuarial gain on plan assets (W1) (74)
Actuarial loss on plan liabilities (W1) 58
Net actuarial gain in OCI (16)

Working 1 FV of assets PV of
liabilities
Rs.000 Rs.000
Opening balance 2,200 2,400
Service cost 500
Interest cost (8% x Rs. 2,400,000) 192
Expected return (8% x Rs. 2,200,000) 176
Contributions paid in 300
Paid to retired members (450) (450)
2,226 2,642
Actuarial gain on plan assets 74
Actuarial loss on plan liabilities 58
Closing balance 2,300 2,700

© Emile Woolf International 218 The Institute of Chartered Accountants of Pakistan


Answers

13.5 UNIVERSAL SOLUTIONS


(a) (i) Defined benefit pension scheme
The employees of a defined benefit scheme will be guaranteed a pension based on
their final salary and their number of years of service. Accordingly, the higher paid
the employee is on retirement and the longer the length of service:
the greater the employee’s pension entitlement and
the greater the liability of the pension fund.
An actuary will advise the company of the cash contributions to be paid into the plan
each year in order to provide the promised pensions. This is a complicated
calculation involving many estimates such as employee mortality, future increases in
salary and expected future investment returns.
The employer has an open-ended liability to make additional contributions should
there be a deficit in the defined benefit pension fund. A deficit may arise, for
example, if salary levels rise more than expected or staff turnover reduces,
increasing service years.
It will be necessary for the actuary to regularly re-value the pension fund’s assets
and liabilities to assess the surplus or deficit position and revise the company’s
contributions.
(ii) The basis to be adopted in measuring scheme assets
Assets should be measured at their fair value. For quoted securities, for example,
this means their market price.
(iii) The basis to be adopted in measuring scheme liabilities
Liabilities should be measured on an actuarial basis (i.e. discounted cash flow),
using the projected unit method.
The projected unit method is an accrued benefits valuation method in which the
scheme’s liabilities reflect projected future earnings. To derive the scheme liabilities,
the expected future pension payments should be discounted at a rate that reflects
the time value of money, for example, using an AA (high quality) corporate bond
rate.
(iv) Actuarial gains and losses
Actuarial gains and losses are deficits or surpluses that arise because:
 events have not coincided with the actuarial assumptions made at the last
valuation (experience gains and losses) or
 the actuarial assumptions have changed.
For example, if the actuary forecast that investment returns were going to be 7% in
a year, but in fact the return actually achieved was only 5%, this would give rise to
an actuarial deficit.
(b)

(i) Statement of financial position - extract

Year 3 Year 4

Defined benefit net liability 200 300

© Emile Woolf International 219 The Institute of Chartered Accountants of Pakistan


Advanced accounting and financial reporting

(ii) Journal
Dr Cr
P&L
Interest cost 10
Current service cost 100
110
OCI 130
Cash 140
Defined benefit net liability 100
Workings
Company
Pension fund
position
Statement of
Liabilities Assets financial
position
Rs. Rs. Rs.
Opening balance 1 January Year 4 (1,200) 1,000 (200)
Interest cost (5%) (60) 50 (10)
Current service cost (100) (100)
Contributions to the pension fund 140 140
Benefits paid out 95 (95)
Amounts recorded by company (1,265) 1,095 (170)
Actuarial difference (balance) (135) 5 (130)
Closing balance 31 Dec Year 4 (1,400) 1,100 (300)

13.6 DHA INTERIORS LTD


Report to the Directors of DHA Interiors Ltd
Terms of Reference
This report sets out the differences between a defined contribution and defined benefit plan, and
the accounting treatment of the company’s pension plans. It also discusses the principles
involved in accounting for warranty claims, and the accounting treatment of those claims.
(a) Pension plans – IAS 19
A defined contribution plan is a pension plan whereby an employer pays fixed contributions
into a separate fund and has no legal or constructive obligation to pay further contributions
(IAS 19 paragraph 7). Payments or benefits provided to employees may be a simple
distribution of total fund assets, or a third party (an insurance company) may, for example,
agree to provide an agreed level of payments or benefits. Any actuarial and investment
risks of defined contribution plans are assumed by the employee or the third party. The
employer is not required to make up any shortfall in assets and all plans that are not
defined contribution plans are deemed to be defined benefit plans.
A defined benefit plan is any plan other than a defined contribution plan. It is the residual
category. An employer’s obligation under a defined benefit plan is to provide the agreed
amount of benefits to current and former employees. The differentiating factor between
defined benefit and defined contribution schemes is in determining where the risks lie. If an
employer cannot demonstrate that all actuarial and investment risk has been shifted to
another party and that its obligations are limited to contributions made during the period,
then the plan is a defined benefit plan. Any benefit formula that is not solely based on the
amount of contributions, or that includes a guarantee from the entity or a specified return,
means that elements of risk remain with the employer and must be accounted for as a
defined benefit plan.

© Emile Woolf International 220 The Institute of Chartered Accountants of Pakistan


Answers

For defined contribution plans, the cost recognised in the period is the contribution payable
in exchange for service rendered by employees during the period. Accounting for a defined
contribution plan is straightforward because the employer’s obligation for each period is
determined by the amount to be contributed for that period. Often, contributions are a
percentage of employee salary in the period as its base. No actuarial assumptions are
required to measure the obligation or the expense.
The employer should account for the contribution payable at the end of each period based
on employee services rendered during that period, reduced by any payments made during
the period. If the employer has made payments in excess of those required, the excess is
a prepaid expense to the extent that the excess will lead to a reduction in future
contributions or a cash refund.
For defined benefit plans, the amount recognised in the statement of financial position is
the present value of the defined benefit obligation (that is, the present value of expected
future payments required to settle the obligation resulting from employee service in the
current and prior periods), as reduced by the fair value of plan assets at the reporting date.
If the balance is an asset, the amount recognised may be limited under IAS 19
Pension Plan 1 is a defined benefit plan as the employer has the investment risk as the
company is guaranteeing a pension based on the service lives of the employees in the
scheme. The employer’s liability is not limited to the amount of the contributions. There is a
risk that if the investment returns fall short the employer will have to make good the
shortfall in the scheme. Pension Plan 2 is a defined contribution scheme because the
employer’s liability is limited to the contributions paid.
(b) Accounting for the two plans
Pension Plan 1
The accounting for the defined benefit plan results in a liability of Rs. 20.5 million as at 31
October 2016, an expense in the statement of profit or loss of Rs. 20.5 million and a
charge in other comprehensive income of Rs. 1.5 million for the year (see Appendix 1).
Pension Plan 2
The company does not recognise any assets or liabilities for the defined contribution
scheme but charges the contributions payable for the period (Rs. 10 million) to operating
profit. The contributions paid by the employees will be part of the wages and salaries cost
and when paid will reduce cash.
Appendix 1
The accounting for the defined benefit plan is as follows:
31 October 2016 1 November 2015
Rs. m Rs. m
Present value of obligation 240 200
Fair value of plan assets
(225) (190)
––––– –––––
Liability recognised 15 10
––––– –––––
Expense in Statement of profit or loss year ended 31 October 2016:
Rs. m
Current service cost 20.0
Net interest expense 0.5
–––––
Expense 20·5
–––––
Analysis of amount in statement of other comprehensive income (OCI):
Rs. m
Actuarial loss on obligation (w2) 29
Actuarial gain on plan assets (w2) (27·5)
–––––
Actuarial loss on obligation (net) 1·5
–––––

© Emile Woolf International 221 The Institute of Chartered Accountants of Pakistan


Advanced accounting and financial reporting

Working 1
Movement in net liability in statement of financial position at
31 October 2016:
Rs. m
Opening liability 10.0
Expense 20·5
Contributions (17.0)
Actuarial loss 1.5
–––––
Closing liability 15.0
–––––
Working 2 – Change in present value of the obligation and fair value of plan assets
Fair value
PV of of plan Net
obligation assets liability
Rs.000 Rs.000 Rs.000
At start of year (200.0) 190.0 (10.0)
Interest expense (5% × 200) (10.0) (10.0)
Interest earned (5% × 270) 9.5 9.5
Net interest (0.5)
Current service cost (20.0) (20.0)
Contributions paid 17.0 17.0
Benefits paid out (given) 19.0 (19.0) 0
Expected year end position (211.0) 197.5 (13.5)
Remeasurement (balancing figure) (29.0) 27.5 (1.5)
Actual year end position (240.0) 225.0 (15.0)

© Emile Woolf International 222 The Institute of Chartered Accountants of Pakistan


Answers

CHAPTER 14: IFRS 2: SHARE BASED PAYMENTS

14.1 TOSHACK LTD


(i) Liability as at 31 December 2016:
Appreciation in price between 1/1/2013 – 31/12/2016
Rs. 31 – Rs. 22 = Rs. 9
No of employees on which rights vest = 75%  1000 = 750
No of rights granted per employee = 50
Liability Rs. 9  750  50 = Rs. 337,500
(ii) Gain between 1/1/2016 – 31/12/2017
(Rs. 28 – Rs. 22)  750  50 = Rs. 225,000
Payment to employees Rs. 112,500
Comment: The transaction would be accounted for as a cash settled share base payment
if the entity has incurred a liability to settle in cash or other asset.

14.2 IFRS 2
(a) (i) The need for accounting standard regulation
Share options are often granted to employees at an exercise price that is higher
than the market price of the shares. Therefore, the options have no intrinsic value to
the company and, prior to the issue of IFRS 2, these transactions were not generally
recognised until such time as the shares were issued. This approach could be seen
as resulting in a distortion of reported results between accounting periods and
leaving liabilities unrecorded.
In addition, the subject of accounting for share-based payments contains a number
of other contentious issues, notably relating to the measurement principles to be
applied in recognising the transactions. If employees agree to stay until their options
vest, the organisation must recognise the service they will provide in return, but how
should this be valued?
IFRS 2 was therefore issued in February 2004 to provide comprehensive guidance
on these matters.
(ii) The three types of share based payments
These can be summarised as follows:
Category Features
Equity-settled share-based The entity pays for goods or services by issuing
payment transactions equity instruments in the form of shares or share
options.
Cash-settled share-based The entity incurs a liability for goods or services
payment transactions and the settlement amount is based on the price
(or value) of the entity’s shares or other equity
instruments.
Share based payments with Transactions where an entity acquires goods or
cash alternatives receives services and either the entity or the
supplier can choose payment to be a cash amount
based on the price (or value) of the entity’s shares
or other equity instruments, or equity instruments
of the entity.

© Emile Woolf International 223 The Institute of Chartered Accountants of Pakistan


Advanced accounting and financial reporting

(b) (i) Assuming all options vest


31 December Year 5 Profit and Equity
loss
Expected outcome
(at grant date value)
500 employees × 100 options 750,000
× Rs. 15 fair value
3 years to vest ×1/3
Year 1 charge 250,000
Balance carried forward 250,000

31 December Year 6 Rs.


Expected outcome (at grant date value)
500 × 100 × Rs. 15 750,000
×2/3
Recognised by the year end 500,000
Minus expense previously recognised (250,000)
Year 2 charge 250,000
Balance carried forward 500,000
31 December Year 7 Rs.
Actual outcome (at grant date value)
500 × 100 × Rs. 15 750,000
Minus expense previously recognised (500,000)
Year 3 charge 250,000
Balance at end of year 3 750,000

(ii) Reflecting revised vesting assumptions


Profit Equity
or loss
31 December Year 5 Rs.
Expected outcome (at grant date value)
85% × 500 × 100 × Rs. 15 637,500
×1/3
Year 1 charge 212,500
Balance carried forward 212,500

31 December Year 6
Expected outcome (at grant date value)
88% × 500 × 100 × Rs. 15 660,000
×2/3
440,000
Minus expense previously recognised (212,500)
Year 2 charge 227,500
Balance carried forward 440,000

31 December Year 7
Actual outcome (at grant date value)
44,300 × Rs. 15 664,500
Minus expense previously recognised (440,000)
Year 3 charge 224,500
Balance at the end 664,500

© Emile Woolf International 224 The Institute of Chartered Accountants of Pakistan


Answers

14.3 SAVAGE LTD


This is an equity-settled share-based transaction and in accordance with IFRS 2 the fair value of
the share options is used to estimate the fair value of the services provided by the employees.
The total fair value is allocated over the three year vesting period and is based on the fair value
at the grant date.
2015
Total expected expense:
1,000 options x Rs. 11 x (300 – 10 – 30) Rs. 2,860,000
Fraction of vesting period by the year end 1/3
Expense in 2015 Rs. 953,333

Dr Statement of profit or loss Rs. 953,333


Cr Equity Rs. 953,333
2016
Total expected expense:
1,000 options x Rs. 11 x (300 -10 – 20 – 15) Rs. 2,805,000
Fraction of vesting period by the year end 2/3
Expense recognised by the year end Rs. 1,870,000
Recognised in 2015 Rs. 953,333
To be recognised in 2016 Rs. 916,667

Dr Statement of profit or loss Rs. 916,667


Cr Equity Rs. 916,667

14.4 YORATH LTD


(a) 2016 equity balance required:
Total expected expense (at end of 2016)
500 options x Rs. 148 x 520 (600 – 20 – 25 – 15 – 20) Rs. 38,480,000
Fraction of vesting period by the year end ¾
Expense recognised by the year end Rs. 28,860,000
Total expected expense (at end of 2015)
500 options x Rs. 148 x 515 (600 – 20 – 25 – 40) Rs. 38,110,000
2
Fraction of vesting period by the year end /4
Expense recognised by the last year end Rs. 19,055,000
To be recognised in 2016 Rs. 9,805,000
Recorded in 2016 financial statements:
Dr Statement of profit or loss – staff costs Rs. 9,805,000
Cr Equity – other reserves Rs. 9,805,000
(b) The sales director is incorrect, despite no cash changing hands, the share options are
issued in exchange for employees providing services to Yorath Ltd. Possibly the options
have been given as a reward for service provided or in lieu of a pay rise or bonus which
would otherwise have been paid in cash. As there is no direct wage cost, we instead must
calculate an equivalent cost of receiving staff services and match this with the revenue that
the staff helps to generate. We do this by estimating the value inherent in the options and
allocating that over the period in which employees must remain with Yorath Ltd, in this
case 4 years.

© Emile Woolf International 225 The Institute of Chartered Accountants of Pakistan


Advanced accounting and financial reporting

The amount chargeable to the statement of profit or loss is based on the fair value of the
share options at the grant date. This is not subsequently remeasured as these share
options represent an equity-settled share-based payment. The equivalent cost will be
updated each year for those employees that are still eligible or expected to be eligible at
the year end to ensure that the amount charged reflects the amount that is expected to
vest.

14.5 QUALTECH LTD


(i) Share-based payment
Total expected expense (at end of 2016)
1,000 options x Rs. 122 x 240 (300 – 25 – 15 – 20) Rs. 29,280,000
2
Fraction of vesting period by the year end /3
Expense recognised by the year end Rs. 19,520,000
Total expected expense (at end of 2015)
1,000 options x Rs. 122 x 235 (300 – 25 – 40) Rs. 28,670,000
1
Fraction of vesting period by the year end /3
Expense recognised by the last year end Rs. 9,556,667
To be recognised in 2016 Rs. 9,963,333
Double entry in 2016:
Dr Statement of profit or loss – staff costs Rs. 9,963,333
Cr Other reserves (equity) Rs. 9,963,333
Being the charge for share-based payment for the year ended 31 December 2016
(ii) Share-based payments that are to be settled in cash would be credited instead to liabilities
in the statement of financial position and the liability would be remeasured using the fair
value of the shares at each year-end date until the end of the vesting period.

14.6 BRIDGE LTD


(i) Statement of profit or loss charge
Total expected expense (at end of 2016)
1,000 options x Rs. 50 x 213 (300 – 20 – 23 – 44) Rs. 10,650,000
2
Fraction of vesting period by the year end /4
Expense recognised by the year end Rs. 5,325,000
Total expected expense (at end of 2015)
1,000 options x Rs. 50 x 215 (300 – 20 – 65) Rs. 10,750,000
¼
Fraction of vesting period by the year end
Expense recognised by the last year end Rs. 2,687,500
To be recognised in 2016 Rs. 2,637,500

Double entry in 2016:


Dr Statement of profit or loss – staff costs Rs. 2,637,500
Cr Other reserves (equity) Rs. 2,637,500
Being the charge for share-based payment for the year ended 31 December 2016
(ii) Share options, such as those granted by Bridge Ltd, are given by an entity in return for
services provided by its employees. In effect the share options are given to the employees
as a form of bonus or reward for these services and are therefore part of the employee’s
remuneration package. The value of these options (or relevant part thereof) must then be
reflected in the staff costs included within the statement of profit or loss.

© Emile Woolf International 226 The Institute of Chartered Accountants of Pakistan


Answers

14.7 CAPSTAN LTD


SARs are an example of a cash-settled share-based transaction and, in accordance with IFRS 2
Share-based payments, are initially measured at fair value at the grant date and subsequently
remeasured to fair value at each year-end. The liability is remeasured and any difference is
charged to the statement of profit or loss as an expense.
2015
Total expected expense (at end of 2015)
1,000 SARs x Rs. 80 x 233 (300 – 32 – 35) Rs. 18,640,000
1
Fraction of vesting period by the year end /3
Liability to be recognised by the year end Rs. 6,213,333
Double entry in 2015:
Dr Statement of profit or loss – staff costs Rs. 6,213,333
Cr Liability Rs. 6,213,333
2016
Total expected expense (at end of 2016)
1,000 SARs x Rs. 120 x 230 (300 – 32 – 28 – 10) Rs. 27,600,000
2
Fraction of vesting period by the year end /3
Liability to be recognised by the year end Rs. 18,400,000
Less opening liability (see above) (Rs. 6,213,333)
To be recognised in 2016 Rs. 12,186,667

Double entry in 2016:


Dr Statement of profit or loss – staff costs Rs. 12,186,667
Cr Liability Rs. 12,186,667

14.8 NEWTOWN LTD


(i) 2015
Total expected expense (at end of 2016)
1,000 SARs x Rs. 110 x 405 (500 – 42 – 28 – 25) Rs. 44,550,000
2
Fraction of vesting period by the year end /3
Liability to be recognised by the year end Rs. 29,700,000
Less opening liability:
Total expected expense (at end of 2015)
1,000 SARs x Rs. 90 x 383 (500 – 42 – 75) Rs. 34,470,000
1
Fraction of vesting period by the year end /3
Liability recognised by the end of 2015 Rs. 11,490,000
To be recognised in 2016 Rs. 18,210,000

Double entry in 2016:


Dr Statement of profit or loss – staff costs Rs. 18,210,000
Cr Liability Rs. 18,210,000

(ii) If a share-based payment was settled in equity rather than cash the implications would be:
Recognition:
There would be a credit to other reserves within equity in the statement of financial
position, rather than a liability. However the debit would still be to staff costs.

© Emile Woolf International 227 The Institute of Chartered Accountants of Pakistan


Advanced accounting and financial reporting

Measurement:
The amount would be initially and subsequently measured using the fair value of the rights
at the grant date rather than re-measured at each year end.

14.9 SINDH TRANSIT LTD


(a) Accounting entries
Accounting entries for year ended 31 December 2016:
Share options
Dr Staff costs (statement of profit or loss) (W1)Rs. 17,820,000
Cr Equity Rs. 17,820,000
Share appreciation rights (SARs)
Dr Staff costs (statement of profit or loss) (W2)Rs. 6,495,000
Cr Liabilities (non-current) Rs. 6,495,000
Working 1: Options
Total expected expense (at end of 2016)
1,000 options x Rs. 220 x 327 (400 – 15 – 22 – 36) Rs. 71,940,000
2
Fraction of vesting period by the year end /4
Expense recognised by the year end Rs. 35,970,000
Total expected expense (at end of 2015)
1,000 options x Rs. 220 x 330 (400 – 15 – 55) Rs. 72,600,000
1
Fraction of vesting period by the year end /4
Expense recognised by the last year end Rs. 18,150,000
To be recognised in 2016 Rs. 17,820,000
Working 2: SARs
Total expected expense (at end of 2016)
500 SARs x Rs. 140 x 327 (400 – 15 – 22 – 36) Rs. 22,890,000
2
Fraction of vesting period by the year end /4
Liability to be recognised by the year end Rs. 11,445,000
Less opening liability:
Total expected expense (at end of 2015)
500 SARs x Rs. 120 x 330 (400 – 15 – 55) Rs. 19,800,000
1
Fraction of vesting period by the year end /4
Liability recognised by the end of 2015 Rs. 4,950,000
To be recognised in 2016 Rs. 6,495,000
(b) In accordance with IFRS 2, the share options and the share appreciation rights are
recognised as an expense in the statement of profit or loss as they are awarded in return
for employee service.
The treatment of each of above stated however is different in the statement of financial
position. The share appreciation rights will result in a future outflow of cash and therefore
represent an obligation and are presented as a liability. The liability should reflect the most
reliable measurement at each balance sheet date and so the total amount payable that is
estimated at each year-end date is estimated using the updated fair values.
The options represent an equity-settled share-based payment and do not meet the
definition of obligation, and so instead the entry is to equity. The equity element is
measured initially and subsequently at the fair value at the grant date.

© Emile Woolf International 228 The Institute of Chartered Accountants of Pakistan


Answers

CHAPTER 15: FINANCIAL INSTRUMENTS: RECOGNITION AND MEASUREMENT

15.1 AJI PANCA LTD


Capital and reserves Rs.
Share capital (Rs. 1 ordinary shares) (W2) 1,625,000
Share premium (W3) -
Retained earnings 6,116,812
7,741,812
Liabilities (W5) 164,751
Workings
(1) Profit for the year
Rs.
Original 508,500
Minus: Finance charges (W5) (14,988)
493,512
(2) Ordinary share capital
Rs.
At 1 January 1,000,000
Issue at full price on 31 March 300,000
1,300,000
Bonus issue on 30 June (1,300,000 ÷ 4) 325,000
1,625,000
(3) Share premium
Rs.
At 1 January 200,000
Issue at full price on 31 March ((300,000  0.30) – 20,000) 70,000
270,000
Bonus issue on 30 June (270,000)
NIL
(4) Retained earnings
Rs.
At 1 January 5,670,300
Minus: Bonus issue on 30 June (325,000 (W2) – 270,000 (W3) (55,000)
Add: Profit for the year (W1) 493,512
Add back: Preference dividends charged to retained earnings
(W5) 8,000
6,116,812
(5) Redeemable preference shares
Rs.
Liability at beginning of year
Year 1 ((100,000  Rs. 1.60) – 2,237)) 157,763
Finance charge at 9.5% 14,988
Interest paid at 4% (8,000)
Liability at end of year 164,751

© Emile Woolf International 229 The Institute of Chartered Accountants of Pakistan


Advanced accounting and financial reporting

15.2 PASSILA LTD


(a) The face value of the debentures
Rs. 100 X 20,000 = Rs. 2,000,000
The amount accrued to the company as proceeds =
Rs. 97.5 X 20,000 = Rs. 1,950,000
(b) The difference between the face value and the market value of the debentures is Rs.
50,000. This is as a result of discount allowed on the issue on the debentures. Discount
on debentures attracts investors.
(c) Nominal interest rate is the rate based specifically on the face value of the loan capital. In
case of Passila Ltd., the nominal interest rate on the debentures is 8% per annum on Rs.
2,000,000.
The effective interest is the rate based on the market value. This is the actual value
collected on issue which can be at par, discount or premium. For Passila Ltd., the
effective interest rate will be 8% of Rs. 1,950,000
(d) The nominal interest payable
Rs. 2,000,000 X 8% X 6 months ÷ 12 months
= Rs. 80,000
(e) (i) The face value of Rs. 2,000,000 will be the most appropriate valuation
to be disclosed in the Statement of financial position. The management may be
interested in the quoted market value or the proceeds, but for the sake of outside
investors who would only be interested in the company having good reputations
devoid of trading losses, it is advisable that the face value be adopted.
(ii) Disclosing the debentures’ liability at face value plus interest payment for five years
may seem proper in the eyes of external investors and credit institutions, but
principally, it would be wrong to credit debentures’ account with both the face value
and the interest payments. An interest payment on debentures is a revenue item
which is debited to the Profit and Loss Account.
(iii) Disclosing debentures’ liability at market value on the Statement of financial position
will amount to disclosure at replacement value. The market value should be
disclosed.

15.3 FINANCIAL INSTRUMENTS


(a) IFRS 9 requires that all financial assets and financial liabilities are recognised at fair value
at initial recognition. Separate guidance is given for the treatment of transaction costs.
When a financial instrument is acquired or issued it must be classified into one of two
asset categories or one of two liability categories. This classification then determines the
subsequent accounting treatment for the item. The two categories are “at amortised cost”
and at “fair value through profit or loss”.
Amortised cost
Amortised cost is calculated as:
Initial cost recognised
Plus: Interest at the effective rate
Minus: Cash received/paid
Fair value
Fair value is defined as “the price that would be received to sell an asset or paid to transfer
a liability in an orderly transaction between market participants at the measurement date”.

© Emile Woolf International 230 The Institute of Chartered Accountants of Pakistan


Answers

Financial assets
A financial asset must be measured at amortised cost if both of the following conditions are
met:
 the asset is held within a business model whose objective is to hold assets in order
to collect contractual cash flows; and
 the contractual terms of the financial asset give rise on specified dates to cash flows
that are solely payments of principal and interest on the principal amount
outstanding.
Any asset which is not measured at amortised cost must be measured at fair value. Any
fair value movement on these financial assets is reported in profit or loss.
In addition an entity is allowed to designate a financial asset that otherwise meets the
amortised cost criteria as FVTPL if it meets certain conditions. Furthermore an entity is
allowed to make an irrevocable decision on initial recognition of equity that any fair value
difference on that equity be recognised in OCI rather than P&L
Financial liabilities
A financial liability must be measured at amortised cost with specific exceptions including:
 Derivatives that are liabilities at the reporting date; and
 Financial liabilities that might arise when a financial asset is transferred but this
transfer does not satisfy the derecognition criteria. .
A company is allowed to designate a financial liability as measured at fair value through
profit or loss. This designation can only be made if:
 it eliminates or significantly reduces a measurement or recognition inconsistency; or
 this would allow the company to reflect a documented risk management strategy.
Any such designation is irrevocable.
(b) (i) 3% Bond
The bond must initially be recorded at its purchase price of Rs. 250,000. The bond
seems to satisfy the amortised cost criteria. The company seem to operate a
business model whose objective is to hold financial assets in order to collect
contractual cash flows and it seems that the cash it will collect will be solely payment
of interest and principle. The market value is not relevant.
Interest will be credited to profit or loss using the effective interest rate, resulting in
finance income of Rs. 24,250 (9.7% × 250,000). The effective rate reflects the total
return received by the investor over the duration of the bond – being the coupon +
Rs. 50,000 premium on redemption. The coupon recorded in the statement of cash
flows is Rs. 9,000 (3% × 300,000).
The difference between the effective interest and the actual coupon is added to the
investment to give an amortised cost at the end of Year 3 of Rs. 265,250 (250,000 +
24,250 – 9,000).

(ii) Equity shares in XYZ


The shares must be classified as FVTPL. They will initially be recorded at their cost
of Rs. 30,000. As they have been classed as ‘fair value through profit or loss’ the
transaction costs must be expensed to profit or loss immediately. At the end of each
reporting period, the shares must be re-measured to their market value, with the
resulting gain or loss being taken to profit or loss.
At 1 January Year 3, the investment has a carrying value of Rs. 34,000. By the 31
December Year 3 this value is now Rs. 35,000. A Rs. 1,000 gain will therefore be
recognised in profit or loss for the year.

© Emile Woolf International 231 The Institute of Chartered Accountants of Pakistan


Advanced accounting and financial reporting

(iii) Convertible bond


A convertible bond is a compound instrument. In essence, issuing a convertible
bond is equivalent to issuing a non-convertible bond plus a call option on the entity’s
shares. Therefore, the bond should be divided into a liability portion and an equity
portion in accordance with the rules in IAS 32.
Note that the investor would have to do something similar
To establish the liability (debt) element, the future cash flows from the bond are
discounted at the normal market rate to establish the value of an equivalent but
redeemable bond.
Using a rate of 7% this gives a net present value of:
20, 000 20, 000 520, 000
 2
 3
 $460, 635
1.07 1.07 1.07
As the bond was issued for Rs. 500,000, it implies that the call option embedded
within the bond was sold for Rs. 39,365 (Rs. 500,000 – 460,635).
 The liability component is measured at amortised cost after initial recognition.
Interest will be recognised at the effective rate of 7%. The difference between the
cash interest paid (4%) and the interest expense recognised will increase the
amortised cost of the liability year on year until the bond is redeemed.

15.4 ESPANOLA LTD


(a) (i) The held to maturity investment will be initially recorded at fair value plus transaction
costs. It will be subsequently measured at each year-end at amortised cost using
the effective interest rate.
(ii) Held to maturity investment -amortised cost using effective interest rate of 7.05%.
Year Opening Effective interest Interest Closing
end balance 7.05% received balance
Rs. Rs. Rs. Rs.
2016 3,200,000 225,600 (180,000) 3,245,600
Investment income - Income from HTM investment Rs. 225,600
Non-current assets - Held to maturity investment Rs. 3,245,600
(b) Held for trading investment
Initial recording:
Dr Current asset investment Rs. 300,000
Cr Bank Rs. 300,000
Being the purchase of shares
Dr Statement of profit or loss Rs. 12,000
Cr Bank Rs. 12,000
Being the write off of the transaction costs to the statement of profit or loss as the
investment is an asset held at fair value through profit or loss
Subsequent measurement
Dr Current asset investment Rs. 40,000
Cr Statement of profit or loss – gain Rs. 40,000
Being the uplift in value and the recording of the gain in the statement of profit or loss

© Emile Woolf International 232 The Institute of Chartered Accountants of Pakistan


Answers

15.5 SANDIA LTD


(i) Available for sale (AFS) Investment initially recorded at fair value plus transactions costs:
Dr AFS Investment (40,000 shares x Rs. 2.68) Rs. 107,200
Cr Bank Rs. 107,200
Being initial recognition of AFS asset
Dr AFS Investment Rs. 5,360
Cr Bank Rs. 5,360
Being 5% commission paid on purchase
The investment is subsequently measured at the fair value of the shares with the gain or
loss calculated as fair value of the investment less its carrying amount.
This is a valuation exercise, not a transaction, so there is no need to account for
commission when calculating the year end valuation [(40,000 x Rs. 2.96) - Rs. 112,560].
Dr AFS Investment Rs. 5,840
Cr Equity – other reserves Rs. 5,840
Being subsequent measurement of AFS asset
(ii) In accordance with IAS 39, all derivative contracts are classified as fair value through profit
and loss, therefore any gain or loss in the value of the derivative contract is taken directly
to the statement of profit or loss. Gains or losses on available for sale investments are
normally recorded through other comprehensive income. However, as hedge accounting
can be applied (because it has been designated as a hedge) then the gain/loss on both the
investment (hedged item) and the derivative contract (hedging instrument) can be offset
within the statement of profit or loss. Hedge accounting (for this fair value hedge) ensures
that the gain/loss on the AFS investment is taken to profit or loss and matched against the
gain/loss on the hedging instrument.

15.6 GEO ALLOYS LTD


(a) (i) The investment should be classified as held to maturity investment because GEO
Alloys Ltd intends to hold the investment until its redemption date. Initially the
investment will be measured at its fair value (which in this case is its cost), plus any
associated issue costs. The initial journal entry required is therefore:
DR: Investment in HTM investment Rs. 4,200,000
CR: Cash Rs. 4,200,000
(ii) Subsequent measurement
Year end Opening Effective interest Interest Closing
30 June balance 8.4% received balance
7% x Rs.
4m
Rs.000 Rs.000 Rs.000 Rs.000
2015 4,200 353 (280) 4,273
2016 4,273 359 (280) 4,352
The investment will be held at Rs. 4,352,000 in the statement of financial position at
30 June 2016.
(b) This forward contract is an example of a derivative and in accordance with IAS 39 such
derivative contracts are classified as an asset or liability held at fair value through profit
and loss. This would mean that at each year end the contract would need to be re-valued
to its fair value (being the difference between the derivative price and the market price of
the underlying asset under the contract). Any gains or losses would usually then be
recorded in profit or loss.

© Emile Woolf International 233 The Institute of Chartered Accountants of Pakistan


Advanced accounting and financial reporting

However this contract is specifically intended to mitigate the risk of future adverse cash
flows as a result of potential increases in raw material prices. This contract is therefore
being used as a cash flow hedge (because it’s being used to fix the price of material to be
acquired in the future on 1 August). In such circumstances IAS 39 has some special hedge
accounting rules. Hedge accounting allows the gains or losses on a derivative contract
being used in a cash flow hedge to be taken to reserves until the cash flow it is designed to
hedge against is recognised in the financial statements. In this case, the gains or losses
will be held in reserves until the year ended 30 June 2017 which is the year in which the
cash flow is actually incurred, and then released to the profit or loss.
In this case, a loss on the derivative of Rs. 40,000,000 (100,000 x Rs.(10,500  10,100))
will be included in reserves at 30 June 2016.

15.7 CASCABEL LTD


(a) At 31 July 2016 this instrument meets the definition of a derivative:
Small or no initial investment.
Its value is dependent on an underlying economic item; exchange rate.
Its settlement will take place at some future date.
As a derivative it should be accounted for as an “asset or liability held at fair value through
profit or loss”. The value of the derivative instrument will be the difference between the
value of the contract when settled compared with the cost of A$2m being purchased at the
spot rate at the year-end date.
Cost of A$2m at a contracted rate of A$0.64 = Rs. 3,125,000
Cost of A$2m at the forward rate of A$0.70 = Rs. 2,857,143
The derivative results in a liability at the year-end date of Rs. 267,857 (Rs. 3,125,000 - Rs.
2,857,143) as the contract has unfavourable terms when compared to the spot rate. The
loss on derivative would be charged to the statement of profit or loss in the year to 31
August 2016.
Recorded as:
Dr Statement of profit or loss (loss on derivative) Rs. 267,857
Cr Liabilities – derivatives Rs. 267,857
(b) If the derivative was designated as a hedging instrument in a cash flow hedge then the
loss of Rs. 267,857 would be recognised in other comprehensive income until the related
cash flow (hedged item) occurred, and shown as a loss in other comprehensive income in
the year ended 31 August 2016. This ensures that the movements in the hedged item and
the hedging item can be offset in the same accounting period.

15.8 FAIR VALUE HEDGE ACCOUNTING


(a) Journals at 31 December 2015

Debit Credit
Forward contract asset 95,000
P&L account – fair value gain 95,000
Being mark-to-market for the derivative

Debit Credit
P&L account – fair value loss on inventory 100,000
Inventory 100,000
Being fair value loss on inventory, attributable to the risk being hedged

© Emile Woolf International 234 The Institute of Chartered Accountants of Pakistan


Answers

(b) Journals at 31 March 2016

Debit Credit
Forward contract asset (142000 – 95000) 47,000
P&L account – fair value gain 47,000
Being mark-to-market for the derivative

Debit Credit
P&L account – further fair value loss on inventory 50,000
Inventory 50,000
Being fair value loss on inventory, attributable to the risk being hedged

Debit Credit
Bank 1,150,000
P&L account 1,150,000
Being sales proceeds

Debit Credit
Bank 142,000
Forward contract 142,000
Being forward contract closed out for cash

Debit Credit
P&L account – cost of sales 850,000
Inventory 850,000
Being inventory carrying value now derecognised upon sale

The results of these journals can be summarised as:

Debit/(Credit)
Cash Derivative Inventory P&L
Inventory brought forward 1,000,000
December:
Change in fair value (FV)
of forward 95,000 (95,000)
Change in FV of inventory (100,000) 100,000
March:
Change in FV of forward 47,000 (47,000)
Change in FV of inventory (50,000) 50,000
Revenue 1,150,000 (1,150,000)
Close out derivative 142,000 (142,000)
Cost of sale (850,000) 850,000
TOTALS 1,292,000 NIL NIL 292,000

© Emile Woolf International 235 The Institute of Chartered Accountants of Pakistan


Advanced accounting and financial reporting

15.9 CASH FLOW HEDGE ACCOUNTING


Fair value of the derivative contract at 31 December:

31 Dec 28 Feb
The cash flow under the contract will be
400,000 * 0.7 = Rs. 280,000 Rs. 280,000
The cash flow available in the market is
400,000 * 0.75 = Rs. 300,000
The cash flow available in the market is
400,000 * 0.80 = Rs. 320,000
Therefore the fair value of the derivative (an asset) is Rs. 20,000 Rs. 40,000

(a) Journals at 31 December

Debit Credit
Forward contract 20,000
Equity – cash flow hedge reserve 20,000
Being fair value change, deferred to equity as an effective cash flow hedge

(b) Journals at 28 February

Debit Credit
Forward contract 20,000
Equity – cash flow hedge reserve 20,000
Being fair value change January and February 2016

Debit Credit
Bank 40,000
Forward contract 40,000
Contract closed with payment from the FX dealer of 400,000 * (0.70-0.80) = Rs. 40,000

Debit Credit
Property, Plant and Equipment 320,000
Bank 320,000
Being initial recognition of purchase price of machine: 400,000 * 0.80 = Rs. 320,000

Debit Credit
Equity – cash flow hedge reserve 40,000
Property, Plant and Equipment 40,000
Being transfer of deferred gains/losses on closure of a cash flow hedge

The machine will therefore be recorded at 320,000 – 40,000 = Rs. 280,000

© Emile Woolf International 236 The Institute of Chartered Accountants of Pakistan


Answers

The results of these journals can be summarised as:


Debit/(Credit)
Cash Derivative Equity PP&E
December:
Change in (FV) of forward 20,000 (20,000)
February:
Change in FV of forward 20,000 (20,000)
Purchase of drilling rig (320,000) 320,000
Basis adjustment 40,000 (40,000)
Close out of derivative 40,000 (40,000)
TOTALS (280,000) - - 280,000

15.10 WATERS LTD


1 Investment in 7% treasury stock 2022
As there is not an intention to hold the investment to maturity, the investment should be
classified as at fair value (with gains and losses recognised in profit or loss).
At initial recognition it will be measured at fair value which is the consideration given of Rs.
208,200.
There is no interest received up to year end (first payment will be received on 31 October
2017)
The market value of the stocks at the reporting date is Rs. 196,140 and the revaluation
loss of Rs. 12,060 will be recognised in profit or loss.
Alternative:
Waters Ltd could choose to recognise finance income in the statement of profit or loss at
6.3%  208,200  4/12 = Rs. 4,372.
The financial asset will then have a carrying amount of Rs. 212,572 (208,200 + 4,372) prior
to remeasuring to the fair value. The market value of the stocks at the reporting date is Rs.
196,140 and the revaluation loss of Rs. 16,432 will be recognised in profit or loss.
Notice that the overall profit impact is the same in each case as *4,372 + (16,432) =
12,060.
2 Futures Prif contract
The derivative will be classified as at fair value through profit or loss.
Initial transaction costs cannot be included as part of the carrying amount and therefore
the fee of Rs. 750 will be immediately charged to profit or loss.
At the reporting date the contract is valued at the fair value of PR1.99/Rs. 1 so the loss is
Rs. 1,269 to be included in profit or loss and as a liability on the statement of financial
position
3 Investment in Gilmour Ltd
This would normally be classified as at fair value (with gains and losses recognised in
profit or loss).
On initial recognition it would be valued at fair value which would be the cost of Rs.
1,212,500. The directly attributable transaction costs (Rs. 35,000) would be expensed to
profit or loss.
At the reporting date the shares will be valued at fair value (Rs. 5.20 per share) ignoring
selling costs = Rs. 1,300,000.
The revaluation gain of Rs. 87,500 will be recognised in profit or loss.

© Emile Woolf International 237 The Institute of Chartered Accountants of Pakistan


Advanced accounting and financial reporting

Alternative:
Waters Ltd could have made an irrevocable election at initial recognition to recognise
gains and losses in other comprehensive income. If this election had been made the
shares would have been measured on initial recognition at the cost of Rs. 1,212,500 plus
directly attributable transaction costs Rs. 35,000 = Rs. 1,247,500.
At the reporting date the shares would then have been be valued at fair value with the
revaluation gain of Rs. 52,500 recognised in other comprehensive income.
4 Amount receivable from Mason
On recording the sale, the revenue needs to be discounted at the imputed rate of interest
of 11%. Revenue recognised on 1 July is therefore Rs. 450,450 (500,000  1.11).
The receivable on the statement of financial position will include the accrued interest
element of Rs. 24,775 (Rs. 450,450 x 0.11 x 6/12) and so will be Rs. 475,225 (Rs. 450,450
+ Rs. 24,775) in total. The accrued interest of Rs. 24,775 will be recognised as finance
income.
The receivable would not be adjusted for any change in interest rates.
5 Investment in 8.5% treasury stock 2018
This would be classified as to be subsequently measured at amortised cost.
On initial recognition, it will be recorded at fair value, the cost of Rs. 107,100.
Finance income will be credited to profit or loss using the gross redemption yield of 5.9%.
Interest recognised in profit or loss will be Rs. 4,213 (Rs. 107,100  5.9%  8/12).
The investment in the statement of financial position at 31 December 2016 will be at Rs.
107,100 plus Rs. 4,213 = Rs. 111,313. (No interest will have been received to date as it is
paid annually in arrears).
The market value is not reflected in the statement of financial position at 31 December
2016 but it would be disclosed in accordance with IFRS 7.
6 Investment in loan notes
The investment has been classified as held for trading so it is accounted for as a financial
asset at fair value through profit or loss.
On acquisition it will be recorded at its cost of Rs. 25,000.
At the reporting date the notes will be revalued to their fair value of Rs. 25,500 with the
Rs. 500 uplift being recognised in profit or loss
7 Selling shares short
On initial recognition, the journal would be:

Rs. Rs.
Dr Cash (10,000  Rs. 3.60) 36,000

Cr Financial liability 36,000

At the reporting date the financial liability must be revalued to its fair value of Rs. 33,000:

Rs. Rs.
Dr Financial liability 3,000
Cr Statement of profit or loss 3,000

© Emile Woolf International 238 The Institute of Chartered Accountants of Pakistan


Answers

15.11 ARIF INDUSTRIES LIMITED


(a) Amortisation table
Interest income @
Opening Expected Closing
15.5%
balance cash flow balance
(Cr P&L)
A B = A x 15.5% C E=A+B+C
2016 104,641,483 16,219,430 (15,000,000) 105,860,913
2017 105,860,913 16,408,441 (15,000,000) 107,269,354
2018 107,269,354 16,626,750 (20,000,000) 103,896,104
2019 103,896,104 16,103,896 (120,000,000) -
(b) Amortisation table with impairment
Interest
Impairment
Opening income @ Expected Closing
loss
balance 15.5% cash flow balance
(Dr P&L)
(Cr P&L)
A B=A x 15.5% C D E=A+B+C+D
2016 104,641,483 16,219,430 (15,000,000) - 105,860,913
2017 105,860,913 16,408,441 (15,000,000) (26,818,338) 80,452,016
2018 80,452,016 12,470,062 (15,000,000) - 77,922,078
2019 77,922,078 12,077,922 (90,000,000) - -

Computation of impairment loss:


Expected cash Discounted rate (effective
Present value
flow (revised) rate = 15.5%)
2018 15,000,000 0.8658 12,987,013
2019 90,000,000 0.7496 67,464,003
Revised present value (recoverable amount) 80,451,016
Existing present value (as at end of 2017) 107,269,354

Impairment loss 26,818,338


(c) Amortisation table with adjustment for future income estimate
Interest Effect of
Opening income @ Expected change in Closing
balance 15.5% cash flow estimate balance
(Cr P&L) (Cr P&L)
A B=A x 15.5% C D E=A+B+C+D
2016 104,641,483 16,219,430 (15,000,000) - 105,860,913
2017 105,860,913 16,408,441 (15,000,000) 1,622,535 108,891,889
2018 108,891,888 16,878,243 (20,000,000) - 105,770,132
2019 105,770,131 16,394,370 (20,000,000) - 102,164,502
2020 102,164,501 15,835,498 (118,000,000) - -

Computation of effect of change in estimate:

Expected cash flow Discounted rate (effective


Discounted
(revised) rate = 15.5%)
2018 (20,000,000) 0.8658 17,316,017
2019 (20,000,000) 0.7496 14,992,223
2020 (118,000,000) 0.6490 76,583,649

Revised present value 108,891,889


Existing present value (as at end of 2017) 107,269,354

Effect of change in estimate 1,622,535

© Emile Woolf International 239 The Institute of Chartered Accountants of Pakistan


Advanced accounting and financial reporting

15.12 QASMI INVESTMENT LIMITED


Qasmi Investment Limited
Journal entries for 31 December 2011 and 2012
Debit Credit
Date Description
Rs. in million
31-Dec-2011 Accrued Interest written off (P&L) 12.00
Accrued Interest - 2010 12.00
(Accrued interest on 12%TFCs for 2010 is no
more receivable, now written off.)
Financial assets (12%
TFCs) W.3 (16.89–12.00) 4.89
Interest income (P&L) 4.89
(Interest income on 12% TFCs at 4.426% for
2011)
Impairment loss (P&L) W.1 19.16
Financial assets (12%
TFCs) 19.16
(Impairment of financial assets (12% TFCs) as
interest for 2010 to 2013 is no more
receivable)
Financial assets(12%
31-Dec-2012
TFCs) W.1 (88.53×16.426%) 14.54
Interest income (P&L) 14.54
(Interest income for
2012)
Financial assets (12%
TFCs) W.2 10.31
Impairment reversal (P&L) 10.31

(Reversal of impairment of financial assets on


rescheduling of payments for TFCs)
W.1 Impairment
Carrying value of 12% TFCs on 31-12-2011 W.3 107.69
–2
PV of future cash flows on 31-12-2011 120×[(1.16426) ] 88.53
Impairment loss 19.16
W.2 Impairment Reversal
Revised carrying amount on rescheduling at lower of (A) and (B) below 113.38
(A) PV of the future cash flow as per the agreed revised schedule 115.00
(B) Amortised cost on impairment reversal date of 31-12-2012 would have
been had the impairment not been recognised. W.3 113.38
Existing carrying amount at 31-12-2012 88.53×1.16426 (103.07)
Impairment reversal 10.31

© Emile Woolf International 240 The Institute of Chartered Accountants of Pakistan


Answers

W.3 Original amortisation schedule


Effective interest @ Cash flow
Cash flow dates Amortised cost
16.426% (Interest @ 12%)
--------------------------------Rs. in million--------------------------------
01-Jan-2009 (100×95%) 95.00
31-Dec-2009 15.60 (12.00) 98.60
31-Dec-2010 16.20 (12.00) 102.80
31-Dec-2011 16.89 (12.00) 107.69
31-Dec-2012 17.69 (12.00) 113.38

© Emile Woolf International 241 The Institute of Chartered Accountants of Pakistan


Advanced accounting and financial reporting

CHAPTER 16: FINANCIAL INSTRUMENTS: PRESENTATION AND DISCLOSURE

16.1 SERRANO LTD


IAS 32 Financial Instruments: Disclosure and Presentation says that the issuer of a compound
(hybrid) instrument (i.e. one that contains both a liability debt and an equity element) should
classify the instrument’s components separately. Thus the advice of Ancho Services is wrong;
convertible loan stock cannot be classified as pure equity. The proceeds of the issue have to be
split between the amount attributable to the conversion rights, which is then classed as equity,
and the balance of the proceeds being classed a liability/debt. There are several methods of
obtaining these amounts, but from the information given in the question these can only be
calculated on a ‘residual value of equity’ basis:

Cash Factor at Present


flows 10% value

Rs.000 Rs.000
Year 1 interest 600 0.91 546
Year 2 interest 600 0.83 498
Year 3 interest 600 0.75 450
Year 4 interest and capital 10,600 0.68 7,208
––––––
Total value of debt component 8,702
Proceeds of the issue 10,000
––––––
Equity component (residual amount) 1,298
––––––

Statement of profit or loss: Rs.


Interest paid (6% of Rs. 10 million) 600,000
Provision for additional finance costs 270,000

((10%  8.702m) – 0.6m) 870,000

Statement of financial position:


Non-current liabilities:
6% Convertible Loan Stock (from above) 8,702,000
Provision for additional finance costs 270,000

8,972,000

Capital and reserves:


Option to convert to equity (from above) 1,298,000

© Emile Woolf International 242 The Institute of Chartered Accountants of Pakistan


Answers

16.2 POBLANO LTD


In the financial statements of Poblano Ltd for the year to 30 September 2016.
In the statement of profit or loss, the finance cost relating to the loan notes is Rs. 640,000.
In the statement of financial position:
Non-current liability for the loan notes = Rs. 9,384,000
Equity component of loan notes = Rs. 856,000.

Workings: Rs. 10 million of loan notes


Discount
Annual factor at Present
Year Cash flows cash flow 7% value
Rs.000 Rs.000
1 Interest 400 0.93 372
2 Interest 400 0.87 348
3 Interest + Redemption 10,400 0.81 8,424
Value as straight loan notes 9,144
Issue price 10,000
Equity component (residual amount) 856

Finance cost: year to 30 September 2016


Rs.000
Total finance cost: 9,144  7% 640
Interest payable on 30 September 2016 (Rs. 10 million  4%) 400
Accrual to add to carrying value of debt 240

Carrying value of loan notes: 30 September 2016


Rs.000
Initial valuation of debt element 9,144
Add accrued interest 240
Carrying amount at 30 September 2016 9,384

16.3 PIQUIN LTD


(a) Convertible instrument
A convertible instrument is considered part liability and part equity. IAS 32 requires that
each part is measured separately on initial recognition. The liability element is measured
by estimating the present value of the future cash flows from the instrument (interest and
potential redemption) using a discount rate equivalent to the market rate of interest for a
similar instrument with no conversion terms. The equity element is then the balance,
calculated as follows:
Rs.
PV of the principal amount Rs. 10m at 7% redeemable in 5 yrs 7,130,000
Rs. 10m x 0.713
PV of the interest annuity at 7% for 5 yrs 2,050,000
(5% x Rs. 10m) x 4.100
Total value of liability element 9,180,000
Equity element (balancing figure) 820,000
Total proceeds raised 10,000,000

© Emile Woolf International 243 The Institute of Chartered Accountants of Pakistan


Advanced accounting and financial reporting

The equity will not be remeasured, however the liability element will be subsequently
remeasured at amortised cost using the effective interest rate of 7%. The total finance cost
for the year ended 31 December 2016 is Rs. 642,600 (7% x 9,180,000). The coupon rate
of interest of 5% has already been charged to profit or loss in the year so a further Rs.
142,600 should be recorded:
Dr Finance costs Rs. 142,600
Cr Non-current liability Rs. 142,600
(b) Preference shares
The substance of the instrument is a debt instrument. IAS 32 requires that any instrument
that contains an obligation to transfer economic benefit be classified as a liability. The
cumulative nature of the returns on the preference shares means that the outflow of benefit
is inevitable. The preference shares would then be classified as debt and would in fact
increase the gearing of the entity.

16.4 AJI LTD


(a) IAS 32 requires that the equity and liability elements within convertible instruments be
initially recognised separately. The initial carrying amount of the liability is estimated by
measuring the fair value of a similar liability that has no equity element. This is achieved by
calculating the present value of the future cash flows associated with the instrument
assuming that it is not converted on redemption (ie: the interest and principal repayment
cash flows) discounted at the prevailing market rate for a similar instrument without
conversion rights. The difference between this amount and the proceeds (ie: the residual)
is recognised as equity.
The bonds are initially recognised as:
Rs.000 Rs.000
Dr Bank (proceeds of issue) 6,000
Cr Liability (W1) 5,609
Cr Equity (W2) 391

Working 1
Liability element Rs.000
PV of the principal (at 9% after 4 years) = (Rs. 6m x 0.708) 4,248
PV of interest of 7% on Rs. 6m for 4 years = (Rs. 6m x 0.07 x 3.24) 1,361
Total value of liability element 5,609
Working 2
Equity element Rs.000
Total proceeds raised on issue 6,000
Total value of liability element (5,609)
Value attributable to equity 391

(b) (i) In accordance with IAS 39, the liability element will be subsequently measured at
amortised cost using the effective interest rate (which in this case is the interest rate
used to discount the principal to PV, ie 9%). The equity element is not subsequently
re-measured.
The interest of Rs. 420,000 (7% x Rs. 6m) has already been paid and recorded. The
additional finance cost is recorded as:
Rs.000 Rs.000
Dr Finance costs (W1) 85
Cr Liability element of bonds 85

© Emile Woolf International 244 The Institute of Chartered Accountants of Pakistan


Answers

(ii) Extracts from statement of financial position Rs.000


Equity and Liabilities
Equity - Other component of equity 391

Liabilities (W1) 5,694

Working 1
Opening balance Finance cost at 9% Interest paid 7% Closing balance
Rs.000 Rs.000 Rs.000 Rs.000
5,609 505 (420) 5,694
Tutorial note:
The total finance cost for the year ended 31 December 2016 is Rs. 505K, however
the interest paid of Rs. 420K has already been recorded so only the difference of
Rs. 85K is recognised.

16.5 CHILTEPIN LTD


The convertible bonds on issue will be recorded as:
Dr Bank Rs. 3,900,000
Cr Liability Rs. 3,729,400
Cr Equity Rs. 170,600
Workings:

Time Narrative Cash flow Discount Present


factor (7%) value
1 to 4 Interest 5% x Rs. 4m x 3.387 3.387 677,400
4 Repayment Rs. 4,000,000 0.763 3,052,000

Fair value of the liability 3,729,400


Fair value of equity component (balancing figure) 170,600

Total fair value (amount raised) 3,900,000

The liability will then be accounted for in accordance with IAS 39, i.e. at amortised cost using the
effective interest rate of 7%.

Opening Finance cost Interest paid Closing


carrying value @ 7% carrying value

Rs. Rs. Rs. Rs.

y/e 30/9/2016 3,729,400 261,058 (200,000) 3,790,458

The interest paid of Rs. 200,000 has already been posted, so the additional Rs. 61,058 is
recorded as:
Dr Finance costs Rs. 61,058
Cr Liability Rs. 61,058

© Emile Woolf International 245 The Institute of Chartered Accountants of Pakistan


Advanced accounting and financial reporting

16.6 HABENERO LTD

(a) The preference shares will be classified as a liability despite being called “shares”. IAS 32
requires us to consider the substance of the instrument in order to determine whether it
should be classified as debt or equity. In this case the 5% dividend payable on the shares
is cumulative which will eventually result in an outflow of economic benefit for Habenero
Ltd and hence represents an obligation. It therefore meets the definition of a liability. Once
the principal amount is classed as a liability, it follows then that any payment associated
with this instrument (in this case the 5% dividend) will be presented as a finance cost and
be charged in arriving at profit for the year.
The ordinary shares have no inherent obligation as they will not be repaid, nor do they
provide any fixed return to the shareholder. Indeed ordinary shares contain only a residual
interest in the profits of the entity (i.e.: after all obligations have been settled) and hence
will be classified as equity. The associated dividend, when paid, will be presented in the
statement of changes in equity as a reduction in retained earnings.

(b) (i) Initial recognition of the HFT investment is at cost and transaction costs are
charged to the statement of profit or loss:
Dr HFT Investment Rs. 1,400,000
Cr Bank Rs. 1,400,000
Being recognition of investment (where Rs. 1,400,000 = Rs. 2.80 x 500,000 shares)
Dr Statement of profit or loss Rs. 7,000
Cr Bank Rs. 7,000
Being write off of transaction costs (where Rs. 7,000 = Rs. 1,400,000 x 0.5%), with
the costs taken to profit or loss rather than included as part of the initial investment
(because of being classified as HFT).
(ii) Subsequent measurement is at fair value with the gain or loss taken to profit or loss:
Dr HFT Investment Rs. 310,000
Cr Statement of profit or loss Rs. 310,000
Being the gain on HFT investment (where Rs. 310,000 = Rs.(3.42 – 2.80) x 500,000
shares), with the gain being recognised in profit for the year.

© Emile Woolf International 246 The Institute of Chartered Accountants of Pakistan


Answers

CHAPTER 18 – IAS 12: INCOME TAXES

18.1 SHAKIR INDUSTRIES


COMPUTATION OF TAX EXPENSE
FOR THE YEAR ENDED DECEMBER 31, 2016
2016
Rs.in
million
Profit before tax 15.80
Add: Inadmissible expenses
Accounting depreciation (Rs. 1.1 million + Rs.0.7 million) 1.80
Financial charges on finance lease 0.15
Penalty paid to SECP 0.70
Provision for gratuity 2.40
5.05

Less: Admissible expenses Rs. m


Tax depreciation 1.65
Lease payments 0.65
Payment of gratuity 1.60
Borrowing cost capitalised 2.30
6.20
Taxable profit for the year 14.65
Current tax expense @ 35% 5.13

COMPUTATION OF DEFERRED TAX EXPENSE


FOR THE YEAR ENDED DECEMBER 31, 2016
Carrying Tax Temp
amount base difference
Rs. m Rs. m Rs. m
Fixed assets – Owned 16.70 13.85 2.85
Fixed assets – Leased 1.80 - 1.80
Capital work in progress 2.30 - 2.30
Provision for gratuity (0.7 + 2.4 – 1.6) (1.50) - (1.50)
Obligation against assets subject to finance
lease (1.20) - (1.20)
Total 4.25
Deferred tax expense @ 35% 1.49

Rs.in
million
Deferred tax liability (Opening) 0.55
Deferred tax expense for the year (balancing figure) 0.94
Deferred tax liability as at December 31, 2016 (Rs. 4.25 million x 35%) 1.49

© Emile Woolf International 247 The Institute of Chartered Accountants of Pakistan


Advanced accounting and financial reporting

18.2 DWAYNE LTD (PART 1)


(a)
Carrying Tax Temporary To OCI
value base difference
Buildings 45,500 17,500
Revaluation 14,500  14,500
60,000 17,500 42,500

Plant 68,000 26,000 42,000


Investments 72,000 65,000 7,000 (1,000)
Dividend income 150 150 -
Loan 20,500 21,000 500
Defined Benefits 1,000  (1,000)
liability
91,000 13,500
@ 28% @28%
25,480 3,780

(b) Deferred taxation


liability
Rs. 000
B
Balance /F 13,500
2
Due to change in rate (13,500 × /30) (900)
28
13,500 x /30 12,600
To OCI (28% x 13,500) 3,780
To statement of profit or loss (as a balancing figure) 9,100
25,480
(c) Journal
Debit Credit
Deferred tax liability 11,980
Movement due to rate change:
2
OCI ((24,000 × 30%) × /30) 480
P&L (balancing figure) 420
Movement due temporary differences arising
during 2013
OCI 3,780
P&L 9,100

18.3 DWAYNE LTD (PART 2)


(a) Rs. 000
DWAYNE 25,480
LARRY FV Tax base
Buildings 600 300 300
Plant 56 25 31
Inventory 152 144 8
Retirement (100)  (100)
benefit
239
@28% 67
(b) 25,547

© Emile Woolf International 248 The Institute of Chartered Accountants of Pakistan


Answers

Deferred
taxation
liability
Rs. 000
B
(c) Balance /F 13,500
2
Due to change in rate (13,500 × /30) (900)
28
13,500 x /30 12,600
To OCI (28% x 13,500) 3,780
To statement of profit or loss (as a balancing figure) 9,100
Due to introduction of a new subsidiary 67
25,547
(d) Goodwill
Rs. 000
Cost 750
Less share of net assets
80% x (778 – 67) (569)
Goodwill arising 181

18.4 COHORT
Note for presentation to partner
Subject: Deferred Taxation
The calculation and presentation of deferred tax is considered by IAS 12 Income taxes. A
company is required to provide deferred tax on all material temporary differences using the full
provision method. Temporary differences arise because there is a difference in timing between
transactions being reflected in the financial statements and the item being taxed.
In light of the recent acquisitions of Legion and Air, deferred tax must be considered for the group
accounts. Additional tax issues arise at the group level that will not have been reflected in the
individual entity’s accounts and these points are outlined below.
Once the temporary differences have been identified, deferred tax must be provided at the tax
rate expected to be effective at the date when the tax is settled. Given this rate is not known
when the differences arise, a provision is made using the rates enacted at that time and the
estimate is then confirmed as tax changes arise.
Air
(a) The acquisition of air mid-year gives rise to a number of issues:
(1) Intangible asset
There is some concern that the acquisition of the database of key customers may
not be allowed for tax purposes but it has nevertheless been included in the tax
calculation on the assumption that a deduction will be allowed by the tax authorities.
If this deduction is not allowed, then an additional tax payment will need to be made
to the authorities, hence it would be prudent to recognise a liability for this amount
(probably classified as current taxation, rather than deferred taxation).
(2) Inter-company sales
When goods are sold between group members, the profits made are seen as
unrealised in the group accounts until the items are sold outside of the group.
However, the tax authorities tax the individual entities, not the group, and so the
profit will be subject to tax at the time of the inter-company sale. The unrealised
profit represents the temporary difference on which deferred tax must be provided.
The goods were sold at a margin of 33⅓%. Goods sold for Rs. 1.8m remain in
inventory at the year end, and hence the unrealised profit, and therefore temporary
difference, is Rs.0.6m.

© Emile Woolf International 249 The Institute of Chartered Accountants of Pakistan


Advanced accounting and financial reporting

(3) Unremitted profits


Un-remitted earnings represent a temporary difference in the group accounts. This
is because the tax base is the cost of the investment, yet in the consolidated
accounts, this cost is uplifted by the post-acquisition un-remitted profits. IAS 12
requires a provision to be made on this timing difference unless the parent controls
the timing of the reversal and it is probable that the difference will not reverse for the
foreseeable future. The payment of dividends is under the control of Cohort, but we
understand that the intention is to realise these un-remitted earnings through future
dividends and hence a provision must be made.
(b) Legion
The acquisition of Legion at the start of the year brings further deferred tax issues in the
group accounts as outlined below.
(1) Fair value through the profit or loss investments
The fair value adjustment has been reflected in the financial statements, yet the gain
is not taxed until the investments are sold. Hence the fair value adjustment of Rs.
4m gives rise to a temporary difference upon which deferred tax must be provided.
As the gain has been taken to profit or loss rather than other comprehensive income
or reserves, the deferred tax must also be expensed to profit or loss.
(2) General allowance
The allowance against the loan portfolio has reduced the carrying value of the loans
but the tax relief is not available until the loan is written off, and hence a temporary
difference is created on the provision. As the tax relief will reduce future tax charges,
the temporary difference of Rs. 2m gives rise to a deferred tax asset. The temporary
difference is accounted for even though there is no expectation that the difference
will reverse in the immediate future. However, a deferred tax asset can only be
reflected to the extent that it is probable that there will be future taxable profits
against which the temporary difference can be relieved.
(3) Unrelieved tax losses
When Legion was acquired, it had unused tax losses brought forward which could,
in principle, give rise to a deferred tax asset. However, it can only be recognised to
the extent that it is believed that the loss can be recovered. Given your belief that
there will not be sufficient future profits, the deferred tax can only be partially
recognised. If the fortunes of Legion change in the future, the deferred tax asset
should then be recognised, leading to a compensating amendment to goodwill.

18.5 MODEL TOWN GROUP


Adjustment to
financial Tax Temporary
statements base difference
Rs.000 Rs.000 Rs.000 Rs.000
Property plant, and equipment 10,000 2,400 7,600
Goodwill 6,000 6,000
Other intangible assets 5,000 0 5,000
Financial assets (cost) 9,000 1,500 9,000 1,500
──────
Total non-current assets 30,000
──────
Trade receivables 7,000 7,500 (500)
Other receivables 4,600 5,000 (400)
Cash and cash equivalents 6,700 6,700 –
────── ──────
Total current assets 18,300
──────
Total assets 48,300
──────

© Emile Woolf International 250 The Institute of Chartered Accountants of Pakistan


Answers

Long term borrowings 10,000 (400) 10,000 400


Deferred tax liability 3,600 3,600 –
Employee benefits 4,000 5,000 1,000
Current tax liability 3,070 3,070 –
Trade and other payables 5,000 4,000 (1,000)
────── ──────
Total liabilities 25,670 13,600
────── ──────
Share capital 9,000 –
Other reserves 4,500 1,500
400
Retained earnings 9,130
──────
Total equity 22,630
──────
Rs.000
Deferred tax liability 15,500 @ 30% 4,650
Deferred tax asset (1,900) @ 30% (570)
──────
Net deferred tax liability 13,600 @ 30% 4,080
Less existing liability (3,600)
──────
Adjustment to deferred tax 480
──────
(i) The financial assets should be valued at fair value with the increase going to OCI (Rs. 1.5
million).
(ii) The bond should be split into its equity and liability elements as per IAS 32.
(iii) As the development costs have been allowed for tax already, it will have a tax base of
zero. Goodwill is measured as a residual and, therefore, the impact is not measured under
IAS 12.
(iv) The accrual for compensation will not be allowed until a later period and, therefore, will
reduce the tax base relating to trade and other payables.

© Emile Woolf International 251 The Institute of Chartered Accountants of Pakistan


Advanced accounting and financial reporting

CHAPTER 19: BUSINESS COMBINATIONS AND CONSOLIDATION

19.1 HELLO
Consolidated statement of financial position as at 31 December 2016
Rs.
Assets
Non-current assets
Property, plant and equipment (225 + 175 + 10 – 2) 408,000
Goodwill (W3) 8,000

Current assets (271 + 157) 428,000


———–
844,000
———–
Equity and liabilities
Shareholders’ equity
Called up share capital 100,000
Retained earnings (W5) 291,800
———–
391,800
Non-controlling interest (W4) 79,200
Current liabilities 373,000
———–
844,000
———–

WORKINGS
(1) Group structure

Hello

60%

Solong
(2) Net assets of Solong Inc
Reporting Date of Post-
date acquisition acquisition
Rs. Rs.
Share capital 100,000 100,000
Retained earnings
Per the question 90,000
Less: Fair value adjustment for
2
depreciation ( /10 × 10,000) (2,000)
88,000 60,000
Fair value adjustment 10,000 10,000
198,000 170,000

© Emile Woolf International 252 The Institute of Chartered Accountants of Pakistan


Answers

(3) Goodwill Rs.


Cost 110,000
Net assets acquired
60%  170,000 (W2) (102,000)
————
8,000
————

(4) Non-controlling interest Rs.


40%  198,000 (W2) 79,200

(5) Retained earnings Rs.


Hello 275,000
Solong (60%  (88,000 – 60,000 (W2)) 16,800
————–
291,800
————–

19.2 HASAN LIMITED


Hasan Limited
Consolidated statement of financial position as at 31 March 2016
Rs.000 Rs.000
Assets
Non-current assets
Property, plant and equipment (W1) 4,020
Goodwill (W4) 480
Software (W1) 1,440
Investments (65 + 210) 275
–––––––––––––
6,215
Current assets
Inventories (W2) 1,274
Trade receivables (524 + 328) 852
Cash and bank (20 + 55 cash in transit) 75
–––––––––––––
2,201
–––––––––––––

Total assets 8,416


–––––––––––––

Equity and liabilities


Capital and reserves
Equity capital 2,000
Reserves
Share premium 2,000
Retained earnings (W3) 2,420
–––––––––––––
4,420
–––––––––––––

6,420
Non-controlling interest (W5) 350
Rs.000 Rs.000
Government grants (230 + 40) 270
Current liabilities
Trade payables (475 + 472) 947
Operating overdraft 27
Income tax liability (228 + 174) 402
–––––––––––––
1,376
–––––––––––––

Total equity and liabilities 8,416


–––––––––––––

© Emile Woolf International 253 The Institute of Chartered Accountants of Pakistan


Advanced accounting and financial reporting

Workings
(W1) Property, plant and equipment
Rs.000
Balance from question – Hasan Limited 2,120
Balance from question – Shakeel Limited 1,990
Fair value adjustment on acquisition (see below) (120)
Over-depreciation re fair value adjustment year to 31 March 2016 30
–––––––––––––

4,020
–––––––––––––

A fair value of the leasehold based on the present value of the future rentals (receivable in
advance) would be the next (non-discounted) payment of the rental plus the final three
years as an annuity at 10%:
Rs.000
PV of rental receipts: Rs. 80,000 + (Rs. 80,000  2.50) 280
Carrying value on acquisition is (400)
–––––––––––––

Fair value reduction of leasehold (120)


–––––––––––––

The depreciation of the leasehold in Shakeel Limited’s accounts would be Rs. 100,000 per
annum. However in the consolidated accounts it should be Rs. 70,000 (Rs. 280,000/4).
This would require a reduction in depreciation of Rs. 30,000 in the consolidated accounts
for the next four years.
Software:
Shakeel Consolidated Difference
Limited’s figures
accounts
Rs.000 Rs.000
Capitalised amount 2,400 2,400
Depreciation to
31 March 2015 (300) 8 year life (480) 5 year life
––––– –––––
Value at date of 180 fair
acquisition 2,100 1,920 value adjustment
Depreciation to 180 additional
31 March 2016 (300) (480) amortisation
––––– –––––
Carrying value
31 March 2016 1,800 1,440
––––– –––––
(W2) Inventories
Rs.000
Amounts given in the question (719 + 560) 1,279
Unrealised profit in inventories (25  25/125) (5)
–––––––––––––

1,274
–––––––––––––

© Emile Woolf International 254 The Institute of Chartered Accountants of Pakistan


Answers

(W3) Retained earnings


Rs.000
Retained profits of Shakeel Limited, 31 March 2016 1,955
Adjustments:
Excess charge for leasehold depreciation 30
Insufficient charge for Software amortisation (180)
Unrealised profit in inventory (W2) (5)
–––––––––––––

Adjusted retained profits at 31 March 2016 1,800


Retained earnings of Shakeel Limited at 1 April 2015 2,200
–––––––––––––

Shakeel Limited: loss for the year (post-acquisition loss) (400)


–––––––––––––

Rs.000
Parent company share of post-acquisition loss (90%) (360)
Hasan Limited reserves at 31 March 2016 2,900
Goodwill impairment (120)
–––––––––––––

Consolidated retained profits at 31 March 2016 2,420


–––––––––––––

(W4) Goodwill
Rs.000
At acquisition date
Shares of Shakeel Limited 1,500
Share premium of Shakeel Limited 500
Retained earnings of Shakeel Limited 2,200
Fair value adjustments:
Leasehold (W1) (120)
Software (W1) (180)
–––––––––––––

3,900
–––––––––––––

Acquired by Hasan Limited (90%) 3,510


Cost of investment 4,110
–––––––––––––

Goodwill at acquisition 600


Impairment 120
–––––––––––––

Goodwill at 31 March 2016 480


–––––––––––––

(W5) Non-controlling interests


Rs.000
Share capital of Shakeel Limited 1,500
Share premium of Shakeel Limited 500
Adjusted retained earnings of Shakeel Limited, 31 March 2016 (W3) 1,800
Fair value adjustments:
Leasehold (120)
Software (180)
–––––––––––––

Total net assets at 31 March 2016 3,500


–––––––––––––

Non-controlling interests (10%) 350


–––––––––––––

© Emile Woolf International 255 The Institute of Chartered Accountants of Pakistan


Advanced accounting and financial reporting

(W6) Elimination of current accounts:


Rs.000
Shakeel Limited’s current account with Hasan Limited per question 75
Deduct cash in transit regarding this balance (15)
–––––––––––––

Adjusted figure to cancel 60


–––––––––––––

(W7) Elimination of intra-group loan:


Rs.000
Investment in Hasan Limited’s books 200
Deduct repayment in transit (40)
–––––––––––––

Non-current liability in Shakeel Limited’s books 160


–––––––––––––

19.3 FLAMSTEED LTD AND HALLEY LTD


(a) An impairment loss is the amount by which the carrying amount of an asset or a cash
generating unit exceeds its recoverable amount.
(b) The following external sources of information may indicate that an asset is impaired.
(i) There are observable indications that the assets value has declined during the
period significantly more than would be expected as a result of the passage of time
or normal use.
(ii) Significant changes with an adverse effect on the entity have taken place during the
period or will take place in the near future, in the technological, market, economic or
legal environment in which the entity operates or in the market to which the asset is
dedicated.
(iii) The carrying amount of the net asset of the entity is more than its market
capitalization.
(iv) The carrying amount of the investment in the separate financial statements exceeds
the carrying amount in the consolidated financial statement of the investee’s net
asset, including associated goodwill, or the dividend exceeds the total
comprehensive income of the subsidiary, joint venture or associates in the period
the dividend is declared.
(v) Market interest rate or other market rate of return on investment have increased
during the period and those increases are likely to affect the discount rate used in
calculating the asset value in use and decrease the assets recoverable amount
materially.
(c) Flamsteed Ltd group: extract of consolidated statement of financial position as at 30 June
2016
Rs.‘000
Assets
Non-Current Assets
Property, Plant and Equipment (100,000 + 80,000) 180,000
Goodwill (WK) 13,468
Intangible-brand name 10,000
203,468
Current Assets
Inventory (6,000 + 16,000) 22,000
Receivables (32,000 + 14,000) 46,000
Cash (4,000 + 0 + 4,000) 8,000
76,000

Total Assets 279,468

© Emile Woolf International 256 The Institute of Chartered Accountants of Pakistan


Answers

Workings
(i) Goodwill
Rs.‘000 Rs.‘000
Consideration transferred 77,468
Fair value of NCI 18,000
Net Asset acquired as represented by: 95,468
Ordinary share capital 50,000
Revaluation surplus on acquisition 10,000
Retained earnings on acquisition 12,000
Intangible assets (brand name) 10,000 (82,000)
13,468
Note
st st
The deferred consideration has been discounted at 7% for 2 years (1 July 2015 – 1 July
2017).

19.4 BRADLEY LTD


Consolidated statement of financial position as at 31 December 2016
Rs.’Million Rs.’Million
Non-current assets
Goodwill (working 1) 120
Land & building (630 + 556 + 140) 1,326
Machinery & equipment (570 + 440) 1,010
2,456
Current assets
Inventory (714 +504 – 24) 1,194
Trade receivables (1,050 + 252 – 50) 1,252
Cash/Bank (316 + 60) 376 2,822
5,278
Ordinary shares of Rs. 1 each 3,000.0
Retained earnings (Working 3) 1,323.2
Non-controlling Int. (Working 4) 376.8
4,700
Current liabilities
Trade payables (440 + 188 - 50) 578
5,278
Workings:
Rs. million
1. Calculation of goodwill:
Fair value of consideration 1,320
Plus fair value of NCI at acquisition 330
Less net acquisition – fair value of
Assets acquired & liability:
Share capital 1,200
Retained Earning 190
Fair value adj at acquisition 140 (1,530)
Goodwill 120

© Emile Woolf International 257 The Institute of Chartered Accountants of Pakistan


Advanced accounting and financial reporting

Rs. million
2. Group structure
960 million
 100
1,200 million 80%
3. Retained earnings:
As per question 1,160 424
Adjustment (unrealised profit) (24)
Pre-acquisition retained earnings (190)
234
Group share of post-acquisition retained
earnings:
(80% x 234) 187.2
1,323.2
4. Non-controlling interest: Rs. million
Fair value of NCI at acquisition 330.0
Plus NCI’s share of post-acquisition
retained earnings (20% x 234) 46.8
376.8
Alternative workings:

(W1) Fair value adjustment:


Dr: Consolidated land & building Rs. 140 million
Cr: Revaluation reserve Rs. 140 million

(W2) Consolidation Schedule


Bliss Ltd Rs. ’M Bradley in NCI Post –
Bliss 80% 40% Acq
(W3)
Rupees in millions
Ordinary share capital 1,200 960 240
Revaluation Res. (W1) 140 112 28
Retained earnings 424 152 84.8 187
────
Net assets acquired 1,224
Cost of acquisition (1,320)
Goodwill (partial value) (96)
Goodwill attribute to NCI (W5) (24) 24
────
Goodwill (fair value) ( 120)
────
Unrealised profit on inventory - (24)
NCI (fair value) 376.8
────
Retained earnings of Bradley Ltd 1,160
Consolidated retained earnings 1,323

© Emile Woolf International 258 The Institute of Chartered Accountants of Pakistan


Answers

(W3) Bradley in Bliss = = 80%

(W4) Bradley’s share of Bliss’s pre-acquisition retained earnings = 80% x


= 80% x Rs. 190m = Rs. 152m

(W5) Goodwill attribute to NCI Rs. m


Fair value of NCI @ date of acquisition 330
Less: fair value of net assets attributable to
NCI (20% x (1,200 + 140 + Rs. 190m)) 306
Goodwill attributable to NCI 24

19.5 X LTD
Consolidated statement of financial position as at 31 December 2016
for the X Ltd Group
All workings in Rs.000
ASSETS Rs.000
Non-current assets
Property, plant and equipment (12,000 + 4,000 + 750(W1)) 16,750
Goodwill (W2) 208
Intangible asset (W1) 90
17,048
Current assets
Inventories (2,200 + 800 -30 (W3)) 2,970
Receivables (3,400 + 900) 4,300
Cash and cash equivalents (800 + 300) 1,100
8,370
Total assets 25,418

EQUITY AND LIABILITIES


Equity
Share capital (Rs. 1 equity shares) 10,000
Retained earnings (W4) 7,893
Total equity attributable to parent 17,893
Non-controlling interest (W5) 1,741
Total equity 19,634
Non-current liabilities
Long term borrowings 2,700
Current liabilities (2,000 + 1,000 + 84) 3,084
Total liabilities 5,784
Total equity and liabilities 25,418

© Emile Woolf International 259 The Institute of Chartered Accountants of Pakistan


Advanced accounting and financial reporting

Workings
1. Fair value adjustments
At acquisition
Movement 31 Dec 2016
date
Rs.000 Rs.000 Rs.000
PPE 800 (50) 750
Inventories 200 (200) -
Intangible assets 150 (60) 90
Liabilities (210) 126 (84)
──── ──── ────
940 (184) 756
──── ──── ────
2. Goodwill
Rs.000 Rs.000
Consideration transferred 3,800
NCI at fair value 1,600
────
5,400
Net assets at fair value:
Share capital 1,000
Retained earnings 3,200
Fair value adjustments 940
────
(5,140)
────
Goodwill on acquisition 260
20% impairment (52)
────
Goodwill at 31 December 2016 208
────
3. Unrealised profit on inventories
Sales of Rs. 300k x 20% x 50% left in inventories at y/e = Rs. 30,000
4. Retained earnings
Rs.000 Rs.000
As per SOFP 7,500 4,000
Pre-acquisition reserves (3,200)
Adjustments arising from movement in
FV adjustments (184)
────
616
────
Group share 75% 462
Unrealised profit on inventory transfer (30)
Goodwill impairment (75% x 52)(W2) (39)
────
Consolidated reserves 7,893
────
5. Non-controlling interests
Rs.000
NCI at acquisition (at fair value) 1,600
25% x post-acquisition retained earnings Rs. 616,000 (W4) 154
Goodwill impairment (25% x 52)(W2) (13)
─────
1,741
─────

© Emile Woolf International 260 The Institute of Chartered Accountants of Pakistan


Answers

19.6 KHAN LIMITED


(a) Khan Limited
Consolidated statement of financial position as at December 31, 2016
Rupees in
ASSETS million
Non-current assets
Property plant & equipment (W- 1) 14,800
Goodwill (W - 2) 100
14,900

Current assets (1,069+ 1,316) 2,385

17,285
EQUITY AND LIABILITIES
Equity
Share capital 6,800
General reserve (W5) 1,975
Retained earnings 3,844
12,619
Non-controlling interest (W8) 2,420
Total equity 15,039

Non-current liabilities
14% Term finance certificates (2,250-1,500) 750

Current liabilities
Accounts payable (445 + 190) 635
Dividend payable (W3) 861
17,285

(b) Khan Limited


Consolidated statement of profit or loss and other comprehensive income for the
year ended December 31, 2016
Rupees in
million
Profit before tax and interest (W4) 4,315
Interest expense (315 - 210) (105)
Profit before tax 4,210
Taxation expense (650 + 474) (1,124)
Profit for the period 3,086
Other comprehensive income -
Total comprehensive income 3,086

Attributable to:
Equity holders of parent Balancing 2,894
Non-controlling interest (W-7) 192
3,086

© Emile Woolf International 261 The Institute of Chartered Accountants of Pakistan


Advanced accounting and financial reporting

(c) Khan Limited


Consolidated statement of retained earnings for the year ended December 31, 2016
Rupees in
million
Balance as at January 1, 2016 (W- 6) 1,700
Total comprehensive income for the year 2016 2,894
Dividends (750)
Balance as at December 31, 2016 3,844
W1 – Property, plant & equipment
Rupees in
million
Cost – KL 16,250
Cost – GL 25,000
Acc. depreciation – KL (9,750)
Acc. depreciation – GL (17,000)
14,500
Fair value adjustment 1,000
Less: Depreciation on increased fair
value (Rs. 1,000 x 10% x 7) (700)
14,800
W2 – Goodwill
Purchase consideration 5,500
Less:
Share capital (75% of 5,000) (3,750)
Retained earnings (75% of 1,000) (750)
General reserve (75% of 200) (150)
FV increase in PPE (1,000 x 75%) (750)
100
W3 - Dividend payable
Ordinary dividend – KL 750
Ordinary dividend - GL (300 x 25%) 75
Preference dividend - GL (60 x 60%) 36
861

W4: Profit before tax and interest


KL 2,865
GL 1,550
Current year depreciation on increased value of
PPE (1,000 x 10%) (100)
4,315
W5: General reserve
General reserve – KL 1,750
General reserve – GL (500 – 200) x 75% 225
1,975
W6: Retained earnings
Rupees in
million
Retained earnings – KL 2,000
Retained earnings – GL (1,200 - 1,000) x 75% 150
Less: Depreciation charge on increased FV (1,000 x 6 x 10% x 75%) (450)
1,700

© Emile Woolf International 262 The Institute of Chartered Accountants of Pakistan


Answers

W7: Non-controlling interest (for statement of comprehensive income)


Share from profit of GL (1,550+210-300-474—120) x 25% 217
Less: Current year depreciation on
increased of PPE (100 x 25%) (25)
192
W8: Non-controlling interest (for statement of financial position)
Rupees in
million
Share capital (5,000 x 25%) 1,250
Preference shares (1,000 x 60%) 600
General reserve (500 x 25%) 125
Opening retained earnings (1,200 x 25%) 300
Comprehensive income for the year (W-7) 192
Increase in FV of building (1,000 x 25%) 250
Less: Depreciation charge on
increased FV (1,000 x 6 x 10% x 25%) (150)
Less: Dividend on ordinary shares (300 x 0.25) (75)
Less: Dividend on preference shares (120 x 0.6) (72)
2,420

© Emile Woolf International 263 The Institute of Chartered Accountants of Pakistan


Advanced accounting and financial reporting

CHAPTER 20: CONSOLIDATED STATEMENTS OF PROFIT OR LOSS AND OTHER


COMPREHENSIVE INCOME

20.1 MILLARD LTD


Millard Ltd: Consolidated profit and loss account for the year ended 31 December 2016
NOTE Rs.’000
Revenue (1) 425,000
Cost of sales (2) (162,600)
Gross profit 262,400
Distribution costs (35,000)
Impairment losses (4) (7,000)
Administrative expenses (28,000)
Operating profit 192,400
Investment income (5) 3,750

Debenture interest (6) (58,750)


Profit before tax 137,400
Tax (52,500)
Profit after tax 84,900
Non-controlling Interests (7) (8,380)
Group profit 76,520
Preference dividends (13,750)
Ordinary dividends (20,000)
Retained profit 42,770
Retained profit 1 January 2016 (8) 72,350
Retained profit 31 December 2016 115,120
Workings:
1 Revenue: Rs.’000
Millard 312,500
Fillmore 125,000
437,500
Less: inter-company group sales 12,500
425,000

2 Cost of Sales: Rs.’000


Millard 125,000
Fillmore 50,000
Inter-company sales (12,500)
Unrealized profits (3) 100
162,600

3 Provision for unrealised profits on inventory Rs.’000


25 2000 400
2016 
125 1
25 1500
2015  (300)
125 1
Charged to profit and loss a/c 100

© Emile Woolf International 264 The Institute of Chartered Accountants of Pakistan


Answers

4 Goodwill on acquisition: Rs.’000 Rs.’000


Cost of investment 67,000
Net assets acquired:
- On ordinary share capital 62,500
- Profit and loss account 12,500
75,000
Percentage acquired 50/62.5 x 100 = 80%
80% of Rs. 75,000 (60,000)
Goodwill (written-off in 2016) 7,000

5 Investment income: Rs.’000


As per draft account 7,950
Inter-company dividend 80% x Rs. 5,250 (4,200)
Investment income 3,750

6 Debenture interest Rs.’000


Parent’s 47,500
Subsidiary 15,000
Inter-company amount 25% x Rs. 15,000 (3,750)
Investment income 58,750

7 Non-controlling interest:
Rs.’000 Rs.’000
Profit after tax 24,500
Less: Preference dividend (4,375) 4,375
20,125 x 20% 4,025
8,400
Non-Controlling Interest share of unrealized profit 20% x Rs. 100 (20)
8,380

8 Retained Profit b/f Rs.’000 Rs.’000


Parent’s 66,750
Subsidiary 19,500
Pre-acquisition (12,500)
Parent’s share 7,000 x
80% 5,600
72,350

© Emile Woolf International 265 The Institute of Chartered Accountants of Pakistan


Advanced accounting and financial reporting

20.2 SHERLOCK
Sherlock Ltd: Consolidated statement of profit or loss and other comprehensive income for the year
ended 31 December 2016.
Rs. m
Revenue 538.0
Cost of sales (383.0)
Gross profit 155.0
Other income 29.0
Administrative costs (30.0)
Other expenses (72.6)
Operating profit 81.4
Finance costs (10.0)
Finance income 15.0
Profit before tax 86.4
Income tax expense (31.0)
Profit for the year 55.4
Other comprehensive income
Revaluation surplus 7.8
Remeasurement 2.0
Loss on cash flow hedge (3.0)
6.8
Total comprehensive income for year 62.2
Profit attributable to:
Rs. m
Owners of the parent (balancing figure) 43.8
Non-controlling interest (W1a) 11.6
55.4
Total comprehensive income attributable to:
Rs. m
Owners of the parent (balancing figure) 51.8
Non-controlling interest (W1a) 10.4
62.2
Workings
W1 Balances for inclusion in the consolidated statement of profit or loss and other
comprehensive income
Sherlock Katie Ltd
Mycroft Ltd Adjustment Total
Ltd (6/12)
Rs. m Rs. m Rs. m Rs. m Rs. m
Revenue 400 115 35 (12) W3 538
Cost of sales (312) (65) (18) 12 W3 (383)
Gross profit
Other income 21 7 1 29
Administrative costs (15) (9) (6) (30)
Other expenses (35) (19) (4)
Goodwill impairment W2 (3)
Pension cost W4 (7.2)
Revaluation W5 (2.4)
Share Katie Ltd W6 (2.0) (72.6)
Operating profit

© Emile Woolf International 266 The Institute of Chartered Accountants of Pakistan


Answers

Sherlock Katie Ltd


Mycroft Ltd Adjustment Total
Ltd (6/12)
Rs. m Rs. m Rs. m Rs. m Rs. m
Finance costs (5) (6) (2)
Cash flow hedge W7 3.0 (10)
Finance income 6 5 4 15
Profit before tax
Income tax expense (19) (9) (3) (31)
Profit for the year 41 22 7
OCI
Revaluation surplus 10
Revaluation W5 (2.2) 7.8
Remeasurement W4 2 2.0
Cash flow hedge W7 (3.0) (3.0)
Total CI for year 50.8 19 7 6.8
W1a Non-controlling interests
Mycroft Ltd Katie Ltd Total
Profit for the year 22 7
NCI percentage holding 40% 40%
NCI 8.8 2.8 11.6

Mycroft Ltd Katie Ltd Total


Total CI for the year W1 19 7
NCI percentage holding 40% 40%
NCI 7.6 2.8 10.4
W2 Goodwill write off
Rs. m
Cost of investment 80
FV of NCI at acquisition 45
125
Fair value of identifiable net assets (110)
Goodwill on acquisition 15
Write off in previous year (20%) (3)

W3 Inter-company trading
The inter-company trading amounts must be eliminated (ie sales and purchases).
There is no adjustment in respect of the loss. The question states that the sale is at fair value.
Therefore the loss is realised. Only unrealised amounts are adjusted on consolidation.
W4 Pension scheme
Amounts charged to profit or loss: Rs. m
Interest (10% of (Rs. 50m – Rs,48m)) 0.2
Current service cost 4.0
Past service cost 3.0
7.2
Amount charged to OCI
Remeasurement 2.0

© Emile Woolf International 267 The Institute of Chartered Accountants of Pakistan


Advanced accounting and financial reporting

W5 Revaluation of plant
Rs. m
Original cost (1 January 2015) 12.0
Depreciation in year ended 31 December 2015 (1.2)
Carrying amount before revaluation at 31 December 2015 10.8
Revaluation recognised in year ended 31 December 2015 2.2
Value at 31 December 2015 13.0
Depreciation in year ended 31 December 2016 (÷9) (1.4)
Carrying amount before revaluation at 31 December 2016 11.6
Fall in value to be recognised 4.6
Value at 31 December 2016 7.0
Dr Cr
Rs. m Rs. m
Plant 4.6
Statement of profit or loss 2.4
Other comprehensive income 2.2
W6 Share Katie Ltd expense
Rs. m
Balance recognised in year ended 31 December 2015
8,000 Katie Ltds  Rs. 100  4 directors  1/4 0.8

Balance recognised in year ended 31 December 2015


8,000 Katie Ltds  Rs. 100  7 directors  2/4 2.8
Charge for the year ended 31 December 2016 2.0
Dr Cr
Rs. m Rs. m
Statement of profit or loss 2.0
Equity 2.0
W7 Cash flow hedge
Mycroft Ltd’s loss on the effective cash flow hedge has been treated incorrectly. The effective part
of any gain or loss on a cash flow hedge should be recognised in other comprehensive income
and accumulated in a cash flow hedge reserve in equity.
The following corrective journal is necessary:
Dr Cr
Rs. m Rs. m
Loss on cash flow hedge
(in other comprehensive income) 3
Finance cost (profit or loss) 3

© Emile Woolf International 268 The Institute of Chartered Accountants of Pakistan


Answers

20.3 FAISAL LIMITED


Faisal Limited
Consolidated statement of financial position as at 31 December 2016
Rs. in million
Non-current assets
Property, plant and equipment (W1) 31,926.00
Accumulated depreciation (W1) (7,491.00)
24,435.00
Goodwill (W6) 1,380.00
Other investments 11,100.00
36,915.00
Current assets
Inventory (W3) 23,740.00
Accounts receivable (6,240 + 2,460 + 6,580  800) 14,480.00
Cash and bank balances (4,920 + 660 + 2,700) 8,280.00
46,500.00
Total assets 83,415.00

Equity and liabilities


Share capital 30,000.00
Retained earnings (W8) 42,379.75
72,384.75
Non-controlling interest (W7) 5,655.25
78,035.00
Current liabilities
Accounts payable (2,760 + 1,980 + 1,440  800) 5,380.00
Total equity and liabilities 83,415.00

Faisal Limited
Consolidated statement of profit or loss
for the year ended 31 December 2016
Rs. in million
Sales (W4) 100,100.00
Cost of sales (W4) (80,991.00)
Gross profit 19,109.00
Operating expenses (3,600 + 2,100 + 5,400) (11,100.00)
8,009.00
Gain on sale of non-current assets (540 – 54) 486.00
Dividend income (1,080 – (80%  600)) 600.00
Profit for the year 9,095.00
Attributable to:
Ordinary shareholders of parent 8,599.75
Non-controlling interest (W9) 495.25
9,095.00

© Emile Woolf International 269 The Institute of Chartered Accountants of Pakistan


Advanced accounting and financial reporting

Workings (all figures in millions of rupees)

(W1) Unrealised profit adjustments; Transfer of non-current asset


Figures in Figures if no
the transfer had Adjustment
accounts been made required
Against SL’s figures:
Cost 144 150 6 Dr
Accumulated depreciation
(144/3 years)  6/12 (24)
150  3 years/5 years (75)
(24) (75) 51 Cr
Carrying amount 120 75
Charge for the year 24 15 9 Cr

Against FL’s figures:


Profit on disposal 54 54 Dr

Double entry in consolidated financial statements Dr Cr


Profit on disposal 54
Depreciation charge for year 9
Property, plant and equipment (cost) 6
Accumulated depreciation 51

NCI in the statement of profit or loss 2.25


NCI in the statement of financial position 2.25
Being the NCI share of the depreciation adjustment (20%  9)

Accumulated
Consolidated balances PP and E depreciation
Rs. millions Rs. millions
FL 22,500 5,760
SL 3,480 420
AIL 5,940 1,260
Adjustments for inter-company
transfer 6 51
31,926 7,491
(W2) Unrealised profit adjustments: Inter-company trading
FL to SL to AIL to Total
AIL AIL FL
Sales 2,400 1,800 3,600 7,800

Inventory held 900 600 1,200


Gross profit percentage on sale 20% 10% 30%
Unrealised profit 180 60 360 600
NCI’s share (based on selling
company’s NCI)
25%  60 15
20%  360 72

© Emile Woolf International 270 The Institute of Chartered Accountants of Pakistan


Answers

Double entry in consolidated financial statements Dr Cr


Cost of sales (closing inventory) 600
Closing inventory in statement of financial position 600

NCI in the statement of financial position 87


NCI in the statement of comprehensive income 87

Impact on consolidated retained earnings (600 – 97) 513

(W3) Consolidated inventory


Rs.
millions
FL 14,460
SL 4,200
AIL 5,680
(600)
23,740

(W4) Consolidated sales and cost of sales


Cost of
Sales sales
Rs. millions Rs. millions
FL 57,600 49,200
SL 16,500 18,000
AIL 33,800 21,000
Inter-company sales (7,800) (7,800)
Unrealised profit 600
Depreciation adjustment on inter-company
transfer of non-current assets (9)
100,100 80,991

(W5) Net assets of SL


At end of
At reporting
acquisition period
Rs. millions Rs. millions
Share capital 12,000 12,000
Retained earnings (3,600) -
8,400 12,000

Net assets of AIL


At end of
At reporting
acquisition period
Rs. millions Rs. millions
Share capital 6,000 6,000
Retained earnings (12,200) 5,400
18,200 11,400

© Emile Woolf International 271 The Institute of Chartered Accountants of Pakistan


Advanced accounting and financial reporting

(W6) Goodwill
SL AIL
Rs. millions Rs. millions
Cost of investment 9,000 10,500
NCI at acquisition
25%  12,000 (W5) 3,000
20%  11,400 (W5) 2,280
12,000 12,780
Net assets acquired (12,000) (11,400)
 1,380

(W7) Non-controlling interest in statement of financial position


SL AIL
Rs. millions Rs. millions
NCI at acquisition
25%  12,000 (W5) 3,000
20%  11,400 (W5) 2,280
NCI’s share of post-acquisition profit / (loss)
25%  (3,600) (W5) (900)
20%  (12,200 – 5,400) (W0) 1,360
2,100 3,640
Total (2,100 + 3,640) 5,740.00
Unrealised profit adjustments
on inter-company sale of inventory (W2) (87.00)
on inter-company sale of non-current asset (W1) 2.25
5,655.25

(W8) Retained earnings


Rs. millions
FL 40,200.00
Share of SL (75%  (3,600)) (2,700.00)
Share of AIL (80%  (12,200 – 5,400) (W5)) 5,440.00
42,940.00
Unrealised profit adjustments
Inter-company sale of inventory (W2) (600.00)
NCI share 87.00
(513.00)

Inter-company sale of non-current asset (W0)


Unrealised profit (W1) (54.00)
Depreciation adjustment (W1) 9.00
NCI share of depreciation adjustment (W1) (2.25)
(47.25)
42,379.25

© Emile Woolf International 272 The Institute of Chartered Accountants of Pakistan


Answers

(W9) Non-controlling interest in statement of profit or loss


SL AIL
Rs. millions Rs. millions
Sales 16,500 33,800
Cost of sales (18,000) (21,000)
Operating expenses (2,100) (5,400)
(3600) 7400
25% 20%
Non-controlling interest (900) 1,480
Total (1,480 – 900) 580.00
Unrealised profit adjustments
on inter-company sale of inventory (W2) (87.00)
on inter-company sale of non-current asset (W1) 2.25
495.25

20.4 GOLDEN LIMITED


Golden Limited
Consolidated statement of profit or loss for the year ended June 30, 2016
Rs. in million
Sales (875 + 350 - 40) 1,185.00
Cost of sales (567 + 206 - 33.6 (W1)) (739.40)
Gross profit 445.60
Selling expenses (33 + 11) (44.00)
Administrative expenses (63 + 40) (103.00)
Interest expenses (30 + 22) (52.00)
Other income (65 - 36) [20 x Rs. 2 x 90%) 29.00
Impairment losses
Goodwill (W2) (9.18)
Investment in associates (W3) (25.80)
Share of loss from associates [(Rs. 82 x 40%)+0.6] (33.40)
Profit before tax 207.22
Income tax expense (73 + 15) (88.00)
Profit for the year 119.22
Attributable to:
Ordinary shareholders of parent 114.26
Non-controlling interest (W4) 4.96
119.22

W1: Adjustment in cost of sales Rs. in million


Intra-group purchases (40.00)
Additional depreciation on machines 4.00
Unrealized profit in inventories 2.40
(33.60)

© Emile Woolf International 273 The Institute of Chartered Accountants of Pakistan


Advanced accounting and financial reporting

W2: Impairment on goodwill Rs. in million


Shares issued (18 x 4/5 x Rs. 20) 288.00
Less: Net assets acquired:
Share capital 200
Pre-acquisition reserves 24
Fair value adjustment (22 + 20 + 3) 45
269
Holding % 90% 242.10
Goodwill 45.90
20% Impairment in goodwill 9.18

Rs. in
W3: Impairment in the value of investment in associates million
Cash paid (6 x 12) 72.00
Less: Post-acquisition losses:
Reserves on acquisition 40
Reserves at June 30, 2016 (108-82) 26
(14)
% holding 40% (5.60)
Elimination of unrealized gain to the extent of GL's share
(Rs. 11.5 x 0.15 / 1.15 x 40%) (0.60)
65.80
Fair value as per impairment testing 40.00
Impairment losses 25.80

W4: Non-controlling interests


Profit of YL 56.00
Less: Additional depreciation (4.00)
Unrealized profit in inventories (2.40)
49.60
Non-controlling interest % 10%
4.96

© Emile Woolf International 274 The Institute of Chartered Accountants of Pakistan


Answers

CHAPTER 21: ASSOCIATES AND JOINT VENTURES

21.1 JOINT ARRANGEMENTS

(a) (i) Joint Operations


A joint operation is a joint arrangement whereby the parties that have joint control of
the arrangement have right to the assets and obligations for the liabilities relating to
the arrangement. Those parties are called joint operators.
(ii) Joint venture
A joint venture is a joint arrangement whereby the parties that have joint control of
the arrangement have right to the net asset of the arrangement. These parties are
referred to as joint venturers.
(b) Elements to be recognised by a joint operator
(i) Its assets and share of any assets held jointly
(ii) Its liabilities and share of any liabilities incurred jointly
(iii) Its revenue from the sale of its share of the output arising from the joint operation
(iv) Its share of the revenue from the sale of the output by the joint operations
(v) Its expenses and share of any expenses incurred jointly.
(c) Characteristics of joint arrangements
(i) The parties are bound by a contractual arrangement
(ii) The contractual arrangement gives two or more of those parties joint control of the
arrangement.

21.2 HELIUM
Consolidated statement of financial position as at 31 December 2016
Rs.000
Assets
Non-current assets
Property, plant and equipment 500
Interest in associate (W6) 51
Goodwill 15
Current assets 605
———
Total assets 1,171
———
Equity and liabilities
Capital and reserves
Share capital 100
Retained earnings (W5) 737
———
837
Non-controlling interest 84
Long-term liabilities 250
———
Total equity and liabilities 1,171
———

© Emile Woolf International 275 The Institute of Chartered Accountants of Pakistan


Advanced accounting and financial reporting

Workings
(1) Group structure

H e liu m

30%
60%

A r s e n ic

S u lp h u r

(2) Net assets


Sulphur
Balance Post
sheet date Acquisition acquisition
Rs.000 Rs.000 Rs.000
Share capital 30 30 –
Retained earnings 180 70 110
—— ——
210 100
—— ——
(3) Goodwill
Sulphuric
Rs.000

Cost of investment 75
Share of net assets acquired
(60%  100 (W2)) (60)
——
15
——
(4) Non-controlling interest
Rs.000

Sulphur (40%  210) 84


——
(5) Retained earnings
Rs.000
Helium 650
Sulphur (60%  110 (W2)) 66
Arsenic (30%  (100 − 30)) 21
——
737
——
(6) Investment in associate
Rs.000
Cost 30
Share of post-acquisition profit (30%  (100 − 30)) 21
——
51
——

© Emile Woolf International 276 The Institute of Chartered Accountants of Pakistan


Answers

21.3 HAMACHI LTD


(a)
Hamachi Ltd
Consolidated statement of financial position as at 31 March 2016
Rs.000 Rs.000
Non-current assets
Property, plant and equipment (8,050 + 3,600) 11,650
Goodwill (W2) 702
Licence (180 – 60) (W3) 120
12,472
Investments
Associate (W6) 717
Others (4,000 + 910 – 3,240 – 630 + 120 FV) 1,160
1,877
14,349
Current assets
Inventory (830 + 340) 1,170
Accounts receivable (520 + 290 – 40) 770
Bank (240 + 40) 280
2,220
Total assets 16,569
Equity and liabilities
Equity attributable to equity holders of the parent:
Ordinary shares of Rs. 1 each 5,000
Retained earnings (W5) 8,415
13,415
Non-controlling interest (W4) 374
13,789
Non-current liabilities
10% Loan notes (500 + 240) 740
Current liabilities
Accounts payable (420 + 960) 1,380
Taxation (220 + 250) 470
Overdraft 190
2,040
Total equity and liabilities 16,569
Workings
(W1) Net assets in subsidiary
At end of
At reporting
acquisition period
Rs.000 Rs.000
Share capital 1,200 1,200
Retained earnings 800 2,300
Fair value adjustment:
Investment property 120 120
Licence 180 180
Amortisation of licence 180/6 x 2yrs (60)
2,300 3,740

© Emile Woolf International 277 The Institute of Chartered Accountants of Pakistan


Advanced accounting and financial reporting

(W2) Goodwill
Rs.000
Cost of investment (Rs. 3  1,200  90%) 3,240
Net assets acquired (90%  2,300) (W1) 2,070
Goodwill 1,170
Less impairment (468)
702
(W3) Unrealised profit in inventory
((2/3 × 65,000) × 30/130) × 30% = Rs. 3,000
Parent sells to associate, therefore reduce group retained earnings and Investment
in associate

(W4) Non-controlling interest


10%  3,740 = Rs. 374
(W5) Retained earnings
Rs.000
Hamachi Ltd 7,500
Saba Ltd – group share post-acquisition
90%  (3,740 – 2,300) 1,296
Anogo Ltd – group share post-acquisition
30%  (600  6/12) 90
Unrealised profit (W3) (3)
Less impairment (468)
8,415
(W6) Investment in associate
Rs.000
Investment at cost 630
Post-acquisition profit (30%  (600  1/2)) 90
Unrealised profit in inventory (3)
717
(b) IAS 28 Investments in Associates and Joint Ventures defines associates. In order for an
investment to be classified as an investment in an associate the investor must have
‘significant influence’ over the investee. Significant influence is presumed to exist where
there is a holding of 20% or more of the voting power unless the investor can clearly
demonstrate that this is not the case. Conversely a holding of less than 20% is presumed
not to be an associate, unless it can be clearly demonstrated that the investor can exercise
significant influence. The voting rights can be held directly or through subsidiaries.
IAS 28 says that a majority holding by one investor does not preclude another investor
having significant influence. An investing company owning a majority holding in another
company normally has control over the investee and would thus class it as a subsidiary. In
normal circumstances it is difficult to see how a company could be controlled by one entity
and be significantly influenced by a different entity unless ‘control’ was passive. The 20%
test is not definitive and the following other evidence should be considered.
Does the investing company:
 have representation on the Board of the investee?
 participate in the policy making processes (operational and financial); have material
transactions with the investee?
 interchange managerial personnel with the investee; or provide technical expertise to
the investee?

© Emile Woolf International 278 The Institute of Chartered Accountants of Pakistan


Answers

21.4 HIDE
Hide
Consolidated statement of profit or loss for the year ended 30 June 2016
Rs.000
Revenue 15,131
Cost of sales and expenses (13,580)
———
Operating profit before tax 1,551
Tax (736)
———
Profit after tax 815
Share of profit of associate (30% of 594) 178
———
Profit for the year 993
———

Profit for the year attributable to members of Hide 963


Non-controlling interest (W2) 30
———
Profit for the year 993
———
Workings
(1) Group structure

H id e

30%
80%
A r r iv e

Seek

(2) Consolidation schedule


Hide Seek (5/12) Adjustment Total
Rs. 000 Rs. 000 Rs. 000 Rs. 000
Revenue 12,614 2,567 (50) 15,131
Cost of sales
Per question (11,318) (2,302) 50
Unrealised profit
50  (25/125) (10) (13,580)

Tax (621) (115) (736)


Profit for the year 150
Non-controlling interest (%) 20%
Non-controlling interest (Rs. 000) 30

© Emile Woolf International 279 The Institute of Chartered Accountants of Pakistan


Advanced accounting and financial reporting

21.5 HARK, SPARK AND ARK


Hark Group
Consolidated statement of financial position as at 31 March 2016
Rs.000 Rs.000
Non-current assets
Property, plant and equipment (working 1) 90,200
Goodwill (working 4) 23,000
Investment in associate (working 6) 9,500
Other investments 650
123,350
Current assets (working 5) 24,300
Total assets 147,650
Equity and liabilities
Equity shares of Rs. 1 each (working 3) 21,000
Share premium (working 3) 42,000
Retained earnings (working 8) 43,730
85,730
106,730
Non-controlling interests (working 7) 7,420
Total equity 114,150
Non-current liabilities
Deferred consideration for Spark shares 5,500
6% loan notes 10,000
7% loan notes 6,000
21,500
Current liabilities: 7,000 + 5,000 12,000
Total equity and liabilities 147,650
Workings
1 Property, plant and equipment (PPE)
Rs.000 Rs.000
Hark 60,000
Spark 31,000
Profit on transfer of machines (3 million – 2 million) 1,000
Less: Depreciation on this amount in accounts of Spark
(1,000/5 years) (200)
Unrealised profit in machines (800)
PPE in consolidated statement of financial position 90,200

2 Deferred consideration
The present value of the deferred consideration at 1 April 2015 is Rs. 6.05 million 
2
1/(1.10) = Rs. 5 million.
During the year to 31 March 2016 there is a finance charge of 10% (= Rs. 500,000) on this
amount, reducing the parent’s share of the consolidated profit.
The deferred consideration at 31 March 2016 is Rs. 5 million + Rs. 500,000 = Rs.
5,500,000. This is payable in just over 12 months and is included in the consolidated
statement of financial position as a non-current liability.

© Emile Woolf International 280 The Institute of Chartered Accountants of Pakistan


Answers

3 Share issues
The share issues to acquire the shares in Spark and Ark are not recorded in the summary
statement of financial position of Hark (as stated in the question).
Share Share
Total capital premium
To acquire the shares in Spark Rs.000 Rs.000 Rs.000
Hark shares issued: (4 million at Rs. 9) 36,000 4,000 32,000
To acquire the shares in Ark
Hark shares issued: (1 million at Rs. 9) 9,000 1,000 8,000
Increase in share capital and share
premium of Hark 5,000 40,000
In summary statement of financial position 16,000 2,000
In consolidated statement of financial
position 21,000 42,000

4 Goodwill
Hark has acquired 4 million/5 million = 80% of the shares of Spark.
At 1 April 2015 the fair value of the net assets of Spark was (share capital plus reserves) =
Rs.(5 + 4 + 16) million = Rs. 25 million
Rs.000
Purchase consideration paid by the parent company
Issue of 4 million shares at Rs. 9 36,000
Deferred consideration 5,000
41,000

Fair value of parent company share of net assets


(80%  Rs. 25 million) 20,000
Purchased goodwill attributable to parent 21,000

Rs.000
Fair value of NCI at acquisition date (1 million shares  Rs. 7) 7,000
NCI share of net assets at this date (20%  Rs. 25 million) 5,000
Purchased goodwill attributable to NCI 2,000
There has been no impairment of goodwill during the year.
Rs.000
Purchased goodwill attributable to parent 21,000
Goodwill attributable to NCI 2,000
Total goodwill in consolidated statement of financial position 23,000
Alternatively, total goodwill could be calculated as follows:
Rs.000
Purchase consideration paid by the parent company 41,000
(see above)
Fair value of NCI at acquisition date 7,000
48,000
Net assets of the subsidiary at the acquisition date 25,000
(at fair value)
Total goodwill (parent and NCI) 23,000

© Emile Woolf International 281 The Institute of Chartered Accountants of Pakistan


Advanced accounting and financial reporting

5 Current assets
The cost of the goods sold by Spark to Hark was Rs. 3,600,000  100/150 = Rs. 2,400,000
and the profit was Rs. 1,200,000.
Since 75% of these goods are in closing inventory, the unrealised profit on intra-group
sales is 75%  Rs. 1,200,000 = Rs. 900,000. Current assets in the consolidated statement
of financial position (inventory) should be reduced by this amount.
The question states that the transaction costs of the acquisition of Spark have not yet been
recorded. These costs reduce the consolidated profit, and also (presumably) reduce the
current assets of Hark.
Current assets on consolidation Rs.000
Hark 18,200
Spark 8,000
Less: unrealised profit in closing inventory (900)
Less: expenses of acquisition of Spark (1,000)
Current assets in consolidated statement of financial position 24,300

6 Investment in associate (Ark)


Since Hark owns 25% of the equity of Ark, it is assumed that Ark is an associated entity.
Rs.000
Cost of investment: 25%  6 million shares  Rs. 6 9,000
Share of post-acquisition retained profit: 25%  Rs. 2 million 500
9,500
7 Non-controlling interests
Rs.000
Share of net assets of Spark at 31 March 2016 (20%  Rs. 28 million) 5,600
Goodwill attributable to NCI (working 4) 2,000
7,600
NCI share of unrealised profit in inventory (20%  Rs. 900,000) (180)
NCI at 31 March 2016: fair value method 7,420

8 Consolidated retained earnings


Rs.000 Rs.000
Hark retained earnings (36,000 + 8,000) 44,000
Spark
Profit for year ended 31 March 2016 3,000
Unrealised profit in closing inventory (900)
2,100
Parent company share (80%) 1,680
Share of post-acquisition retained profits of Ark 500
(25%  Rs. 2 million)
Costs of acquisition of Spark (expensed) (1,000)
Additional finance costs: deferred consideration (500)
Unrealised profit in machines (working 1) (800)
Loss on other (800 – 650) (150)
Consolidated retained earnings at 31 March 2016 43,730

© Emile Woolf International 282 The Institute of Chartered Accountants of Pakistan


Answers

21.6 P, S AND A
P Group
Consolidated statement of financial position as at 31 December Year 5
Assets
Non-current assets Rs.
Property, plant and equipment (450,000 + 240,000) 690,000
Goodwill (W3) 45,000
Investment in associates (W5) 168,800
903,800
Current assets
Inventory (70,000 + 90,000 – 10,000) 150,000
Other current assets (20,000 + 110,000 + 130,000) 260,000
Total assets 1,313,800
Equity and liabilities
Equity
Share capital 100,000
Share premium 160,000
Consolidated accumulated profits (W6) 711,300
Attributable to equity holders of the parent 971,300
Non-controlling interest in S (W4) 102,500
Total equity 1,073,800
Long-term liabilities (40,000 + 20,000) 60,000
Current liabilities (100,000 + 80,000) 180,000
Total equity and liabilities 1,313,800

Workings
P owns 75% of the equity of S and 30% of the equity of A. Therefore S is a subsidiary and A is
an associate.
W1: Net assets summary
Calculate the net assets of S and A at the acquisition date and at the end of the reporting period.
At this stage, make any fair value adjustments and eliminate the unrealised profit in inventory.
At date of At date of Post-
Net assets of S consolidation acquisition acquisition
Rs. Rs. Rs.
Equity shares 200,000 200,000 -
Share premium 80,000 80,000 -
Accumulated profits (per question) 140,000 60,000 80,000
410,000 340,000

W2: Unrealised profit on inter-company trading


Sale by S to P: Rs. 40,000 × 33.33/133.33 = Rs. 10,000.
Dr Cr
Consolidated accumulated profits (75%) 7,500
Non-controlling interest (25%) 2,500
Consolidated inventory 10,000

© Emile Woolf International 283 The Institute of Chartered Accountants of Pakistan


Advanced accounting and financial reporting

Sale by P to A: Rs. 16,000 × 33.33/133.33 = Rs. 4,000.


P’s share: Rs. 4,000  30% = Rs. 1,200
Dr Cr
Consolidated accumulated profits (75%) 1,200
Investment in associate 1,200
W3: Goodwill on acquisition of S
Rs.
Cost of the acquisition 320,000
Non-controlling interest at acquisition (25%  340,000 (W1)) 85,000
405,000
Less: Fair value of identifiable net assets at acquisition (W1) (340,000)
Goodwill at acquisition 65,000
Minus: Impairment to date (20,000)
Balance carried forward 45,000

W4: Non-controlling interest in S

Rs.
Non-controlling interest at acquisition (25%  340,000 (W1)) 85,000
Share of post-acquisition profits (25%  80,000 (W1)) 20,000
Unrealised profit (W2) (2,500)
102,500

W5: Investment in associate

Rs.
Investment at cost 140,000
P’s share of post-acquisition accumulated profits 30,000
(30%  (250,000 – 150,000)
Unrealised profit (W2) (1,200)
168,800

W6: Consolidated accumulated profits.

Rs.
Accumulated profits of P 650,000
P’s share of post-acquisition profits of S
(75% × Rs. 70,000 (W2)) 60,000
Unrealised profit (sale by S to P (W3)) (7,500)
P’s share of post-acquisition accumulated profits (W5) 30,000
Unrealised profit (W2) (1,200)
Impairment of goodwill (20,000)
Consolidated accumulated profits 711,300

© Emile Woolf International 284 The Institute of Chartered Accountants of Pakistan


Answers

21.7 H LTD GROUP


Consolidated statement of profit or loss and other comprehensive income for the H Ltd group for
the year ended 31 May 2016
Rs.000
Revenue (6,000 + 3,000) 9,000
Cost of sales (4,800 + 2,400) (7,200)
Gross profit 1,800
Distribution costs (64 + 32) (96)
Administrative expenses (336 + 168) (504)
Finance costs (30 + 15) (45)
Share of profit of associate (30% x 100) 30
Profit before tax 1,185
Income tax expense (204 + 102) (306)
PROFIT FOR THE YEAR 879
Other comprehensive income:
Revaluation of PPE (200 + 100) 300
Actuarial gain on pension plan assets 40
Actuarial loss on pension plan liabilities (52)
Gain on AFS investment 14
Tax effect of other comprehensive income (42 + 21) (63)
Share of OCI of associate (net of tax) (30% x 24) 7
Other comprehensive income for the year, net of tax 246
TOTAL COMPREHENSIVE INCOME FOR THE YEAR 1,125

Profit for the period attributable to: Rs.000


Owners of the parent entity 822.4
Non-controlling interests (20% x 283) 56.6
879

Total comprehensive income attributable to:


Owners of the parent entity 1052.6
Non-controlling interests (20% x 362) 72.4
1,125

© Emile Woolf International 285 The Institute of Chartered Accountants of Pakistan


Advanced accounting and financial reporting

CHAPTER 22: BUSINESS COMBINATIONS ACHIEVED IN STAGES

22.1 STEP ACQUISITION


The profits of AS since the investment was acquired (all retained) are Rs. 40 million (= Rs. 300m
– Rs. 260m). During this period, H held 25% of the equity of AS and it is assumed that AS is an
associate. Profits attributable to H for the year are therefore Rs. 10 million (= 25%  Rs. 40
million).
(i) Total gain or profit attributable to the investment in AS
Rs. m
Initial investment in associate at cost 80
Share of post-investment retained profits 10
90
Fair value of investment at 30 June 95
Gain recognised when step acquisition occurs 5

The total gain/profit recognised for the year from the investment in AS is therefore Rs. 10
million + Rs. 5 million = Rs. 15 million.

(ii) Total goodwill on acquisition


Rs. m
Fair value of shares that gave control (40%) 160
Fair value of previous investment (25%) 95
255
Fair value of NCI at acquisition 120
375
Net assets of AS at 30 June 300
Total goodwill 75

(iii) Goodwill attributable to NCI


Rs. m
Fair value of investment in 25% of AS (35%  300) 105
Goodwill attributable to NCI (balancing figure) 15
Total NCI 120

22.2 A LTD
(a) Treatment of B Ltd
IFRS 3 Business combinations requires goodwill on acquisition to be calculated at the date
control is gained. The second acquisition gives A Ltd a 75% holding and therefore control
over B Ltd. The simple investment of 15% will be derecognised and the 75% holding will
be fully consolidated as a subsidiary in the group financial statements. The goodwill will be
calculated as the cost of the 60% acquired in the year plus the fair value of the previously
held interest of 15%, compared with the fair value of the net assets at the date of
acquisition (1 April 2016).

© Emile Woolf International 286 The Institute of Chartered Accountants of Pakistan


Answers

(b) Consolidated statement of financial position for the A Ltd Group as at 31 September
2016.
A Ltd
ASSETS Rs.000
Non-current assets
Property, plant and equipment (22,000+5,000) 27,000
Goodwill (W orking 1) 405
27,405
Current assets
Inventories (6,200+800– 40 (Working 2)) 6,960
Receivables (6,600+1,900) 8,500
Cash and cash equivalents (1,200+300) 1,500
16,960
Total assets 44,365

EQUITY AND LIABILITIES


Equity
Share capital (Rs. 1 equity shares) 20,000
Retained earnings (W orking 3) 8,629
Other components of equity (W orking 6)
28,629
Non-controlling interest (Working 4) 1,604
Total equity 30,233

Non-current liabilities
5% Bonds 2015 (Working 5) 4,032
Current liabilities (8,100+2,000) 10,100
Total liabilities 14,132
Total equity and liabilities 44,365

Working 1 Goodwill Rs.000 Rs.000


Consideration transferred for the 60% 2,900
Fair value of 15% holding at 1 April 2016 800
Fair value of non-controlling interest 1,250
4,950
Net assets acquired:
Share capital 1,000
Retained earnings (5,000- 1,500) 3,500 (4,500)
450
Impaired by 10% (45)
Net value of goodwill 405

Working 2 Unrealised profit on inventories Rs.000


Sales from B Ltd to A Ltd 400
50% in inventories 200
Profit margin 20% - adjust Inventories and retained earnings of B 40
Ltd

© Emile Woolf International 287 The Institute of Chartered Accountants of Pakistan


Advanced accounting and financial reporting

Working 3 Retained earnings A Ltd B Ltd


Rs.000 Rs.000
As at 30 September 2016 7,500 5,000
Pre-acquisition (5,000 – (3,000 x 6/12)) (3,500)
Less unrealised profit of B Ltd (working 2) (40)

1,460

Group share 75% 1,095


Group share of impairment (75% x 45) (34)
Additional finance costs on bonds (working 5) (132)
Group profit on derecognition of AFS Investment
– gain to date of deemed disposal 1 April 2016 (800 - 600) 200

Consolidated retained reserves 8,629

Working 4 Non-controlling interest Rs.000


Fair value at 1 April 2016 1,250
Plus 25% adjusted post-acquisition reserves 1,460 (working 3) 365
Less NCI share of goodwill impairment (25% x 45) (11)

NCI at 30 September 2016 1,604

Working 5 Bonds – amortised


Rs.000 Rs.000 Rs.000 Rs.000
cost
Opening Effective rate Interest paid Value at 30
value 8.5%
5% x Rs. 4m September
To 30 September 2016 3,900 332 (200) 4,032

The difference of Rs. 132,000 must be added to the value of the bond liability and
deducted from
A Ltd’s retained earnings.
Working 6 Other reserves and AFS investment
IFRS 3 requires that the 15% simple investment be derecognised and on derecognition any
gain/loss would be considered realised. The gain of Rs. 200,000 (FV of Rs. 800,000 at
date of derecognition less the investment cost of Rs. 600,000) represents the group gain
and will be included in the consolidated reserves.
The balance on other reserves again relates to the treatment of the investment in the
parent’s own accounts and the gains on the AFS investment (B Ltd) and not relevant for
the group accounts – as the B Ltd has been fully consolidated.

© Emile Woolf International 288 The Institute of Chartered Accountants of Pakistan


Answers

22.3 X LTD GROUP


(a) (i) X Ltd group: Consolidated statement of profit or loss and other
comprehensive income for the year ended 31 December 2016

Rs.000
Revenue (1,200 + 290) 1,490
Cost of sales (810 + 110 + 4 (W1)) (924)
Gross profit 566
Operating expenses (100 + 40 + 9 (W2)) (149)
417
Investment income
(50 – intra group dividend 40 (80% x 50)) 10
Finance costs (45 + 10) (55)
Share of associate’s profit (40% x 30) 12
Profit before tax 384
Income tax expense (80 + 30) (110)
Profit for the year 274
Other comprehensive income
Revaluation of property, net of tax (60 + 20) 80
Share of associate’s OCI (40% x 10) 4
Other comprehensive income for the year, net of tax 84
Total comprehensive income 358

Profit for the year attributable to:


Owners of the parent (274 – 17 (W3)) 257
Non-controlling interest 17
274
Total comprehensive income attributable to:
Owners of the parent (358 – 21 (W3)) 337
Non-controlling interest 21
358
(ii) Consolidated statement of changes in equity for the year ended 31 December
2016
X Ltd NCI Total
group
Rs.000 Rs.000 Rs.000
Equity at 1 January 2016 (W4)/(W5) 1,868 216 2,084
Total comprehensive income for the year 337 21 358
Dividends (100) (100)
Dividend paid to NCI (20% x 50) (10) (10)
Equity at 31 December 2016 2,105 227 2,332

© Emile Woolf International 289 The Institute of Chartered Accountants of Pakistan


Advanced accounting and financial reporting

Workings

(W1) Net assets of subsidiary

Acquisition 1 Jan 31 Dec


1 Jan 2010 2016 2016
Rs.000 Rs.000 Rs.000
Share capital 200 200 200
Retained reserves 420 640 (bal) 710
620 840 910
Fair value adjustment 60 60 60
Accumulated additional depreciation
on FV
adjustment (60/15 yrs = 4 per yr) (12) (16)
Accumulated impairment of goodwill
(W2) (30) (39)
Adjusted net assets 680 858 915
Post-acquisition retained reserves to
1 Jan/31 Dec 178 235

(W2) Goodwill

Rs.000 Rs.000
Consideration transferred 620
NCI at fair value 180
800
Net assets acquired:
Share capital 200
Retained earnings 420
Fair value adjustment 60 (680)
120
Impairment 2015 (25%) (30)
90
Impairment 2016 (10% of carrying value) (9)

(W3) Non-controlling interest

PFY TCI
Rs.000 Rs.000
Profit for year/TCI of Y Ltd 100 120
Less impairment of goodwill in the year (W2) (9) (9)
Less depreciation on FV adjustment for the year
(W1) (4) (4)
87 107
20% NCI share 17 21

© Emile Woolf International 290 The Institute of Chartered Accountants of Pakistan


Answers

(W4) Group equity attributable to parent at 1 January 2016

Rs.000
Parent’s equity at 1 January 2016 as per SOCIE 1,700
Plus share of post-acquisition retained reserves of Y Ltd to 142
1 January 2016 (80% x 178 (W1))
Plus share of post-acquisition retained reserves of Z Ltd to
1 January 2016 (40% x(500-435))
26
Equity attributable to parent at 1 January 2016 1,868

(W5) Group equity attributable to NCI at 1 January 2016

Rs.000
At acquisition 180
Plus share of post-acquisition retained reserves to 1 January
2016 (20% x 178 (W1)) 36
Equity attributable to NCI at 1 January 2016 216

(b) (i) Additional acquisition of shares


The purchase of the additional 10% of Y Ltd’s share capital is treated as a
transaction between owners of the entity, as NCI reduces and parent’s share
increases. No additional goodwill is calculated as X Ltd already controls Y Ltd and
goodwill is only calculated when control is attained. Any difference between the
consideration paid by X Ltd and the reduction in the NCI is adjusted through group
retained earnings.
(ii)

Adjustment to parent’s equity Rs.000


Consideration transferred 120
Reduction in NCI at 1 January 2017 (50% x Rs. 227,000) (114)
Adjustment to retained earnings – debit 6

(c) Additional investment in Z Ltd


The additional 20% investment will give X Ltd the majority holding of Z Ltd’s ordinary
shares. This gives the presumption of control, unless there is evidence to the contrary and
once control is attained Z Ltd will be treated as a subsidiary and fully consolidated.
Goodwill on acquisition is calculated at 1 January 2017 and the existing investment will be
restated to FV at the date of acquisition

22.4 PLAIN LTD


Consolidated statement of financial position as at 31 March 2016
Rs. m
Assets
Tangible non-current assets (W4) 1,745
Intangible non-current assets – goodwill 45 – 15 (W3) 30
Investment in associate (W5) 95
Held to maturity investment (W6) 50
Current assets (477 + 190 - 250) 417
2,337

© Emile Woolf International 291 The Institute of Chartered Accountants of Pakistan


Advanced accounting and financial reporting

Consolidated statement of financial position as at 31 March 2016


Rs. m
Equity and liabilities
Share capital of Rs. 1 800
Share premium 150
Revaluation reserve 90
Retained earnings (W8) 169
1,209
Non-controlling interest (W9) 99
1,308
Non-current liabilities (640 + 30) 670
Current liabilities (214 + 130) 344
Pension liability (W7) 15
2,337
Workings
(W1) Retained profits of Stripes
Rs. m
Fair value of net assets at 1 April 2015 460
Fair value adjustment for land (25)
Carrying value of net assets 435
Share capital plus share premium (260)
Therefore retained earnings at 1 April 2015 175

Rs. m
Carrying value of net assets at 1 April 2013 325
Share capital plus share premium (260)
Therefore retained earnings at 1 April 2013 65
(W2) Gain or loss on acquiring control of Stripes
1 April 2015 Rs. m Rs. m
Fair value of initial investment in Stripes at 1 April 2015 150
Initial cost of investment 120
Share of retained earnings 1 April 2013 – 1 April 2015 33
(= 30%  (175 – 65) –see W1
Carrying value of investment in associate 153
Loss recognised on gaining control of Stripes (3)
This loss has not yet been recognised in the individual financial statements of Plain; it must
therefore be included in the calculation of group reserves (see Working 8).

(W3) Goodwill in Stripes at acquisition


Rs. m
Fair value of initial investment at acquisition 150
Cost of additional shares 260
Total cost 410
Fair value of net assets acquired (80%  460) 368
Goodwill at acquisition attributable to Plain 42
Goodwill attributable to NCI 3
Total goodwill at acquisition date 45

© Emile Woolf International 292 The Institute of Chartered Accountants of Pakistan


Answers

Goodwill in statement of financial position: There has been impairment of Rs. 15 million in
goodwill. This is apportioned between the interests of the equity owners of Plain and NCI
in the ratio 80:20.
Impairment of goodwill attributable to parent = Rs. 15m  80% = Rs. 12 million
Impairment of goodwill attributable to NCI = Rs. 15m  20% = Rs. 3 million.

(W4) Tangible non-current assets


Rs. m
Plain 1,280
Stripes 440
Fair value adjustment 25
1,745
(W5) Investment in associate – Spots
Rs. m
Cost 60
Group share of post-acquisition profit
(324 – (200 – 16)) × 25% 35
95

Or Rs. m
Share of net assets (25% × 324) 81
Fair value adjustment (25% × 16) 4
Goodwill [60 – (200 × 25%)] 10
95
(W6) Held to maturity investment
Rs. m
Amortised cost
Cost 54
Less: Discount (20/5) (4)
50
Tutorial note: It is not correct to recognise interest on a straight line basis. It is used here
as a simplification. IAS 39 requires the recognition of interest using the effective rate.

(W7) Pension
Rs. m
Scheme assets
Cash 250
Expected return 26
Actuarial gain (bal fig) 26
Fair value of scheme assets 302
Scheme liabilities
Current service cost 276
Interest cost 41
Present value of obligation 317

Pension scheme liability (317 – 302) 15


Expense in profit or loss
Current service cost 276
Interest cost 41
Expected return (26)
Expense in profit or loss 291

© Emile Woolf International 293 The Institute of Chartered Accountants of Pakistan


Advanced accounting and financial reporting

(W8) Consolidated retained earnings


Rs. m
Plain (given) 390
Stripes post-acquisition retained earnings
(210 – 175 (W1)) × 80% 28
Loss on acquiring control (W2) (3)
Goodwill impairment attributable to parent (W3) (12)
Share of post-acquisition profits of associate (W5) 35
Discount on investment (W6) (4)
Pension cost (W7) (291)
Actuarial gain (W7) 26
169
(W9) Non-controlling interest in Stripes
Rs. m
Book value (20% × 470) 94
Fair value adjustment (20% × 25) 5
Goodwill (3 – impairment 3) (W3) 0
99

22.5 MANGO LTD


Mango Ltd
Consolidated statement of financial position as at 31 March 2016
Rs. m
Non-current assets
Non-current assets (3,295 + 2,000 + 226 + 2) 5,523.0
Goodwill (W2) 89.0
Current assets (1,685 + 861) 2,546.0
Total assets 8,158.0

Equity and liabilities


Share capital 850.0
Retained earnings (W4) 3,405.9
Other components of equity (W5) 257.0
4,512.9
Non-controlling interest (W3) 649.1
5,162.0
Non-current liabilities (1,895 + 675) 2,570.0
Current liabilities (320 + 106) 426.0
Total equity and liabilities 8,158.0

© Emile Woolf International 294 The Institute of Chartered Accountants of Pakistan


Answers

Workings
(W1a) Net assets in subsidiary at acquisition – before measurement period
adjustments
At end of reporting At
period acquisition
Rs. m Rs. m
Share capital 1,020 1,020
Retained earnings 980 900
Other components of equity 80 70
1,990

Fair value adjustments:


Contingent liability (6)
Property 266
260
Fair value of net assets (given) 2,250

The total fair value adjustment of Rs. 260 million above is taken as a balancing
figure as is the fair value adjustment that relates to property.
The amount in respect of the contingent liability and an amount within the property
adjustments is subsequently found to be incorrect. This information came to light in
the measurement period. Therefore, they retrospectively adjust th e carrying amount
of goodwill. In this case the easier approach is to calculate goodwill using the
corrected figures.
(W1b) Net assets in subsidiary at acquisition – after measurement period
adjustments
At At acquisition
consolidation
Rs. m Rs. m
Share capital 1,020 1,020
Retained earnings 980 900
Reduction of depreciation recognised on the
buildings
(Rs. 40/20 years) 2
Adjustment re recognition of the provision 5
987 900
Other components of equity 80 70
1,067 1,990
Fair value adjustments:
Contingent liability (6 – 1) (5) (5)
Property (266 – 40) 226 226
Fair value of net assets 2,308 2,211

The contingent liability at acquisition is recognised for consolidation purposes as a


deduction from the net assets of the subsidiary. It is not recognised in the
subsidiary’s own accounts as it does not pass the IAS 37 recognition criteria.

© Emile Woolf International 295 The Institute of Chartered Accountants of Pakistan


Advanced accounting and financial reporting

The contingent liability has evolved into a provision by the date of consolidation.
This means that it is recognised as a liability and the amount has been exp ensed in
the subsidiary’s own financial statements. The adjustment made above (Dr Fair
value adjustment and Cr Retained earnings) is made because the expense which
has been recognised by the subsidiary since the date of acquisition relates to an
amount that has already been recognised in the consolidation workings at
acquisition.
(W2) Goodwill
Rs. m
Cost of investment 975
Fair value of initial holding 705
Fair value of NCI at date of acquisition 620
2,300
Net assets acquired (W1b) (2,211)
89
(W3) Non-controlling interest
Rs. m
Fair value of NCI at date of acquisition 620.0
NCI’s share of post-acquisition growth in:
Retained earnings (30% of (987 – 900)) 26.1
Other components in equity (30% of (80 – 70)) 3.0
649.1
(W4) Retained earnings
Rs. m
Mango Ltd’s balance as per the question 3,340.0
fair value adjustment re initial holding (705 – 700) 5.0
Share of post-acquisition growth (70% of (987 – 900)) 60.9
3,405.9
(W5) Other components of equity
Rs. m
Mango Ltd’s balance as per the question 250.0
Share of post-acquisition growth (70% of (80 – 70)) 7.0
257.0

© Emile Woolf International 296 The Institute of Chartered Accountants of Pakistan


Answers

CHAPTER 23: COMPLEX GROUPS

23.1 PARVEZ LTD

(a) Parvez Ltd: Consolidated statement of profit or loss for the year ended 31 December
2016
Rs. 000
Revenue (W4) 92,360.0

Cost of sales (W4) (28,123.0)


Gross profit 64,237.0
Distribution costs (3,325 + 2,137 + 1,900) (7,362.0)
Administrative expenses (3,475 + 950 + 1,900) (6,325.0)
Operating profit 50,550.0
Interest paid (325.0)
Profit before tax 50,225.0
Tax (17,931.0)
Profit after tax 32,294.0

Attributable to:
Ordinary shareholders of parent 28,580.8
Non-controlling interest (W9) 3,713.2
32,294.0

(b) Parvez Ltd: Consolidated statement of financial position as at 31 December 2016


Rs. 000
Goodwill (W6) 3,963.5
Property, plant and equipment
(35,483 + 24,273 + 13,063) 72,819.0
Current assets (W3) 19,446.0
96,228.5
Share capital 8,000.0
Retained earnings (W8) 56,641.3
Non-controlling interest (W7) 8,453.2
Current liabilities (13,063 + 10,023 + 48) 23,134.0
96,228.5

(W1) Group structure

© Emile Woolf International 297 The Institute of Chartered Accountants of Pakistan


Advanced accounting and financial reporting

(W2) Unrealised profit adjustments: Inter-company trading


Vazir to Saad to Total
Saad Parvez
Sales 480,000 260,000 740,000

Inventory held 75,000 60,000


Unrealised profit adjustment  25/125  33.3/133.3
Unrealised profit 15,000 15,000 30,000

NCI’s share (based on selling


company’s NCI)
10%  15 1,500
28%  15 4,200

Double entry in consolidated financial statements Dr Cr


Cost of sales (closing inventory) 30,000
Closing inventory in statement of financial position 30,000

NCI in the statement of financial position 5,700


NCI in the statement of comprehensive income 5,700

Impact on consolidated retained earnings (30 – 5.7) 24,300


(W3) Consolidated current assets
Rs. 000
Parvez 1,568
Saad 9,025
Vazir 8,883
Unrealised profit (W2) (30)
19,446
(W4) Consolidated sales and cost of sales
Cost of
Sales sales
Rs. 000 Rs. 000
Parvez 45,600 18,050
Saad 24,700 5,463
Vazir 22,800 5,320
Inter-company sales (740) (740)
Unrealised profit 30
92,360 28,123
(W5) Net assets summary
At date of At date of Post-acquisition
Saad Ltd consolidation acquisition
Rs. 000 Rs. 000
Share capital 3,000 3,000
Retained earnings 24,075 1,425 22,650
27,068 4,425

© Emile Woolf International 298 The Institute of Chartered Accountants of Pakistan


Answers

At date of At date of Post-acquisition


Vazir Ltd consolidation acquisition
Rs. 000 Rs. 000
Share capital 2,000 2,000
Retained earnings 19,898 950 18,948
21,898 2,950
(W6) Goodwill
In Saad Ltd In Vazir Ltd
Rs. 000 Rs. 000
Cost 6,650.0
90% × 3,800 3,420.0
NCI at acquisition
10% × 4,425 W5 442.5
28% × 2,950 W5 826.0
7,092.5 4,426.0
Net assets at acquisition (4,425.0) (2,950.0)
2,667.5 1,296.0
Total 3,963.5
(W7) Non-controlling interest (statement of financial position)
Rs. 000 Rs. 000
NCI at acquisition
Saad Ltd: 10% × 4,425 (W5) 442.5
Vazir Ltd: 28% × 2,950 (W5) 826.0
NCI’s share of post-acquisition profits
10% × 22,650 (W5) 2,265.0
28% × 18,948 (W5) 5,305.4
NCI in Saad Ltd ’s share of net assets of Vazir
Ltd
10% × 3,800 (380.0)
2,327.5 6,131.4
Total 8,458.9
Unrealised profit (W2) (5.7)
8,453.2

(W8) Consolidated retained earnings carried forward


Rs. 000
All of Parvez Ltd
Per the question 22,638.0
Share of Saad Ltd
90%  22,650 (W5) 20,385.0
Share of Vazir Ltd
72% × 18,948 (W5) 13,642.6
Unrealised profit (W2) (24.3)
56,641.3

© Emile Woolf International 299 The Institute of Chartered Accountants of Pakistan


Advanced accounting and financial reporting

(W9) Non-controlling interest (statement of profit or loss)


Saad Ltd Vazir Ltd
Rs. 000 Rs. 000
Profit after tax 10,760 9,439
10% 28%
Non-controlling interest 1,076 2,642.9

Total (1,076 + 2,642.9) 3,718.9


Unrealised profit (W2) (5.7)
3,713.2

23.2 HASAN, RIAZ AND SIDDIQ


Hasan Limited: Consolidated statement of financial position as at 31 December 2016
Rs.
Goodwill (W6) 26,250
Property, plant and equipment
(1,102,500 + 271,950 + 122,550) 1,497,000

Inventories (W4) 783,520


Receivables
(241,920  8,000 W2) + 129,680 + 29,750 – 17,500 W2) 375,850
Cash and bank balances
((88,200 + 8,000 W2) + 4,725 + 8,105) 109,030
2,791,650
Share capital 1,750,000
Retained earnings (W8) 181,795
Other reserves (W9) 402,500
Non-controlling interest (W7) 191,625
Payables
(95,480 + (86,645 + 12,500 W2) + 88,605 – 17,500 W2) 265,730
2,791,650

(W1) Group structure

© Emile Woolf International 300 The Institute of Chartered Accountants of Pakistan


Answers

Non-controlling
interest
Hasan’s interest (balance)
In Riaz Ltd 75% 25%
In Siddiq Ltd (40% + (75%  20%)) 55% 45%

(W2) Individual company adjustments: Transaction before the year-end not yet
accounted for
Books of Riaz
Purchase of inventory from Hasan Dr Cr
Closing inventory 12,500
Payable (to Hasan) 12,500

Books of Hasan
Cash received from Riaz Dr Cr
Cash 8,000
Receivable (from Hasan) 8,000
The inter-company balances can be reconciled as follows after these adjustments
have been processed:
Hasan’s Riaz’s
financial financial
statements statements
Receivable Payable
Given in the question
Receivable from Riaz (note 5 in the
question) 25,500
Payable to Hasan (note 7 in the question) 5,000
Cash from Riaz (8,000)
Purchase from Hasan 12,500
17,500 17,500

These balances must be cancelled out on consolidation as follows:


Dr Cr
Consolidated payables 17,500
Consolidated receivables 17,500
(W3) Unrealised profit adjustments: Inter-company trading

Inventories held by Riaz purchased from Hasan


Rs.
Purchased 30 December 12,500
Purchased previously 10,400
Purchased previously 22,900
Mark up adjustment  25/125
Unrealised profit 4,580

© Emile Woolf International 301 The Institute of Chartered Accountants of Pakistan


Advanced accounting and financial reporting

Double entry in consolidated


financial statements Dr Cr
Cost of sales (closing inventory) 4,580
Closing inventory in statement of
financial position 4,580

There is no double entry for the NCI as all sales were from the parent.
(W4) Consolidated inventories
Rs.
Hasan 526,610
Riaz (163,290 + 12,500 (W2) 175,790
Vazir 85,700
Unrealised profit (W3) (4,580)
783,520

(W5) Net assets summary


At date of At date of Post-
Riaz Ltd consolidation acquisition acquisition
Rs. Rs.
Share capital 420,000 420,000
Accumulated profits 17,500 35,000 (17,500)
Other reserves 70,000 nil 70,000
437,500 455,000

At date of At date of Post-acquisition


Siddiq Ltd consolidation acquisition
Rs. Rs.
Share capital 175,000 175,000
Accumulated losses (17,500) (35,000) 17,500
157,500 140,000
(W6) Goodwill
In Riaz In Siddiq Ltd
Ltd
Rs. Rs.
Cost 367,500 49,000
75% × 24,500 18,375
67,375
NCI at acquisition
25% × 455,000 W5 113,750
45% × 140,000 W5 63,000
481,250 130,375
Net assets at acquisition (455,000) (140,000)
26,250 (9,625)

The balance for Siddiq is a gain on a bargain purchase.

© Emile Woolf International 302 The Institute of Chartered Accountants of Pakistan


Answers

(W7) Non-controlling interest (statement of financial position)


Rs. Rs.
NCI at acquisition
25% × 455,000 W5 113,750
45% × 140,000 W5 63,000
NCI’s share of post-acquisition profits
25% × (17,500) (W5) (4,375)
45% × 17,500 (W5) 7,875
NCI’s share of other reserves
25% × 70,000 (W5) 17,500
NCI in Riaz Ltd ’s share of net assets of
Siddiq Ltd
25% × 24,500 (6,125)
120,750 70,875
Total 191,625
(W8) Consolidated retained earnings carried forward
Rs.
All of Hasan Ltd
Per the question 180,250
Unrealised profit (W2) (4,580)
175,670
Share of Riaz Ltd
75% × (17,500) (W5) (13,125)
Share of Siddiq Ltd
55% × 17,500 (W5) 9,625
Gain on bargain purchase (W6) 9,625
181,795
(W9) Consolidated other reserves
Rs.
All of Hasan Ltd
Per the question 350,000
Share of Riaz Ltd
75% × 70,000 (W5) 52,500
402,500

© Emile Woolf International 303 The Institute of Chartered Accountants of Pakistan


Advanced accounting and financial reporting

CHAPTER 24: DISPOSAL OF SUBSIDIARIES

24.1 PATCHE LTD


(a) (i) Statement of profit or loss and other comprehensive incomes for the year ended 30
June 2016
Rs.’m
Operating profit 390
Profit on disposal of shares (W1) 210
Profit before tax 600
Income tax (120)
Profit for the year 480
Other comprehensive income 60
540

(ii) Statement of changes in equity for the year ended 30 June 2016
Share Retained Total
capital earning
Rs.’m Rs.’m Rs.’m
Balance b/f (opening Balance) 1,500 660 2,160
Profit for the year - 480 480
Balance c/d 1,500 1,140 2,640

(b) Consolidated statement of profit or loss and other comprehensive incomes for the
year ended 30 June 2016
Rs.’m
Profit before tax (390 + 180) 570
Income tax expenses (120 + 60) (180)
Profit for the year 390
Other comprehensive income (60 + 30) 90
Total comprehensive income 480
Attributable to
Non-controlling interest (W2) 30
Members of the parent (480 – 30) 450
480

(c) Consolidated statement of financial position as at 30 June 2016


Non-current assets: Rs.’m

Property plant and equipment (1605 + 534) 2,139


Goodwill (W3) 240
2,379
Current assets:
Inventories (960 + 570) 1,530
Trade receivables (750 + 525) 1,275
Cash and bank (240 + 267) 507
3,312
5,691

© Emile Woolf International 304 The Institute of Chartered Accountants of Pakistan


Answers

Equity attributable to owners of the parent


Share capital 1,500.0
Reserves (W4) 1,432.5
2,932.5
Non-controlling interest (W6) 472.5
3,405
Current liabilities:
Trade payables (885 + 513) 1,398
Income tax payables (240 + 180 + 90 (W1)) 510
Provisions (285 + 93) 378
2,286
5,691
Workings:
1. Gain on disposal of shares in parent’s separate financial statement:-
Rs.’m
Fair value of consideration received 480
Less: Original cost of shares (765 x 20%/85%) (180)
Parent gain 300
Less tax on parent’s gain (30%) (90)
210
2. Non-controlling interest (NCI)
Profit for the year: Rs.’m
9
Pre-disposal periods = /12 x 120m x 15% = 13.5
3
Post-disposal periods = /12 x 120m x 35% = 10.5 24
Other comprehensive income
9
Pre-disposal periods = /12 x 30m x 15% = 3.375
3
Post-disposal periods = /12 x 30m x 35% = 2.625 6
30
3. Goodwill
Rs.’m
Consideration transferred (285 + 480) 765
Non-Controlling interest at fair value 135
Less:
Fair value of identifiable net assets
at acquisition:
Share capital 600
Pre-acquisition reserve 60 (660)
240
4. Consolidated reserves
Rs.’m
All of Patche
Per question at year-end 930.00
Adj. to equity on disposal (W5) 217.50
Tax on parent gain (W1) (90.00)
1,057.50

© Emile Woolf International 305 The Institute of Chartered Accountants of Pakistan


Advanced accounting and financial reporting

Groups’ share of post-acquisition reserve of Somers:


85% (412.5 (see below) x 85%) 350.63
65% (37.5 (see below) x 65%) 24.37
1,432.50
Post-acquisition profits of Somers
Rs.’m
Per question at year-end 510.0
Profit since the date of disposal (when Somers is a 65%
subsidiary): Rs. 150m  3/12 (37.5)
Balance on reserves at the date of disposal 472.5
Balance on reserves at the date of acquisition (60.0)
Post-acquisition reserves up to the date of disposal (for
which period Somers was an 85% subsidiary) 412.5
5 Adjustment to equity on disposal of shares in group financial statement
Rs.’m
Fair value on consideration received 480.00
Increase in NCI in net asset and goodwill at disposal
(196.8 x 20%/15%) (262.50)
217.50
6 Non-controlling interest
Rs.’m
NCI @ acquisition 135.00
NCI share of post-acquisition reserve:
Somers (412.5 x 15%) 61.88
Somers (37.5 x 35%) 13.12
Increase in NCI (W5) (214.5 + 48) 262.50
472.50

24.2 DISPOSAL
Rs. Rs.
million million
Consideration from sale of shares 960
Fair value of retained shares in Spool 100
1,060
Net assets of Spool at carrying value 800
Minus: non-controlling interest de-recognised (10%  800) (80)
720
Gain on sale of shares 340

None of the assets of Spool have been re-valued, therefore there is no balance on a revaluation
reserve; therefore none of this gain should be transferred directly to retained earnings and not
reported in profit or loss.
There is no information to suggest that a reclassification adjustment is required to reclassify
income previously reported as other comprehensive income as profit or loss.
The total gain of Rs. 340 million on disposal of the shares should therefore be recognised in profit
or loss for the period.
Hoo will recognise an investment in Spool in its statement of financial position in accordance with
the requirements of IAS 39. On initial recognition, this investment should be valued at Rs. 100
million.

© Emile Woolf International 306 The Institute of Chartered Accountants of Pakistan


Answers

24.3 PART DISPOSAL

The disposal of 10% of the shares in S leaves P with a controlling interest; therefore the disposal
of the shares should be accounted for as an equity transaction between owners of the group. No
gain or loss is recognised in the consolidated financial statements of P.
It is assumed that the profits of S for the year were Rs. 200 million (all retained; therefore Rs. 900
million - Rs. 700 million). At 30 June it is assumed that profits for the year to date were Rs. 100
million (= Rs. 200 million  6/12); therefore the net assets of S at this date were Rs. 800 million.
P NCI
Rs. m Rs. m
Before the share sale (80%  800) 640 (20%) 160
After the share sale (70%) 560 (30%) 240
Change in interest in S - 80 + 80
The shares were sold for Rs. 94 million adding to the assets in P’s statement of financial position.
The transaction should therefore be accounted for in equity as follows:
Debit: Cash Rs. 94 million
Credit: NCI Rs. 80 million
Credit: Reserves attributable to P (= gain = balance) Rs. 14 million
Rs. million Rs. million
Post-acquisition profit attributable to S (see above) 200
Less: Impairment of goodwill (8)
Recognised profit 192
Attributable to equity owners of P
1 January – 30 June (80%  200  6/12) 80
1 July – 31 December (70%  200  6/12) 70
Goodwill impairment (8)
Attributable to NCI 142
1 January – 30 June (20%  200  6/12) 20
1 July – 31 December (30%  200  6/12) 30
50
192

24.4 THE A GROUP


A‘s original investment in C was 90% of C’s 400,000 shares (360,000 shares). During the year A
has disposed of 350,000 of these shares, which reduces the investment from subsidiary status to
that of a ‘simple’ investment.
A Group
Consolidated statement of financial position as at 31 December Year 4
Rs.000
Goodwill (W2, B only) 428
Investment in C at fair value 44
Other net assets (W4) 6,661
7,133
Equity
Share capital 1,500
Accumulated profits attributable to owners of A (working 1) 5,347
Equity attributable to owners of A 6,847
Non-controlling interest: 20%  (1,260 + 170) 286
Total equity 7,133

© Emile Woolf International 307 The Institute of Chartered Accountants of Pakistan


Advanced accounting and financial reporting

Statement of profit or loss for year ended 31 December Year 4


Working
A B Group
Rs.000 Rs.000 Rs.000
Operating profit 1,200 250 1,450
Minus: Dividend from B (16) nil (16)
1,184 250 1,434
Gain on disposal of C (W2) 237 237
Profit before tax 1,421 250 1,671
Tax (360) (60) (420)
Profit after tax 1,061 190 1,251

Attributable to:
Equity owners of A (1,061 + 80%  190) 1,213
Non-controlling interest: 20% × 190 38
1,251
Workings
(1) Movement on consolidated reserves attributable to owners of parent
A B C Group
Rs.000 Rs.000 Rs.000 Rs.000
At 31 December Year 3 (W5) 3,300 272 612 4,184
Profit for year attributable to A 1,213
Dividends paid by A (50)
At 31 December Year 4 5,347
(2) Disposal of shares in C, with loss of control
Gain to parent Rs.000 Rs.000
Net assets of C at date of disposal: de-recognised 1,400
Purchased goodwill in C de-recognised
(see working 3) 472
1,872
Minus: Non-controlling interest de-recognised
(10%  1,400) (140)
Assets attributable to A de-recognised 1,732
Fair value of investment retained 44
Sale proceeds 1,925
1,969
Total gain on disposal of shares 237
Since there has been no revaluation of non-current assets and there is no information
about any reclassification adjustments that might be required, it is assumed that this entire
gain should be included in profit or loss for the year.

(3) Calculation of goodwill


B C
Rs.000 Rs.000
Cost of Investment 1,164 1,120
Less: Group share of the fair value of the net assets at
acquisition
80% × (500 + 420) (736)
90% × (400 + 320) (648)
428 472

© Emile Woolf International 308 The Institute of Chartered Accountants of Pakistan


Answers

(4) Other net assets


Rs.000
A’s net assets as 1 January Year 4 2,516
B’s net assets at 1 January Year 4 1,260
A’s retained profit year ended 31 December Year 4 790
B’s retained profit year ended 31 December Year 4 170
Proceeds of disposal of C 1,925
6,661

(5) Calculation of post-acquisition retained profits b/f attributable to A


Rs.000
A As given in the question 3,300
B and C Group share of post-acquisition
B 80%  (760 - 420) 272
C 90%  (1,000 - 320) 612
Total 4,184

24.5 BARTLETT LTD


Consolidated statement of profit or loss for the year ended
31 December 2016

Total
Rs.
Revenue (1,926,500 + 396,200 + 260,800) 2,583,500

Cost of sales (1,207,200 + 202,950 + 193,100) (1,603,250)


Gross profit 980,250
Net operating expenses (400,100 + 152,650 + 52,650) (605,400)
Operating profit 374,850
Profit on disposal of
operations (W1) 66,360
Profit on ordinary
activities before taxation 441,210
Tax on profit on ordinary
activities (110,000 + 4,750 + 13,750) (128,500)
Profit for the year 312,710

Profit attributable to: Rs.


Owners of the parent 286,162
Non-controlling interests (W2) (11,458)
Profit for the year 312,710
Workings
(1) Profit on disposal of Lymon
Recognise: Rs.
Proceeds 212,000

© Emile Woolf International 309 The Institute of Chartered Accountants of Pakistan


Advanced accounting and financial reporting

Derecognise:
Net assets of subsidiary
Net assets at January 2016 140,000
Profit to 1 July 2016 (6/12 x 20,600) 10,300
150,300
Non-controlling interest (20%) (30,060)
(120,240)
Unimpaired goodwill (25,400)
Profit on disposal 66,360

(2) Non-controlling interests


In Lymon Inc 20% x (6/12 x Rs. 20,600) 2,060
In Zeigler Inc 35% x (6/12 x Rs. 53,700) 9,398
11,458

© Emile Woolf International 310 The Institute of Chartered Accountants of Pakistan


Answers

CHAPTER 26: FOREIGN CURRENCY

26.1 DND LIMITED


Date Description Dr. Cr.
Rs. Rs.
1-Jul-16 Advance to suppliers 1,210,000
Cash 1,210,000
(Amount paid on signing the contract.
Exchange rate was Rs. 60.5/US$)
30-Sep-16 Advance to suppliers 3,050,000
Cash 3,050,000
(Amount paid on delivery. Exchange rate
was Rs. 61/US$)
30-Sep-16 PPE in transit/ CWIP 6,090,000
Advance to suppliers 4,260,000
Payable to suppliers 1,830,000
(Recording of asset on the delivery date as
risk and rewards are transferred to the
company)
31-Dec-16 Exchange loss 6,000
Payable to suppliers 6,000
(Adjustment of exchange rate as of
reporting date. Exchange rate was Rs.
60.5/US$)
31-Jan-17 Property, plant and Equipment 6,090,000
PPE (In transit/ in progress) 6,090,000
(Transfer the new plants and machineries to
Property, Plant and Equipment)
31-Jan-17 Payable to suppliers 1,836,000
Exchange loss (Bal.) 9,000
Cash 1,845,000
(Final payment to supplier. Exchange rate
was Rs. 61.5/US$1)

26.2 STARLIGHT LIMITED


(a) Starlight Limited. Translated Profit and Loss Account
QR’000 Rate Rs.’000
Turnover 344,880 32 10,778
Cost of Sales (249,710) 32 (7,804)
Gross Profit 95,170 2,974
Expenses (29,490) 32 (922)
Profit Before Tax 65,680 (2,052)
Taxation (17,325) 32 (541)
Profit After Tax 48,355 1,511
Interim dividend (16,300) 32 (509)
Retained profit for the year 32,055 1,002

© Emile Woolf International 311 The Institute of Chartered Accountants of Pakistan


Advanced accounting and financial reporting

(b) (i) Calculation of goodwill


QR’000 QR’000
Cost of investment:
(Rs. 2,500,000 x 30 QR) 75,000
Less net assets acquired:
Share capital 20,250
Pre–acquisition reserves 49,300
69,550
Group share 80% thereon (55,640)
Goodwill in QR 19,360
Translated to rupees at the closing rate:
QR 19,360,000 ÷ 33 = Rs. 586,667
(ii) Non–controlling Interest in statement of profit or loss:
20% x Rs. 1,511,000 = Rs. 302,200

26.3 PERCEPT LTD


(a) Translation of financial position of Trint Ltd as at 31 December 2016
YEN YEN RATE
Rs.’000 Rs.’000 Rs.’000
Property plants and equipment 12,375 5 2,475
Financial assets 1,250 5 250
Current assets 8,250 5 1,650
21,875 4,375

Share capital 5,000 6 833


Retained earnings 4,500 6 750
Fair value adjustment 2,875 6 479
Post-acquisition (Balancing figure) 3,000 1,013
Non-current liabilities 4,000 5 800
Current liabilities 2,500 5 500
21,875 4,375

Note: Fair value of property,


plant and equipment: = 12,375 Yen – 5,000 yen (equity)
- 4,500 Yen (pre-acquisition profit) = 2,875 Yen
(b) Goodwill
Yen Rate Rs.
‘000 ‘000
Cost of acquisition 6,900 6 1,150
Fair value (NCI) 6,250 6 1,042
13,150 2,192
Net asset acquired 12,375 6 (2,063)
775 6 129

Retranslated at the closing rate 775 5 155


Exchange gain on retranslation of goodwill 26

© Emile Woolf International 312 The Institute of Chartered Accountants of Pakistan


Answers

Parent’s share (70% x 26) = 18.2


Non-controlling interest share (30% x 26) = 7.8
26
(c) Exchange rate difference arising on re-translation of Trint Ltd’s net assets
Rs.’000
Difference from translation of opening net assets
Opening rate: 12,375,000 Yen at 6 2,062.5
Closing rate: 12,375,000 Yen at 5 2,475.0
Exchange gain 412.5
Difference arising from translation of profit
Average rate (2,000 Yen at 5.5) 364.0
Closing rate (2,000 Yen at 5) 400.0
Exchange gain 36.0
Total exchange gain 448.5

Rs.’000
Parent’s share of the exchange gain (70% of 448.5) 314.0
Non-controlling interest share of the exchange gain (30% x 448.5) 134.5
448.5

26.4 ORLANDO
(a) Year to June Year 4
The revenue and the receivable for the sale of €96,000 should be translated at the spot
rate of 0.8 = $120,000
The capital expenditure of €1m should also be translated at the spot rate of 0.8:
Debit Property, plant and equipment $1,250,000
Credit: Payables $1,250,000.
The receipt on 12 June relating to the receivable is translated at the rate at that date of 0.9.
This generates cash of $106,667 to settle a receivable of $120,000. Hence an exchange
loss of $13,333 is recognised in profit or loss.
The non-current asset is not re-translated at the year end, but the outstanding payable (a
monetary item) must be re-stated to the year end exchange rate of 0.7. This gives a year-
end payable balance of $1,428,571. This has increased from the initial $1,250,000;
therefore an exchange loss of $178,571 will be recognised in profit or loss.

(b) Year to June Year 5


When the payable is settled after the year end at the spot rate of 0.8, it results in a
payment of $1,250,000. There is an exchange gain of $178,571 compared with the
carrying value at the end of Year 4.

26.5 MANCASTER AND STOCKPOT


Part A
(1) (a) Functional currency
Functional currency is the currency of the primary economic environment in which
the entity operates. It reflects the underlying transactions, events and conditions that
affect the company. It is not simply the currency of the country where the company
is based. For example, if a company is incorporated and listed in Pakistan but
operates in the South African mine fields, then its functional currency will be the
South African rand, not rupees, because the rand is the currency in which it
generates and spends the most cash.

© Emile Woolf International 313 The Institute of Chartered Accountants of Pakistan


Advanced accounting and financial reporting

(b) Presentation currency


Presentation currency is the currency in which the financial statements are
presented. Continuing the example of the company referred to above, since the
company is listed in Pakistan it will present and file its accounts in rupees.
Transactions and balances measured in a currency other than the presentation
currency will have to be translated into the presentation currency for reporting
purposes. So the sales and operating costs incurred in South Africa will need to be
translated from Rand into Sterling.
(2) Factors to be considered when determining the functional currency of an overseas
subsidiary
The primary economic environment in which an entity operates is defined in IAS 21 as the
one in which it primarily generates and expends cash.
Primary indicators
An entity must consider the following factors in determining its functional currency:
 the currency that mainly influences sales prices for goods and services
 the country whose competitive forces and regulations mainly determined the sales
prices of its goods and services
 the currency that mainly influences labour, material and other costs of goods or
services.
Additional indicators
The following factors may also provide evidence:
 the currency in which funds from financing activities are generated (i.e. currency
used for issuing debt and equity)
 the currency in which surplus cash is invested.
Part B
(a) Translation: Statement of financial position of Stockpot at 31 March Year 4
EU000 Rate $000
Property, plant and equipment 30,000 2.2 13,636
Inventories 18,000 2.2 8,182
Trade receivables 15,000 2.2 6,819
Trade payables (10,400) 2.2 (4,727)
Bank overdraft (7,600) 2.2 (3,455)
Non-current liabilities (20,000) 2.2 (9,091)
25,000 11,364
Issued capital 15,000 3.0 5,000
Pre-acquisition reserves 5,000 3.0 1,667
20,000 6,667
Post-acquisition reserves 5,000 balancing figure 4,697
25,000 11,364

Mancaster Group: Consolidated statement of financial position at


31 March Year 4
$000 $000
Non-current assets
Goodwill (see workings) 682
Property, plant and equipment (20,000 + 13,636) 33,636

© Emile Woolf International 314 The Institute of Chartered Accountants of Pakistan


Answers

$000 $000
Current assets:
Inventories (10,000 + 8,182) 18,182
Trade receivables (10,000 + 6,819) 16,819
35,001
69,319
$000
Capital and reserves:
Issued capital 9,000
Accumulated profits (see workings) 16,205
25,205
Non-controlling interest (see workings) 2,841
28,046
Non-current liabilities:
Loans (10,000 + 9,091) 19,091
Current liabilities:
Bank overdraft (6,100 + 3,455) 9,555
Trade payables (7,900+4,727) 12,627
22,182
69,319
(b) Translation: Statement of profit or loss of Stockpot for year ended 31 March Year 4
EU000 Rate $000
Revenue 60,000 2.3 26,087
Cost of sales (30,000) 2.3 (13,043)
Gross profit 30,000 13,044
Operating expenses (16,000) 2.3 (6,957)
Operating profit 14,000 6,087
Interest payable (2,000) 2.3 (870)
Profit before tax 12,000 5,217
Tax (4,200) 2.3 (1,826)
Profit after tax 7,800 3,391
The statement of profit or loss has been translated at the average rate as an
approximation to the actual (historical) rate. The closing rate is not allowed under IAS 21.
Mancaster Group: Consolidated statement of profit or loss for the year ended 31
March Year 4
$000
Revenue (50,000 + 26,087) 76,087
Cost of sales (25,000 + 13,043) (38,043)
Gross profit 38,044
Operating expenses (15,000 + 6,957) (21,957)
Operating profit 16,087
Interest payable (1,000 + 870) (1,870)
Profit before tax 14,217
Tax (3,600 + 1,826) (5,426)
Profit after tax 8,791
Attributable to
Equity holders of the parent 7,943
Non-controlling interest (25% × 3,391) – see translation 848
8,791

© Emile Woolf International 315 The Institute of Chartered Accountants of Pakistan


Advanced accounting and financial reporting

Workings
(1) Goodwill at date of acquisition
$000 Rate EU000
Cost of investment 5,500 3 16,500
Minus: Share of net assets acquired:
Share capital (translated at 3.0) 5,000
Accumulated profits (translated at 3.0) 1,667
6,667
Group share (75%) 5,000 3 (15,000)
Goodwill 500 1,500
Re-stated to closing rate: (1,500/2.2) 682
Translation gain on goodwill – to group
reserves 182
(2) Consolidated accumulated profits
$000
Mancaster: 12,500
Stockpot: group share of post-acquisition profits (75% ×
4,697) – see translation of statement of financial position
3,523
Translation gain on goodwill 182
16,205
(3) Non-controlling interest
$000
Non-controlling share of net assets at 31 March Year 4 :
(25% × 11,364) – see translation of Stockpot statement of
financial position 2,841

26.6 A, B AND C
A group: Summarised consolidated statement of profit or loss and other comprehensive income
for the year ended 30 September 2016
Rs.000
Revenue (4,600 +3,385(W1)) 7,985
Costs and expenses (3,700+2,462(W1)) (6,162)
Share of associate’s profit (W3) 160
Profit before tax 1,983
Income tax expense (200+231(W1)) (431)
Profit for the year 1,552

Other comprehensive income


Revaluation gains net of tax (200+185(W1)) 385
Share of associate’s OCI (W3) 28
Forex gain in year (W4) 803
Total other comprehensive income 1,216
Total comprehensive income 2,768

© Emile Woolf International 316 The Institute of Chartered Accountants of Pakistan


Answers

Rs.000
Profit for year attributable to: 1,414
Equity holders of the parent
Non-controlling interest (W5) 138
1,552
Total comprehensive income attributable to:
Equity holders of the parent 2,432
Non-controlling interest (W5) 336
2,768
Consolidated statement of financial position for the A group as at 30 September 2016
Rs.000
Assets
Non-current assets
Property, plant and equipment (7,000 + 6,349 (W1)) 13,349
Goodwill (W2) 635
Investment in associate (W6) 1,220
15,204

Current assets (3,000 + 3,175 (W1)) 6,175


Total assets 21,379
Equity and liabilities
Equity attributable to the parent
Share capital 2,000
Retained reserves (W8) 13,522
15,522
Non-controlling interest (W7) 1,476
Total equity 16,998
Current liabilities (2,000 + 2,381(W1)) 4,381
Total equity and liabilities 21,379

Rate @ avge
W1 Translation of B
A$000 rate Rs.000
Statement of profit or loss and other
comprehensive income
Revenue 2,200 Rs./A$0.65 3,385
Cost of sales and expenses (1,600) Rs./A$0.65 (2,462)
Profit before tax 600 923
Income tax (150) Rs./A$0.65 (231)
Profit for year 450 692
Other comprehensive income:
Revaluation gains on PPE Total OCI 120 Rs./A$0.65 185
120 185
Total comprehensive income 570 877
Statement of financial position
Non-current assets
Property, plant and equipment 4000 @CR A$0.63 6349

© Emile Woolf International 317 The Institute of Chartered Accountants of Pakistan


Advanced accounting and financial reporting

Rate @ avge
W1 Translation of B
A$000 rate Rs.000
Current assets 2,000 @CR A$0.63 3,175
Total assets 6,000 9,524
Share capital 1000 @HR A$0.50 2000
Pre-acquisition reserves 1800 @HR A$0.50 3600
Post-acquisition reserves 1,700 Bal fig 1,543
Total equity 4,500 7143
Current liabilities 1,500 @CR A$0.63 2,381
Equity and liabilities 6,000 9524

W2 Goodwill A$000 Rate Rs.000


Consideration transferred 2,600 Rs./A$0.50 5,200
NCI @ FV 600 Rs./A$0.50 1,200
Net assets acquired:
Share capital (1,000) Rs./A$0.50 (2,000)
Retained earnings (1,800) Rs./A$0.50 (3,600)
Goodwill at 1 October 2013 400 800
Forex loss (balancing figure) (237)
Goodwill at 30 September 2015 400 Rs./A$0.71 563
Forex gain (balancing figure) 72
Goodwill at 30 September 2016 400 Rs./A$0.63 635

W3 Share of associate’s profit/OCI Rs.000


Share of associate’s PFY (40% x Rs. 400,000) 160
Share of associate’s other comprehensive income (40% x Rs. 70,000) 28

W4 FOREX gains/losses in the year Rs.000


Closing net assets @ CR (A$4,500,000/0.63) or from W1 7,143
Less opening net assets @ OR ((A$4,500,000 less TCI (5,535)
A$570,000)/0.71)
Less TCI for year @ average rate (A$570,000/0.65) (877)
Forex gain on translation of subsidiary’s net assets 731
Plus Forex gain on translation of goodwill 72
Total Forex gains on translation of subsidiary 803
W5 NCI share of Profit/Total comp income PFY TCI
Rs.000 Rs.000
Subsidiary’s PFY/TCI (W1) 692 877
20% share 138 175
Forex gain on translation of subsidiary (20% x
Rs. 803,000) 161
138 336

© Emile Woolf International 318 The Institute of Chartered Accountants of Pakistan


Answers

W6 Investment in associate Rs.000


Investment at cost 900
Plus share of post-acquisition reserves 40% x
(Rs. 1,500,000 -Rs. 700,000) 320
1,220

W7 Non-controlling interest Rs.000


NCI on acquisition (W2) 1,200
NCI share of post-acquisition reserves of subsidiary
(20%xRs. 1,543,000(W1)) 309
NCI share of net FOREX losses on translation of goodwill
(20% x Rs.(237,000-72,000)) (33)
NCI at 30 September 2016 1,476

W8 Reserves A B
Rs.000 Rs.000
As per SOFP 12,100 5,143

Less pre-acquisition reserves (W1) (3,600)


1,543
Group share 80% x Rs. 1,543,000 1,234
Group share of associate’s post-acquisition reserves 320
(W6)
Group share of net FOREX losses on translation of
goodwill (80% x Rs.(237,000-72,000)) (132)
Group reserves 13,522

26.7 OMEGA LIMITED


Omega Limited
Extract from Statement of comprehensive income for the year ended 31 December 2013
 Profit for the year: Rupees
Dividend received from
AWL (IFRS 9,B5.7.5.1) (20,000*10*15%*26.5) 795,000
Transfer of FV gain reserve of 31-12-2012, on derecognition
of AWL investment W.1 500,000
FV / exchange gains on valuation of AWL shares on 1-6-
2013 W.1 2,124,000
Loss on de-recognition of AWL' shares W.1 ( 308,000)
 Other comprehensive income:
FV gain/(loss) on investment available for sale W.1 693,000
Exchange gain on investment available for sale W.1 225,225

© Emile Woolf International 319 The Institute of Chartered Accountants of Pakistan


Advanced accounting and financial reporting

W-1 FV per Gain /


No. of Investment
Date share (loss) Remarks
shares
AED AED Conv.@ Rupees Rupees
1-May-2012 20,000 12.00 240,000 25.00 6,000,000
31-Dec-2012 20,000 13.00 260,000 25.00 6,500,000 500,000 FV gain
1-Jun-2013 22,000 14.00 308,000 28.00 8,624,000 2,124,000 Gain on
(20,000x1.1) valuation of
AWL on its
acquisition
by HL
1-Jun-2013 16,500 18.00 297,000 28.00 8,316,000 Loss on de-
(22,000/4*3) recognition
(308,000)
of
AWL
shares
31-Dec-2013 16,500 19.50 321,750 28.00 9,009,000 693,000 FV gain
31-Dec-2013 16,500 19.50 321,750 28.70 9,234,225 225,225 Exchange
gain
3,234,225

26.8 PARENT COMPANY LIMITED


(i) Parent Company Limited
Consolidated statement of financial position as at 30 June 2014
Rs. in million
Assets
Non-current assets
Property, plant and equipment 4,200+3,500+250×17.3 12,025.00
Intangible assets (W-1) 796+1,730 2,526.00
Current assets 3,500+4,000+450×17.3 15,285.00
29,836.00
Equity and liabilities
Equity attributable to owners of PL:
Ordinary shares capital 6,000.00
Retained earnings (W-4) 5,565.15
[(W-1) 253+(W-2) 813.20]] ×
Exchange reserve 75% 799.65
12,364.80
Non-controlling interest W-4 (731.20+2,050) 2,781.20
15,146.00
Current liabilities 4,700+4,800+300×17.3 14,690.00
29,836.00
(ii) Parent Company Limited
Consolidated statement of other comprehensive income
For the year ended 30 June 2014
Rs. in million
Other comprehensive income:
Items that may be translated to profit or loss:
 Exchange gain on translation of goodwill W-1 55.00
 Exchange gain on translating of foreign
operations W-2 195.80
250.80

© Emile Woolf International 320 The Institute of Chartered Accountants of Pakistan


Answers

W-1: Goodwill and exchange gain


LS FS
thereon
---------- Rs. in million ----------
Purchase consideration 2,000 (300 × 15) 4,500
NCI fair value on acquisition date 540 (90 × 15) 1,350
2,540 5,850
Net assets on acquisition (120+260)
date (1,800/1.2+250) (1,750) × 15 (4,200)
Estimated liability for a pending claim 6 -
Goodwill on acquisition date i.e. 1 July
2012 1,650
Impairment on 30 June 2014 - (10 × 17.3) (173)
Goodwill as at 30 June 2014 1,477
Goodwill as at 30 June 2014 @ Rs. 17.30 796 (100×17.3) 1,730
Exchange gain reserve as 30 June 2014 253
(148.5 ×
Exchange gain reserve as 30 June 2013 75%) (198)
Exchange gain for the year 55

W-2: Exchange reserve on translation of FS foreign operations


CU in Conversion Rs. in
million @ million
Net assets as at 30 June 2014 400.00 17.30 6,920.00
Net assets as at 30 June 2013
(400-30+18) 388.00 16.80 6,518.40
Profit for the year 30.00 17.00 510.00
Dividend paid during the year
(120×15%) (18.00) 16.90 (304.20)
400.00 6,724.20

Exchange gain for the year ended 30 Jun


2014 195.80
(463.05/75
Exchange reserve as at 30 June 2013 %) 617.40
Exchange gain on foreign operations as at 30
June 2014 813.20

W-3: FS retained earnings CU in Conversion Rs. in


million @ million
Net assets as at 30 June 2014 400.00 17.30 6,920.00

Net assets on acquisition date (280.00) 15.00 (4,200.00)


Post-acquisition retained earnings as at 30
June 2014 including OCI item of exchange
gain 120.00 2,720.00
Exchange gain to be classified to OCI W-2 (813.20)
Post-acquisition retained earnings as at 30
June 2014 1,906.80

© Emile Woolf International 321 The Institute of Chartered Accountants of Pakistan


Advanced accounting and financial reporting

W-4: Consolidated retained earnings and Retained


NCI - LS NCI - FS
NCI earnings
----- Rs. in million -----
Balance as at 30 June 2014 3,500.00 - -
NCI fair value on acquisition date - 540.00 1,350.00
Post-acquisition profit – LS
(650×80%) 520.00 130.00 -
Post-acquisition profit – FS
(1,906.8×75%) 1,430.10 - 476.70
LS earnings used for bonus issue
240.00
(300×80% /20%) 60.00 -
Liability paid in May 2014 booked on acq.
4.80
(6×80%/20%) 1.20 -
Exchange gain on translation of FS
- 266.55
(W-2) 1,066.20×25% -
Goodwill impairment
(173×75%/25%) (129.75) - (43.25)
5,565.15 731.20 2,050.00

© Emile Woolf International 322 The Institute of Chartered Accountants of Pakistan


Answers

CHAPTER 27: IAS 7: STATEMENTS OF CASH FLOWS

27.1 EVERNEW LTD


Consolidated statement of cash flows for the year ended 31 December 2016

Rs.’000 Rs.’000
Profit before taxation 138,960
Adjustment for non-cash items:
Depreciation charges 72,720
Profit on disposal of subsidiary (W.1) (5,040)
Interest expenses (payable) 10,080
Operating profit before working
Capital changes 216,720
Changes in working capital
Increase in inventory (W2) (28,800)
Increase in Receivables (W2) (32,400)
Increase in Creditors (W2) 25,200
(36,000)
Cash generated from operations 180,720
Income tax paid (W.3) (37,080)
Net cash flow from operating activities 143,640
Cash flow from investing activities:
Purchases of non-current assets (W4) (111,240)
Sales of Pastit Limited (W5) 41,040
Net cash used in investing activities (70,200)
Cash flow from financing activities:
Redemption of 10% debenture (W6) (18,000)
Dividend paid to non-controlling interest (W7) (3,600)
Interest paid (10,080)
Net cash used in financing activities (31,680)
Net increase in cash & cash equivalent 41,760
Cash & cash equivalent b/f (14,400 – 36,000) (21,600)
Cash & cash equivalent c/f 20,160
Cash & cash equivalent c/f is represented by:
Cash in hand 63,360
Bank overdraft (43,200)
20,160

© Emile Woolf International 323 The Institute of Chartered Accountants of Pakistan


Advanced accounting and financial reporting

Workings
(W1) Profit on disposal of subsidiary:
The entire 80% shareholding was sold.
Rs.’000
Net asset of subsidiary sold (shown in the question) 43,200
Sales proceeds 39,600
Less Net asset sold x 80% = (80% x Rs. 43,200) 34,560
Profit on disposal of subsidiary 5,040

(W2) Movement in Working Capital


31/12/16 Add Less Bal. Cash flow
Rs.’000 disposal 31/12/15 statement
Rs.’000 Rs.’000 Rs.’000
Inventory 180,000 14,400 165,600 28,800
Receivables 151,200 18,000 136,800 32,400
Trade creditors (108,000) (10,800) (93,600) (25,200)
(W3) Income Tax Paid
Taxation
Rs.’000 Rs.’000
Tax on disposal 2,160 Balance b/f 39,240
Cash/Bank 37,080 Tax for the year – P & L 46,800
Balance c/f 46,800
86,040 86,040
(W4) Non-current assets
Non-current assets
Rs.’000 Rs.’000
Balance b/f 360,000 Disposal 28,800
Cash/Bank 111,240 Depreciation – (P & L) 72,720
Balance c/f 369,720
471,240 471,240
W5 Cash flow from sale of Pastit Limited
Rs.’000
As per question 39,600
Add Bank overdraft of Pastit Limited on disposal 1,440
41,040

(W6) Movement on debenture


Rs.’000
Balance b/f at 01/01/2015 90,000
Disposal of subsidiary (3,600)
Cash paid (bal. figure) (18,000)
Balance c/f at 31/12/2016 68,400

© Emile Woolf International 324 The Institute of Chartered Accountants of Pakistan


Answers

(W7) Non-controlling interest

Rs.’000 Rs.’000
Disposal 8,640 B/d 41,400
Dividend paid to NCI 3,600 P&L 7,200
B/d 36,360
48,600 48,600

27.2 BELLA
Statement of cash flows for the year ended 31 March Year 6
Rs.000 Rs.000
Cash flows from operating activities
Profit before taxation 4,617
Adjustments for:
Depreciation (Working 1) 300
Loss on disposal of non-current asset (800 – 700) 100
Interest expense 60
––––––––
5,077
Increase in inventories (280 – 100) (180)
Decrease in trade and other receivables (1,350 – 1,290) 60
Increase in trade payables (430 – 275) 155
––––––––
Cash generated from operations 5,112
Interest paid (Working 3) (45)
Income taxes paid (Working 4) (185)
––––––––
Net cash from operating activities 4,882
Cash flows from investing activities
Purchase of property, plant and equipment (6,000)
Proceeds from the sale of property, plant and equipment 700
Purchase of intangible assets (800 – 300) (500)
––––––––
Net cash used in investing activities (5,800)
Cash flows from financing activities
Proceeds from the issue of share capital (Working 2) 865
Issue of long-term loan (600 – 500) 100
Dividends paid (350)
––––––––
Net cash inflow from financing activities 615
––––––
Net decrease in cash and cash equivalents (303)
Cash and cash equivalents at the beginning of the period 45
––––––
Cash and cash equivalents at the end of the period (55 – 313) (258)
––––––

© Emile Woolf International 325 The Institute of Chartered Accountants of Pakistan


Advanced accounting and financial reporting

Workings
(1) Property, plant and equipment (PPE)
Rs.000 Rs.000
PPE at net book value (NBV) at end of year 12,900
PPE at NBV at beginning of year 8,000
Disposals during the year at NBV (800)
(7,200)
5,700
Depreciation charge for the year (balancing figure) 300
PPE acquired during the year 6,000

(2) Share capital and premium


Share Share Total
capital premium
Rs.000 Rs.000 Rs.000
At end of year 1,900 95 1,995
At beginning of year 1,100 30 (1,130)
Cash receipts from share issue 865

(3) Interest payable


Rs.000
Accrued interest at beginning of year 25
Interest charge in profit and loss 60
85
Accrued interest at end of year (40)
Interest payments in the year 45
(4) Current tax payable
Rs.000
Tax payable at beginning of year 325
Tax charge in profit and loss 400
725
Tax payable at end of year (540)
Tax payments in the year 185

27.3 BISHOP GROUP

(a)
Statement of cash flows for year ended 31 December 20X2
Rs.000 Rs.000
Cash flows from operating activities (Note 1) 2,282
Interest paid (120 + 205) (325)
Dividends received 90
Taxation paid (W2) (117)
–––––––
Net cash flows from operating activities 1,930

Cash flows from investing activities:


Payments to acquire tangible non-current assets (W3) (4,996)
Receipts from sale of tangible non-current assets 810

© Emile Woolf International 326 The Institute of Chartered Accountants of Pakistan


Answers

Statement of cash flows for year ended 31 December 20X2


Rs.000 Rs.000
Purchase of investments (300)
–––––––
Net cash used in investing activities (4,486)

Cash flows from financing activities:


Proceeds of share issue 3,824
Additional loans (1,200 – 800 – 25) 375
Capital payments under finance leases (W4) (150)
Dividends paid to NCI (W1) (295)
Equity dividends paid (600)
–––––––
Net cash provided by financing activities 3,154
–––––––
Net increase in cash and cash equivalents 598
Effect of exchange rate movements 53
–––––––
651
Cash and cash equivalents brought forward 169
–––––––
Cash and cash equivalents carried forward 820
–––––––
Notes to the statement of cash flows
Reconciliation of operating profit to net cash inflow from operating activities
Rs.000
Operating profit 2,849
Depreciation 1,200
Profit on sale of non-current assets (810 – 720) (90)
Increase in inventories (6,135 – 5,740 – 117) (278)
Increase in receivables (5,720 – 4,380 – 339) (1,001)
Decrease in payables (1,420 – 1,760 – 58) (398)
Net cash flows from operating activities 2,282
Workings
(1)
Non-controlling interest
Rs.000 Rs.000
Dividend paid to NCI 295 Balance b/fwd 2,500
Balance c/fwd 2,800 Statement of profit or 420
loss
Exchange gain
(20% × 875) 175
3,095 3,095

(2)
Tax
Rs.000 Rs.000
Tax paid 117 B/fwd current tax 167
C/fwd current tax 700 B/fwd deferred tax 400
C/fwd deferred tax 550 Statement of profit or loss 800
1,367 1,367

© Emile Woolf International 327 The Institute of Chartered Accountants of Pakistan


Advanced accounting and financial reporting

(3) Non-current assets

Rs.

Opening NBV 7,520

Depreciation (1,200)

Disposals at NBV (720)

New finance leases 700

Exchange rate gains 424

Purchase for cash 4,996

Closing NBV 11,720

(4)

Obligations under finance leases

Rs.000 Rs.000

Cash paid 355 Balance b/f < 1 year 50

Balance c/f < 1 year 110 Balance b/f > 1 year 250

Balance c/f > 1 year 740 Finance charge in profit or loss 205

Non current asset additions 700

1,205 1,205

The payment of Rs. 355,000 is split as Rs. 205,000 interest and Rs. 150,000 capital as
payments are made in arrears and hence the year end payment pays off the year’s finance
cost.
(b) The statement of profit or loss and statement of financial position are based on the
accruals concept whereas the statement of cash flows is based on the cash concept. Cash
is the 'life blood' of the company and is therefore critical to an entity’s survival. Without
cash to pay suppliers, the work force and other payables, the company will cease to
operate, irrespective of how profitable it is.
Shareholders need to know that a company is viable and has the resources to continue,
and perhaps expand, operations. Suppliers need to know they will be paid and customers
need to know the company is in a position to continue operations.
Profit may be significantly affected by the choice of accounting policies made by a
company. This means it is more subjective than cash and more open to manipulation.
However, the statement of cash flows itself may be subject to window dressing, for
example by delaying payment of suppliers until after year end. The auditor needs to be
involved in this respect to ensure the shareholders and other users receive meaningful
information.
The statement of cash flows gives additional information not provided by the other financial
statements.

© Emile Woolf International 328 The Institute of Chartered Accountants of Pakistan


Answers

27.4 THE GRAPE GROUP


Group statement of cash flows for the year ended 31 March Year 4.

Rs.000
Cash flows from operating activities
Net profit before taxation 9,550
Adjustments for:
Depreciation (Note 1) 1,176
Loss on sale of assets 18
Income from associate (139)
Interest expense 552
Operating profit before working capital changes 11,157
Increase in inventories (1,127 – 139) (988)
Increase in receivables (273 – 85) (188)
Increase in payables (203 – 68) 135
Cash generated from operations 10,116
Interest paid (552)
Income taxes paid (W3) (2,400)
Net cash from operating activities 7,164

Cash flows from investing activities

Acquisition of subsidiary net of cash acquired (346 – 3) (343)


Purchase of property, plant and equipment (1,875 – 315) (1,560)
Proceeds from sale of property, plant and equipment (W1) 156
Dividends received from associate (W2) 93
Net cash used in investing activities (1,654)

Cash flows from financing activities Rs.000


Proceeds from issuance of share capital (675 + 519 - 152) 1,042
Repayment of loan notes (990)
Dividends paid (2,100)
Net cash used in financing activities (2,048)
Net increase in cash and cash equivalent 3,462
Cash and cash equivalents at beginning of period 1,728
Cash and cash equivalents at end of period 5,190

Notes to the statement of cash flows


(1) Major non-cash transactions
During the year the group purchased a subsidiary undertaking. Part of the consideration
for the acquisition was in the form of shares. Further details of the acquisition are given
below.

© Emile Woolf International 329 The Institute of Chartered Accountants of Pakistan


Advanced accounting and financial reporting

(2) Purchase of subsidiary undertaking


Rs.000
Net assets acquired:
Property, plant and equipment 315
Inventories 139
Receivables 85
Cash at bank and in hand 3
Payables (68)
474
Goodwill 24
498
Satisfied by:
Shares allotted 152
Cash 346
498

Workings
(1) Proceeds from sale of property, plant and equipment
Rs.000
Cost of assets sold 429
Accumulated depreciation (255)
Loss on sale (18)
Proceeds 156

(2) Dividends received from associate

Interest in associate
Rs.000 Rs.000
Balance b/d 1,920 Dividends received from 93
associates
Share of associates' profit 139 Balance c/d 1,966
after tax
2,059 2,059

(3) Taxation

Taxation
Rs.000 Rs.000
Cash paid 2,400 Balance b/d 2,400
Balance c/d 2,950 Statement of profit or loss 2,950
5,350 5,350

© Emile Woolf International 330 The Institute of Chartered Accountants of Pakistan


Answers

CHAPTER 28: IAS 33: EARNINGS PER SHARE

28.1 AIRCON LTD


(a) Earnings Per Share

1 , 854
2016 = Rs. = Rs. 1.01
1 , 818
1 , 584 6 . 06
2015 = x = Rs. 1.69
900 6 . 30
Workings
1. Calculation of theoretical ex-rights price
1 share at Rs. 6.30 each 6.30
2 rights issue for every 1 at Rs. 5.94 11.88
3 shares for 18.18
Price per share = = Rs. 6.06

2. Weighted average number of shares


6
1 April – 30 Sept. 2015 = 900m x /12 x = 467.8
6
1 Oct. – 31 March 2016 = 2,700 x /12 = 1,350
1,818

(b) Report
To: Mr Hamad

From: Management Accountant

Date: 15 April 2016

Subject: Evaluating the changes in EPS of Aircon Ltd


The key factors which has led to changes in the EPS of Aircon Ltd. are as follows:
Revenue and profitability. Revenue increased by Rs. 2,700 million (18%) last year, but
the gross profit and net profit ratios have not increased proportionately.
The gross profit percentage fell from 40% to 37% in 2016, while the net profit
percentage remained constant at 10%.
Factors responsible for the decline might be due to the inability of the entity to maintain
good profit margin coupled with the failure to also maintain good control over operating
expenses.
The more funds realised from the rights issue did not lead to any significant increase in
return on capital employed which fell from 43% (2,880/6,606) in 2015 to 25%
(3,240/12,780) in 2016.
Capital employed: raising over Rs. 5,760 million of new finance was largely used to
acquire intangible assets.
It is hoped that this asset will start generating substantial returns in the near future.
EPS has therefore fallen from Rs. 1.69 in 2015 to Rs. 1.01 in 2016.

© Emile Woolf International 331 The Institute of Chartered Accountants of Pakistan


Advanced accounting and financial reporting

Signed
Management Accountant
APPENDIX TO THE REPORT
The ratios that are relevant to discussion and evaluation of changes in EPS of Aircon Ltd
are those that relate to profitability and return on capital employed.
The effect of the rights issue should also be considered in the discussion in relation to how
the funds raised through the shares were employed.
TABLE OF RATIOS
(i) Change in revenue 18 ,000  15 ,300
= x 100 = 18% Increase
18 ,000

2016 2015
(ii) Costs of sales/revenue 11 ,340 6 ,120
= 63% = 40%
18 ,000 15 ,300

(iii) Gross profit % 6 , 600 6 ,120


= 37% = 40%
18 ,000 15 ,300

(iv) Net profit % 1,854


= 10% = 10%
18 ,000

(v) Operating expenses % 3 , 420 3 , 420


= 19% = 22%
18 ,000 15 ,300

(vi) Interest payable/sales 540 576


= 3% = 4%
18 ,000 15 ,300

(vii) Taxation/sales 846 720


= 5% = 5%
18 ,000 15 ,300

(viii) Capital employed 3 , 240 2 ,880


=25% = 43%
9 ,180  3 , 600 3 ,006  3 ,600

(ix) Assets/turnover 18 ,000 15 ,300


= 1.41 = 2.32
12 ,780 6 , 606

Relevance of EPS to shareholders


(i) The EPS is used to compute the price earning (P/E) ratio, a major market indicator
to determine how successful a company has been operating.
(ii) The price earning figure is a multiple of the EPS, where the multiple represents the
number of years’ earnings required to recoup the price paid for the share.
(iii) Rising trend in EPS is a more accurate performance indicator than rising trend in
profit after tax. The investor should consider the future economic conditions of an
entity with some other ratios such as dividend cover and ROCE.
(iv) EPS is a measure of performance from the existing and potential investors’
perspective.
(v) EPS show the amount available to each ordinary shareholder thereby indicating the
potential returns on individual investment.
(vi) EPS is used to compare the activities of two entities in the same industry.

© Emile Woolf International 332 The Institute of Chartered Accountants of Pakistan


Answers

28.2 CACHET LTD


BASIC EPS DILUTED
EPS
i. No change in share capital
PAT - Pref Div 69,000 1,380
= 20,700
No. of shares
= 3.27 N/A
───── ────
ii. Bonus issue on 30 Sept. 2016:
No. of shares before bonus issue 20,700
Bonus (1 for 4) 5,175
─────
No. of shares after bonus issue 25,875
─────
PAT - Pref Div 69,000 1,380
=
No. of shares
25,875
= 2.61 N/A
───── ────
iii. Rights issue on 1 Oct. 2016
Before rights issue 5 shares 1.80 20,700 9.00 37,260
Rights issue (1 for 5) 1 share 1.20 4,140 1.20 4,968
─── ───── ──── ─────
After rights issue 6 shares 3.00 24,840 10.20 42,228
─── ───── ──── ─────
Theoretical ex-right price (Rs. 10.20/6) 1.70 1.70
──── ─────

20,000  /1.7 = 21,176


1.8
Bonus element of issue increases shares to
20,700  /5 = 24,840
6
Full price element of issue increases shares to
Weighted average number of shares in issue
21,176  /12
9
15,882
24,840  /12
3
6,210
22,092

EPS
PAT - Pref Div 69,000 1,380
= .
No. of shares
22,

28.3 MARY
Rs.
2 existing shares have a cum rights value of (2  Rs. 4) 8
1 new share is issued for 1
––
3 new shares have a theoretical value of 9
––

Theoretical ex-rights prices = Rs. 9/3 = Rs. 3

© Emile Woolf International 333 The Institute of Chartered Accountants of Pakistan


Advanced accounting and financial reporting

Weighted
average
Number Time Bonus Rights
number of
of shares factor fraction fraction
Date shares
1 January Brought
forward
5,000,000 1/12 6/5 4/3 666,667
1 February Bonus issue
(1 for 5)
1,000,000
–––––––––

6,000,000 2/12 4/3 1,333,333


1 April Rights issue
(1 for 2)
3,000,000
–––––––––

9,000,000 2/12 1,500,000


1 June Issue at full
market price

800,000
–––––––––

31 December Carried 9,800,000


forward
7/12 5,716,667
––––––––––

9,216,667
––––––––––

Earnings for Year 5 are (3,362,000 – 600,500 – 800,000) Rs. 1,961,500

EPS Year 5 = 1,961,500/9,216,667 = Rs.0.21 or 21 paisa

EPS Year 4 (adjusted) = Rs.0.32 × 3/4 × 5/6 = Rs.0.20 or 20 paisa

28.4 MANDY
Adjusted total earnings

Rs. Rs.

Reported earnings 2,579,000 1,979,000

Add back interest saved

(1,000,000  7%) 70,000


(1,000,000  7%  9/12) 52,500

Minus tax at 30% (21,000) (15,750)

49,000 36,750

Adjusted total earnings 2,628,000 2,015,750

© Emile Woolf International 334 The Institute of Chartered Accountants of Pakistan


Answers

Number of shares
Year 4 Number of
shares
1 January Brought forward 5,000,000
Dilutions:
Share options (W) 200,000
Convertible shares (1,000,000 ÷ 100  30) 300,000
––––––––––
31 December 5,500,000
––––––––––

Year 3 Weighted
average
Number of Time number of
Date shares factor shares
1 January Brought forward 5,000,000
Share options: dilution (W) 125,000
5,125,000 3/12 1,281,250
1 April Convertibles: dilution 300,000
5,425,000 9/12 4,068,750
5,350,000
Diluted EPS
Year 4 = 2,628,000/5,500,000 = Rs.0.48 or 48 paisa
Year 3 = 2,015,750/5,350,000 = Rs.0.38 or 38 paisa
Working
Cash receivable on exercise of all the options = 500,000 × Rs. 3 = Rs. 1,500,000

Year 4
Number of shares this would buy at full market price in Year 4 = Rs. 1,500,000/5 = 300,000
shares
Shares
Options 500,000
Minus number of shares at fair value (300,000)
––––––––
Net dilution 200,000
––––––––

Year 3
Number of shares this would buy at full market price in Year 3 = Rs. 1,500,000/4 = 375,000
shares
Shares
Options 500,000
Minus number of shares at fair value (375,000)
––––––––
Net dilution 125,000
––––––––

© Emile Woolf International 335 The Institute of Chartered Accountants of Pakistan


Advanced accounting and financial reporting

28.5 AAZ LIMITED


a) Step 1: Ranking in order of dilution

Increase in
Earnings per
Increase in no. of
incremental Rank
earnings ordinary
shares
shares
Rs. Rs.
Convertible Debentures
Increase in earnings
(Rs. 7.5m x 70%) 5,250,000 1.75 3
Increase in shares 3,000,000
Convertible Preference
Shares
Increase in earnings
2,450,000 0.61 2
Increase in shares 4,000,000
Options
Increase in earnings -
Increase in shares - 1
(1.5m x 1.1 / 11) 150,000

Step 2: Testing for dilutive effect


Profit from
operations
attributable to
ordinary Ordinary
shareholders Shares EPS Effect
Rs. Rs.
Basic Earnings per share *125,380,000 85,220,000 1.471 -
Options (Rank 1) - 150,000
125,380,000 85,370,000 1.469 Dilutive

Convertible preference
shares (Rank 2) 2,450,000 4,000,000
127,830,000 89,370,000 1.430 Dilutive

Convertible debentures
(Rank 3) 5,250,000 3,000,000
Anti-
133,080,000 92,370,000 1.44 Dilutive
*Rs. 127,830,000 – Rs. 2,450,000 = Rs. 125,380,000

© Emile Woolf International 336 The Institute of Chartered Accountants of Pakistan


Answers

b) AAZ Limited
Notes to the financial statements for the year ended December 31, 2016

EARNINGS PER SHARE


2016
Basic alternative to ordinary share holders
Profit (Rupees) 125,383,000

Weighted average number of ordinary shares outstanding


during the year 85,220,000

Earnings per share - basic (Rupees) 1.47

Diluted
Profit after taxation (Rupees) 127,833,000

Weighted average number of ordinary shares, options and


convertible preference shares outstanding during the year 89,370,000

Earnings per share - diluted (Rupees) 1.430

Because diluted earnings per share is increased when taking the convertible
preference shares into account (from Rs. 1.430 to Rs. 1.44), the convertible
debentures are anti-dilutive and are ignored in the calculation of diluted
earnings per share.

28.6 ABC LIMITED


ABC Limited
Notes to consolidated financial statements for the year ended March 31, 2016
2016
Rs. in
'000
Earnings per share basic
Profit after tax and non-controlling interest (15,000-2,000) 13,000
Dividend paid during the year to ordinary shareholders
-
(Rs. 4,000)
10% Cumulative preference dividend for 2015 (Rs. 2,000) -
10% Cumulative preference dividend for 2016 (2,000)
Dividend declared on 12% non-cumulative preference shares for
2016 (2,400)
Profit available for distribution to ordinary share holders 8,600

Diluted earnings per share


Profit available for distribution to ordinary share holders 8,600
Effect of dividend declared on 12% non-cumulative preference
shares convertible into ordinary shares on or before December 31,
2017 2,400
11,000

© Emile Woolf International 337 The Institute of Chartered Accountants of Pakistan


Advanced accounting and financial reporting

Weighted average number of ordinary shares W1 13,146


12% Non-cumulative preference shares convertible to
ordinary shares on or before December 31, 2017 W3 1,771
Weighted average number of ordinary shares - diluted 14,917
Antidiluted earning per share Rs. 0.74
W1: Weighted average ordinary shares outstanding for "Basic EPS"

Bonus Weighted
Number of Time average
Date fractions
shares factor number of
(W3) shares
1 April 2015 to 30 June  6/5
2015 10,000,000 × 3/12 1.00833 3,024,990
1 July
Conversion of cumulative
prefs at a premium of Rs.
2 per share
(500,000  10/12) 416,667
 6/5 
1 July to 30 September 10,416,667 × 3/12 1.00833 3,151,031
1 October
Rights issue 1,200,000

30 September to 31
December 11,616,667 × 3/12  6/5 3,485,000
1 January
Bonus issue (20%) 2,323,333
1 January to 31 March 13,940,000 × 3/12 3,485,000
Weighted average 13,146,021

W2: Calculation of bonus adjustment factor


No. of Rs. in
@ Rs.
shares '000
Bonus element with right issue
Outstanding shares before the exercise of rights at
fair value 10,417 12.50 130,213
Rights issued at a premium of Rs. 1.5 1,200 11.50 13,800
11,617 144,013

Rs.
Actual cum rights price per share 12.5000
Theoretical ex-right value per share (144,013/11,617) ÷ 12.3967
Adjusting factor 1.00833

Bonus issued on January 01, 2016 (20%)


Adjusting factor (6 shares for 5 shares) 1.2

© Emile Woolf International 338 The Institute of Chartered Accountants of Pakistan


Answers

W3: Diluted EPS

Number of Earnings
shares (Rs.) EPS (Rs.)
Basic EPS 13,146,021 8,600,000 0.65
Dilution:
Non-cumulative prefs in issue for the
year (W4)at a premium of Rs. 2 per
share (for the whole year)
2,000,000  10/12  12/12 1,666,667
Add back dividend paid to non-
cumulative prefs in issue at the year-
end 2,400,000

Non-cumulative prefs actually


converted in the year (for the part of
the year before conversion)
(500,000  10/12)  3/12
i.e. 416,667  3/12 104,167
1,770,834
Adjusted figures 14,916,855 11,000,000 0.74

Diluted EPS: Rs. 11,000,000/14.917 million = Rs.0.74 per share


The non-cumulative preference shares are anti-dilutive

W4: Non-cumulative prefs in issue at the year-end


This can be found from the information about the dividend.
Rs. 2,400,000 is 12% of the nominal value of the shares.
Therefore, the nominal value is Rs. 20,000,000 (Rs. 2,400,000/0.12).
Therefore the number of shares (at Rs. 10 per share) is 2,000,000

28.7 ALPHA LIMITED


Alpha Limited
Extracts from consolidated profit and loss account for the year ended 31 December
2013
Rs. in '000
Profit for the year W.1 (49,462.16+26,950) 76,412.12
Profit attributable to
• Owners of Alpha Limited 76,412.12-5,390 71,022.12
• Non-controlling interest 26,950*20% 5,390
76,142.12
Earnings per share: Rupees
• Basic W.2 72.10
• Diluted W.2 53.39

© Emile Woolf International 339 The Institute of Chartered Accountants of Pakistan


Advanced accounting and financial reporting

W-1 Profit for the year AL ZL


(Rs. in '000)
Profit after tax 60,000.00 25,000.00
Cash dividend received from ZL (net of tax)
• Final dividend for 2012 (35,000*15%*80%)*90
% (3,780.00)
• Interim dividend for 2013 (35,000*1.16*12%*80%
)*90% (3,507.84)
FV gain on ZL's investment [67,000-
property (40.35) (59,000+5,000)]*65% 1,950
Cost of defined benefit gratuity (8,000-
sch. (19.120) 3,000)*65% (3,250.00)
49,462.16 26,950

W-2 Basic / diluted EPS:


Basic
Weighted Basic/
/Diluted
average Diluted
earnings
shares in EPS
(Rs. in
'000 (Rs.)
'000)
Weighted average No. of shares:
1-Jan- Balance
2013 80,000/100, 800×7/12 800
1-Jan- Bonus issue at 20%
2013 (800*20%), 160×7/12 160
960

1-Aug- Shares issued under


2013 employees' share option
scheme
(60*5/12) 25
(960+60)×5/12
Basic earnings per share (EPS) 985 71,022 72.10
Shares from assumed conversions:
1-Aug- Convertible 12% bonds (5
2013 shares for 4 bonds)
(30,000/100*5/4),
(30,000*0.12*0.65) 375 2,340
1-Aug- Shares for no consideration
2013 issued under employees'
share option.
(250-150)/250*60*7/12 (IAS
33.45) 14 -
Diluted earnings per share (EPS) 1,374 73,362 53.39

© Emile Woolf International 340 The Institute of Chartered Accountants of Pakistan


Answers

CHAPTER 29: ANALYSIS AND INTERPRETATION OF FINANCIAL STATEMENTS

29.1 ALPHA LIMITED AND OMEGA LIMITED


(a)
Alpha Limited Omega Limited
(i) Current ratio
Current assets 620,000 504,000
Current liabilitie s 240,000 440,000
= 2.58:1 = 1.15:1
(ii) Acid test
Current assets - stock 340,000 332,000
Current liabilitie s 240,000 440,000
= 1.42:1 = 0.75:1
(iii) Creditors ratio
Average creditors 180,000 344,000
 365  365  365
Purchases or cost of sale 1,120,000 1,342,000
= 59 days = 94 days
Collection period/Receivables
(iv)
Ratio
Average debtors 310,000 300,000
 365  365  365
Sales 1,440,000 1,720,000
= 79 days = 64 days
(v) Earnings per share
PAT 60,000 48,000
No. of ord. shares 600,000 200,000
= 0.1/share = 0.24/share
(b) Comments on comparative analysis of the two companies.
Based on the ratios computed above:
(i) In terms of working capital and liquidity, Alpha Limited is in a better position to
honour its obligations as they fall due because its current ratio and acid test ratio are
higher than those of Omega Limited.
(ii) Omega Limited’s payment period is better than that of Alpha Limited’s because
Omega Limited uses supplier’s funds to finance its operation.
(iii) Omega Limited’s collection period is also better than that of Alpha Limited. It
extends shorter credit period to its customers than Alpha Limited.
(iv) Omega Limited’s credit policy is better than that of Alpha Limited. This is because
there is 30 days difference between its payments period and collection periods
compared with Alpha Limited that had a longer collection period than its payment
period.
(v) Omega Limited’s EPS is better than that of Alpha Limited by 0.14 (0.24 – 0.10).
Omega Limited’s shareholders will be happier than those of Alpha Limited.

© Emile Woolf International 341 The Institute of Chartered Accountants of Pakistan


Advanced accounting and financial reporting

29.2 COOK LIMITED


(a) The ratios computed from Cook Limited financial statement for the years ended 31
December, 2015 and 2016 revealed that the company’s profitability has fallen in 2016 in
comparison with the performance in 2015.
(i) The gross profit percentage of 60% in 2015 has decreased to 40% in 2016. The net
profit percentage has also fallen from 33.6% in 2015 to 11.07% in 2016. The fall in
the profitability of the company might be due to the new sales manager’s price-
reduction policy.
(ii) The return on capital employed also dropped from 30.5% in 2015 to 17.5% in 2016.
The increased market resulting from the new sales policy did not translate into
increased profitability as hoped for.
(iii) The company appears solvent. There is no shortage of liquid assets.
Considerable amount of fund is being tied down with 76 days credit period given to
receivables under the new sales policy adopted in 2016 as compared to 34 days in 2015.

Profitability ratios 2016 2015


(i) Gross profit percentage 2,240 1,080
  100   100
Gross Profit 5,600 1,800
  100  60 %
 40 %
Sales

(ii) Net profit percentage 620 604


  100   100
Net Profit 5,600 1,800
  100  11 . 1 %  33 . 6 %
Sales

(iii) Return on capital employed


620  384 604  24
Profit before loan interest   100   100
(ROCE)  2,534  3,200 1,858  200
Shareholde r' s funds plus
 17 . 5 %  30 . 5 %
long  term loans

(iv) Return on equity or Net Profit 620  1 00 604


  100   100
Equity 2,53 4 1,858
 28 . 4 %  32 . 5 %

Liquidity ratios 2016 2015


(i) Current ratio
1,642 250
Current Assets   4 . 41 : 1   3 .5 : 1
 372 72
Current Liabilitie s

(ii) Acid test ratio


1,642  476 250 - 60
Current Assets - Stock   3 :1   2 .6 : 1
 372 72
Current Liabilitie s

(iii) Receivables collection period


1,166 166
 x 365  76 days   365  33 . 6 %
Trade Debtors
 x 365 5,600 1,800
Credit Sales

(iv) Creditors payment period


350 72
  365  38 days   365  36.5 days
Trade Creditors 720
  365 3,360
Credit Purchase

© Emile Woolf International 342 The Institute of Chartered Accountants of Pakistan


Answers

(b) Amount of cash that should be released

1,166
Receivables collection period   365  76 days
5,600

If the collection period is reduced to 45 days,


The new receivables figure would be:-
45
 Rs. 1,166m = Rs. 690m
76

The amount that would be released


= Rs. (1,166 - 690)m = Rs. 476m
or
Receivables collection period in 2016

1,166
=  365  75.99 days  76 days
5,600

Imposition of 45 days

x x
45 days = 365
5,600 1

365x
45 days =
5,600

365x = 252,000
252,000
x=
365

x = Rs. 690m
Amount of cash to be released will be
= (Rs. 1,166 – Rs. 690)m = Rs. 476m

29.3 FITZROY LIMITED


Statement of financial position as at 31 December, 2016

Rs.’000 Rs.’000 Rs.’000


Non-current assets
Land & Building 5,000
Furniture & Fittings 1,450
Motor vehicles 2,300
8,750
Current assets
Inventory 1,800
Receivables (W3) 1,650
Cash (W5) 300
(W1) 3,750

© Emile Woolf International 343 The Institute of Chartered Accountants of Pakistan


Advanced accounting and financial reporting

Rs.’000 Rs.’000 Rs.’000


Less: Current liabilities
Creditors (W9) 900
Dividend (W10) 600
1,500
Net current assets 2,250
Total assets less current liabilities (balance) 11,000
Financed by: Rs.’000
Ordinary shares (W11) 5,100
7½ Debenture (W12) 3,800
Reserves:
Retained profits (balance) 2,100
11,000

Profit and loss accounts for the year ended 31 December 2016

Note Rs.’000
Turnover (W5) 12,925

Operating profit before interest & tax (W6) 2,585


Interest expense (W13) (285)
Profit before Tax 2,300
Taxation (given) (400)
Profit after tax 1,900
Dividend – ordinary (W10) (600)
Retained profit for the year 1,300
Retained profit b/f (given) 800
Retained profit c/f 2,100
Workings

All figures are in thousand

(W1) Non-current assets = 70% of total assets

8750 100
 Total assets = x
70 1

= Rs. 12,500

Current assets = Rs. 12,500 – 8,750 = Rs. 3,750

Current Assets 3750 2 .5


(W2) Current ratio = = =
Current Liabilitie s CL 1

3750 100
 Current liabilities = x = Rs. 1,500
2 .5 1

© Emile Woolf International 344 The Institute of Chartered Accountants of Pakistan


Answers

Cash 0 .2 Cash
(W3) Cash ratio = = =
CL 1 1500

 Cash = Rs. 300

(W4) Receivables = (3,750 – 1,800 – 300) = Rs. 1,650

Debtorsx 365
(W5) Average collection period =
Sales

1650 x 365
46.596 =
Sales

1650 x 365
Sales = = Rs. 12,925
46 . 576

(W6) Profit margin = 1/5  Gross profit = Rs. 12,925 x 1/5

= Rs. 2,585

(W7) Cost of sales = Rs. 12,925 – Rs. 2,585 = Rs. 10,340

(W8) Cost of sales = Opening inventory + purchases – closing inventory

Rs. 10,340 = 0 + purchases – 1800

Purchases = 10,340 + 1800 = Rs. 12,140

Trade creditors 365


(W9) Creditors payment period = 
Purchases 1

27 . 06 x 12140
Trade creditors = = Rs. 900
365

(W10) Current liabilities = Trade creditors + Dividend

 Dividends = 1,500 – 900 = Rs. 600

600
(W11) Ordinary shares = = Rs. 5,100
0 . 1176

Total cash flow


(W12) Cash flow ratio =
Total debts

Total cash flow = profit + depreciation

2585  972
i.e. 0.6711 =
Totaldebts

3557
Total debts = = 5,300
0 . 6711

 7½% debenture = 5,300 – 1,500 = Rs. 3,800

(W13)  Interest expenses 7½% x 3800 = Rs. 285

© Emile Woolf International 345 The Institute of Chartered Accountants of Pakistan


Advanced accounting and financial reporting

29.4 TRAVELWELL LTD


(a) Ratios
2016 2015
Return on year-end capital employed (24 + 9.75)
/(144 + 130) 12.3% 8.5%
Net asset turnover 310 1.2 times 1.5 times
/(144 + 130)
Gross profit margin (given) 20% 16.7%
Net profit margin (before tax) 24 8.0% 5.7%
/300
Current ratio 46 1.3 3.0
/36
Inventory holding period (29/240)  365 44 days 35 days
Trade receivables collection period (17/300)  365 21 days 16 days

Trade payables payment period (28/240)  365 43 days 31 days

Gearing ratio 130 47.4% nil


/(144 + 130)
(b) The acquisition of Rondel Ltd’s net assets
At the beginning of the year Travelwell Ltd acquired the net assets of Rondel Ltd. The
revenue contributed by the purchase of these assets was Rs. 90 million, which explains
the entire increase of 43% in sales revenue for Travelwell Ltd for the year.
The acquisition also added Rs. 40 million to gross profit, and without this the gross profit of
the company would have fallen in 2016 compared with the previous year. The gross profit
(60,000 – 40,000)
margin would have been /(300,000 – 90,000) = 9.5%, substantially lower than the
16.7% achieved in the previous year.
Clearly, the Chief Executive Officer has selected ratios and other performance
measurements that suggest excellent performance; however, his report is misleading
because it fails to explain the effects of the acquisition of Rondel Ltd’s net assets. The
acquisition cost Rs. 130 million which was the equivalent of 92% of Travelwell Ltd’s capital
employed just before the acquisition occurred.
A more appropriate analysis of performance and financial position should consider a wider
range of ratios and should also allow for the effects of the acquisition of Rondel Ltd’s
business.
Return on capital employed
ROCE improved from 8.5% in the previous year to 12.3% in the current year, but the
improvement is attributable to the acquisition of the net assets of Rondel Ltd. There were
no disposals of non-current assets during the year. This means that the year-end net
assets of Rondel Ltd’s business that have been incorporated in the Travelwell Ltd
statement of financial position can be estimated as follows:
Rs.000
Net assets at acquisition (balancing figure) 118,000
Goodwill 12,000
Cost of the acquisition (given) 130,000
Plus post-acquisition increase in net assets:
Pre tax profit for the year (given) 29,000
Taxation (@25%) (7,250)
21,750
Net assets at 30 September 2016 151,750

© Emile Woolf International 346 The Institute of Chartered Accountants of Pakistan


Answers

29
This suggests that the ROCE from Rondel Ltd’s business was 19.1% ( /151.75). The high
ROCE from Rondel Ltd’s business will explain the rise on ROCE for the company s a
whole.
Profitability
As indicated earlier, gross profit would have fallen in 2016 but for the gross profit
contributed by the net assets of Rondel Ltd and the gross profit from the ‘original’ business
of Travelwell Ltd was lower in 2016 than in 2015 (9.5% compared with 16.7%).
There would also have been a loss of Rs. 5 million before tax except for the profit of Rs. 29
million contributed by Rondel Ltd’s business.
It is likely however that the finance costs of Rs. 9.75 million in the current year, resulting
from the issue of the loan notes, were due to a need to borrow to acquire the assets of
Rondel Ltd. If so, it would be more appropriate to assess the profit before tax from Rondel
Ltd’s business as Rs. 19.25 million (= Rs. 29 million – Rs. 9.75 million finance charge) and
the profit before tax from Travelwell Ltd’s other business as Rs. 4.75 million (= Rs. 5 million
loss + Rs. 9.75 million).
profit before tax
Using these adjusted figures, this suggests that the /sales ratio for Travelwell Ltd’s
4.75
other business was 2.3% (= /(300 – 90), which is much worse than the previous year.
Net asset turnover
Net asset turnover fell in 2016 to 1.1 times compared with 1.5 times in 2015. The net asset
turnover from the business of Rondel Ltd was only 0.59 times (= Rs. 90 million/Rs. 151.75
million), which means that the acquisition of the net assets of Rondel Ltd contributed
significantly to the fall.
Financial position
The change in the financial position of Travelwell Ltd can be assessed by looking at the
gearing ratio and working capital ratios.
Gearing
At 30 September 2015, Travelwell Ltd had no gearing. Gearing was 47.4% one year later.
This is due to the issue of the loan notes, presumably to contribute towards financing the
acquisition of Rondel Ltd’s not assets. Higher gearing creates greater financial risk, in the
sense that any fall in profits before interest will have a much greater proportional effect on
earnings per share.
Liquidity
The current ratio has fallen from 3.0 times to 1.3 times. The fall is attributable largely to the
change from having a bank balance of Rs. 28 million at 30 September 2015 to a bank
overdraft of Rs. 2 million one year later. This net cash outflow of Rs. 30 million is exactly
equal to the cash used to acquire the net assets of Rondel Ltd (= Rs. 130 million cost
minus loan notes issued Rs. 100 million). The fall in liquidity is therefore possibly not a
matter of concern.
Working capital
There has been an increase in the inventory turnover period from 35 to 44 days, but this is
largely offset by the increase in the trade creditors payment period from 31 to 44 days
(since the values of inventory and trade payables in the statement of financial position are
roughly equal). There has been a slight increase in the average collection period by trade
receivables. On balance, the change in working capital has not affected the financial
position of the company significantly.
Dividends
The company paid a dividend for the year of Rs. 15 million, up from Rs. 12 million the
previous year (Rs. 15 million 
100
/125). Since profit after tax is Rs. 18 million, dividend
cover is just 1.20 times, which is quite low, and retained profits are only Rs. 3 million. It
might therefore be argued that the 25% increase in dividends was perhaps excessive.

© Emile Woolf International 347 The Institute of Chartered Accountants of Pakistan


Advanced accounting and financial reporting

Conclusion
The acquisition of the net assets of Rondel Ltd appears to have contributed very
favourably to the financial performance of Travelwell Ltd. However the contribution from
Rondel Ltd’s assets should not hide the fact that there has been some deterioration in the
performance of Travelwell Ltd’s other business. This is a problem that management need
to consider.

29.5 SACHAL LIMITED


(a) To: Board of Directors
From: Chief Financial Officer
Date: December 8, 2017
Subject: Financial and Operating Performance of Waris Limited
As requested, I have analyzed the financial performance of Waris Limited (WL) with
the industry with a view to evaluate the feasibility of launching a takeover bid. My
analyses of each category of ratios is as follows:
Profitability Ratios
The gross profit ratio is near to the highest while the operating profit is near to the
lowest as compared to similar companies. It indicates that key issue which is
affecting WL’s profitability is its lack of control over operating expenses. The positive
aspect of this situation is that we may be able to improve the profitability just by
controlling the operating expenses without being required to make significant
changes in the current operations of WL.
Return on shareholders’ equity is around the average prevailing in the industry. This
ratio is obviously, related to operating profit and as discussed above it can be
improved by exercising greater control over operating expenses, after take over.
Working Capital Ratios
WL’s working capital ratios specially the current ratio indicates that the company’s
liquidity position is in line with the industry average. Hence, it seems that the
company’s working capital is being appropriately managed although there may be
some room for improvement.
The inventory turnover is among the lowest in the industry which shows that sound
inventory management policies are in place.
However, the level of receivables is among the highest in the industry. The possible
causes of the situation may be as follows:
 Poor efforts in making collections
 Lack of proper credit control policies or slackness in their implementation.
 Chances of bad debts which may not have been provided.
 Sales to related parties.
 Fictitious sales.
We need to seek appropriate explanations and investigate the matters if possible.
Gearing Ratios
The debt equity ratio is on the higher side but can be restructured after acquisition.
However, the interest cover is only 1.3. It is among the lowest in the industry and is
indicative of a high degree of risk as the profits are barely able to cover the interest
charges. Even a slight decline in the profitability of the company may have highly
adverse impact on the company’s bottom line.

© Emile Woolf International 348 The Institute of Chartered Accountants of Pakistan


Answers

Investor Ratios
Earnings per share is on the lower side. However, it can be improved by improving
profits as discussed while comparing performance ratios. WL’s dividend payout is the
lowest (22.2%) in terms of percentage among other similar companies. Generally,
past history of dividend payouts is not relevant to our bid decision. However, low
dividend may also be on account of liquidity problems and we should consider this
aspect.
Conclusion
The company’s performance indicates a mixed trend. However, it may be concluded
that below average performance, (wherever applicable) can be improved by revisiting
the situation and bringing about necessary changes in the policies.
(b) Following additional information could have been useful for a better analysis of the
situation:
(i) Any recent audited or management accounts.
(ii) Comparison of accounting policies following by the companies in the same
industry and the possible impact thereof on the above ratios.
(iii) Expected growth in future earnings
(iv) Alternative investment opportunities
(v) Effect of synergy
(vi) WL’s market reputation;
(vii) Quality of human resource within the company;
(viii) Research and development activities
(ix) Legal framework and industry risks
The figures given in the question suggest that company had the funds in addition to
sale proceeds to pay for cost associated with PIB investment. Therefore, Present
Value has been taken as Rs. 104,641,483 (Rs. 100,000,000 + Rs. 4,641,483).

29.6 OPAL INDUSTRIES LIMITED


Accounting treatment for various investments in first separate financial statements
In accordance with IAS 27, in separate financial statements, investments in subsidiaries, joint
ventures and associates should be valued either at cost or fair value in accordance with IFRS 9.
As OIL is preparing its first separate financial statements for the year ended 30 June 2014, there
is a change in accounting policy as investments in AL, an associate company, would be treated
in 2014 at cost as against the previous basis of equity accounting. Accordingly, comparative
figures would be restated to incorporate this change in accounting policy.
(i) Opal Industries Limited
Statement of changes in equity for the year ended 30 June 2014
Retained earnings
Rs. in million
Balance as at 30 June 2012 465.00
Profit for the year - restated W-1 1,251.60
Final dividend for the year ended 30 June 2012:
- Cash dividend at 20% 2,500×20% (500.00)
- Bonus issue at 15% 2,500×15% (375.00)
Balance as at 30 June 2013 - restated 841.60
Profit for the year W-1 1,454.80
Final cash dividend at 25% for the year ended 30
June 2013 2,875×25% (718.75)
Balance as at 30 June 2014 1,577.65

© Emile Woolf International 349 The Institute of Chartered Accountants of Pakistan


Advanced accounting and financial reporting

(ii) Opal Industries Limited


Notes to the financial statements or the year ended 30 June 204
1 - Long term investments:
2013 2014 2014 2013 (Restated)
Description
Number of shares Rs. in million
Subsidiary and associated
companies-at cost
- 975,000 GL 195.00 -
240,000 240,000 AL 50.00 50.00

Others - Available-for sale


BL
70,000 70,000 7.70 11.20
(70,000×110), (70,000×160)
310,000 1,285,000 252.70 61.20

The company holds 65% and 30% and 10% ownership interest in GL, AL and BL
respectively.
W-1: OIL profit for the year after taking effect of investee companies:
2013
2014
(Revised)
Rs. in million
Profit for the year 1,450.00 1,260.00
AL - Associated company:
Reversal of previously booked profit (28×30%) - (8.40)
Dividend for the year ended 30 June 2013
(80×30%×16%) 3.84 -
BL - Available for sale:
Dividend for the year ended 30 June 2013
(70×10%×18%) 1.26
(7.7-8) OR (11.2-8)-(7.7-
Investment impairment 11.2) (0.30) -
1,454.80 1,251.60

© Emile Woolf International 350 The Institute of Chartered Accountants of Pakistan


Answers

CHAPTER 30: SUNDRY STANDARDS AND INTERPRETATIONS

30.1 GUJRANWALA FOODS LIMITED


(a)
Chickpea Apricot Dates Onion Total
Rs.’000 Rs.’000 Rs.’000 Rs.’000 Rs.’000
Sales: 200 300 750 250 1,500
Add: Subsidies 40 80 60 60 240
Own consumption 45 40 75 20 180
285 420 885 330 1920
Cost of sales
Opening inventory 40 40 70 - 150
Add: Purchases 75 150 300 80 605
Closing inventory (60) (110) (300) (60) (530)
55 80 70 20 225
Gross output value 230 340 815 310 1,695

Less expenses:
Casual labour 16
Regular workers 24
Land preparation 64
Hire of tractors 48
Depreciation: irrigation 80
Depreciation: farms equipment 60
(292)
1,403

Workings:
12
Casual labour /15 x Rs. 20,000 = Rs. 16,000
12
Regular workers /15 x Rs. 30,000 = Rs. 24,000
12
Land preparation /15 x Rs. 80,000 = Rs. 64,000
12
Hire of tractors /15x Rs. 60,000 = Rs. 48,000

Depreciation:
00,000
Irrigation cost
9
Farm equipment 20% x 400,000 x /12 = Rs. 80,000
(b) (i) Biological assets are living plants and animals.
(ii) Biological transformation relates to the process of growth and degeneration that
can cause changes of a quantitative or qualitative nature in a biological asset.
(iii) Harvest is the detachment of produce from a biological asset or cessation of a
biological asset’s life process.

© Emile Woolf International 351 The Institute of Chartered Accountants of Pakistan


Advanced accounting and financial reporting

30.2 WAH AGRIPROD LTD


(a) Wah Agriprod Ltd: Statement of profit or loss and other comprehensive income for the year
ended 31 December 2016.
Notes Rs.‘000
Revenue (wi) 851,400
Cost of sales (wii) (495,532)
Gross profit 355,868
Distribution cost (31,950)
Administrative expenses (wiii) (79,100)
Profit from operations 244,818
Investment income 18,250
Finance cost (1,020)
Fair value loss on financial instrument
(40,500 – 39,700) (800)
Profit before tax 261,248
Income tax expenses (86,750)
Profit for the year 174,498
Other comprehensive income:
Fair value gain on leasehold property 12,333
Total comprehensive income for the year 186,831

Working Notes:
Rs.
Revenue
(i) Rs. 855,000 – Rs. 3,600 value of returnable goods = 851,400
(ii) Cost of sales:
Opening inventory 85,075
Purchases 503,600
Depreciation of plant and equipment
(0.15 of Rs. 98,800 – Rs. 28,800) 10,500
Amortization of leased property
(Rs. 3,833 + Rs. 4/208 of Rs. 99,000) 5,237
Closing inventory
(Rs. 106,000 + 0.8 of Rs. 3,600) (note iv) (108,880)
495,532
(iii) Admin expenses .
Per the question 104,400
Less dividend = Rs. 115,00 x 2 x Rs. 2.2 x Rs.0.05 (25,300)
79,100

(iv) Closing inventory: As per question 106,000


Sales return at cost (80% of 3,600) 2,880
108,880

© Emile Woolf International 352 The Institute of Chartered Accountants of Pakistan


Answers

(b) Statement of changes in equity for the year ended 31 December 2016
Revaluati
Share Share on Retained
capital premium surplus earnings Total
Rs.’000 Rs.’000 Rs.’000 Rs.’000 Rs.’000
Balance as at 105,000 6,400 - 55,600 167,000
1/1/2016
Issue of 10,000 2,000 - - 12,000
shares
Profit for the - - - 174,498 174,498
year
Fair value gain - - 12,333 - 12,333
Realised - - (237) 237 -
during the
year
Dividend paid - - - (25,300) (25,300)
Balance 115,000 8,400 12,096 205,035 340,531
31/12/2016

Revaluation of leased property Rs.‘000 Rs.‘000


Cost as at 1/1/2016 125,000
Accumulated depreciation as at 1/1/2016 35,000
For the eight months to 1/9/2016:
8/12 of 125,000 ÷ 25 years 3,333 38,333
86,667
Value of leased property 99,000
(12,333)
(c) (i) A financial asset is any asset that is cash, an equity instrument of another entity or a
contractual right to receive cash or another financial asset from another entity; or to
exchange financial assets or financial liabilities with another entity under conditions
that are potentially favourable to the entity.
(ii) The classes of financial assess and how each is valued are as follows:
Classification Measurement
 Financial assets at fair value Fair value through profit or loss
through profit or loss
 Held-to-maturity investments Amortised cost
 Loans and receivables Amortised cost
 Available-for-sale Fair value through other
comprehensive income

(d) A biological asset is defined as a living animal or plant.


Entities are required to recognise biological assets or agricultural produce when and only
when, all of the following conditions are met:
(i) The entity controls the asset as a result of past events;
(ii) It is probable that future economic benefits associated with the asset will flow to the
entity; and
(iii) The fair value or cost of the asset can be measured reliably.

© Emile Woolf International 353 The Institute of Chartered Accountants of Pakistan


Advanced accounting and financial reporting

30.3 HELIOS GROUP


(a) Helios Ltd
Consolidated statement of financial position as at 31 December, 2016
Non-current assets Rs.’000 Rs.’000

Biological assets - Dairy livestock: Immature 40,000


- Dairy livestock: mature 50,000
- Plantation 20,000 110,000
─────
Property, plant and equipment
(600,000 + 450,000 + 60,000 – 40,000 – 50,000) 1,020,000
Investments (800,000 – 600,000) 200,000
Goodwill (W3) 260,000
───────
1,590,000
Current assets
Inventories (160,000 + 150,000 – 20,000) 290,000
Trade receivables (120,000 + 280,000) 400,000
Cash and cash equivalents (20,000 + 50,000) 70,000 760,000
───── ───────
2,350,000
───────
Equity and liabilities
Equity
Ordinary shares capital 160,000
Share premium 40,000
Group reserves (W5) 730,000
───────
Parent equity 930,000
Non-controlling interests (W4) 220,000
───────
Total equity 1,150,000

Non-current Liabilities
Loan notes (600,000 + 170,000) 770,000

Current liabilities
Trade payables (310,000 + 120,000) 430,000 1,200,000
───── ───────
Total equity and liabilities 2,350,000
───────

Workings (W)

(1) Group Structure


Helios Ltd
70%
NCI = 30%
Sol Ltd

© Emile Woolf International 354 The Institute of Chartered Accountants of Pakistan


Answers

(2) Net assets of Sol Ltd


Acquisition Reporting Post-
date date acquisition
Rs.’000 Rs.’000 Rs.’000
Ordinary shares 120,000 120,000 -
Share premium 20,000 20,000 -
Reserves 300,000 500,000 200,000
Fair value adjustment (land) 60,000 60,000
Fair value of net assets 500,000 700,000 200,000
(3) Goodwill in Sol Ltd
Rs.’000
Purchase consideration 600,000
Fair value of NCI at acquisition 160,000
760,000
Fair value of net assets at acquired (500,000)
Goodwill 260,000
(4) NCI at reporting date
Rs.’000
Fair value of NCI at acquisition 160,000
Share of post-acquisition reserves
(30% x Rs. 200,000) 60,000
220,000
(5) Group reserves
Rs.’000
Helios Ltd 590,000
Share of post-acquisition
(70% x Rs. 200,000) 140,000
730,000
(b) Measurement of harvested agricultural products. Agricultural products harvested from an
entity’s biological assets shall be measured at its fair value less costs to sell at the point of
harvest. Such measurement is the cost at that date when applying it as inventories or
another applicable standard.

30.4 FASHION BLUE ENTERPRISES


(a) Fashion Blue Enterprises: Statement reconciling the inventory balance
Recorded Physical
Balance Count
Rs. 000 Rs. 000
Balance prior to adjustment 73,410 71,400
Add: Goods sold, but not dispatched by 31 December 2017 300
Less: Goods held on behalf of third parties (200)
Add: Unrecorded purchases 410
Add: Goods purchased, in transit at 30 December 2017 400
Add: Goods at Sialkot undercasted 90
Less: Unrecorded purchase return (2,500)
Add: Adjustment of sales to Saleem now recorded at cost (780
 /130)
30
180
Less: Inventory shortfall / loss (balancing figure) (110)
(b) Value of inventory at 31 December 2017 71,690 71,690

© Emile Woolf International 355 The Institute of Chartered Accountants of Pakistan


Advanced accounting and financial reporting

No adjustment required for goods costing Rs. 310,000 sold on credit to Skims Industries Ltd.
The value of inventory that should be recorded in the Statements of financial position is Rs.
71,690 thousand

(c) Adjustments in the books of Fashion blue enterprises


Debit Credit
Rs. 000 Rs. 000
(i) Inventory 300
Cost of goods sold 300

(iii) Inventory 410


Account payable – Mustafa & Co. 410
Being: Correction of unrecorded purchases

(iv) Account payable (Ali Garments) 2,500


Inventory 2,500
Being: Correction of unrecorded purchases
return

(viii) Inventory account 180


Cost of sales 180
Being: Sales to Saleem recorded at sale price
instead of cost now adjusted

Part (a) Inventory losses / write downs (P&L) 110


Inventory 110
Being: Unexplained difference / inventory theft /
inventory damage

30.5 KHAN LIMITED


Khan Limited: Inventory Reconciliation Statement
Rs. Rs.
Inventory as per physical inventory at July 14, 2017 185,000
(i) Less: Purchases for July 1 to 14, 2017 included in the
physical inventory:
Payment against purchases 48,000
Adjustments to the above

Last year purchases (5,000)

Purchased and not delivered yet (6,000)


Purchased and returned July 07 (2,000) (35,000)
(ii) Add: Sales for July 1 to 14, 2017 excluded from the physical
inventory:
Collection against sales 60,000
Adjustments to the above

© Emile Woolf International 356 The Institute of Chartered Accountants of Pakistan


Answers

Khan Limited: Inventory Reconciliation Statement


Rs. Rs.
Last year sales (1,500)
Sold and not delivered (2,800)
Sold and returned (760)
54,940
(v) Sales on account 10,000
64,940
(iii) Profit element (25% of 64,940) (16,235)
Cost of sales during 1-14 July 48,705
(iv) No adjustment is required for purchase of goods on June
28 for Rs., 6,000, as already included in inventory on 14 July,
2017 
(vi) Adjustment of net realizable value not to be accounted for at
June 30, as damage pertains to subsequent period. Cost of
goods over and above NRV (9,000 – 6,000) 3,000
(vii) Error in carry forward of page total (1,000)
Error in casting 200
(viii) Goods held on consignment (2,200)
198,705

30.6 AFRIDI
Statement showing the amount of physical inventory as on March 31, 2017
Rs.
Inventory as on December 31, 2017 (W1) 140,025
Add: Purchases for the quarter (W2) 145,360
285,385
Less: Adjusted Cost of sales (W3) (100,345)
100
Less: goods given in charity ( /120 of Rs. 2,100) (1,750)
Physical inventory balance as on March 31, 2017 183,290
Working - 1
Inventory as on December 31, 2016
Inventory as valued previously 140,525
Add: Cost of 1,000 items recorded at Re. 0.50 per item instead of Rs. 10 per item. 9,500
150,025
Less: error in carry forward of a page total (10,000)
Actual inventory as on December 31, 2016 140,025

Working – 2
Purchases for the quarter ended March 31, 2017
Total of invoices from Jan. 01 to Mar. 31, 2017 as per purchased day book 138,560
Add: Goods purchased before march 31, 2017 but recorded in April 2017 37,000
Less : Invoices pertaining to Goods received before December 31, 2017 (28,000)
Less : Purchase of ceiling fan (2,200)
145,360

© Emile Woolf International 357 The Institute of Chartered Accountants of Pakistan


Advanced accounting and financial reporting

Working - 3
Cost of sales for the quarter
Total of sales invoices raised from January 01 to March 31, 2017 151,073
Add: Goods dispatched before March 31, 2017 but invoiced in April 2017 25,421
Less: Goods dispatched before December 31, 2016 but invoiced during the quarter
ended March 31, 2017 (38,240)
Less: sales return during the quarter (12,800)
Less: Sale invoice recorded twice (5,760)
Net sales 119,694
Add: Discount allowed (6,000 × 1.20 = 7,200 × 10%) 720
Sales before discount 120,414
Less: gross margin of 20% on cost (120,414*20/120) (20,069)
100,345

© Emile Woolf International 358 The Institute of Chartered Accountants of Pakistan


Answers

CHAPTER 31: IFRS 1: FIRST TIME ADOPTION OF IFRS

31.1 IFRS 1
(a) The first IFRS reporting period was the year ended 31 December 2016, and the date of
Transition to IFRS was 1 January 2015.
(b) The procedures which must be followed in order to prepare the Financial
Statements for the year ended 31 December 2016, are as follows:
(i) Choice of accounting policies to be included as part of notes to the Financial
Statements
(ii) Preparation of the opening IFRS Statements of Financial Position by applying the
following rules, except in cases where IFRS grants exemptions and /or prohibits
retrospective application:
Recognise all assets and liabilities required by IFRS
Not recognise assets and liabilities not permitted by IFRS
Reclassify all assets and liabilities and equity in accordance with IFRS
Measure all assets and liabilities in accordance with IFRS
Any gains and losses arising from this exercise should be recognised immediately in
retained earnings as at January 2016
(iii) Since IAS 1 requires that at least one year of comparative prior period financial
information be presented, the opening Statement of Financial Position will be 1
January 2015, if not earlier.
(iv) Preparation of full IFRS Financial Statements for the year ended 31 December
2016, which should include:
 three statements of financial position
 two statements of comprehensive income
 two separate statements of profit or loss (if presented)
 two statements of cash flows
 two statements of changes in equity
 related notes, including comparative information
(c) The reconciliation which the company must include in its financial statements for the year
ended 31 December 2102, to explain how the transition from previous GAAP to IFRS
affect the reported financial position, financial performance and cash flows are as follows:
(i) Reconciliation of equity reported under previous GAAP to equity under IFRS both
(a) at the date of the opening IFRS Statement of Financial Position and (b) the end
of the last annual period reported under the previous GAAP.
(ii) Reconciliation of Total Comprehensive Income under IFRS for the last annual period
reported under the previous GAAP to Total Comprehensive Income under IFRS for
the same period.
(iii) Explanation of material adjustments that were made, in adopting IFRS for the first
time, to the Statement of Financial Position, Statement of profit or loss and
Statement of Cash Flows.
(iv) If errors in previous GAAP financial statements were discovered in the course of
transaction to IFRS, those must be separately disclosed.
(v) If the entity recognised or reversed any impairment losses in preparing its opening
IFRS Statement of Financial Position, these must be disclosed.
(vi) Appropriate explanations if the entity has elected to apply any of the specific
recognition and measurement exemptions permitted under IFRS 1, for example, if
the entity used fair values as deemed cost.

© Emile Woolf International 359 The Institute of Chartered Accountants of Pakistan


Advanced accounting and financial reporting

(d) Contents of a typical statement of changes in equity are as follows:


(i) Total comprehensive Income for the period, showing separately amounts
attributable to owners of the parent and to non-controlling interests.
(ii) For each component of equity, the effect of retrospective application or retrospective
restatement recognised in accordance with IAS 8.
(iii) For each component of equity, a reconciliation between the carrying amount at the
beginning and end of the period, separately disclosing
 Profit or loss
 Other comprehensive income
 Transactions with owners in their capacity as owners showing separately,
contributions by and distributions to owners, and changes in ownership
interests in the subsidiaries that do not result in a loss of control.

© Emile Woolf International 360 The Institute of Chartered Accountants of Pakistan


Answers

CHAPTER 32 – SPECIALISED FINANCIAL STATEMENTS

32.1 IFRS FOR SMES


(a) (i) IFRS, were not designed specifically for a class of companies – quoted or unquoted.
IFRSs give financial statements enhanced reliability, relevance and credibility and
result in fair presentation.
(ii) SMEs’ would wish to comply with IFRSs for consistency and easy comparability with
similar SMEs’ both within their own country and internationally.
(iii) The objectives of general purpose Financial Statements are basically appropriate for
SMEs and bigger publicly quoted companies alike. Given this, one set of IFRS that
would be used nationally and internationally is desirable.
(iv) The cost burden of applying the full set of IFRSs may not be justified on the basis of
user needs. The purpose and usage of the Financial Statements, and the nature of
the accounting expertise available to the SMEs, will not be the same as for listed
companies. These may provide justification for a separate set of IFRS for SMEs.
(v) Where attention is devoted to adopt local Generally Accepted Accounting Principle
(GAAP) for SMEs on a national basis and IFRSs for listed companies, this practice
of adopting GAAP for SMEs may differ between countries when applied by such
SMEs thus making comparability of Financial Statements difficult across national
boundaries.
(b) (i) Most SMEs have a narrower range of users than publicly quoted companies. Users
of SMEs Financial Statements are basically owners, lenders and suppliers of the
merchandize and tax authorities.
(ii) In deciding the form of modification to be made to IFRSs, the needs of these users
must be taken into consideration. Also the financial burden imposed by the
compliance with IFRSs by these SMEs must be identified.
(iii) There will have to be a relaxation of some of the measurement and recognition
criteria in IFRSs in order to achieve the reduction in the costs and burden of
implementation.
(iv) Some disclosures, such as Earnings Per Share, segment reporting, etc, may not be
relevant to SMEs and therefore may not be needed.
(v) A review of these disclosures requirements in IFRSs will be required to assess their
appropriateness for SMEs.
(c) IFRSs for SMEs would not necessarily deal with all the recognition and measurement
issues facing an entity but the key issues should revolve around the nature of the
recognition, measurement and disclosure of the transactions of SMEs. In the case where
the item is not dealt with by the standards, there are three alternatives:
(i) the entity can look to the full IFRSs to resolve the issue.
(ii) management judgement can be used with reference to the framework and
consistency with other IFRSs for SMEs.
(iii) existing practice could be used.

© Emile Woolf International 361 The Institute of Chartered Accountants of Pakistan


Advanced accounting and financial reporting

32.2 AKMAL GENERAL INSURANCE LIMITED

© Emile Woolf International 362 The Institute of Chartered Accountants of Pakistan


Answers

32.3 MAHFOOZ GENERAL INSURANCE LIMITED


Mahfooz General Insurance Limited
Statement of claims for the year ended June 30, 2016
Business underwritten inside Pakistan

Rupees in million

Reinsurance
and the

Net claims
Reinsurance and other

Reinsurance and other

expense
Outstanding recoveries in

recoveries received

recoveries revenue
Claims expenses
Total claims paid

claims respect of
outstanding
Class
claims
Opening

Opening
Closing

Closing
2016

Direct and
facultative
Fire and
property
damage 900 600 500 800 600 500 350 450 350
Marine,
aviation &
transport 450 400 450 500 300 300 400 400 100
Motor 1,150 900 750 1,000 850 700 550 700 300
Accident and
health 250 300 150 100 160 150 80 90 10

Total 2,750 2,200 1,850 2,400 1,910 1,650 1,380 1,640 760
Treaty
Proportional 13 10 12 15 - - - - 15

2,763 2,210 1,862 2,415 1,910 1,650 1,380 1,640 775

© Emile Woolf International 363 The Institute of Chartered Accountants of Pakistan


Advanced accounting and financial reporting

32.4 DEE GENERAL INSURANCE LIMITED


Dee General Insurance Limited
Consolidated statement of expenses for the year ended 31 December 2016

Deferred

Net commission

from reinsurers
Commissions

management

Underwriting

underwriting
Commission
commissions

expense

expense

expense
Other
Class

Net
Opening

Closing
Direct and
(Rs. in million)
Facultative
Fire and
property 321.41 148.79 160.43 309.77 165.28 475.07 270.44 204.61
damage
Marine,
aviation and 126.87 11.31 5.68 132.50 139.96 272.46 5.70 266.76
transport
Motor 215.00 128.50 114.23 229.27 499.93 729.20 12.72 716.48
Miscellaneous 90.94 38.59 35.17 94.36 172.70 267.06 82.40 184.66
1,372.
754.22 327.19 315.51 765.90 977.87 1,743.77 371.26
51
Treaty
Proportional 0.30 - - 0.30 0.13 0. 43 - 0.43
1,372.
Grand total 754.52 327.19 315.51 766.20 978.00 1,744.20 371.26
94

32.5 BANK LATEEF BANK LIMITED


8 Lendings to financial institutions
Notes 2016 2015
Rs. in million

Call money lending 8.2 850 1,200


Repurchase agreement lending
(reverse repo) 8.3 2,100 2,850
2,950 4,050

8.1 Particulars of lending


In local currency 2,840 3,900
In foreign currencies 110 150
2,950 4,050
8.2 These are unsecured lendings to financial institutions, carrying mark up ranging from
15% to 17% (2015: 10% to 12 % and will mature latest by October 2016.
8.3 These are short term lendings to various financial institutions and are secured against
government securities shown in note 8.4 below. These carry mark up at rates ranging
from 9.5% to 13.2 % (2015:8% to 10.5 %) and will mature on various dates, latest by
October 2016.

© Emile Woolf International 364 The Institute of Chartered Accountants of Pakistan


Answers

8.4 Securities held as collateral against lending to financial institutions


Rs. in million
2016 2015
Held Further Held Further
by given as by given as
bank collateral Total bank collateral Total
Market treasury
bills 1,650 - 1,650 1,850 - 1,850
Pakistan
investment
bonds 450 - 450 1,000 - 1,000
2,100 - 2,100 2,850 - 2,850

Market value of the above as at September 30, 2016 amounted to


Rs. 2,250 million 2015: 2,930 million).

32.6 SECURED BANK LIMITED


Secured Bank Limited
Notes to the financial statements for the year ended December 31, 2016
2016 2015
Rs. in million
9. INVESTMENTS BY SEGMENTS
Federal government securities
Market treasury bills 366 309
Pakistan investment bonds 69 61
Government of Pakistan bonds (USD/Euro) 26 30
Investments in associated undertakings 9 8
Fully paid ordinary shares
Listed companies 6 5
Unlisted companies 2 3
Bonds, participation term certificates & term finance
certificates
Listed securities 19 30
Unlisted securities 260 210
Other investments
Overseas government securities 60 52
Investments of mutual funds 32 28
Others 19 29
868 765
Less: Provisions for diminution in value of investments 9.1 (45) (39)
Net investments 823 726
2016 2015
9.1 Particulars of provision for diminution in value of
Rs. in million
investments
Opening balance 39 28
Charge for the year 17 12
Impairment / (reversals) (6) 2
Amounts written off (5) (3)
6 11
Closing balance 45 39

© Emile Woolf International 365 The Institute of Chartered Accountants of Pakistan


Advanced accounting and financial reporting

32.7 AL-AMIN BANK LIMITED


Al-Amin Bank Limited
Notes to the Financial Statements for the year ended XX/XX/XX

1.1 Particulars on non-performing advances


Advances include Rs. 5,000 million which, as detailed below, have been placed
under non-performing status:
2016
Category of Amount outstanding Provisions required and held
classification Domestic Overseas Total Domestic Overseas Total
Rs. in million
Other assets
especially
mentioned 100 - 100 5 - 5
Sub-standard 400 260 660 70 50 120
Doubtful 840 - 840 530 - 530
Loss 3,400 - 3,400 3,345 - 3,345
Total 4,740 260 5,000 3,950 50 4,000

1.2 Particulars of provision against non-performing advances


2016
Specific General Total
Rs. in million
Opening balance 3,320 65 3,385
Charge for the year 802 40 842
Amounts written off (50) - (50)
Reversals (90) - (90)
Exchange adjustments 18 - 18
Closing balance 4,000 105 4,105

32.8 BLUE-CHIP ASSET MANAGEMENT LIMITED


Rs. in '000'
Net assets at beginning of the year 350,050
Cash received / receivable on issuance of 765,900 units 85,015
Cash paid / payable on redemption of 717,480 units (77,488)
7,527
357,577
Element of (income) / loss and capital gains included in prices of
units issued less those in units redeemed – net (2,685)
Net income for the year (recognized income for the year) 65,325
Net assets at end of the year 420,217
Working
Sold Redeemed
No. of Units 765,900 717,480

Rupees in 000
Par value of units @ Rs. 100 76,590 71,748
Sale proceed / redemption value 85,015 77,488
Element of (income) / loss (8,425) 5,740
Net element (2,685)

© Emile Woolf International 366 The Institute of Chartered Accountants of Pakistan


Answers

32.9 A-ONE ASSET MANAGEMENT FUND LIMITED


A-One Asset Management Fund Limited
Statement of movement in unit holders' fund for the year ended March 31, 2016
2016
Rs. in
million
Net assets at the beginning of the year 27,000
Issue of 100 million units 3,500
Redemption of 95 million units (3,277)
223
27,223
Element of income and capital gains included in prices of units issued /
redeemed transferred to income statement (173)
Net unrealized appreciation of re-measurement of investments classified
as available for sale (1,800-1,200-480) 120
Capital gains 400
Net unrealized appreciation on re-measurement of investments classified
as financial assets at fair value through profit or loss (2,500-2,200) 300
Other net income for the year 3,000
Final distribution for the year ended March 31, 2015 at Rs. 4 per unit
(900*4) (3,600)
100
Net assets at the end of the year 27,270

32.10 IAS 26
(a) The differences between 1AS 26 - Accounting and Reporting by Retirement Benefit Plan
and IAS 19 - Employee Benefits are:
(i) IAS 26 addresses the financial reporting considerations for the benefit plan itself as
the reporting entity while IAS 19 deals with employers’ accounting for the cost of
such benefits as they are earned by the employees
(ii) These standards are thus somewhat related, but there will not be any direct inter-
relationship between the amounts reported in benefit plan financial statements and
amounts reported under IAS 19 by employers.
(iii) IAS 26 differs from IAS 19, Employee Benefits, in allowing a choice of measurement
based either on current salary levels or projected salary levels. IAS 19 requires an
actuarial valuation to be based on the latter, whereas IAS 26 requires valuation
based on present value of promised retirement benefits.
(b) Defined Benefit Plan (DBP)
Defined benefit plans are retirement benefit plans under which amounts to be paid as
retirement benefits are determined by reference to a formula usually based on employees’
earnings and/or years of service.
(c) Defined Contribution Plan (DCP)
Defined contribution plans are retirement benefit plans under which amounts to be paid as
retirement benefits are determined by contributions to a fund together with investment
earnings thereon.
(d) Actuarial present value of promised retirement benefits: This is the present value of the
expected payments by a retirement benefit plan to existing and past employees
attributable to the service already rendered.

© Emile Woolf International 367 The Institute of Chartered Accountants of Pakistan


Advanced accounting and financial reporting

32.11 SOGO LIMITED


(a) SOGO Limited
Staff Gratuity Fund
Statement of net assets available for benefits as at December 31, 2016
Note 2016
Rupees
ASSETS
Investments 3 159,033,144
Receivable from SOGO Limited 1,147,150
Cash at bank in current accounts 17,930,120
178,110,414
LIABILITIES
Due to outgoing members 4,301,017
Accrued expenses 3,822
Withholding tax payable 61,251
4,366,090
NET ASSETS 173,744,324
REPRESENTED BY:
Members' Fund
(Rs. 142,472,122 + Rs. 27,712,441) 170,184,563
Surplus on re-measurement of investments available for
sale 3,559,761
173,744,324
(b) SOGO Limited
Staff Gratuity Fund
Statement of changes in net assets available for benefits for the year ended
December 31, 2016
Income
Contribution during the year 10,623,106
Profit from investments 23,389,251
Dividend income 2,696,399
Liabilities no more payable 3,450,000
40,158,756
Expenditure
Transferred / paid to outgoing members (12,432,973)
Bank charges (3,342)
Audit fee (10,000)
(12,446,315)
Net Income for the year 27,712,441

© Emile Woolf International 368 The Institute of Chartered Accountants of Pakistan


Profit / Profit /
Balance Principal Balance
Addition interest Fair value interest
as at realized as at
during the accrued gain / realized

© Emile Woolf International


July 01, during the June 30,
year during the (loss) during the
2015 year 2016
year year

HELD TO MATURITY

Government Securities

Defense Saving Certificates 87,812,855 - 21,376,809 - (1,600,000) (5,456,000) 102,133,664

Unlisted Securities and Deposits


-

369
Term Finance Certificates 19,943,656 5,000,000 1,655,223 (12,873,068) (1,893,722) 11,832,089

Term Deposit 6,414,058


-
119,341,142 5,000,000 23,389,251 (19,773,068) (7,577,514) 120,379,811

AVAILABLE FOR SALE

Listed Securities

SUN Ltd. 8,220,957 9,373,936 (784,518) - 16,810,375

PEACE Ltd. 587,169 - 317,728 - 904,897

NIT Units 16,911,510 - 4,026,551 - - 20,938,061

25,719,636 9,373,936 - 3,559,761 - - 38,653,333

145,060,778 14,373,936 23,389,251 3,559,761 (19,773,068) (7,577,514) 159,033,144

The Institute of Chartered Accountants of Pakistan


Answers
Advanced accounting and financial reporting

CHAPTER 36: ETHICAL ISSUES IN FINANCIAL REPORTING

36.1 ETHICAL ISSUES


The range of comments made by Arif raises questions over his ethical behaviour and
professional standards.
A chartered accountant should be unbiased when involved in preparing and reviewing financial
information. A chartered accountant should prepare financial statements fairly, honestly, and in
accordance with relevant professional standards and must not be influenced by considerations of
the impact of reported results.
Arif’s failings
Arif appears to be influenced by the need to achieve a specified level of profit. This is not
appropriate and calls his integrity into question.
In addition Arif’s professional competence seems to be suspect. His comment on not being up to
date on all of the little technicalities in IFRS s suggests that he has not maintained a level of
professional competence appropriate to his professional role.
ICAP members have a responsibility to engage in continuing professional development in order
to ensure that their technical knowledge and professional skills are kept up to date. Arif should
seek continuing professional development activities and improve his knowledge on ethical
standards. Furthermore, it might be expected that as Waheed’s superior he should set an
example to Waheed and guide him in his responsibilities. Clearly this is not happening.
As a member of ICAP Arif should be aware of the ICAP code of ethics. Arif should know of the
danger of self-interest threats and intimidation threats to himself and to others. His attempt to
influence the outcome of a fellow professional by applying such a threat to that individual is very
unprofessional.
Waheed’s ethical issues
Waheed faces a self-interest threat, in that there is the possibility of a bonus provided the
earnings per share figure remains the same as last year. Arif has also suggested that he can
influence the Board’s decision over employing him as a replacement finance director – another
self-interest threat to Waheed. Both of these threats must be ignored.
Arif’s comments imply that his application of professional responsibility is lacking. This may
extend into the way in which the current financial statements have been prepared. Waheed must
be very careful (as always) to carry out the review with all due care.
Waheed should first discuss his recommendations with Arif and remind him of his professional
responsibilities to ensure that the accounting standards are correctly followed. If the financial
statements are found to contain errors or incorrect accounting treatment then they must be
amended. If Arif refuses to amend the draft financial statements if necessary Waheed should
discuss the matter with other board members (including non- executives and the audit
committee, if possible). Further action might include consulting with ICAP.

36.2 SINDH INDUSTRIES LTD


(a) Financial reporting issues
Revenue
IAS 18 Revenue sets out the rules to be followed in recognising revenue.
The fact that the customer cannot cancel the contract is not relevant to the recognition of
revenue. Revenue from providing a service is recognised according to the stage of
completion subject to satisfying criteria set out in IAS 18. In the absence of other
information the revenue in this contract should be recognised over the life of the contract
as time progresses. As the contract was only signed just before the year end, none of the
revenue can be recognised in 2016.

© Emile Woolf International 370 The Institute of Chartered Accountants of Pakistan


Answers

The credit for the amount received should be recognised as a liability. This represents the
obligation that the company has to provide the service over the next two years.
The fact that the customer cannot cancel the contract is not relevant to the recognition of
revenue. If Sindh Industries failed to provide the service they would be sued for restitution.
Therefore the revenue can only be recognised as the service is provided.
New factory
Borrowing costs directly attributable to construction of an asset which necessarily takes a
substantial period to get ready for its intended use should be capitalised as part of the cost
of that asset under IAS 23 Borrowing Costs. IAS 23 states that the capitalisation of
borrowing costs should commence when three conditions are all met for the first time:
borrowing costs are being incurred, expenditure is being incurred and activities to prepare
the asset are being undertaken. Although borrowing costs were incurred throughout the
year and expenditure was incurred from 1 February 2016 (the date the land was
purchased), construction only started on 1 June 2016. Therefore this is the date on which
capitalisation commences.
Capitalisation ceases when substantially all of the activities required to make the asset
ready for use/sale have been completed, that is on 30 September 2016. (The actual date
on which the factory was brought into use is irrelevant.) Therefore the period of
capitalisation should be four months.
Where construction is financed from general borrowings, the calculation of the amount to
be capitalised should be based on the weighted average cost of borrowings. This is:
(Rs. 1,000,000 × 9.75%) + (Rs. 1,750,000 × 10%) + (Rs. 2,500,000 × 8%)/ (Rs. 1,000,000
+ Rs. 1,750,000 + Rs. 2,500,000) = 9%
Therefore the amount capitalised should be 9% × Rs. 4.5 million (land Rs. 1.8 million plus
construction costs Rs. 2.7 million) × 4/12 = Rs. 135,000. The total cost of the factory
should be measured at Rs. 4,635,000 (Rs. 1.8 million plus Rs. 2.7 million, plus Rs.
135,000). The amount that has been recognised in the statement of financial position
should be reduced by Rs. 315,000 (Rs. 450,000 – Rs. 135,000). Finance costs recognised
in profit or loss should be increased by Rs. 315,000.
Land should not be depreciated because it has an indefinite life. Under IAS 16 Property,
Plant and Equipment depreciation charges should start when the asset becomes available
for use, from 1 October 2016 in this case. Depreciation of Rs. 35,000 ((Rs. 2.7 million, plus
(Rs. 135,000 × 2.7/4.5) ÷ 20) × 3/12) should be recognised in profit or loss for the year
ended 31 December 2016 and the carrying amount of the asset reduced by the same
amount to Rs. 4.6 million.
Useful life of the blast furnace
Depreciation of the blast furnace has been based on an estimated useful life of 20 years.
This is at variance with a report by a qualified expert. The asset valuation specialist treats
the furnace as being made up of two components, the main structure and the lining, which
must be replaced at regular five yearly intervals over the life of the asset. This is the
approach required by IAS 16. The uncertainties inherent in business mean that many
items in financial statements cannot be measured with certainty, but estimates should
always be made using the most up to date and reliable information. Where estimates have
been prepared by professionals with relevant qualifications, then it is nearly always most
appropriate to use those estimates. Therefore in accordance with the valuer’s report the
main structure of the furnace should be depreciated over 15 years from 1 January 2016
and the lining should be depreciated over five years from that date.
The reassessment of the estimated lives of assets is a change in accounting estimate,
rather than a change in accounting policy (IAS 8 Accounting Policies, Changes in
Accounting Estimates and Errors). Changes in accounting estimate should be dealt with on
a prospective basis. This is achieved by including the effect of the change in profit or loss
in current and future periods. The additional depreciation should be calculated as:

© Emile Woolf International 371 The Institute of Chartered Accountants of Pakistan


Advanced accounting and financial reporting

Rs.000
Revised depreciation: main structure 140
((Rs. 3.5m – Rs. 1.4m)/15 years)
lining (Rs. 1.4m/5 years) 280
420
Current depreciation (Rs. 3.5m/20 years) (175)
Additional depreciation 245

IAS 8 requires the disclosure of the nature and amount of the effect of the change in the
estimate of useful lives on the profit for the year.
(b) Revised financial statements
Statement of profit or loss extract for the year ended 31 December 2016
Borrowing Blast
Draft Revenue costs furnace Revised
Rs.000 Rs.000 Rs.000 Rs.000 Rs.000
Profit before tax 2,500 (1,000) (315)+ (35) (245) 905

Statement of financial position at 31 December 2016


Borrowing Blast
Draft Revenue costs furnace Revised
Rs.000 Rs.000 Rs.000 Rs.000 Rs.000
Non-current
assets
Property, plant and
equipment 12,000 (315) + (35) (245) 11,405
Current assets 3,500 3,500
Total assets 15,500 14,905

Share capital 2,000 2,000


Retained earnings 6,000 (1,000) (315) + (35) (245) 4,405
Equity 8,000 6,405
Non-current
liabilities 5,000 500 5,500
Current liabilities 2,500 500 3,000
Total equity and
liabilities 15,500 14,905
(c) Ethical issues
It is noticeable that all the adjustments required reduce profit. This and the background to
the previous finance director’s resignation suggest serious problems.
It is not clear who actually prepared the draft financial statements. If they were prepared by
more junior staff in the absence of a finance director, some of the adjustments (for
example, the calculation of borrowing costs to be capitalised) could be the result of
genuine errors or lack of accounting knowledge. However, it seems reasonably clear that
the managing director has attempted to influence the treatment of the revenue and the
estimated useful life of at least one significant non-current asset. (Note: the directors have
reviewed the useful lives of several items of plant and machinery and it is possible that
other assets besides the furnace are being depreciated over unrealistically long periods.)

© Emile Woolf International 372 The Institute of Chartered Accountants of Pakistan


Answers

It seems almost certain that the previous finance director resigned as a result of pressure
from the managing director (and possibly from other members of the Board) to present the
financial statements in a favourable light. The directors intend to seek a stock market
listing in the near future. Therefore they have clear motives for manipulating the profit
figure and also (perhaps) for making controversial decisions before the financial
statements come under much greater scrutiny as a result of the listing. The job title of
financial controller is also significant. It suggests that the role has been downgraded and
that the person holding it has less authority than the rest of the Board.
Possible courses of action:
 Discuss with the managing director the financial reporting standards that apply to
the transactions and explain the implications of non-compliance. If the managing
director is himself a member of a professional body then it might be worth pointing
out to him that he himself is bound by an ethical code.
 Advise him that as a Chartered Accountant you are bound by the ICAP code of
ethics, and that you would not be prepared to compromise your views of the figures
he has prepared for career advancement.
 Consider speaking to the other directors (or audit committee if there is one) and
seeking their support.
 If all of these actions produce a negative response then it would be appropriate to
consult the ICAP ethical handbook and/or the Institute.
 If all else fails then consider seeking alternative employment.

36.3 SOHAIB AND OMAR


The ICAP Ethical Code sets out fundamental principles which should guide the behaviour of an
accountant in all of his or her professional and business activities.
The code applies to full members and to student members who required to conduct themselves
to the highest standards in all professional engagements.
The fact that no fee is involved is irrelevant. Omar is calling upon Sohaib's professional expertise
as an accountant, and therefore this is a professional engagement. The Ethical Code therefore
applies.
It is important that accountants are guided not only by the specific terms and examples provided
in the Code, but also by the underlying spirit of the Code.
A key fundamental principle of the Code is that of integrity: a professional accountant must be
straightforward and honest in all professional and business relationships. Following the spirit of
the Code, the source of the financial information about Gethsemene is questionable, and Sohaib
should not produce a report that uses it without further investigation.
It is important in professional relationships with clients that the chartered accountant should avoid
the threat of familiarity. This does not preclude a professional relationship with someone who is a
friend, but care must be taken that the friendship does not present a familiarity threat in the
context of the work undertaken.
Sohaib should explain to Omar that he cannot use Gethsemene's draft financial statements
without obtaining permission directly from Gethsemene.
Also he should establish a proper, professional relationship with Omar for the purposes of this
work. This could involve setting out the nature of the work to be done, and Sohaib's obligations in
respect of it, in an engagement letter.
If Sohaib is in need of advice, he could seek it from a more experienced chartered accountant in
his firm (if applicable; no details of Sohaib's current employment are supplied in the scenario) or
he could contact the ICAP to discuss the matter confidentially.

© Emile Woolf International 373 The Institute of Chartered Accountants of Pakistan


Advanced accounting and financial reporting

36.4 ABBAS AND BASHIR


As a chartered accountant, Abbas is expected to follow the guidance set out in the ICAP's Code
of Ethics. This is a principles-based code, and chartered accountants are expected to be able to
identify threats to compliance with the code's principles, to evaluate the significance of the threats
and to implement safeguards to eliminate the threats or to reduce them to an acceptable level. It
appears that Abbas has identified a potential threat to compliance, but is currently struggling to
evaluate the significance of the threat and to formulate a plan to address any significant threat.
The fundamental principles in the Code of Ethics include objectivity and confidentiality. It is
expected that a chartered accountant should not allow bias, conflict of interest or undue influence
by others to override his or her judgement. Also, a chartered accountant is expected to keep their
business dealings with clients strictly confidential.
In the current situation, there are possible threats to objectivity and confidentiality. In his role as a
consultant to BigShop Limited and Nourish Limited who have an interdependent business
relationship, Abbas may be party to confidential and commercially sensitive information about
both companies. The extent to which this is a problem depends upon the nature of the work that
Abbas does for these clients.
If it is limited to the analysis of publicly available information (as in the analysis conducted in the
above report) there is unlikely to be a problem. However, if he is party to confidential information,
as seems likely, a conflict of interest may exist. Abbas could find it difficult to retain objectivity if
he is dealing with both clients, and he would have to be very careful about confidentiality. The
next step for Abbas is to assess the significance of the threats. It is not possible to determine
conclusively from the information given whether or not these threats exist. Abbas must do this for
himself, but he may be able to obtain advice from other chartered accountants in his
organisation.
Because Insight Ltd employs multi-disciplinary teams of professionals, it is possible that the firm
has experience of these conflicts, and policies to deal with them. Abbas should make himself
familiar with the firm's policies and assess the extent to which they do, or do not address, the
threats he has identified.
In the absence of firm's policies to deal with such issues, Abbas may be obliged to decline
involvement in the work relating to one or other of the clients. He should discuss the ethical issue
with his team leader, Bashir. Because he is not himself a chartered accountant, he may not be
familiar with the Ethical Code governing Abbas's activities as a chartered accountant, and he
may need to explain it to him.
If Abbas is experiencing difficulties because of an absence of policies within his firm, or because
there is no one available with whom he can discuss ethical issues, it would be appropriate to call
the ICAP's ethics helpline for the opportunity to discuss the issues and obtain advice.

© Emile Woolf International 374 The Institute of Chartered Accountants of Pakistan

S-ar putea să vă placă și